You are on page 1of 283

Sports Medicine Scored and Recorded Self-Assessment Examination 2019

1. Based on the injury shown on the axial MRI scan of the shoulder in Figure 1, what other
pathology should be closely examined for during surgery?

a. Subscapularis tear
b. Supraspinatus tear
c. Superior labral anterior-posterior (SLAP) tear
d. Bankart tear

Preffered Answer : 1
The axial MRI scan reveals a subluxated biceps tendon. In the study by Koh and associates, 85%
of patients with a biceps subluxation on MRI were found to have a subscapularis tear at the time
of arthroscopy. These are not always obvious on the MRI, and close inspection of the leading
edge/upper border of the subscapularis tendon at the time of arthroscopy is necessary. Although
supraspinatus tears, SLAP tears, and Bankart tears can all occur in conjunction with a biceps
subluxation, none have been shown to be strongly correlated with this pathology, nor as specific
to this pathology.

2. Figure 1 is the radiograph of a 31-year-old man who had left shoulder pain after a fall during a
snowboarding jump. Residual displacement of 5 mm after closed reduction is most likely to result
in

Figure 1
a. nonunion.
b. osteonecrosis.

UI // UNAIR // UNPAD // UNHAS // UNS // UGM // UB // UNUD // USU


Sports Medicine Scored and Recorded Self-Assessment Examination 2019

c. altered rotator cuff mechanics.


d. normal shoulder function.

Preffered Answer 3
Humerus fractures account for 11% of all fractures among snowboarders and are the second-
most-common upper-extremity fracture after radius fractures (48%). Surgical fixation is
recommended for fractures with residual displacement >5 mm, or >3 mm in active patients
involved in frequent overhead activity. Malunion can result in a mechanical block to shoulder
abduction or external rotation and altered rotator cuff mechanics, causing weakness. A rich
arterial network provides a favorable healing environment for greater tuberosity fractures.
Consequently, nonunion and osteonecrosis are uncommon.

3. A 23-year-old student complains of recurrent left shoulder instability. He first dislocated his
shoulder in high school while playing lacrosse and was managed with physical therapy. A second
dislocation occurred one year later while skiing. He has since sustained two more dislocations
and says that his shoulder feels ―loose.‖ Examination reveals grade II anterior load and shift,
positive apprehension and relocation tests, and normal rotator cuff strength. An MRI arthrogram
is ordered and surgical treatment is recommended. What factor would most strongly represent an
indication for a procedure including bone augmentation (e.g. Latarjet) rather than a soft-tissue-
only stabilization (isolated labral repair/capsulorrhaphy)?

a. Patient’s intention to resume lacrosse and other contact sports after surgery
b. Presence of a 270° labral tear
c. 2-cm ―on-track‖ Hill-Sachs lesion
d. Anterior bony loss measuring 30% of inferior glenoid width

Preffered Answer: 4
There is much debate in the literature regarding optimal techniques for treatment of shoulder
instability. Barring other factors or concomitant pathology, however, there is no persuasive
literature to suggest routine use of bone augmentation for contact athletes. Extensive labral
involvement (here specifically implying posterior labral involvement, as well) will require a more
extensive repair but does not, in and of itself, suggest the necessity for glenoid bone
augmentation. A large Hill-Sachs lesion may be an indication for glenoid augmentation, primarily
if it is in a location/orientation that engages the anterior glenoid rim. These are referred to as ―off-
track‖ lesions. Of these choices, the strongest indication for a Latarjet coracoid transfer or similar
bone augmentation (other options include iliac crest autograft or distal tibial allograft) is high-
grade glenoid bone loss. Classically, this is performed through an open approach, although
arthroscopic techniques are increasing in popularity. Although the critical amount of bone loss is
debated, most surgeons and studies suggest a cut-off of approximately 20% to 25%, above
which isolated soft-tissue stabilization alone is less likely to be successful in the long-term.

4. A 32-year-old volleyball player has dull posterior shoulder pain. An examination reveals
moderate external rotation weakness with his arm at his side but normal strength on
supraspinatus isolation. Deltoid and supraspinatus bulk appear normal, although there appears
to be mild infraspinatus atrophy. Sensation is normal throughout the shoulder and shoulder
girdle. What is the most likely diagnosis?

a. Calcified transverse scapular ligament


b. Parsonage-Turner syndrome

UI // UNAIR // UNPAD // UNHAS // UNS // UGM // UB // UNUD // USU


Sports Medicine Scored and Recorded Self-Assessment Examination 2019

c. Spinoglenoid notch cyst


d. Quadrilateral space syndrome

Preffered Answer: 3
This clinical scenario describes a patient with an isolated injury affecting the infraspinatus muscle.
The anatomic location of such a lesion would be at the spinoglenoid notch, at which the
suprascapular nerve may be compressed distal to its innervation of the supraspinatus but
proximal to the infraspinatus innervation. A calcified transverse scapular ligament would also
affect the suprascapular nerve but is proximal to the innervation of both muscles. Quadrilateral
space syndrome would affect innervation of the deltoid (and teres minor). Parsonage-Turner
syndrome is a more diffuse, and often severely painful, brachial plexus neuropathy.

5. Figure 1 is an MRI scan of the right hip of a 19-year-old woman with a 6-month history of right
groin pain. She was diagnosed with a stress fracture and was treated with 3 months of limited
weight bearing. Figure 2 is a repeat MRI scan in which the edema pattern changed minimally but
the pain worsened. Ibuprofen alleviates most of her pain. What is the best next step?

Figure 1 Figure 2

a. Hip arthroscopy with labrum repair


b. MRI arthrogram
c. Percutaneous screw fixation
d. CT scan with fine cuts
Preffered Answer: 4
An osteoid osteoma is a benign bone tumor. Osteoid osteomas tend to be small—typically <1.5
cm. Regardless of their size, they cause a large amount of reactive bone to form around them,
and they make a new type of abnormal bone material called osteoid bone. This osteoid bone,
along with the tumor cells, forms the nidus of the tumor, which is easily identified on CT scans.

6. When reconstructing the anterior cruciate ligament (ACL) with autograft, what is the most
common source of surgical failure?

a. Graft choice
b. Tunnel position
c. Tibial fixation
d. Femoral fixation

UI // UNAIR // UNPAD // UNHAS // UNS // UGM // UB // UNUD // USU


Sports Medicine Scored and Recorded Self-Assessment Examination 2019

Preffered Answer: 3
Technical failure is the most common reason for ACL reconstruction failure. Tunnel position is
the most frequent cause for technical failure. Malpositioning of the tunnel affects the length of the
graft, causing either decreased range of motion or increased graft laxity. Although graft choice is
an important factor when planning an ACL reconstruction, overall outcomes with autograft
tissues are fairly similar. Fixation of the graft at the femoral or tibial end is not as important as
tunnel position.

7. An otherwise healthy 31-year-old man has had right knee pain for the past 9 months. His former
physician administered a cortisone injection and ordered 6 months of physical therapy. The
patient later had an arthroscopy with debridement of the right knee by another physician and
completed another course of physical therapy. The patient received minimal relief from these
treatments and still is not able to walk longer distances or go on hikes. On examination, he is a
healthy appearing male with a body mass index of 24 kg/m 2. He has a small effusion, minimal
quadriceps atrophy, no tenderness about the knee, full range of motion, stable to varus and
valgus stress at 30° of flexion, a grade 1 Lachman test, and a normal posterior drawer. Figures 1
through 4 are his arthroscopic views, radiograph and MRI scan from his prior surgical procedure.
What is the next most appropriate step in treatment?

Figure 1 Figure 2

Figure 3 Figure 4

a. Bracing with physical therapy focusing on quadriceps/vastus medialis obliquus (VMO) and
hamstring strengthening

UI // UNAIR // UNPAD // UNHAS // UNS // UGM // UB // UNUD // USU


Sports Medicine Scored and Recorded Self-Assessment Examination 2019

b. Osteotomy
c. Osteochondral allograft to femoral condyle
d. Arthroscopy with femoral condyle microfracture

Preffered Answer: 3
The patient has a symptomatic cartilage lesion of his medial femoral condyle, which has not
responded to nonsurgical measures, and he failed a prior arthroscopy with debridement. Based
on his examination and imaging, he is ligamentously stable, has normal mechanical alignment,
and has intact menisci, making him a candidate for a cartilage restoration procedure. The
accompanying MRI also indicates subchondral bone involvement with increased T2 signal
underlying the cartilage defect. Osteochondral allograft is the only choice that addresses both the
cartilage defect, as well as compromised subchondral bone. Depending on lesion size,
osteochondral autograft transfer may also be considered, but this is not presented as an answer
choice.

Given the radiographic finding of neutral mechanical alignment, bracing would be less effective,
and the patient has already tried extensive physical therapy. Lack of malalignment also excludes
tibial osteotomy as a preferred answer choice. Microfracture is best for small cartilage lesions
without significant bone marrow involvement.

8. A collegiate lacrosse player is struck on the head by an opposing player’s stick. She is initially
unresponsive. She regains consciousness within 2 minutes but remains confused and
uncooperative, complaining of head and neck pain. This is her second concussion of the
calendar year. Initial management should consist of

a. calculation of Glasgow Coma Scale score.


b. evaluation with a sideline assessment tool, such as the SCAT-3.
c. urgent hospital transfer for CT scan.
d. stabilization of the cervical spine and placement of a collar

Preffered Answer: 4
This patient has sustained a significant concussion or minor brain injury. Although all answer
options reflect important steps in her management, the initial primary concern in any player who
is confused or combative is protection of the cervical spine until formal clearance can be
performed. This patient requires immediate immobilization, collar placement, and, ultimately,
transportation to a hospital. Cervical immobilization should be achieved before transport, given
her complaints of neck pain and inability to provide a reliable examination

9. Figures 1 and 2 are the MR arthrogram images of a 16-year-old, right-hand-dominant baseball


player who injured his left shoulder 4 weeks ago during a game. He now has pain, weakness,
and the inability to swing a bat and can no longer do push-ups. He denies prior injury to his left
shoulder. Radiographs are unremarkable.

Which of the four muscles of the rotator cuff provides the most resistance to this patient's
direction of instability?

UI // UNAIR // UNPAD // UNHAS // UNS // UGM // UB // UNUD // USU


Sports Medicine Scored and Recorded Self-Assessment Examination 2019

a. Subscapularis
b. Supraspinatus
c. Infraspinatus
d. Teres minor
Preffered Answer: 1
Posterior shoulder instability is a rare form of instability that often presents with pain rather than
feelings of instability. It often occurs in young athletes during activities that put the shoulder in an
―at-risk position‖ (flexion, adduction, internal rotation). Repetitive microtrauma can lead to posterior
shoulder instability such as seen in football linemen. Swinging a bat or golf club places the lead
arm in a flexed, adducted, and internally rotated position, which can lead to posterior translation of
the humeral head that is forcibly reduced in follow-through, as seen in this patient.

The glenohumeral joint relies on static and dynamic stabilizers. Static stabilizers help prevent
instability at the end ranges of motion when the ligaments are taut. Dynamic stabilizers work to
prevent subluxation at midranges of motion, at which the ligaments are lax. The rotator cuff is
integral as a dynamic stabilizer of the shoulder. It works through a process called concavity
compression. The four muscles of the rotator cuff compress the humeral head into the concavity of
the glenoid-labrum. This prevents the humeral head from subluxing during the midranges of
motion. Of the four rotator cuff muscles, the subscapularis is most important at preventing
posterior subluxation.

This patient has posterior instability, and various surgical techniques may be indicated depending
on findings. Arthroscopic labral repair is indicated for anterior instability. Arthroscopic posterior
labral repair is indicated for this patient because he has a posterior labral tear and posterior
instability. If a patient has ligamentous laxity (not seen in this scenario because sulcus and
Beighton sign findings would be negative), a posterior capsular shift with rotator interval closure is
indicated. If a patient has excessive glenoid retroversion (not seen in this scenario with 5 degrees
of retroversion), a posterior opening-wedge osteotomy is appropriate.

The most common complication seen after arthroscopic posterior labral repair is stiffness, followed
by recurrent instability and degenerative joint disease.

10. Figures 1 and 2 are the MR arthrogram images of a 16-year-old, right-hand-dominant baseball
player who injured his left shoulder 4 weeks ago during a game. He now has pain, weakness, and
the inability to swing a bat and can no longer do push-ups. He denies prior injury to his left
shoulder. Radiographs are unremarkable.

UI // UNAIR // UNPAD // UNHAS // UNS // UGM // UB // UNUD // USU


Sports Medicine Scored and Recorded Self-Assessment Examination 2019

The patient fails an extensive course of physical therapy and is unable to return to baseball. He
and his orthopaedic surgeon elect to proceed with surgery. During a repeat evaluation, he has
negative sulcus and Beighton sign findings, and radiographs show 5° of glenoid retroversion. What
is the most appropriate surgical plan?

Figure 1 Figure 2

a. Arthroscopic infraspinatus tenodesis


b. Arthroscopic posterior labral repair
c. Arthroscopic capsular shift and rotator interval closure
d. Posterior glenoid opening-wedge osteotomy

Preffered Answer: 1
Posterior shoulder instability is a rare form of instability that often presents with pain rather than
feelings of instability. It often occurs in young athletes during activities that put the shoulder in an ―at-
risk position‖ (flexion, adduction, internal rotation). Repetitive microtrauma can lead to posterior
shoulder instability such as seen in football linemen. Swinging a bat or golf club places the lead arm in
a flexed, adducted, and internally rotated position, which can lead to posterior translation of the
humeral head that is forcibly reduced in follow-through, as seen in this patient.

The glenohumeral joint relies on static and dynamic stabilizers. Static stabilizers help prevent
instability at the end ranges of motion when the ligaments are taut. Dynamic stabilizers work to
prevent subluxation at midranges of motion, at which the ligaments are lax. The rotator cuff is integral
as a dynamic stabilizer of the shoulder. It works through a process called concavity compression. The
four muscles of the rotator cuff compress the humeral head into the concavity of the glenoid-labrum.
This prevents the humeral head from subluxing during the midranges of motion. Of the four rotator
cuff muscles, the subscapularis is most important at preventing posterior subluxation.

This patient has posterior instability, and various surgical techniques may be indicated depending on
findings. Arthroscopic labral repair is indicated for anterior instability. Arthroscopic posterior labral
repair is indicated for this patient because he has a posterior labral tear and posterior instability. If a
patient has ligamentous laxity (not seen in this scenario because sulcus and Beighton sign findings
would be negative), a posterior capsular shift with rotator interval closure is indicated. If a patient has
excessive glenoid retroversion (not seen in this scenario with 5 degrees of retroversion), a posterior
opening-wedge osteotomy is appropriate.

UI // UNAIR // UNPAD // UNHAS // UNS // UGM // UB // UNUD // USU


Sports Medicine Scored and Recorded Self-Assessment Examination 2019

The most common complication seen after arthroscopic posterior labral repair is stiffness, followed by
recurrent instability and degenerative joint disease.

11. Figures 1 and 2 are the MR arthrogram images of a 16-year-old, right-hand-dominant baseball
player who injured his left shoulder 4 weeks ago during a game. He now has pain, weakness, and
the inability to swing a bat and can no longer do push-ups. He denies prior injury to his left
shoulder. Radiographs are unremarkable.

If present, what is the most likely complication after surgical treatment in this scenario?

a. Recurrent instability
b. Degenerative joint disease
c. Shoulder stiffness
d. Axillary nerve injury
Preffered Answer: 1
Multiple studies have confirmed that cam or pincer anatomy is commonly present in
asymptomatic hips. According to a large systematic review, cam deformities are present in
approximately one-third of asymptomatic hips in young adults, and the proportion is higher than
50% in the subgroup of athletes.

Ganz and associates proposed that femoral acetabular impingement is the root cause of
osteoarthritis in most nontraumatic, nondysplastic hips, and functional improvement with surgical
correction of the deformity has been demonstrated. Despite the link between cam deformity and
hip osteoarthritis, a corresponding link between the correction of the deformity and prevention of
osteoarthritis has never been proven.

The results of cam deformity correction, typically including repair of the degenerative labral tear,
are much poorer when substantial joint space loss is present. A typical joint space cutoff of 2 mm
or less is used to recommend against hip preservation surgery.

UI // UNAIR // UNPAD // UNHAS // UNS // UGM // UB // UNUD // USU


Sports Medicine Scored and Recorded Self-Assessment Examination 2019

12. A 24-year-old collegiate pitcher has had increasing pain over his medial elbow for 3 months. He
has point tenderness over his medial epicondyle and reproduction of his symptoms with a valgus
stress test. What phase of the throwing cycle most likely will reproduce his symptoms?
a. Early cocking
b. Late cocking
c. Acceleration
d. Deceleration

Preffered Answer: 1
Highly cross-linked polyethylene makes material resistant to adhesive wear. Abrasive wear from
third bodies does not decrease wear. The fatigue strength of such material is inferior to that of
traditional polyethylene, and its resistance to creep is the same, if not lower, than that of
traditional polyethylene.
13. A 9-year-old girl, who is an avid soccer player, has intermittent spontaneous snapping in her left
knee that has worsened. There is no reported trauma or prior surgeries to her knee. Despite
working with her trainer, she has developed anterior-based knee pain and lacks full extension.
Her knee skin is unremarkable, but there is fullness to palpation on the lateral aspect of her knee.
Her range of motion demonstrates a lack of 15° of terminal extension and ligamentous
examination is unremarkable. Considering possible surgical treatments for this patient, what is
the most appropriate surgical treatment?

a. Arthroscopic lateral release with reconstruction of medial patellofemoral ligament


b. Growth plate sparing anterior cruciate ligament reconstruction
c. Arthroscopic meniscal saucerization
d. Microfracture versus stabilization of osteochondral lesion

Preffered Answer: 3
Surgical intervention of discoid meniscus is based on symptomatic patients. Complete discoid
menisci are typically stable but are expected to have >4.5 times incidence of surgical
intervention. Saucerization of symptomatic discoid meniscus is associated with better results with
younger patients with increases of poor outcomes in adult-aged patients. Meniscal transplant
may be an option, although long-term results are unknown.

14. Figure 1 is the MR image of a high school soccer player who sustained a right knee injury during
a game while making a cut toward the ball. He felt a pop and his leg gave way. During physical
examination, as the knee is moved from full extension into flexion with an internal rotation and
valgus force, you notice a "clunk" within the knee. What is the most likely biomechanical basis for
the "clunk"?

UI // UNAIR // UNPAD // UNHAS // UNS // UGM // UB // UNUD // USU


Sports Medicine Scored and Recorded Self-Assessment Examination 2019

Figure 1

a. In extension with internal rotation/valgus force, the medial tibial plateau is subluxated; with
flexion, the medial tibial plateau reduces.
b. In extension with internal rotation/valgus force, the medial tibial plateau is reduced; with
flexion, the medial tibial plateau subluxates.
c. In extension with internal rotation/valgus force, the lateral tibial plateau is reduced; with
flexion, the lateral plateau subluxates.
d. In extension with internal rotation/valgus force, the lateral tibial plateau is subluxated; with
flexion, the lateral plateau reduces.
Preffered Answer : 4
This patient sustained an isolated anterior cruciate ligament (ACL) injury based upon the
mechanism described and examination findings. The finding that produces the ―clunk‖ is
the pivot-shift maneuver, which is positive in a knee with an incompetent ACL. With an ACL-
deficient knee in full extension and internal rotation, the lateral tibial plateau subluxates anteriorly.
As the knee is flexed, the lateral tibial plateau slides posteriorly into a reduced position, causing
an audible clunk. Response D correctly describes the pathomechanics that result in the
audible clunk heard during the pivot-shift maneuver. Responses A and B are incorrect because
they describe the medial tibial plateau, which is not part of the pathomechanics of the pivot shift.
Response C is incorrect because in extension, the lateral tibial plateau is subluxated, not
reduced.

15. A 9-year-old boy is injured while playing soccer. His examination reveals painful range of motion
between 5° and 75°. There is tenderness on the medial side of his knee. There is no effusion, a
grade 1A Lachman test, and severe pain over the medial epicondyle of the knee. Varus stress is
negative and pain is elicited with valgus stress. Initial radiographs were negative for abnormality.
What is the next diagnostic step?

a. Repeat radiographs while the patient is weight bearing


b. Ultrasonography of the lower extremity and calf
c. Stress radiographs
d. CT scan
Preffered Answer: 3
This patient likely has a physeal injury to the distal femoral physis. Stress radiographs or an MRI
scan will most reliably reveal this diagnosis. The growth plate, when injured, is most commonly
fractured through the hypertrophic zone of cartilage, its weakest point. This patient is optimally
treated in a cylindrical or long-leg cast.

UI // UNAIR // UNPAD // UNHAS // UNS // UGM // UB // UNUD // USU


Sports Medicine Scored and Recorded Self-Assessment Examination 2019

16. A 9-year-old boy is injured while playing soccer. His examination reveals painful range of motion
between 5° and 75°. There is tenderness on the medial side of his knee. There is no effusion, a
grade 1A Lachman test, and severe pain over the medial epicondyle of the knee. Varus stress is
negative and pain is elicited with valgus stress. Initial radiographs were negative for abnormality.
Stress radiographs show a 2-mm medial physeal widening with valgus stress. What is the best
initial treatment strategy for this patient

a. Femoral medial collateral ligament repair, extraphyseal


b. Arthroscopically assisted medial collateral ligament repair
c. Crutch ambulation without immobilization and weight bearing as tolerated
d. Protected weight bearing with cast immobilization

Preffered Answer: 4
This patient likely has a physeal injury to the distal femoral physis. Stress radiographs or an MRI
scan will most reliably reveal this diagnosis. The growth plate, when injured, is most commonly
fractured through the hypertrophic zone of cartilage, its weakest point. This patient is optimally
treated in a cylindrical or long-leg cast.

17. A 9-year-old boy is injured while playing soccer. His examination reveals painful range of motion
between 5° and 75°. There is tenderness on the medial side of his knee. There is no effusion, a
grade 1A Lachman test, and severe pain over the medial epicondyle of the knee. Varus stress is
negative and pain is elicited with valgus stress. Initial radiographs were negative for abnormality.
What is the most likely area of injury?

a. Femoral attachment of the medial collateral ligament


b. Tibial attachment of the medial collateral ligament
c. Hypertrophic zone of the growth plate
d. Proliferative zone of the growth plate

Preffered Answer : 3
This patient likely has a physeal injury to the distal femoral physis. Stress radiographs or an MRI
scan will most reliably reveal this diagnosis. The growth plate, when injured, is most commonly
fractured through the hypertrophic zone of cartilage, its weakest point. This patient is optimally
treated in a cylindrical or long-leg cast.

18. Figures 1 through 4 are the CT scans and intraoperative image of a 17-year-old boy who
sustained a gunshot wound to his knee. What is the most appropriate definitive surgical
management for his articular cartilage defect?

UI // UNAIR // UNPAD // UNHAS // UNS // UGM // UB // UNUD // USU


Sports Medicine Scored and Recorded Self-Assessment Examination 2019

Figure 1 Figure 2 Figure 3

Figure 4

a. Microfracture
b. Autologous chondrocyte implantation
c. Osteochondral allograft transfer
d. Dejour trochleoplasty

Preffered Answer: 3
he images show a full-thickness cartilage defect with significant bony involvement >4 cm2.
Microfracture should be considered for lesions <2 cm2 without an underlying osseous defect.
Autologous chondrocyte implantation, although used for lesions between 1 and 10 cm2, should
be restricted for defects with minimal (<8 mm depth) bone loss. Osteochondral allograft transfer
with the mosaicplasty technique (transfer of multiple plugs) would be well-suited for this large
defect with significant osseous involvement. Dejour trochleoplasty is performed for patellar
instability to correct trochlear dysplasia and would not be indicated in this case.

19. A 24-year-old former high school wrestler had anterior cruciate ligament (ACL) reconstruction
with hamstring autograft 6 years ago. He now experiences daily instability of his knee with
routine activities including walking. Examination reveals a grade 3+ Lachman test with a soft

UI // UNAIR // UNPAD // UNHAS // UNS // UGM // UB // UNUD // USU


Sports Medicine Scored and Recorded Self-Assessment Examination 2019

endpoint, varus laxity at 30°, and a positive dial test at 30° that dissipates at 90° of knee flexion.
He has mild medial joint line tenderness. When walking, there is a slight varus thrust. What
treatment is most likely to lead to a successful outcome?

a. Hamstring autograft
b. Revision ACL reconstruction and posterior cruciate ligament (PCL) reconstruction
c. Revision ACL reconstruction and posteromedial corner reconstruction
d. Revision ACL reconstruction and posterolateral corner reconstruction

Preffered Answer: 4
This patient underwent an ACL reconstruction that has now failed. Based on his examination, he
also has a posterolateral corner injury. Because this concomitant injury was not treated, the
patient had undue strain on his graft, resulting in ultimate failure. Hamstring grafts are as
effective as other graft types for ACL reconstruction. The medial meniscus provides secondary
stabilization to the knee; however, this patient has a missed lateral ligamentous injury, and
meniscus tears do not result in the development of a varus thrust. An unrecognized PCL tear
likely results in mild-to-moderate medial and patellofemoral osteoarthritis without significant
lateral laxity and thrust.

20. Figures 1 and 2 are the MRI scans of a 35-year-old right-hand dominant man who has right
elbow pain after trying to lift a large television at home. An examination reveals ecchymosis, an
abnormal hook test, and altered biceps muscle contour. What treatment is most likely to result in
a satisfactory and predictable outcome?

Figure 1 Figure 2

a. Period of immobilization followed by physical therapy


b. Local corticosteroid injection
c. Surgical repair
d. Platelet-rich plasma (PRP)

Preffered Answer: 3
Figures 1 and 2 show a full thickness distal biceps tendon rupture with proximal retraction.
Edema is seen along the course of the distal biceps tendon, and the axial cut demonstrates the
absence of tendon at the radial tuberosity. The sagittal cut demonstrates the stump of the

UI // UNAIR // UNPAD // UNHAS // UNS // UGM // UB // UNUD // USU


Sports Medicine Scored and Recorded Self-Assessment Examination 2019

proximally retracted biceps tendon. The biceps muscle contour is abnormal in appearance,
demonstrating the classic ―popeye‖ deformity. Nonsurgical treatment options result in predictable
loss of supination and elbow flexion strength that is not desirable. A local corticosteroid injection
would not improve strength, and there is no evidence to support the use of a PRP injection.

21. Figures 1 and 2 are the radiographs of a 20-year-old college multisport athlete who has had
longstanding pain in his left hip. He denies any specific event that initiated his pain, but he notes
that he had hip problems when he was an infant. He denies pain with activities of daily living, but
he believes his pain is increasingly limiting his ability to exercise. He localizes the pain to his
groin. He denies low-back or buttock pain or pain that radiates down his leg.

What examination findings are most consistent with the pathology seen in the radiographs?

a. Pain with resisted hip flexion


b. Pain with a half sit-up, plus tenderness at the pubic ramus
c. Pain with a combination of hip flexion, adduction, and internal rotation
d. Tenderness to palpation at the greater trochanter

Preffered Answer: 3

This patient has cam-type femoroacetabular impingement. Decreased internal rotation and a
positive impingement test (forced flexion, adduction, and internal rotation) are classic findings.
The lack of pain with resisted hip flexion makes hip flexor strain unlikely, and the lack of
tenderness at the greater trochanter renders trochanteric bursitis unlikely. Although athletic
pubalgia can be a source of long-standing groin pain, he lacks the pain with a resisted sit-up and
tenderness along the pubic ramus that is frequently noted in patients with pubalgia. His
radiographs reveal a focal femoral neck prominence consistent with cam impingement, although
pistol grip deformities and flattening of the lateral femoral head are often present as well. His MRI
scan shows a labral tear, which is common in cam impingement. Surgical treatment for cam
impingement can be effective for symptomatic patients. Even among high-level athletes, open
surgical dislocation of the hip has been shown to have good results. Most patients with cam
impingement can be treated with arthroscopic osteoplasty and achieve results comparable with
those realized with open surgical dislocation. The literature describes success in terms of
athletes returning to sports (even professional athletes) to be approximately 90% after
arthroscopic treatment. Byrd and Jones described five patients who developed transient
neurapraxias that resolved uneventfully. The patients in his series who had concomitant

UI // UNAIR // UNPAD // UNHAS // UNS // UGM // UB // UNUD // USU


Sports Medicine Scored and Recorded Self-Assessment Examination 2019

microfracture had a 92% return to sports within the follow-up period. Cam impingement has long
been thought to be associated with a history of a slipped capital femoral epiphysis. The capitis in
these patients is displaced posteriorly, resulting in a prominent anterior femoral neck and
decreased hip internal rotation. Pincer impingement is associated with a deep acetabulum, such
as protrusion acetabula and acetabular retroversion. A patient who underwent a periacetabular
osteotomy can develop a more retroverted acetabulum as well.

22. Figures 1 and 2 are the radiographs of a 20-year-old college multisport athlete who has had
longstanding pain in his left hip. He denies any specific event that initiated his pain, but he notes
that he had hip problems when he was an infant. He denies pain with activities of daily living, but
he believes his pain is increasingly limiting his ability to exercise. He localizes the pain to his
groin. He denies low-back or buttock pain or pain that radiates down his leg.

What is the most likely diagnosis for the source of this patient's pain?

Figure 1 Figure 2

a. Cam-type femoroacetabular impingement


b. Pincer-type femoroacetabular impingement
c. Hip flexor strain
d. Athletic pubalgia

Preffered Answer: 1
This patient has cam-type femoroacetabular impingement. Decreased internal rotation and a
positive impingement test (forced flexion, adduction, and internal rotation) are classic findings.
The lack of pain with resisted hip flexion makes hip flexor strain unlikely, and the lack of
tenderness at the greater trochanter renders trochanteric bursitis unlikely. Although athletic
pubalgia can be a source of long-standing groin pain, he lacks the pain with a resisted sit-up and
tenderness along the pubic ramus that is frequently noted in patients with pubalgia. His
radiographs reveal a focal femoral neck prominence consistent with cam impingement, although
pistol grip deformities and flattening of the lateral femoral head are often present as well. His MRI
scan shows a labral tear, which is common in cam impingement. Surgical treatment for cam
impingement can be effective for symptomatic patients. Even among high-level athletes, open
surgical dislocation of the hip has been shown to have good results. Most patients with cam
impingement can be treated with arthroscopic osteoplasty and achieve results comparable with
those realized with open surgical dislocation. The literature describes success in terms of
athletes returning to sports (even professional athletes) to be approximately 90% after
arthroscopic treatment. Byrd and Jones described five patients who developed transient
neurapraxias that resolved uneventfully. The patients in his series who had concomitant

UI // UNAIR // UNPAD // UNHAS // UNS // UGM // UB // UNUD // USU


Sports Medicine Scored and Recorded Self-Assessment Examination 2019

microfracture had a 92% return to sports within the follow-up period. Cam impingement has long
been thought to be associated with a history of a slipped capital femoral epiphysis. The capitis in
these patients is displaced posteriorly, resulting in a prominent anterior femoral neck and
decreased hip internal rotation. Pincer impingement is associated with a deep acetabulum, such
as protrusion acetabula and acetabular retroversion. A patient who underwent a periacetabular
osteotomy can develop a more retroverted acetabulum as well.

23. Figures 1 and 2 are the radiographs of a 20-year-old college multisport athlete who has had
longstanding pain in his left hip. He denies any specific event that initiated his pain, but he notes
that he had hip problems when he was an infant. He denies pain with activities of daily living, but
he believes his pain is increasingly limiting his ability to exercise. He localizes the pain to his
groin. He denies low-back or buttock pain or pain that radiates down his leg.

Images from an MRI scan of this patient's left hip are shown in Figures 3 through 5. What is the
most likely cause of his acute pain?

Figure 1 Figure 2

Figure 3 Figure 4

UI // UNAIR // UNPAD // UNHAS // UNS // UGM // UB // UNUD // USU


Sports Medicine Scored and Recorded Self-Assessment Examination 2019

Figure 5

a. Significant cartilage loss on the acetabulum


b. Labral tear
c. Femoral neck stress fracture
d. Tendinopathy of the rectus femoris

Preffered Answer: 2
This patient has cam-type femoroacetabular impingement. Decreased internal rotation and a
positive impingement test (forced flexion, adduction, and internal rotation) are classic findings.
The lack of pain with resisted hip flexion makes hip flexor strain unlikely, and the lack of
tenderness at the greater trochanter renders trochanteric bursitis unlikely. Although athletic
pubalgia can be a source of long-standing groin pain, he lacks the pain with a resisted sit-up and
tenderness along the pubic ramus that is frequently noted in patients with pubalgia. His
radiographs reveal a focal femoral neck prominence consistent with cam impingement, although
pistol grip deformities and flattening of the lateral femoral head are often present as well. His MRI
scan shows a labral tear, which is common in cam impingement. Surgical treatment for cam
impingement can be effective for symptomatic patients. Even among high-level athletes, open
surgical dislocation of the hip has been shown to have good results. Most patients with cam
impingement can be treated with arthroscopic osteoplasty and achieve results comparable with
those realized with open surgical dislocation. The literature describes success in terms of
athletes returning to sports (even professional athletes) to be approximately 90% after
arthroscopic treatment. Byrd and Jones described five patients who developed transient
neurapraxias that resolved uneventfully. The patients in his series who had concomitant
microfracture had a 92% return to sports within the follow-up period. Cam impingement has long
been thought to be associated with a history of a slipped capital femoral epiphysis. The capitis in
these patients is displaced posteriorly, resulting in a prominent anterior femoral neck and
decreased hip internal rotation. Pincer impingement is associated with a deep acetabulum, such
as protrusion acetabula and acetabular retroversion. A patient who underwent a periacetabular
osteotomy can develop a more retroverted acetabulum as well.

24. A 13-year-old right-hand dominant pitcher was treated for Little League shoulder. What finding
increases his risk of recurrence?

a. Hyperlaxity
b. Rotator cuff weakness
c. Increased height
d. Glenohumeral internal rotation deficit

UI // UNAIR // UNPAD // UNHAS // UNS // UGM // UB // UNUD // USU


Sports Medicine Scored and Recorded Self-Assessment Examination 2019

Preffered Answer: 4
Little League shoulder is a physeal injury increasingly seen in young throwers. The primary
treatment is refraining from throwing with rehabilitation, followed by a throwing program. The risk
of recurrence is approximately 7%. The risk of recurrence is three times higher in athletes with
glenohumeral internal rotation deficit. Hyperlaxity, rotator cuff weakness, and increased height
have not been shown to correlate with recurrent symptoms.

25. Figure 1 is the T2 axial MRI scan of a 21-year-old man who was injured while playing for his
college football team. His pain was aggravated with blocking maneuvers and alleviated with rest,
and he had to stop playing because of the pain. What examination maneuver most likely will
reproduce his pain?

a. Forward elevation in the scapular plane


b. External rotation and abduction
c. Flexion, adduction, and internal rotation
d. Flexion and abduction

Preffered Answer : 3
This patient has a mechanism of injury and MRI scan consistent with a posterior labral tear and
posterior instability. Flexion, adduction, and internal rotation produce a net posterior vector on the
glenohumeral joint and should reproduce this patient's symptoms. Pain or instability with the arm
elevated in the scapular plane describes an impingement sign. Pain or instability with the arm in
external rotation and abduction describes the apprehension sign. Pain or instability with the arm
in flexion and abduction is a nonspecific finding.
26. Chronic traumatic encephalopathy (CTE) is a neurodegenerative disease that is characterized by

a. onset most often by age 30.


b. a temporary state of neuronal and axonal derangement.
c. manifestations of affect such as apathy, irritability, and suicidal ideation.
d. absence of gross pathological brain changes upon autopsy.

Preffered Answer: 4
CTE is a neurodegenerative disease that occurs years or decades after recovery from acute or
postacute effects of head trauma. The exact relationship between concussion and CTE is not
entirely clear; however, early behavioral manifestations of CTE have been described by family
and providers to include apathy, irritability, and suicidal ideation. For some patients, cognitive
difficulty such as poor episodic memory and executive function may be the first signs of CTE.

UI // UNAIR // UNPAD // UNHAS // UNS // UGM // UB // UNUD // USU


Sports Medicine Scored and Recorded Self-Assessment Examination 2019

Onset most often occurs in midlife after athletes have completed their sports careers, with mean
age of onset at 42 years. The effects on the brain are degenerative, leading to a permanent state
of derangement. Autopsy findings demonstrate multiple gross pathological findings. The
condition is more common among contact athletes.

27. A 19-year-old female field hockey player sustains a right ankle injury last night during a game.
The patient is on crutches and reports that she has not been able to put any weight on her right
ankle since the injury. She was running alongside with another player when her right ankle ―gave
out‖ and she twisted it, falling to the ground. Physical examination reveals discoloration similar to
a hematoma and significant swelling around the lateral ankle area. Pain is elicited during
palpation of the anterior talofibular ligament.

What test should be performed to aid in this diagnosis?

a. Thompson test
b. External rotation stress test
c. Anterior drawer test
d. Squeeze test

Preffered Answer: 3
The anterior drawer test is performed with the ankle in 10° of plantar flexion, which results in the
greatest amount of translation. The test investigates the integrity of the anterior talofibular
ligament with a key distance of translation being 8 to 10 mm. While the patient is sitting and has
her knees flexed over the edge of a table or bench, the physician uses one hand to stabilize the
distal leg and with the other hand applies an anterior force to the heel in an attempt to gap the
talus anteriorly from under the tibia. The anterior talofibular ligament and calcaneofibular
ligament are both compromised based on the examination findings. The anterior drawer test
result reflects injury to the anterior talofibular ligament and a possible injury to the calcaneofibular
ligament. A lateral talar tilt test angle measurement >15° degrees reflects a rupture of both
anterior talofibular ligament and calcaneofibular ligaments. The diagnosis is a severe lateral
ligament complex sprain. This is optimally managed with early mobilization and a guided
rehabilitation program that emphasizes proprioceptive stability.

28. A 19-year-old female field hockey player sustains a right ankle injury last night during a game.
The patient is on crutches and reports that she has not been able to put any weight on her right
ankle since the injury. She was running alongside with another player when her right ankle ―gave
out‖ and she twisted it, falling to the ground. Physical examination reveals discoloration similar to
a hematoma and significant swelling around the lateral ankle area. Pain is elicited during
palpation of the anterior talofibular ligament.

Radiographs of the player’s right ankle confirm there are no fractures. With a lateral talar tilt test
result of 19°, which additional structure is most likely damaged?

a. Deltoid ligament
b. Calcaneofibular ligament
c. Anterior tibiofibular ligament
d. Posterior tibiofibular ligament

Preffered Answer: 2

UI // UNAIR // UNPAD // UNHAS // UNS // UGM // UB // UNUD // USU


Sports Medicine Scored and Recorded Self-Assessment Examination 2019

The anterior drawer test is performed with the ankle in 10° of plantar flexion, which results in the
greatest amount of translation. The test investigates the integrity of the anterior talofibular
ligament with a key distance of translation being 8 to 10 mm. While the patient is sitting and has
her knees flexed over the edge of a table or bench, the physician uses one hand to stabilize the
distal leg and with the other hand applies an anterior force to the heel in an attempt to gap the
talus anteriorly from under the tibia. The anterior talofibular ligament and calcaneofibular
ligament are both compromised based on the examination findings. The anterior drawer test
result reflects injury to the anterior talofibular ligament and a possible injury to the calcaneofibular
ligament. A lateral talar tilt test angle measurement >15° degrees reflects a rupture of both
anterior talofibular ligament and calcaneofibular ligaments. The diagnosis is a severe lateral
ligament complex sprain. This is optimally managed with early mobilization and a guided
rehabilitation program that emphasizes proprioceptive stability.

29. A 19-year-old female field hockey player sustains a right ankle injury last night during a game.
The patient is on crutches and reports that she has not been able to put any weight on her right
ankle since the injury. She was running alongside with another player when her right ankle ―gave
out‖ and she twisted it, falling to the ground. Physical examination reveals discoloration similar to
a hematoma and significant swelling around the lateral ankle area. Pain is elicited during
palpation of the anterior talofibular ligament.

What is the most appropriate course of action for this patient’s condition?

a. Early mobilization and a guided proprioceptive and strengthening


b. Extended immobilization in a cast
c. Surgical intervention
d. Weight bearing as tolerated in an ankle brace for 6 weeks

Preffered Answer: 1
The anterior drawer test is performed with the ankle in 10° of plantar flexion, which results in the
greatest amount of translation. The test investigates the integrity of the anterior talofibular
ligament with a key distance of translation being 8 to 10 mm. While the patient is sitting and has
her knees flexed over the edge of a table or bench, the physician uses one hand to stabilize the
distal leg and with the other hand applies an anterior force to the heel in an attempt to gap the
talus anteriorly from under the tibia. The anterior talofibular ligament and calcaneofibular
ligament are both compromised based on the examination findings. The anterior drawer test
result reflects injury to the anterior talofibular ligament and a possible injury to the calcaneofibular
ligament. A lateral talar tilt test angle measurement >15° degrees reflects a rupture of both
anterior talofibular ligament and calcaneofibular ligaments. The diagnosis is a severe lateral
ligament complex sprain. This is optimally managed with early mobilization and a guided
rehabilitation program that emphasizes proprioceptive stability.

30. Figures 1 and 2 are the radiograph and MRI scan of a 16-year-old boy who injured his right knee
by a lateral side impact while playing football. The MRI indicates what structure was most likely
injured?

UI // UNAIR // UNPAD // UNHAS // UNS // UGM // UB // UNUD // USU


Sports Medicine Scored and Recorded Self-Assessment Examination 2019

a. Lateral collateral ligament


b. Tibial spine
c. Medial meniscus
d. Anterior cruciate ligament (ACL)

Preffered Answer : 4
This is a rupture of the anterolateral ligament complex and a portion of the IT band. This injury is
highly correlated with a complete ACL injury. In the MRI, the curvilinear or elliptic bone fragment
(Segond fracture) projected parallel to the lateral aspect of the tibial plateau, the lateral capsular
sign, is seen. The lateral capsular sign is also associated with ACL tears. Thus, this is an MRI
showing a complete ACL tear.

31. Figures 1 and 2 are intrasurgical photographs from the posterolateral viewing portal that were
taken at the beginning and end of a right shoulder arthroscopic procedure performed on a 54-
year-old man. This technique demonstrates superior results compared with traditional
arthroscopic techniques when evaluating which outcome?

a. Time to healing
b. Retear rate
c. Functional outcome scores
d. Postsurgical pain scores

Preffered Answer: 2

UI // UNAIR // UNPAD // UNHAS // UNS // UGM // UB // UNUD // USU


Sports Medicine Scored and Recorded Self-Assessment Examination 2019

The images reveal a medium-sized tear of the rotator cuff. As more clinical studies are published
comparing double-row with single-row rotator cuff repair, it has become clear that the retear rate
is lower with a double-row construct for small and medium-sized tears. This may be attributable
to the stronger time-zero repair construct that double-row repair provides. No study to date has
demonstrated a significant difference in clinical outcomes (functional and pain scores at any
time) or time to healing between the two techniques.

32. A 16-year-old football player is participating in the second session of two-a-day preseason
practices. He complains of dizziness and fatigue. He is brought to the sideline by the athletic
trainer where examination demonstrates confusion and disorientation. Ambient temperature is
82°F. What would be the next most appropriate step in his treatment?

a. Rapid cooling via ice bath, cold water and fans


b. Transportation via ambulance to a local emergency department
c. Rapid rehydration via oral and IV fluids
d. Immediate administration of acetaminophen or other anti-pyretics

Preffered Answer: 1
Heat exhaustion and heat stroke reflect varying degrees of heat illness, with both marked by
increased heat production with impaired heat dissipation. Heat exhaustion typically involves a
core body temperature between 37°C (98.6°F) and 40°C (104°F) and usually presents with
heavy sweating, as well as nausea; vomiting; headache; fainting; weakness; and cold or clammy
skin. Fatigue, malaise, and dizziness may occur, but necessary to the diagnosis is normal
mentation and stable neurologic status. Heat stroke is defined by a core body temperature >40°C
(>104°F) and disturbances of the central nervous system, such as confusion, irritability, ataxia,
and even coma. Heat exhaustion is a less urgent scenario and can usually be treated with rest,
elevation, and rehydration. Heat stroke, confirmed here by the presence of mental status
changes, is a more critical situation. The most important immediate step is rapid body cooling
through whatever means are available, as this has been clearly shown to improve outcomes.
Ideally, a whole body ice bath would be used, with ice towels, ice packs, cold water, and air fans
all utilized if needed. Emergency department transportation and rehydration may be considered
as well but are not as important as immediate lowering of body temperature. Anti-pyretics have
no role in this process.

33. A coach of three football teams—the B team, junior varsity team, and varsity team—wants to
study the average times in the 40-yard dash for his players. Which test would help him determine
if the mean 40-yard dash times for the athletes on one team are different from those on the other
teams?

a. Independent 2-sample t test


b. Analysis of variance (ANOVA)
c. Chi-square test
d. Fisher's exact test

Preffered Answer: 2
Data collected in research studies fall into one of two categories—continuous or discrete.
Continuous data can be displayed on a curve. Examples include height, weight, and time

UI // UNAIR // UNPAD // UNHAS // UNS // UGM // UB // UNUD // USU


Sports Medicine Scored and Recorded Self-Assessment Examination 2019

recorded in a 40-yard dash. Discrete data represent data that fall into specific categories such as
gender or the presence or absence of a risk factor.

ANOVA is used to determine statistical significance in mean values of continuous data when
there are more than two independent samples. The 2-sample t test compares mean values of
continuous data between two independent groups. The Chi-square test and Fisher's exact tests
are tests used to analyze discrete data.

34. Figures 1 and 2 are the MRI scans of a 57-year-old man who dislocated his left shoulder after a
fall while playing tennis. On examination, he had full passive shoulder range of motion, but he
was unable to actively elevate his injured shoulder. Sensation was intact to light touch over the
lateral shoulder. What is the most likely etiology of his shoulder weakness?

a. Axillary nerve injury


b. Cervical radiculopathy involving the C6 nerve root
c. Massive rotator cuff tear with loss of the transverse force couple
d. Long head of the biceps tendon rupture with loss of superior stabilizing effect

Preffered Answer: 3
This patient has a massive rotator cuff tear resulting in disruption of the transverse force couple
between the subscapularis anteriorly and the infraspinatus and teres minor posteriorly. These
muscles provide dynamic shoulder stability throughout active elevation, and loss of the force
couple produces a pathologic increase in translation of the humeral head and decreased active
abduction. Active shoulder elevation <90 degrees in the presence of full passive motion is termed
pseudoparalysis. The most common neurologic deficit after shoulder dislocation is isolated injury
to the axillary nerve. This patient's sensory examination suggests that the axillary nerve is intact.
Cervical radiculopathy is less common after shoulder dislocation but has been reported.
Conflicting evidence exists regarding the contribution of the long head of the biceps tendon to
glenohumeral stability. One study reported minimal electromyographic activity in the biceps
during ten basic shoulder motions.

35. A 53-year-old man sustains a fall while skiing. He experiences immediate pain and deformity in
his lower leg just above his ski boot top. Radiographs of his left lower leg are shown in Figures 1
and 2. After discussing operative management with the patient, you choose to proceed with an
intramedullary nail. Where should blocking screws be placed to prevent the characteristic
deformity of this fracture?

UI // UNAIR // UNPAD // UNHAS // UNS // UGM // UB // UNUD // USU


Sports Medicine Scored and Recorded Self-Assessment Examination 2019

a. Posterior and medial aspect of the proximal fragment


b. Anterior aspect of the proximal fragment and medial half of the distal fragment
c. Posterior and lateral aspect of the proximal fragment
d. Anterior and lateral aspect of the proximal fragment

Preffered Answer : 2
The patient has sustained a proximal third tibia and fibula fracture, sometimes referred to as a
"boot top" fracture when sustained while alpine skiing. These fractures frequently fall into apex
anterior and valgus angulation as shown in the radiographs. Great care needs to be taken during
intramedullary nailing to avoid fixing the fracture in a malreduced position. The use of Poller
blocking screws in the posterior and lateral aspects of the proximal fragment can help to avoid
this complication (posterior screw prevents flexion deformity while lateral screw prevents valgus
deformity). Additional technical pearls include starting with a more lateral entry point for the
guidewire, placing a provisional unicortical plate prior to nailing to maintain the reduction, or
using a suprapatellar entry nail to allow for fixation with the knee in a semi-extended position.

36. What is the most common complication after surgical management of chronic exertional
compartment syndrome (CECS) in the pediatric (≤18 years) population?

a. Recurrent CECS
b. Infection
c. Neurologic dysfunction
d. Hematoma or seroma formation

Preffered Answer : 1
In the pediatric population, CECS most commonly presents in females involved in running sports.
In this cohort, recurrence occurs at a rate of 18%. Wound complications are the next most
common at a rate of 11.2%.

37. What do the T2-weighted, fat-saturated MRI scans shown in Figures 1 through 4 reveal?

UI // UNAIR // UNPAD // UNHAS // UNS // UGM // UB // UNUD // USU


Sports Medicine Scored and Recorded Self-Assessment Examination 2019

a. Posterior cruciate ligament (PCL) tear, isolated


b. PCL tear and medial meniscus tear
c. Anterior cruciate ligament (ACL) tear, isolated
d. ACL tear and medial meniscus tear

Preffered Answer: 4
The MRI scans show that edema is noted on the femoral insertion of the ACL consistent with a
high-grade or complete ACL tear. The ACL is not visualized on the sagittal view, although the
torn meniscus can be seen in the notch. On the coronal image, there is an empty lateral wall sign
indicating proximal disruption of the ACL. The medial meniscus images show a disruption of
normal meniscus morphology consistent with a bucket handle medial meniscus tear. Note the
appearance on the sagittal MRI scan of what appears to be a second soft-tissue density in line
with the PCL. This "double PCL" sign is highly indicative of a displaced medial meniscus tear
rather than a displaced lateral meniscus tear.

38. A 19-year-old collegiate offensive lineman injures his left elbow in a scrimmage. He reports
reaching out with his left arm to prevent the defensive player from getting around him, and, as he
grabbed the player, his elbow was forced into extension. He had immediate pain and weakness

UI // UNAIR // UNPAD // UNHAS // UNS // UGM // UB // UNUD // USU


Sports Medicine Scored and Recorded Self-Assessment Examination 2019

and heard a ―pop.‖ He has mild swelling in the antecubital fossa and a prominent-appearing
biceps muscle belly. His hook test result is abnormal at the elbow.

Which type of contraction of the involved muscle most likely resulted in this lineman's injury?

a. Eccentric
b. Concentric
c. Isometric
d. Isokinetic

Preffered Answer: 1
This patient had an eccentric muscle contraction (muscle lengthening while contracting) of his
biceps muscle while trying to stop a defender from getting around him. This in turn caused failure
of the distal biceps tendon, as evidenced by pain in the antecubital fossa, lack of elbow
supination strength, and his positive biceps active test finding (supination/pronation of the
forearm showing no motion of the biceps muscle belly). Eccentric contractors have the highest
potential for building strength but also are at highest risk for injury. Concentric (muscle shortening
with contraction), isometric (no change in muscle length with contracture), and isokinetic
(constant velocity of muscle contraction with a variable force) do not describe the mechanism
detailed.

The loss of distal biceps attachment will result in loss of elbow supination strength in flexion (the
biceps is the only supinator to cross the elbow) while still retaining elbow flexion (albeit
weakened) because of the other elbow flexors (brachioradialis and brachialis). Consequently,
treatment should be anatomic repair of the distal biceps insertion, which can be performed with a
2-incision or 1-incision technique. Although all of the listed complications have been reported
with these techniques, LABC neuropraxia is by far the most common. Radiographs show that this
athlete’s injury was repaired using a 1-incision technique with a cortical fixation device and a
radial bone tunnel. This technique has gained favor because of its decreased incidence of
heterotopic ossification and radioulnar synostosis compared with the 2-incision technique. The
most troubling complication for most surgeons is the development of a posterior interosseous
nerve (PIN) palsy, which this patient clearly demonstrates in addition to the more common
LABCN upon postsurgical examination. Because the LABCN injury is typically a neuropraxia
from retraction, a period of observation is indicated. PIN injury can result from excessive traction
during surgical exposure or from entrapment by the fixation button.

Considering the anatomy of the PIN, successful recovery of the nerve typically progresses based
on the distance from the origin of the nerve to the muscle indicated. The extensor indicis proprius
(EIP) muscle is the most distal muscle innervated and can be expected to recover last. First to
return would be the EDC followed by the ECU, EDQ, and, finally, the EIP.

39. A 19-year-old collegiate offensive lineman injures his left elbow in a scrimmage. He reports
reaching out with his left arm to prevent the defensive player from getting around him, and, as he
grabbed the player, his elbow was forced into extension. He had immediate pain and weakness
and heard a ―pop.‖ He has mild swelling in the antecubital fossa and a prominent-appearing
biceps muscle belly. His hook test result is abnormal at the elbow.

The most substantial functional deficit that may develop if no surgical treatment is provided is

a. elbow flexion strength.

UI // UNAIR // UNPAD // UNHAS // UNS // UGM // UB // UNUD // USU


Sports Medicine Scored and Recorded Self-Assessment Examination 2019

b. elbow supination strength.


c. lack of terminal extension at the elbow.
d. decrease of elbow pronation strength.

Preffered Answer: 2
This patient had an eccentric muscle contraction (muscle lengthening while contracting) of his
biceps muscle while trying to stop a defender from getting around him. This in turn caused failure
of the distal biceps tendon, as evidenced by pain in the antecubital fossa, lack of elbow
supination strength, and his positive biceps active test finding (supination/pronation of the
forearm showing no motion of the biceps muscle belly). Eccentric contractors have the highest
potential for building strength but also are at highest risk for injury. Concentric (muscle shortening
with contraction), isometric (no change in muscle length with contracture), and isokinetic
(constant velocity of muscle contraction with a variable force) do not describe the mechanism
detailed.

The loss of distal biceps attachment will result in loss of elbow supination strength in flexion (the
biceps is the only supinator to cross the elbow) while still retaining elbow flexion (albeit
weakened) because of the other elbow flexors (brachioradialis and brachialis). Consequently,
treatment should be anatomic repair of the distal biceps insertion, which can be performed with a
2-incision or 1-incision technique. Although all of the listed complications have been reported
with these techniques, LABC neuropraxia is by far the most common. Radiographs show that this
athlete’s injury was repaired using a 1-incision technique with a cortical fixation device and a
radial bone tunnel. This technique has gained favor because of its decreased incidence of
heterotopic ossification and radioulnar synostosis compared with the 2-incision technique. The
most troubling complication for most surgeons is the development of a posterior interosseous
nerve (PIN) palsy, which this patient clearly demonstrates in addition to the more common
LABCN upon postsurgical examination. Because the LABCN injury is typically a neuropraxia
from retraction, a period of observation is indicated. PIN injury can result from excessive traction
during surgical exposure or from entrapment by the fixation button.

Considering the anatomy of the PIN, successful recovery of the nerve typically progresses based
on the distance from the origin of the nerve to the muscle indicated. The extensor indicis proprius
(EIP) muscle is the most distal muscle innervated and can be expected to recover last. First to
return would be the EDC followed by the ECU, EDQ, and, finally, the EIP.

40. A 19-year-old collegiate offensive lineman injures his left elbow in a scrimmage. He reports
reaching out with his left arm to prevent the defensive player from getting around him, and, as he
grabbed the player, his elbow was forced into extension. He had immediate pain and weakness
and heard a ―pop.‖ He has mild swelling in the antecubital fossa and a prominent-appearing
biceps muscle belly. His hook test result is abnormal at the elbow.

The athlete undergoes repair of the injury, and postsurgical radiographs are shown in Figures 1
and 2. At his first postsurgical visit, he reports no pain but describes weakness in his hand and
decreased sensation over his lateral forearm. Upon examination, he has decreased 2-point
discrimination over the lateral forearm and an inability to actively extend his thumb and fingers at
the metacarpophalangeal joints. He can extend at the finger interphalangeal joints. He can
extend his wrist weakly, and it deviates radially as he extends. His distal sensation is intact.
Considering his examination findings, which two nerves are injured?

UI // UNAIR // UNPAD // UNHAS // UNS // UGM // UB // UNUD // USU


Sports Medicine Scored and Recorded Self-Assessment Examination 2019

a. PIN and radial nerve


b. PIN and lateral antebrachial cutaneous nerve (LABCN)
c. Median nerve and LABCN
d. Radial nerve and LABCN

Preffered Answer: 2
This patient had an eccentric muscle contraction (muscle lengthening while contracting) of his
biceps muscle while trying to stop a defender from getting around him. This in turn caused failure
of the distal biceps tendon, as evidenced by pain in the antecubital fossa, lack of elbow
supination strength, and his positive biceps active test finding (supination/pronation of the
forearm showing no motion of the biceps muscle belly). Eccentric contractors have the highest
potential for building strength but also are at highest risk for injury. Concentric (muscle shortening
with contraction), isometric (no change in muscle length with contracture), and isokinetic
(constant velocity of muscle contraction with a variable force) do not describe the mechanism
detailed.

The loss of distal biceps attachment will result in loss of elbow supination strength in flexion (the
biceps is the only supinator to cross the elbow) while still retaining elbow flexion (albeit
weakened) because of the other elbow flexors (brachioradialis and brachialis). Consequently,
treatment should be anatomic repair of the distal biceps insertion, which can be performed with a
2-incision or 1-incision technique. Although all of the listed complications have been reported
with these techniques, LABC neuropraxia is by far the most common. Radiographs show that this
athlete’s injury was repaired using a 1-incision technique with a cortical fixation device and a
radial bone tunnel. This technique has gained favor because of its decreased incidence of
heterotopic ossification and radioulnar synostosis compared with the 2-incision technique. The
most troubling complication for most surgeons is the development of a posterior interosseous
nerve (PIN) palsy, which this patient clearly demonstrates in addition to the more common
LABCN upon postsurgical examination. Because the LABCN injury is typically a neuropraxia
from retraction, a period of observation is indicated. PIN injury can result from excessive traction
during surgical exposure or from entrapment by the fixation button.

Considering the anatomy of the PIN, successful recovery of the nerve typically progresses based
on the distance from the origin of the nerve to the muscle indicated. The extensor indicis proprius
(EIP) muscle is the most distal muscle innervated and can be expected to recover last. First to
return would be the EDC followed by the ECU, EDQ, and, finally, the EIP.

UI // UNAIR // UNPAD // UNHAS // UNS // UGM // UB // UNUD // USU


Sports Medicine Scored and Recorded Self-Assessment Examination 2019

41. A 20-year-old healthy female endurance athlete has lower leg pain and dorsal foot paresthesias
after running for 30 minutes. She has seen another physician and has been ruled out for a bone
stress injury. She has tried extensive nonsurgical measures such as shoe modification and an
extended period without running. You suspect chronic exertional compartment syndrome and
perform intramuscular compartment pressure measurements at three separate time points with
the following results:

Baseline 1 Minute 5 Minutes


Anterior 7 32 25
Lateral 8 29 23
Superficial Posterior 12 25 17
Deep Posterior 14 22 16

The patient decides to pursue surgical intervention. Which compartments should be released?

a. Anterior and lateral


b. Anterior, lateral, and deep posterior
c. Anterior, lateral and superficial posterior
d. Lateral and superficial posterior

Preffered Answer: 1
The diagnostic criteria for chronic exertional compartment syndrome is pressure >15 mm Hg at
rest, or >30 mm Hg at 1 minute post exercise, or >20 mm Hg at 5 minutes post-exercise. The
anterior and lateral compartments are the only ones that meet strict diagnostic criteria for chronic
exertional compartment syndrome. The superficial posterior compartment, although close to
meeting criteria, is not responsible for the patient's symptoms and falls below current thresholds
for diagnosis.

42. Figures 1 and 2 are the most recent radiographs of an 18-year-old high school student who
sustains an anterior shoulder dislocation playing recreational football. He has a low Beighton
score on physical examination. He was closed reduced and underwent a course of physical
therapy but had a second dislocation playing recreational basketball. What is the most
appropriate course of treatment, with the lowest complication rate, to prevent further dislocation?

a. Arthroscopic Bankart procedure

UI // UNAIR // UNPAD // UNHAS // UNS // UGM // UB // UNUD // USU


Sports Medicine Scored and Recorded Self-Assessment Examination 2019

b. Physical therapy
c. SAWA shoulder brace
d. Latarjet procedure

Preffered Answer : 1
The patient has recurrent instability and is at a high rate of further dislocations due to his young
age. Therefore, therapy and bracing are unlikely to decrease his dislocation rate. The
radiographs are normal, and there is no Hill-Sachs lesion or bony Bankart lesion. His instability
severity index score is 3, and; therefore, a bony procedure such as Latarjet is not necessary.
Furthermore, the rate of complication following a Latarjet procedure, especially nerve injury and
hardware problems, exceeds that of arthroscopic Bankart repair.

43. A football player injures his knee when he is tackled and falls awkwardly. He does not note any
discreet ―pop,‖ but pain prevents him from returning to the game. An effusion is noted the
following day and an MRI scan is ordered. Selected images are shown in Figures 1 through 3.
Based on these images, physical examination findings likely include

a. positive Lachman test, normal posterior drawer, positive pivot shift.


b. positive Lachman test, positive posterior drawer, negative pivot shift.
c. normal Lachman test, positive posterior drawer, positive pivot shift.
d. normal Lachman test, positive posterior drawer, negative pivot shift.

Preffered Answer : 4
The images provided reveal a posterior cruciate ligament (PCL) disruption with an intact anterior
cruciate ligament (ACL). Common diagnostic findings for a PCL tear include a positive posterior
drawer test, positive reverse pivot shift, positive quadriceps active test, and positive posterior
sag. A positive Lachman test, which would indicate a torn ACL, would not be expected to be
positive. A false-positive result for a Lachman test can arise with a torn PCL because of the
overall increased anterior-posterior translation; this must be avoided by careful attention to initial
resting position and station of the knee.

44. A 40-year-old man who is a manual laborer has had 3 years of worsening medial-sided left knee
pain that has inhibited his ability to work. He reports undergoing a left subtotal medial

UI // UNAIR // UNPAD // UNHAS // UNS // UGM // UB // UNUD // USU


Sports Medicine Scored and Recorded Self-Assessment Examination 2019

meniscectomy 10 years ago. He has been treated with nonsteroidal anti-inflammatory drugs and
two different corticosteroids, with the most recent injection given 1 month ago. Each injection
provided him with a few weeks of pain control. His medical history is unremarkable, and he has
smoked 20 cigarettes per day for the last 15 years. His BMI is 22. On examination, he has varus
alignment of the involved leg and medial joint line tenderness and no lateral or patellofemoral
pain. His knee range of motion is 3° shy of full extension to 130° of flexion. He has negative
Lachman and posterior drawer test results. He demonstrates no lateral thrust with ambulation.

What imaging study is most appropriate to determine treatment options for this patient?

a. Full-length weight-bearing radiographs of both legs


b. MRI scan of the left knee
c. CT scan of the left knee
d. Ultrasonography of the left leg

Preffered Answer: 1
This patient has a classic presentation of postmeniscectomy medial compartment arthritis. The
appropriate diagnostic study is weight-bearing radiographs to confirm the diagnosis. An MRI
scan will reveal medial compartment arthritis but will not provide information about alignment. A
CT scan would be appropriate to detect an occult fracture; however, this condition is not
suspected in this clinical scenario. Ultrasonography can provide information about fluid collection
around the knee or a deep vein thrombosis; however, these conditions also are not suspected in
this clinical scenario.
Because the patient has a correctable deformity (gaps 3 mm with valgus stress), and his
symptoms are localized to the involved compartment, a trial of a medial unloader brace is
appropriate both diagnostically and therapeutically. If unloading the medial compartment resolves
the patient’s symptoms, he would be an excellent candidate for an osteotomy. An MRI scan may
be obtained to evaluate ligamentous integrity or to evaluate degenerative involvement of the
lateral and patellofemoral compartment for presurgical planning of an osteotomy; however, the
integrity of the medial meniscus has no clinical importance in a patient with severe medial
compartment arthritis. A repeat corticosteroid injection is not indicated within 1 month of his last
injection, and referral to pain management is not appropriate with other options available to help
this patient.
A VPHTO is the appropriate intervention considering the patient’s young age, high-functional
occupation, examination, radiographic findings, and response to medial unloader bracing. A
revision knee arthroscopy would be appropriate for a recurrent medial meniscus tear, but not
appropriate in a patient with severe medial compartment arthritis. The patient’s young age and
high functional requirements are contraindications to TKA. The presence of severe arthritis is a
contraindication to medial meniscus transplant.
The patient is a candidate for a VPHTO. The technical options include a medial opening-wedge
or a lateral closing-wedge osteotomy. Both techniques have advantages and disadvantages;
however, a medial opening-wedge osteotomy is contraindicated in a smoker because of concern
for nonunion. As a result, current smoking history is the only factor listed that would influence the
technique used. The history of prior arthroscopy has no relevance in the decision about which
type of osteotomy is appropriate. Normal BMI is between 18.5 and 24.9, so this patient’s BMI is
considered normal and would not affect the surgical technique (if this patient were obese, a
lateral closing-wedge osteotomy would be considered, but this is controversial). His age of 40
years is an indication for HTO but does not influence technique.
45. A 40-year-old man who is a manual laborer has had 3 years of worsening medial-sided left knee
pain that has inhibited his ability to work. He reports undergoing a left subtotal medial
meniscectomy 10 years ago. He has been treated with nonsteroidal anti-inflammatory drugs and

UI // UNAIR // UNPAD // UNHAS // UNS // UGM // UB // UNUD // USU


Sports Medicine Scored and Recorded Self-Assessment Examination 2019

two different corticosteroids, with the most recent injection given 1 month ago. Each injection
provided him with a few weeks of pain control. His medical history is unremarkable, and he has
smoked 20 cigarettes per day for the last 15 years. His BMI is 22. On examination, he has varus
alignment of the involved leg and medial joint line tenderness and no lateral or patellofemoral
pain. His knee range of motion is 3° shy of full extension to 130° of flexion. He has negative
Lachman and posterior drawer test results. He demonstrates no lateral thrust with ambulation.

What is the most appropriate next step in treatment?

a. Repeat corticosteroid injection


b. Trial of a medial unloader brace
c. MRI scan of the knee to evaluate for recurrent medial meniscus tear
d. Referral to pain management

Discussion : 2
This patient has a classic presentation of postmeniscectomy medial compartment arthritis. The
appropriate diagnostic study is weight-bearing radiographs to confirm the diagnosis. An MRI
scan will reveal medial compartment arthritis but will not provide information about alignment. A
CT scan would be appropriate to detect an occult fracture; however, this condition is not
suspected in this clinical scenario. Ultrasonography can provide information about fluid collection
around the knee or a deep vein thrombosis; however, these conditions also are not suspected in
this clinical scenario.

Because the patient has a correctable deformity (gaps 3 mm with valgus stress), and his
symptoms are localized to the involved compartment, a trial of a medial unloader brace is
appropriate both diagnostically and therapeutically. If unloading the medial compartment resolves
the patient’s symptoms, he would be an excellent candidate for an osteotomy. An MRI scan may
be obtained to evaluate ligamentous integrity or to evaluate degenerative involvement of the
lateral and patellofemoral compartment for presurgical planning of an osteotomy; however, the
integrity of the medial meniscus has no clinical importance in a patient with severe medial
compartment arthritis. A repeat corticosteroid injection is not indicated within 1 month of his last
injection, and referral to pain management is not appropriate with other options available to help
this patient.

A VPHTO is the appropriate intervention considering the patient’s young age, high-functional
occupation, examination, radiographic findings, and response to medial unloader bracing. A
revision knee arthroscopy would be appropriate for a recurrent medial meniscus tear, but not
appropriate in a patient with severe medial compartment arthritis. The patient’s young age and
high functional requirements are contraindications to TKA. The presence of severe arthritis is a
contraindication to medial meniscus transplant.

The patient is a candidate for a VPHTO. The technical options include a medial opening-wedge
or a lateral closing-wedge osteotomy. Both techniques have advantages and disadvantages;
however, a medial opening-wedge osteotomy is contraindicated in a smoker because of concern
for nonunion. As a result, current smoking history is the only factor listed that would influence the
technique used. The history of prior arthroscopy has no relevance in the decision about which
type of osteotomy is appropriate. Normal BMI is between 18.5 and 24.9, so this patient’s BMI is
considered normal and would not affect the surgical technique (if this patient were obese, a
lateral closing-wedge osteotomy would be considered, but this is controversial). His age of 40
years is an indication for HTO but does not influence technique.

UI // UNAIR // UNPAD // UNHAS // UNS // UGM // UB // UNUD // USU


Sports Medicine Scored and Recorded Self-Assessment Examination 2019

46. A 40-year-old man who is a manual laborer has had 3 years of worsening medial-sided left knee
pain that has inhibited his ability to work. He reports undergoing a left subtotal medial
meniscectomy 10 years ago. He has been treated with nonsteroidal anti-inflammatory drugs and
two different corticosteroids, with the most recent injection given 1 month ago. Each injection
provided him with a few weeks of pain control. His medical history is unremarkable, and he has
smoked 20 cigarettes per day for the last 15 years. His BMI is 22. On examination, he has varus
alignment of the involved leg and medial joint line tenderness and no lateral or patellofemoral
pain. His knee range of motion is 3° shy of full extension to 130° of flexion. He has negative
Lachman and posterior drawer test results. He demonstrates no lateral thrust with ambulation.

The patient is provided with a medial unloader brace that provides substantial pain relief, and he
is able to work while wearing the brace. After 4 months, he returns to work and reports that while
the brace enables him to work, it is uncomfortable. Consequently, his symptoms return when he
is not wearing the brace, and he is requesting a surgical intervention for his problem. What is the
most appropriate surgical treatment?

a. Valgus-producing high tibial osteotomy (VPHTO)


b. Repeat knee arthroscopy
c. Total knee arthroplasty (TKA)
d. Medial meniscus transplant

Preffered Answer: 1
This patient has a classic presentation of postmeniscectomy medial compartment arthritis. The
appropriate diagnostic study is weight-bearing radiographs to confirm the diagnosis. An MRI
scan will reveal medial compartment arthritis but will not provide information about alignment. A
CT scan would be appropriate to detect an occult fracture; however, this condition is not
suspected in this clinical scenario. Ultrasonography can provide information about fluid collection
around the knee or a deep vein thrombosis; however, these conditions also are not suspected in
this clinical scenario.

Because the patient has a correctable deformity (gaps 3 mm with valgus stress), and his
symptoms are localized to the involved compartment, a trial of a medial unloader brace is
appropriate both diagnostically and therapeutically. If unloading the medial compartment resolves
the patient’s symptoms, he would be an excellent candidate for an osteotomy. An MRI scan may
be obtained to evaluate ligamentous integrity or to evaluate degenerative involvement of the
lateral and patellofemoral compartment for presurgical planning of an osteotomy; however, the
integrity of the medial meniscus has no clinical importance in a patient with severe medial
compartment arthritis. A repeat corticosteroid injection is not indicated within 1 month of his last
injection, and referral to pain management is not appropriate with other options available to help
this patient.

A VPHTO is the appropriate intervention considering the patient’s young age, high-functional
occupation, examination, radiographic findings, and response to medial unloader bracing. A
revision knee arthroscopy would be appropriate for a recurrent medial meniscus tear, but not
appropriate in a patient with severe medial compartment arthritis. The patient’s young age and
high functional requirements are contraindications to TKA. The presence of severe arthritis is a
contraindication to medial meniscus transplant.

UI // UNAIR // UNPAD // UNHAS // UNS // UGM // UB // UNUD // USU


Sports Medicine Scored and Recorded Self-Assessment Examination 2019

The patient is a candidate for a VPHTO. The technical options include a medial opening-wedge
or a lateral closing-wedge osteotomy. Both techniques have advantages and disadvantages;
however, a medial opening-wedge osteotomy is contraindicated in a smoker because of concern
for nonunion. As a result, current smoking history is the only factor listed that would influence the
technique used. The history of prior arthroscopy has no relevance in the decision about which
type of osteotomy is appropriate. Normal BMI is between 18.5 and 24.9, so this patient’s BMI is
considered normal and would not affect the surgical technique (if this patient were obese, a
lateral closing-wedge osteotomy would be considered, but this is controversial). His age of 40
years is an indication for HTO but does not influence technique.

47. A 40-year-old man who is a manual laborer has had 3 years of worsening medial-sided left knee
pain that has inhibited his ability to work. He reports undergoing a left subtotal medial
meniscectomy 10 years ago. He has been treated with nonsteroidal anti-inflammatory drugs and
two different corticosteroids, with the most recent injection given 1 month ago. Each injection
provided him with a few weeks of pain control. His medical history is unremarkable, and he has
smoked 20 cigarettes per day for the last 15 years. His BMI is 22. On examination, he has varus
alignment of the involved leg and medial joint line tenderness and no lateral or patellofemoral
pain. His knee range of motion is 3° shy of full extension to 130° of flexion. He has negative
Lachman and posterior drawer test results. He demonstrates no lateral thrust with ambulation.

The patient is offered a VPHTO. What aspect of his history will determine the most appropriate
VPHTO technique?

a. Prior arthroscopy
b. Current smoking history
c. BMI of 22
d. Age of 40

Preffered Answer : 2
This patient has a classic presentation of postmeniscectomy medial compartment arthritis. The
appropriate diagnostic study is weight-bearing radiographs to confirm the diagnosis. An MRI
scan will reveal medial compartment arthritis but will not provide information about alignment. A
CT scan would be appropriate to detect an occult fracture; however, this condition is not
suspected in this clinical scenario. Ultrasonography can provide information about fluid collection
around the knee or a deep vein thrombosis; however, these conditions also are not suspected in
this clinical scenario.

Because the patient has a correctable deformity (gaps 3 mm with valgus stress), and his
symptoms are localized to the involved compartment, a trial of a medial unloader brace is
appropriate both diagnostically and therapeutically. If unloading the medial compartment resolves
the patient’s symptoms, he would be an excellent candidate for an osteotomy. An MRI scan may
be obtained to evaluate ligamentous integrity or to evaluate degenerative involvement of the
lateral and patellofemoral compartment for presurgical planning of an osteotomy; however, the
integrity of the medial meniscus has no clinical importance in a patient with severe medial
compartment arthritis. A repeat corticosteroid injection is not indicated within 1 month of his last
injection, and referral to pain management is not appropriate with other options available to help
this patient.

A VPHTO is the appropriate intervention considering the patient’s young age, high-functional
occupation, examination, radiographic findings, and response to medial unloader bracing. A

UI // UNAIR // UNPAD // UNHAS // UNS // UGM // UB // UNUD // USU


Sports Medicine Scored and Recorded Self-Assessment Examination 2019

revision knee arthroscopy would be appropriate for a recurrent medial meniscus tear, but not
appropriate in a patient with severe medial compartment arthritis. The patient’s young age and
high functional requirements are contraindications to TKA. The presence of severe arthritis is a
contraindication to medial meniscus transplant.

The patient is a candidate for a VPHTO. The technical options include a medial opening-wedge
or a lateral closing-wedge osteotomy. Both techniques have advantages and disadvantages;
however, a medial opening-wedge osteotomy is contraindicated in a smoker because of concern
for nonunion. As a result, current smoking history is the only factor listed that would influence the
technique used. The history of prior arthroscopy has no relevance in the decision about which
type of osteotomy is appropriate. Normal BMI is between 18.5 and 24.9, so this patient’s BMI is
considered normal and would not affect the surgical technique (if this patient were obese, a
lateral closing-wedge osteotomy would be considered, but this is controversial). His age of 40
years is an indication for HTO but does not influence technique.

48. Figure 1 is the MRI scan of a patient with recurrent knee instability, which persists after a period
of nonsurgical treatment. Anatomic reconstruction of the torn ligament is recommended. What
radiographic finding is the most important independent predictor of recurrent instability following
surgery?

a. Tibial tubercle to trochlear groove (TT-TG) distance


b. Patella alta
c. Tibial slope
d. Trochlear
dysplasia Preffered
Answer : 4

The MR image is consistent with an episode of patellar instability with concomitant bruising of the
medial patellar facet and lateral femoral condyle. The medial patellofemoral ligament appears torn
and attenuated. Kita and associates reported that severe trochlear dysplasia is the most important
predictor of residual patellofemoral instability after isolated medial patellofemoral ligament
reconstruction. An increased TT-TG affected outcomes of patients with type D trochlear dysplasia
(Dejour classification). Wagner and associates also found that high degrees of trochlear dysplasia
correlate with poor clinical outcome due to graft overload in dysplastic situations. Other studies by
Nelitz and associates and Matsushita and associates have also suggested that TT-TG distance did
not reliably correlate with clinical outcome. Tibial slope would not affect recurrent patellar
instability.

UI // UNAIR // UNPAD // UNHAS // UNS // UGM // UB // UNUD // USU


Sports Medicine Scored and Recorded Self-Assessment Examination 2019

49. Figure 1 is an arthroscopic view of the intercondylar notch of a right knee from an anterolateral
portal. What is the main function of the structure delineated by the black asterisks?

Figure 1

a. Resist anterior translation during knee flexion


b. Resist posterior translation during knee flexion
c. Resist rotatory loads during knee flexion
d. Resist rotatory loads during knee extension
Preffered Answer : 4
The structure shown is the posterolateral bundle of the anterior cruciate ligament (ACL). This
bundle is optimally positioned in the knee to resist rotatory forces during terminal knee extension.
"Resist anterior translation during knee flexion" best describes the anteromedial bundle. "Resist
rotatory loads during knee flexion" is unlikely because the posterolateral bundle is tightest during
knee extension. The posterior cruciate ligament, not the ACL, functions to resist posterior
translation.

50. A 19-year old Division 1 offensive lineman sustains an ankle injury during a game. He has pain
with weight-bearing and is unable to return to the game. Figures 1 through 5 are his radiographs
taken the next day. What is the best next step?

a. Immobilization

UI // UNAIR // UNPAD // UNHAS // UNS // UGM // UB // UNUD // USU


Sports Medicine Scored and Recorded Self-Assessment Examination 2019

b. Syndesmotic fixation
c. Physical therapy
d. Obtain an MRI scan

Preffered Answer: 2
The radiographs reveal medial clear space widening and an oblique proximal fibula fracture (best
seen on the lateral view overlapping the tibia). This is consistent with an unstable syndesmotic
injury. Operative repair of the syndesmosis with reduction and fixation is warranted.
Immobilization is the best option for a stable syndesmotic injury. Physical therapy and MRI are
not warranted given the findings on the radiographs.

51. A 16-year-old boy falls while playing soccer. He reports that his knee buckled when he planted
his leg to kick a ball. He noticed an obvious deformity of his knee, which spontaneously resolved
with a ―clunk.‖ He could not finish the game but was able to bear weight with a limp. He has had
two similar episodes but has never sought medical attention. An initial examination demonstrated
an effusion, tenderness at the proximal medial collateral region and medial patellofemoral
retinaculum, decreased range of motion, and patella apprehension. A lateral patellar glide
performed at 30° of flexion was 3+. He was otherwise ligamentously stable, and there were no
other noteworthy findings.

What do Figures 1 and 2 reveal?

Figure 1 Figure 2

a. Medial femoral condyle physeal widening


b. An osseous or osteochondral loose fragment
c. Osgood-Schlatter disease
d. A patella nondisplaced fracture

Preffered Answer : 2
This patient’s examination and history indicate recurrent patellar dislocations. Radiographs show
an osseous or osteochondral loose fragment. There is no evidence of an obvious nondisplaced
fracture or physeal changes. In the setting of suspected patella dislocation or subluxation with
loose fragment seen on radiograph, an MRI is indicated. Lateral release alone is seldom
indicated in a knee that is normal before injury. The examination and MRI do not indicate a need

UI // UNAIR // UNPAD // UNHAS // UNS // UGM // UB // UNUD // USU


Sports Medicine Scored and Recorded Self-Assessment Examination 2019

for medial collateral ligament repair. Treatment should consist of arthroscopy or arthrotomy and
attempted internal fixation of this fragment. If fixation is not possible, the loose body can be
removed. Normal TT-TG values, an increased lateral patellar glide, and a history of recurrent
patellar dislocations after trauma suggest MPFL incompetence and the need for reconstruction.

52. A 16-year-old boy falls while playing soccer. He reports that his knee buckled when he planted
his leg to kick a ball. He noticed an obvious deformity of his knee, which spontaneously resolved
with a ―clunk.‖ He could not finish the game but was able to bear weight with a limp. He has had
two similar episodes but has never sought medical attention. An initial examination demonstrated
an effusion, tenderness at the proximal medial collateral region and medial patellofemoral
retinaculum, decreased range of motion, and patella apprehension. A lateral patellar glide
performed at 30° of flexion was 3+. He was otherwise ligamentously stable, and there were no
other noteworthy findings.

Figures 3 and 4 are this patient's proton density fat-saturated MR images. His tibial tubercle-
trochlear groove (TT-TG) distance is 12 mm, and he has normal limb-alignment film findings.
Treatment at this stage should include

Figure 1 Figure 2

a. hinged knee bracing, protected weight bearing, and physical therapy.


b. anteromedialization of the tibial tubercle.
c. internal fixation and medial patellofemoral ligament (MPFL) reconstruction.
d. arthroscopic lateral retinacular release.

Preffered Answer : 3
This patient’s examination and history indicate recurrent patellar dislocations. Radiographs show
an osseous or osteochondral loose fragment. There is no evidence of an obvious nondisplaced
fracture or physeal changes. In the setting of suspected patella dislocation or subluxation with
loose fragment seen on radiograph, an MRI is indicated. Lateral release alone is seldom
indicated in a knee that is normal before injury. The examination and MRI do not indicate a need
for medial collateral ligament repair. Treatment should consist of arthroscopy or arthrotomy and
attempted internal fixation of this fragment. If fixation is not possible, the loose body can be
removed. Normal TT-TG values, an increased lateral patellar glide, and a history of recurrent
patellar dislocations after trauma suggest MPFL incompetence and the need for reconstruction.

UI // UNAIR // UNPAD // UNHAS // UNS // UGM // UB // UNUD // USU


Sports Medicine Scored and Recorded Self-Assessment Examination 2019

53. Figure 1 is the axial cut MRI scan of a 35-year-old woman who has had posteriorly based right
hip pain for 3 months. Examination demonstrates full and symmetric range of motion between
the right and left hips, negative impingement test, but reproduction of her pain with passive
extension of the right hip. Which muscle is indicated by the arrow?

Figure 1

a. Piriformis
b. Obturator internus
c. Superior gemellus
d. Quadratus femoris

Preffered Answer: 4
This patient has ischiofemoral impingement, in which there is abnormal contact between the
lesser trochanter and the lateral border of the ischium. Patients typically present with posteriorly
based hip pain and do not respond to intra-articular diagnostic injections. Examination can
demonstrate pain with long strides, pain with palpation over the area, as well as reproduction of
symptoms with the patient in the contralateral decubitus position and taking the affected hip into
passive extension (ischiofemoral impingement test). MRI demonstrates a narrowed ischiofemoral
space, as well as increased signal within the quadratus femoris muscle. The diagnosis can be
confirmed with a diagnostic injection into this area. Treatment is typically nonsurgical, with
surgical intervention consisting of resection of the lesser trochanter reserved for refractory cases.

54. A 17-year-old high school football player sustains a neck injury in a game. During the initial on-
field assessment, the team physician removes the player’s helmet, and the athlete is log-rolled to
the supine position while the physician manually stabilizes his cervical spine. An examination
demonstrates tenderness to palpation over the cervical spine and neurologic deficits in bilateral
upper and lower extremities. Shoulder pads prohibit proper placement of a hard cervical collar,
and the athlete is immobilized on a spine board and transported to the emergency department
via ambulance. Comprehensive evaluation in the emergency department reveals a bilateral facet
dislocation of C5 on C6. The on-field intervention most likely to cause a neurologic injury is
a. ailure to place a hard cervical collar.
b. helmet removal prior to examination.
c. transfer to a spine board prior to transport.
d. log-rolling the athlete to the supine position.

Preffered Answer: 2
Complete immobilization of the cervical spine is critical for athletes with a suspected cervical
spine or spinal cord injury. The spinal cord in the subaxial spine is especially sensitive to motion,

UI // UNAIR // UNPAD // UNHAS // UNS // UGM // UB // UNUD // USU


Sports Medicine Scored and Recorded Self-Assessment Examination 2019

and removal of protective gear such as the helmet and shoulder pads presents an unacceptable
risk for progressive neurologic injury in the setting of a potentially unstable cervical spine injury.
Removal of the face mask alone is typically performed to improve access to an athlete's airway.
Protective equipment often prevents proper placement of a hard cervical collar, and the spine
board offers a variety of options for safe cervical spine immobilization of helmeted athletes
without a hard cervical collar. The log-roll and lift-and-slide techniques allow for the safe transfer
of an athlete to a spine board while maintaining appropriate manual stabilization of the cervical
spine.

55. Figures 1 and 2 are the radiographs of a 24-year-old male wrestler who underwent surgery for
recurrent shoulder dislocations using coracoid autograft. At his first postoperative visit, the
patient complains of decreased sensation on the lateral aspect of his forearm. The patient’s
symptoms are most likely due to injury of the

Figure 1 Figure 2
a. axillary nerve.
b. musculocutaneous nerve.
c. median nerve.
d. radial nerve.

Preffered Answer : 2
The patient has undergone a Latarjet procedure as shown in the radiographs. After harvesting
the coracoid graft, care must be taken to not place too much tension on or dissect excessively
near the musculocutaneous nerve. The nerve is encountered 5 cm distal to the coracoid as it
enters the conjoint tendon. The lateral antebrachial cutaneous nerve is the terminal branch of the
musculocutaneous nerve and; therefore, injury can cause decreased sensation in the lateral
forearm.

56. A 19-year-old running back lands directly on his anterior knee after being tackled. He has mild
anterior knee pain, a trace effusion, a 2+ posterior drawer, a grade 1+ stable Lachman, no valgus
laxity, and negative dial tests at 30 degrees and 90 degrees. What is the best treatment strategy
at this time?
a. Physical therapy with a focus on quadriceps strengthening
b. Physical therapy and delayed posterior cruciate ligament (PCL) reconstruction
c. PCL reconstruction
d. PCL and posterolateral corner reconstruction

UI // UNAIR // UNPAD // UNHAS // UNS // UGM // UB // UNUD // USU


Sports Medicine Scored and Recorded Self-Assessment Examination 2019

Preffered Answer: 1
This patient has likely sustained an isolated PCL injury. The examination is consistent with a
grade II injury to the PCL. In this scenario, the best initial option is nonsurgical treatment and
return to play as symptoms subside and strength improves. Physical therapy with a focus on
quadriceps strengthening and delayed PCL reconstruction is not the answer because this patient
can likely be treated without surgery. The absence of valgus laxity and negative dial testing
findings suggest that an injury to the posteromedial and posterolateral corners has not occurred.
Initial nonsurgical treatment is indicated for this patient. If he completes rehabilitation and
experiences persistent disability with anterior and/or medial knee discomfort or senses the knee
is "loose," PCL reconstruction should be considered at that time.

57. Figures 1 and 2 are the radiographs of a 55-year-old man who has a 3-year history of right
shoulder pain. He has maximized nonoperative management and is interested in operative
treatment. He had an open Bankart repair 20 years ago and did well until a few years ago. What
is most important to know when deciding on the best surgical treatment for this patient?

Figure 1 Figure 2

a. Range of motion
b. Infraspinatus strength
c. Activity level
d. Quality of the subscapularis

Preffered Answer : 4
The radiographs show severe osteoarthritis of the shoulder. The best surgical option would be an
arthroplasty. The major determining factor for which type of arthroplasty to choose is the integrity
of the rotator cuff tendons. Although the radiographs do not reveal any obvious signs of rotator
cuff failure, the patient had had an open Bankart 20 years ago. The integrity of the subscapularis
tendon following a previous open shoulder procedure is crucial in deciding which type of
arthroplasty to consider. Range of motion and infraspinatus strength do not affect the decision-
making process, assuming the rotator cuff is intact. Activity level can be important when deciding
whether to proceed with an arthroplasty, but it is not as important as the rotator cuff integrity
when choosing which arthroplasty to use.

58. A 47-year-old man who is an avid tennis player and laborer has had one year of shoulder pain
and weakness. His pain occurs at night and radiates to the deltoid laterally. The patient denies

UI // UNAIR // UNPAD // UNHAS // UNS // UGM // UB // UNUD // USU


Sports Medicine Scored and Recorded Self-Assessment Examination 2019

any anterior based pain. He reports no prior surgeries and has been managed with steroid
injections and physical therapy. On examination, he has full passive motion with significant
weakness with external rotation. His neurologic examination is unremarkable. MRI evaluation
reveals a posterior-superior rotator cuff tear with Goutallier grade 4 fatty infiltrate in the
supraspinatus and infraspinatus with retraction beyond the glenoid. He is concerned about the
lack of rotation of his arm and reports that this disability creates significant disability with his
occupation as a mason. What is the best next step?
a. Shoulder scope and subacromial decompression
b. Tendon transfer
c. Total shoulder arthroplasty
d. Reverse total shoulder arthroplasty

Preffered Answer: 2
In younger active patients, tendon transfer is considered a preferable treatment option. The
patient has failed a course of nonoperative management. Subacromial decompression may offer
pain relief but may not be advisable in a patient with rotator cuff deficient shoulder. A total
shoulder arthroplasty requires functionality of the supraspinatus and infraspinatus. A reverse total
shoulder is an option to alleviate pain and perhaps improve forward flexion height and strength;
however, reverse arthroplasty would not improve external rotation in this patient, and there is
concern for longevity of the implant in younger patient populations.

59. A 24-year-old collegiate pitcher has had increasing pain over his medial elbow for 3 months. He
has point tenderness over his medial epicondyle and reproduction of his symptoms with a valgus
stress test. Which phase of the throwing cycle shown in Figure 1 will most likely reproduce his
symptoms?

Reproduced with permission from Poss R (ed): Orthopaedic Knowledge Update 3. Rosemont, IL.
American Academy of Orthopaedic Surgeons, 1990, pp 293-302.

a. A
b. B
c. C
d. D

Preffered Answer: 3
This patient is experiencing soreness over his medial (ulnar) collateral ligament. Valgus overload
is likely to reproduce his symptoms and is most pronounced during the late cocking phase of the
throwing cycle. In windup, very little elbow torque is required. In early cocking, the arm is getting

UI // UNAIR // UNPAD // UNHAS // UNS // UGM // UB // UNUD // USU


Sports Medicine Scored and Recorded Self-Assessment Examination 2019

loaded, and maximum valgus is not yet achieved at the elbow. In acceleration and deceleration,
more force is generated at the level of the shoulder joint.

60. Figure 1 is the MR image of a 36-year-old athlete who is tackled from behind and falls forward
onto his left knee. He has pain, swelling, and stiffness. Examination includes a moderate
effusion, positive quadriceps active test, and normal Lachman test finding.

The injured structure is composed of an

Figure 1

a. anterolateral bundle that is tight in flexion and a posteromedial bundle that is tight in
extension.

b. anterolateral bundle that is tight in extension and a posteromedial bundle that is tight in
flexion.

c. anteromedial bundle that is tight in flexion and a posterolateral bundle that is tight in
extension.

d. anteromedial bundle that is tight in extension and a posterolateral bundle that is tight in
flexion.

Preffered Answer: 1
The clinical description and MR image point to an injury to the posterior cruciate ligament (PCL).
This ligament is thought to be primarily composed of anterolateral and posteromedial bundles,
with the former tightening in flexion and the latter in extension. Because of alterations in knee
kinematics and increased varus alignment in PCL insufficiency, contact stresses and cartilage
loads increase in the patellofemoral and medial compartments. Although good outcomes may be
obtained with transtibial, open inlay, and arthroscopic inlay techniques, one major difference is
the creation of the ―killer-turn‖ during the transtibial approach. This sharp turn in the graft as it
emerges from the tibia appears to lead to more pronounced attenuation and thinning of the graft
during cyclic loading.

The scenario describes a patient with chronic PCL and posterolateral corner (PLC) injury, as
evidenced by the varus thrust and abnormal Dial test finding. A valgus-producing osteotomy may
be effective, and, in fact, may be the only treatment necessary to address chronic PLC injury.
Accordingly, an opening lateral osteotomy would not be appropriate. Of the remaining responses,
an osteotomy that increases tibial slope would also address the PCL deficiency by reducing
posterior tibial sag.

UI // UNAIR // UNPAD // UNHAS // UNS // UGM // UB // UNUD // USU


Sports Medicine Scored and Recorded Self-Assessment Examination 2019

Vascular injury is an uncommon, but potentially devastating, complication associated with PCL
surgery and may occur regardless of the technique used. Numerous strategies have been
described to reduce the risk, including use of a posteromedial accessory incision to allow finger
retraction of the popliteal neurovascular bundle, oscillating drills to prevent excessive soft-tissue
entanglement, and tapered (rather than square) drill bits that may minimize cut-out of sharp
edges as drilling reaches the posterior tibial cortex. Knee extension lessens, rather than
increases, the distance between the posterior tibia and the neurovascular bundle and increases,
not lessens, risk for vascular injury.

61. Figure 1 is the MR image of a 36-year-old athlete who is tackled from behind and falls forward
onto his left knee. He has pain, swelling, and stiffness. Examination includes a moderate
effusion, positive quadriceps active test, and normal Lachman test finding.

Left untreated, injury to this structure most likely will lead to degenerative changes in

a. medial and lateral compartments.


b. medial and patellofemoral compartments.
c. lateral and patellofemoral compartments.
d. the patellofemoral compartment only.

Preffered Answer : 2
The clinical description and MR image point to an injury to the posterior cruciate ligament (PCL).
This ligament is thought to be primarily composed of anterolateral and posteromedial bundles,
with the former tightening in flexion and the latter in extension. Because of alterations in knee
kinematics and increased varus alignment in PCL insufficiency, contact stresses and cartilage
loads increase in the patellofemoral and medial compartments. Although good outcomes may be
obtained with transtibial, open inlay, and arthroscopic inlay techniques, one major difference is
the creation of the ―killer-turn‖ during the transtibial approach. This sharp turn in the graft as it
emerges from the tibia appears to lead to more pronounced attenuation and thinning of the graft
during cyclic loading.

The scenario describes a patient with chronic PCL and posterolateral corner (PLC) injury, as
evidenced by the varus thrust and abnormal Dial test finding. A valgus-producing osteotomy may
be effective, and, in fact, may be the only treatment necessary to address chronic PLC injury.
Accordingly, an opening lateral osteotomy would not be appropriate. Of the remaining responses,
an osteotomy that increases tibial slope would also address the PCL deficiency by reducing
posterior tibial sag.

Vascular injury is an uncommon, but potentially devastating, complication associated with PCL
surgery and may occur regardless of the technique used. Numerous strategies have been

UI // UNAIR // UNPAD // UNHAS // UNS // UGM // UB // UNUD // USU


Sports Medicine Scored and Recorded Self-Assessment Examination 2019

described to reduce the risk, including use of a posteromedial accessory incision to allow finger
retraction of the popliteal neurovascular bundle, oscillating drills to prevent excessive soft-tissue
entanglement, and tapered (rather than square) drill bits that may minimize cut-out of sharp
edges as drilling reaches the posterior tibial cortex. Knee extension lessens, rather than
increases, the distance between the posterior tibia and the neurovascular bundle and increases,
not lessens, risk for vascular injury.

62. Figure 1 is the MR image of a 36-year-old athlete who is tackled from behind and falls forward
onto his left knee. He has pain, swelling, and stiffness. Examination includes a moderate
effusion, positive quadriceps active test, and normal Lachman test finding.

If the patient chooses surgical reconstruction, he should be advised that, when compared with a
transtibial technique, the tibial inlay technique has been shown to provide

a. stronger initial graft fixation.


b. more anatomic positioning of tibial fixation.
c. more natural knee kinematics during deep flexion.
d. more graft protection during cyclic loading.

Preffered Answer : 4
The clinical description and MR image point to an injury to the posterior cruciate ligament (PCL).
This ligament is thought to be primarily composed of anterolateral and posteromedial bundles,
with the former tightening in flexion and the latter in extension. Because of alterations in knee
kinematics and increased varus alignment in PCL insufficiency, contact stresses and cartilage
loads increase in the patellofemoral and medial compartments. Although good outcomes may be
obtained with transtibial, open inlay, and arthroscopic inlay techniques, one major difference is
the creation of the ―killer-turn‖ during the transtibial approach. This sharp turn in the graft as it
emerges from the tibia appears to lead to more pronounced attenuation and thinning of the graft
during cyclic loading.

The scenario describes a patient with chronic PCL and posterolateral corner (PLC) injury, as
evidenced by the varus thrust and abnormal Dial test finding. A valgus-producing osteotomy may
be effective, and, in fact, may be the only treatment necessary to address chronic PLC injury.
Accordingly, an opening lateral osteotomy would not be appropriate. Of the remaining responses,
an osteotomy that increases tibial slope would also address the PCL deficiency by reducing
posterior tibial sag.

UI // UNAIR // UNPAD // UNHAS // UNS // UGM // UB // UNUD // USU


Sports Medicine Scored and Recorded Self-Assessment Examination 2019

Vascular injury is an uncommon, but potentially devastating, complication associated with PCL
surgery and may occur regardless of the technique used. Numerous strategies have been
described to reduce the risk, including use of a posteromedial accessory incision to allow finger
retraction of the popliteal neurovascular bundle, oscillating drills to prevent excessive soft-tissue
entanglement, and tapered (rather than square) drill bits that may minimize cut-out of sharp
edges as drilling reaches the posterior tibial cortex. Knee extension lessens, rather than
increases, the distance between the posterior tibia and the neurovascular bundle and increases,
not lessens, risk for vascular injury.

63. Figure 1 is the MR image of a 36-year-old athlete who is tackled from behind and falls forward
onto his left knee. He has pain, swelling, and stiffness. Examination includes a moderate
effusion, positive quadriceps active test, and normal Lachman test finding.

This patient elects nonsurgical treatment and later experiences persistent instability. Examination
reveals an asymmetric Dial test finding and a varus thrust during ambulation. Which osteotomy
and correction appropriately addresses this chronic instability pattern?

a. Distal femoral/opening lateral wedge osteotomy


b. Distal femoral/closing lateral wedge osteotomy
c. High tibial osteotomy; opening medial wedge with increased tibial slope
d. High tibial osteotomy; closing lateral wedge with decreased tibial slope

Preffered Answer : 3
The clinical description and MR image point to an injury to the posterior cruciate ligament (PCL).
This ligament is thought to be primarily composed of anterolateral and posteromedial bundles,
with the former tightening in flexion and the latter in extension. Because of alterations in knee
kinematics and increased varus alignment in PCL insufficiency, contact stresses and cartilage
loads increase in the patellofemoral and medial compartments. Although good outcomes may be
obtained with transtibial, open inlay, and arthroscopic inlay techniques, one major difference is
the creation of the ―killer-turn‖ during the transtibial approach. This sharp turn in the graft as it
emerges from the tibia appears to lead to more pronounced attenuation and thinning of the graft
during cyclic loading.

The scenario describes a patient with chronic PCL and posterolateral corner (PLC) injury, as
evidenced by the varus thrust and abnormal Dial test finding. A valgus-producing osteotomy may
be effective, and, in fact, may be the only treatment necessary to address chronic PLC injury.

UI // UNAIR // UNPAD // UNHAS // UNS // UGM // UB // UNUD // USU


Sports Medicine Scored and Recorded Self-Assessment Examination 2019

Accordingly, an opening lateral osteotomy would not be appropriate. Of the remaining responses,
an osteotomy that increases tibial slope would also address the PCL deficiency by reducing
posterior tibial sag.

Vascular injury is an uncommon, but potentially devastating, complication associated with PCL
surgery and may occur regardless of the technique used. Numerous strategies have been
described to reduce the risk, including use of a posteromedial accessory incision to allow finger
retraction of the popliteal neurovascular bundle, oscillating drills to prevent excessive soft-tissue
entanglement, and tapered (rather than square) drill bits that may minimize cut-out of sharp
edges as drilling reaches the posterior tibial cortex. Knee extension lessens, rather than
increases, the distance between the posterior tibia and the neurovascular bundle and increases,
not lessens, risk for vascular injury.

64. Figure 1 is the MR image of a 36-year-old athlete who is tackled from behind and falls forward
onto his left knee. He has pain, swelling, and stiffness. Examination includes a moderate
effusion, positive quadriceps active test, and normal Lachman test finding.

Risk for vascular injury during transtibial drilling for reconstruction of this injury is increased by

a. accessory incisions.
b. use of tapered drill bits.
c. use of oscillating drills.
d. greater knee extension.

Preffered Answer : 4
The clinical description and MR image point to an injury to the posterior cruciate ligament (PCL).
This ligament is thought to be primarily composed of anterolateral and posteromedial bundles,
with the former tightening in flexion and the latter in extension. Because of alterations in knee
kinematics and increased varus alignment in PCL insufficiency, contact stresses and cartilage
loads increase in the patellofemoral and medial compartments. Although good outcomes may be
obtained with transtibial, open inlay, and arthroscopic inlay techniques, one major difference is
the creation of the ―killer-turn‖ during the transtibial approach. This sharp turn in the graft as it
emerges from the tibia appears to lead to more pronounced attenuation and thinning of the graft
during cyclic loading.

UI // UNAIR // UNPAD // UNHAS // UNS // UGM // UB // UNUD // USU


Sports Medicine Scored and Recorded Self-Assessment Examination 2019

The scenario describes a patient with chronic PCL and posterolateral corner (PLC) injury, as
evidenced by the varus thrust and abnormal Dial test finding. A valgus-producing osteotomy may
be effective, and, in fact, may be the only treatment necessary to address chronic PLC injury.
Accordingly, an opening lateral osteotomy would not be appropriate. Of the remaining responses,
an osteotomy that increases tibial slope would also address the PCL deficiency by reducing
posterior tibial sag.

Vascular injury is an uncommon, but potentially devastating, complication associated with PCL
surgery and may occur regardless of the technique used. Numerous strategies have been
described to reduce the risk, including use of a posteromedial accessory incision to allow finger
retraction of the popliteal neurovascular bundle, oscillating drills to prevent excessive soft-tissue
entanglement, and tapered (rather than square) drill bits that may minimize cut-out of sharp
edges as drilling reaches the posterior tibial cortex. Knee extension lessens, rather than
increases, the distance between the posterior tibia and the neurovascular bundle and increases,
not lessens, risk for vascular injury.

65. A 15-year-old male ice hockey player is hit in the chest by a puck and immediately falls to the
ground unconscious. What has been shown to predict survival in the treatment of this condition?
a. Use of chest protectors
b. Time to initiation of chest compressions
c. Lower velocity of the puck at impact
d. Time to defibrillation

Preffered Answer : 4
The hockey player is suffering from commotio cordis, in which a cardiac arrhythmia occurs after a
sudden blunt impact to the chest. Treatment of commotio cordis is defibrillation. As the time to
defibrillation increases, the likelihood of survival decreases. In animal models, chest protectors
have not shown efficacy against ventricular fibrillation. The velocity of the projectile (most
commonly baseball, hockey puck or lacrosse ball) has also not been shown to alter survival.

66. Figures 1 through 6 reveal the radiographs and MR images of a 30-year-old man who has a 1-
year history of atraumatic medial-sided left knee pain refractory to nonsurgical measures. What is
the most appropriate treatment?

UI // UNAIR // UNPAD // UNHAS // UNS // UGM // UB // UNUD // USU


Sports Medicine Scored and Recorded Self-Assessment Examination 2019

a. Distal femoral varus osteotomy


b. Autologous chondrocyte implantation (ACI)
c. Fresh osteochondral allograft (OCA) transplantation
d. Arthroscopic microfracture

Preffered Answer : 3
The images illustrate a large unstable osteochondral lesion of the medial femoral condyle.
Radiographs and MR images clearly show deep subchondral bone involvement. The appropriate
choice of surgery is OCA transplantation, which is indicated for primary treatment of large
cartilage lesions, osteochondral lesions, and salvage procedure from failed prior cartilage
surgery. Correction of mechanical axis malalignment, ligamentous insufficiency, and meniscal
deficiency should also be addressed. ACI alone or an arthroscopic microfracture procedure
would not address the bone defect, leaving an uneven articular surface. Although an osteotomy
may be a viable choice, a distal femoral varus osteotomy would increase the contact pressure in
the medial compartment and worsen the situation.

The histologic anatomy of articular cartilage is well described. The superficial layer or lamina
splendens contains a small amount of proteoglycan with collagen fibrils arranged parallel to the
articular surface. In contrast, the deep zone contains the largest-diameter collagen fibrils,
oriented perpendicular to the joint surface, and the highest concentration of proteoglycans.

UI // UNAIR // UNPAD // UNHAS // UNS // UGM // UB // UNUD // USU


Sports Medicine Scored and Recorded Self-Assessment Examination 2019

67. A hockey player had a puck hit his foot. Radiographs taken immediately after the game were
negative. He still has persistent pain 5 days after the injury and difficulty weight bearing. What is
the best next step?
a. Repeat radiographs
b. Full clearance to return to play
c. Bone scan
d. MRI scan

Preffered Answer : 4
Ice hockey injuries demand a thorough assessment because they have the potential to be
significant. In hockey players, bone injuries in the foot and ankle can be missed or improperly
diagnosed through routine radiographic imaging. MRI can display bone injuries that are not found
radiographically; this is because some fractures and contusions involve the medial ankle and
midfoot bones.

68. A 19-year-old male collegiate rower has a 3-month history of right shoulder pain. There was no
inciting trauma prior to the onset of his pain. He also complains of weakness, particularly in
abduction and overhead activity. Examination reveals no range-of-motion deficits. Strength
testing of the right shoulder demonstrates 4/5 motor strength in forward elevation and abduction.
His Beighton hypermobility score is 3/9. Figure 1 shows his scapular position during a wall
pushup maneuver. An EMG would likely reveal damage to what nerve?

a. Long thoracic nerve


b. Cranial nerve XI (spinal accessory nerve)
c. Suprascapular nerve
d. Axillary nerve

Preffered Answer : 1
Figure 1 reveals medial scapular winging secondary to weakness of the serratus anterior, which
is innervated by the long thoracic nerve. Damage to the long thoracic nerve can occur via
repetitive stretching, compression, or iatrogenic injury during a surgical procedure. Lateral
thoracic winging is caused by weakness of the trapezius, which is innervated by cranial nerve XI
(spinal accessory nerve). The direction of scapular winging is judged by the upper medial border
of the scapula. Observation of a period of at least 6 months with serratus anterior strengthening
while the nerve recovers is the mainstay of treatment for medial scapular winging.

69. Figure 1 is the MR image of a 14-year-old football player who injured his right knee during a
game. He describes feeling a "pop" and then needing help walking off the field. His knee is
visibly swollen. Knee range of motion is between 0° and 70°. What is the most appropriate
treatment option?

UI // UNAIR // UNPAD // UNHAS // UNS // UGM // UB // UNUD // USU


Sports Medicine Scored and Recorded Self-Assessment Examination 2019

Figure 1

a. Open reduction and internal fixation of the lateral condyle


b. Microfracture of the chondral defect
c. Immediate anterior cruciate ligament (ACL) reconstruction
d. Delayed ACL reconstruction

Preffered Answer : 4
The MR image shows bone bruises (―kissing contusions‖) consistent with an ACL tear. During
the ACL subluxation event, the posterolateral tibial plateau subluxes anteriorly, making contact
with the mid portion of the lateral femoral condyle and resulting in this characteristic bone bruise
pattern on MRI. Randomized clinical trials comparing early accelerated versus nonaccelerated
rehabilitation programs have demonstrated no significant differences in long-term results with
regard to function, reinjury, and successful return to play. These studies did not address timing of
return to play with an early accelerated rehabilitation program. At 2 and 3 years postsurgically,
there are no differences in laxity, number of graft failures, or KOOS scores.
70. Figure 1 is the MR image of a 14-year-old football player who injured his right knee during a
game. He describes feeling a "pop" and then needing help walking off the field. His knee is
visibly swollen.

The patient undergoes surgery to repair/reconstruct the damaged structure and has no
postsurgical complications and begins physical therapy rehabilitation. The boy and his parents
stress they want to ―get the therapy over with as fast as possible" to expedite his return to sports,
and the surgeon and rehabilitation team consider their request. Compared with nonaccelerated
rehabilitation, patients who follow an early accelerated rehabilitation protocol experience

UI // UNAIR // UNPAD // UNHAS // UNS // UGM // UB // UNUD // USU


Sports Medicine Scored and Recorded Self-Assessment Examination 2019

a. increased laxity.
b. increased risk for graft failure.
c. no differences in long-term results.
d. lower Knee Injury and Osteoarthritis Outcome Score (KOOS).

Preffered Answer: 3
The MR image shows bone bruises (―kissing contusions‖) consistent with an ACL tear. During
the ACL subluxation event, the posterolateral tibial plateau subluxes anteriorly, making contact
with the mid portion of the lateral femoral condyle and resulting in this characteristic bone bruise
pattern on MRI. Randomized clinical trials comparing early accelerated versus nonaccelerated
rehabilitation programs have demonstrated no significant differences in long-term results with
regard to function, reinjury, and successful return to play. These studies did not address timing of
return to play with an early accelerated rehabilitation program. At 2 and 3 years postsurgically,
there are no differences in laxity, number of graft failures, or KOOS scores.

71. Figure 1 is the MRI scan of a 19-year-old man who has an acute anterior shoulder dislocation.
The bony fragment occupies 10% of the glenoid articular surface. What is the most appropriate
treatment?

Figure 1

a. Open structural iliac crest graft


b. Open reduction and internal fixation
c. Arthroscopic coracoid transfer
d. Arthroscopic repair incorporating the bone lesion

Preffered Answer : 4
The MRI scan shows a bony Bankart lesion involving <20% of the glenoid joint surface. A recent
series reported high success rates after arthroscopic treatment when the defect is incorporated
into the repair. Anterior bony deficiencies occupying >25% to >30% of the glenoid joint surface
treated with soft-tissue repair only are associated with high recurrence rates. In these patients,
an open or arthroscopic coracoid transfer or structural iliac crest graft should be considered.
Open reduction and internal fixation has been reported for treatment of large acute glenoid rim
fractures but is not recommended for recurrent anterior shoulder instability in the setting of a 10%
glenoid rim fracture.

UI // UNAIR // UNPAD // UNHAS // UNS // UGM // UB // UNUD // USU


Sports Medicine Scored and Recorded Self-Assessment Examination 2019

72. A 14-year-old gymnast misses her dismount off of the uneven bars, hits the mat face first, and
loses consciousness for about 15 seconds. She is dazed and confused for several minutes. She
does not complain of pain; numbness; or weakness, and she is moving all extremities without
deficit. The athlete and coach want to go back to competition that day. How should they be
advised?

a. Concussion precludes same-day return to play.


b. Order an urgent MRI scan; if findings are normal, she can return to competition.
c. Order neurocognitive testing; if findings are normal, she can return to competition.
d. If she is symptom-free after a 15-minute exertional test, she may return to competition.

Preffered Answer : A
The National Collegiate Athletic Association's (NCAA) 2011 revised health and safety guidelines
regarding concussion management recommend no return to play on the same day of an injury. In
particular, athletes sustaining a concussion should not return to play the same day as their injury.
Before resuming exercise, athletes must be asymptomatic or returned to baseline symptoms at
rest and have no symptoms with cognitive effort. They must be off of medications that could
mask or alter concussion symptoms. Neurocognitive testing can be a helpful tool in determining
brain function even after all symptoms of concussion have resolved. With a comparison baseline
test, this evaluation, in conjunction with a physician's examination, may reduce risk for second
impact syndrome. The athlete's clinical neurologic examination findings (cognitive, cranial nerve,
balance testing) must return to baseline before resuming exercise. Research has shown that
among youth athletes, it may take longer for tested functions to return to baseline (compared with
the recovery rate in adult athletes). Brain MRI scan has no role in evaluating athletes for return to
play in this situation.
73. Figures 1 through 3 are the weight-bearing radiograph and MRI scans of a 27-year-old man who
twisted his knee coming down awkwardly from a lay-up during a basketball game. He felt a sharp
stabbing pain in the posterior aspect of his knee at the time of the injury. Physical examination
reveals a trace effusion, full range of motion but pain with hyperflexion >90° degrees and
tenderness over the affected joint line. What is the most appropriate treatment at this time?

UI // UNAIR // UNPAD // UNHAS // UNS // UGM // UB // UNUD // USU


Sports Medicine Scored and Recorded Self-Assessment Examination 2019

Figure 1 Figure 2

Figure 3

a. Lateral meniscus repair


b. Corticosteroid injection and physical therapy
c. Medial meniscus repair
d. Unloader brace

Preffered Answer : 3
The MRI scan shows a posterior horn medial meniscus root avulsion with bony edema at the
tibial root insertion. The radiograph shows no significant degenerative changes. If left untreated,
posterior meniscal root tears lead to progressive degenerative changes as a result of the altered
tibiofemoral contact pressures and areas. Nonsurgical treatment including injections, physical
therapy, and unloader braces are more appropriate in the older patient with pre-existing
advanced degenerative changes.

74. A 26-year-old weightlifter has increasing pain in his left shoulder for 4 months. Nonsurgical
treatment consisting of anti-inflammatory medication, corticosteroid injections, and rest fails to
alleviate his symptoms. He undergoes an arthroscopic distal clavicle resection with excision of

UI // UNAIR // UNPAD // UNHAS // UNS // UGM // UB // UNUD // USU


Sports Medicine Scored and Recorded Self-Assessment Examination 2019

the distal 8 mm of clavicle (Mumford procedure). Three months after surgery, he reports mild
pain and popping by his clavicle. His clavicle demonstrates mild posterior instability on
examination without any obvious deformity on his radiographs. What structures were
compromised during his excision?
a. Anterior and superior acromioclavicular joint ligaments
b. Posterior and superior acromioclavicular joint ligaments
c. Conoid ligament
d. Trapezoid ligament

Preffered Answer: 2
The posterior and superior acromioclavicular ligaments provide the most restraint to posterior
translation of the acromioclavicular joint and must be preserved during a Mumford procedure.
Anterior and superior acromioclavicular joint ligaments are the opposite of the preferred response
and prevent anterior translation of the clavicle. Injuries to the conoid and trapezoid ligaments are
more pronounced with grade III or higher acromioclavicular separations, with superior migration
of the clavicle relative to the acromion.

75. Augmentation of a Broström repair with the mobilized lateral portion of the extensor retinaculum
(Gould modification) is expected to produce
a. higher risk for iatrogenic nerve injury.
b. decreased ankle range of motion 6 weeks after surgery.
c. no significant biomechanical difference in initial ankle stability.
d. a significantly lower incidence of osteoarthritis on long-term follow-up.

Preffered Answer : C
Multiple biomechanical studies have investigated the contribution of the Gould modification with
the Broström anatomic repair for chronic ankle instability. No studies to date have demonstrated
a statistically significant difference in initial ankle stability with inclusion of the Gould modification
or augmentation of the repair with a mobilized lateral portion of the extensor retinaculum. No
clear association exists between the Broström-Gould repair technique and risk for nerve injury,
postsurgical range of motion, or incidence of osteoarthritis on long-term follow-up.

76. A 12-year-old boy has a head-on head collision while playing soccer. He had no loss of
consciousness but has persistent headaches for 2 weeks. The patient is now back to school and
has no headaches. What is the best next step?

a. Return to full soccer activity


b. Start light aerobic activity
c. Obtain baseline neuropsychological testing
d. MRI scan of the brain

Preffered Answer : B
Mild traumatic brain injury is common in the adolescent child. Neuropsychological examination is
widely used but, in this case, the patient is asymptomatic and has no baseline testing. There is a
limited role for MRI in the recovery process of concussions. Furthermore, higher levels of
physical/cognitive activity should be avoided due to their potential to increase total recovery time.
In this scenario, a graduated return to activity is most appropriate thus, the next appropriate step
is to start light aerobic activity.

UI // UNAIR // UNPAD // UNHAS // UNS // UGM // UB // UNUD // USU


Sports Medicine Scored and Recorded Self-Assessment Examination 2019

77. Figure 1 is the MR image of a 43-year-old man who has left shoulder pain and weakness after a
fall. An examination reveals active forward elevation at 120° and positive Yergason and lift-off
test examination findings. Arthroscopy reveals that the articular surfaces of the glenohumeral
joint have a normal appearance without significant degenerative changes. What is the most
appropriate treatment at this time?

a. Rotator cuff repair and biceps tenodesis


b. Rotator cuff repair and loose body removal
c. Latissimus dorsi transfer
d. Bankart repair

Preffered Answer : 1
The MR image shows medial subluxation of the biceps tendon, which can be confused with an
articular loose body. In the clinical scenario of biceps instability/subluxation, the rationale
regarding tenodesis is to address the painful dislocation and subluxation of the biceps tendon
from the bicipital groove. Biceps tendon subluxation is most frequently associated with
subscapularis tendon pathology, which is indicated by the MRI and by a positive lift-off test.

The MR image does not show a loose body or Bankart lesion. Patients with irreparable rotator
cuff tears with a severe external rotation deficit and a deficient teres minor may experience a
better functional result with latissimus dorsi transfer.

78. What factor highly correlates with poor outcomes after surgery for femoroacetabular
impingement?

a. Age <20
b. Tonnis grade 2
c. Prominence of the femoral head in cam impingement
d. The patient is a professional athlete

Preffered Answer : 2
A systematic review of case studies looking at the results of surgical treatment for
femoroacetabular impingement shows good results for most patients, with the exception of those

UI // UNAIR // UNPAD // UNHAS // UNS // UGM // UB // UNUD // USU


Sports Medicine Scored and Recorded Self-Assessment Examination 2019

with preoperative radiographs showing osteoarthritis or Outerbridge grade III or grade IV


cartilage damage noted intraoperatively. Both Byrd and Jones and Philippon and associates
have shown good surgical results for this condition among professional athletes. Likewise,
Fabricant and associates demonstrated good surgical results among adolescent patients with an
average age of 17.6 years.

79. Figures 1 and 2 are the T2-weighted MR images of a 54-year-old woman with medial knee pain
and catching of 6 months’ duration. Which treatment option is most likely to be associated with a
favorable outcome?

a. Physical therapy
b. Meniscal repair
c. Menisectomy
d. Reconstruction

Preffered Answer : 2
MR images reveal a posterior horn root tear of the medial meniscus. LaPrade and associates
found that outcomes after posterior meniscal root repair significantly improved postoperatively
and patient satisfaction was high, regardless of age or meniscal laterality. Patients aged <50
years had outcomes similar to those of patients ≥50 years, as did patients who underwent medial
versus lateral root repair. In patients undergoing pullout fixation for posterior medial meniscus
root tear, Chung and associates (in ―Pullout Fixation of Posterior Medial Meniscus Root Tears‖)
found that patients with decreased meniscus extrusion at postoperative 1 year have more
favorable clinical scores and radiographic findings at midterm follow-up than those with increased
extrusion at 1 year. Krych and associates found that nonoperative treatment of medial meniscus
posterior horn root tears is associated with poor clinical outcome, worsening arthritis, and a
relatively high rate of arthroplasty at 5-year follow-up. Reconstruction would have no role in the
setting of a reparable meniscal root tear.

80. Figures 1 through 4 are selected sagittal MR images of an otherwise healthy 20-year-old
collegiate football running back who was tackled during a game and has immediate onset of right
knee pain. Video analysis of the injury shows that his flexed knee impacted the field. He is not
able to return to play. On examination in the training room the following morning, he has a
moderate effusion, no patellar instability, minimal joint line tenderness, and is stable to varus and
valgus stress at 30° of knee flexion. A dial test is also negative. He has increased laxity in the
anterior to posterior direction. What is the most appropriate next step in treatment?

UI // UNAIR // UNPAD // UNHAS // UNS // UGM // UB // UNUD // USU


Sports Medicine Scored and Recorded Self-Assessment Examination 2019

a. Rehabilitation initially focused on closed chain quadriceps strengthening


b. Rehabilitation initially focused on hamstring strengthening
c. Anterior cruciate ligament (ACL) reconstruction using autograft tissue
d. Posterior cruciate ligament (PCL) reconstruction using autograft tissue

Preffered Answer : 1
This athlete sustained an isolated PCL injury. The mechanism of injury is typical for a PCL injury.
When a PCL injury is identified, one must rule out other ligamentous injuries to the knee. The
patient has a stable examination to varus and valgus and a negative dial test, so the lateral
collateral, medial collateral, and posterolateral corner (respectively) are intact. It is common to
have increased anterior to posterior translation in isolated PCL injuries, even with an intact ACL,
as the tibia will rest posterior to the medial femoral condyle.

Treatment of isolated PCL injuries is typically nonoperative, with an initial focus on quadriceps
strengthening. Hamstring strengthening and rehabilitation is added at a later time, as this places
increased stress on the healing PCL. The images reveal an isolated PCL injury with intact
menisci and ACL, ruling out ACL reconstruction using autograft tissue and PCL reconstruction
using autograft tissue.

81. Figure 1 is the radiograph of a 21-year-old college lacrosse player who has a 2-year history of
progressive left groin pain that is exacerbated by activity. Pain is preventing him from
participating with his team. Examination reveals a fit man without tenderness to palpation around
the hip. No clicking or popping occurs with hip range of motion. Strength of all muscles about the
hip is normal, but there is some mild pain with resisted hip flexion and hip adduction. While lying
supine, progressive hip flexion with internal rotation and adduction reproduces his groin pain.
What is the most likely cause of this patient's pain?

UI // UNAIR // UNPAD // UNHAS // UNS // UGM // UB // UNUD // USU


Sports Medicine Scored and Recorded Self-Assessment Examination 2019

Figure 1

a. Femoroacetabular impingement (FAI)


b. Osteoarthritis of the sacroiliac joint
c. Intra-articular loose body
d. Trochanteric bursitis

Preffered Answer : 1
This clinical scenario describes a patient with FAI attributable to pincer (acetabular) deformity.
This form of FAI, which involves prominence of the anterosuperior acetabular lip, may be more
common among women. Decreased range of motion and pain occur secondary to the abutment
of the femoral head against the acetabular labrum and rim. Hip flexion, combined with adduction
and internal rotation, recreates this contact and causes pain, but CAM or pincer etiology remains
unknown.

The differential diagnosis of hip pain in a young athlete includes femoral neck stress
reaction/fracture, sacroiliac arthritis, intra-articular loose body, trochanteric bursitis, osteitis pubis,
and hernia. No information presented in this scenario suggests any of these causes. Diagnosis of
FAI is best performed via MR imaging, with an arthrogram increasing the sensitivity and
specificity for labral pathology. Ultrasonography may be useful in the diagnosis of dysplasia or for
dynamic assessment of a snapping hip, but ultrasonography is not commonly used to diagnose
labral pathology.

Although concomitant chondral lesions of the femoral head are uncommon, the forced leverage
of the anterosuperior femoral neck upon the anterior acetabulum may result in a ―contra-coup‖
chondral injury on the posteroinferior acetabulum. This is the most common location of chondral
lesions in this scenario. Without bony resection to prevent further impingement, this patient will
continue to experience symptoms. Because there is no evidence of femoral neck prominence
(CAM lesion), there is no indication for osteoplasty of the femoral neck; resection of the pincer
lesion is necessary. This will often require take-down of the labrum in this location. If possible,
iatrogenic or traumatic labral tears should subsequently be repaired after pincer debridement
because the labrum has important functions for hip stability and maintenance of the suction seal
of the joint.

82. Figure 1 is the radiograph of a 21-year-old college lacrosse player who has a 2-year history of
progressive left groin pain that is exacerbated by activity. Pain is preventing him from
participating with his team. Examination reveals a fit man without tenderness to palpation around
the hip. No clicking or popping occurs with hip range of motion. Strength of all muscles about the

UI // UNAIR // UNPAD // UNHAS // UNS // UGM // UB // UNUD // USU


Sports Medicine Scored and Recorded Self-Assessment Examination 2019

hip is normal, but there is some mild pain with resisted hip flexion and hip adduction. While lying
supine, progressive hip flexion with internal rotation and adduction reproduces his groin pain.

The patient participates in physical therapy for 8 weeks with his team's trainer but notes little
improvement. What is the most appropriate next diagnostic step to determine the cause of his
pain?

Figure 1
a. Diagnostic arthroscopy of the hip
b. Hip bone scan
c. Hip MRI arthrogram
d. Hip ultrasonography

Preffered Answer : 3
This clinical scenario describes a patient with FAI attributable to pincer (acetabular) deformity.
This form of FAI, which involves prominence of the anterosuperior acetabular lip, may be more
common among women. Decreased range of motion and pain occur secondary to the abutment
of the femoral head against the acetabular labrum and rim. Hip flexion, combined with adduction
and internal rotation, recreates this contact and causes pain, but CAM or pincer etiology remains
unknown.

The differential diagnosis of hip pain in a young athlete includes femoral neck stress
reaction/fracture, sacroiliac arthritis, intra-articular loose body, trochanteric bursitis, osteitis pubis,
and hernia. No information presented in this scenario suggests any of these causes. Diagnosis of
FAI is best performed via MR imaging, with an arthrogram increasing the sensitivity and
specificity for labral pathology. Ultrasonography may be useful in the diagnosis of dysplasia or for
dynamic assessment of a snapping hip, but ultrasonography is not commonly used to diagnose
labral pathology.

Although concomitant chondral lesions of the femoral head are uncommon, the forced leverage
of the anterosuperior femoral neck upon the anterior acetabulum may result in a ―contra-coup‖
chondral injury on the posteroinferior acetabulum. This is the most common location of chondral
lesions in this scenario. Without bony resection to prevent further impingement, this patient will
continue to experience symptoms. Because there is no evidence of femoral neck prominence
(CAM lesion), there is no indication for osteoplasty of the femoral neck; resection of the pincer
lesion is necessary. This will often require take-down of the labrum in this location. If possible,
iatrogenic or traumatic labral tears should subsequently be repaired after pincer debridement
because the labrum has important functions for hip stability and maintenance of the suction seal
of the joint.

UI // UNAIR // UNPAD // UNHAS // UNS // UGM // UB // UNUD // USU


Sports Medicine Scored and Recorded Self-Assessment Examination 2019

83. Figure 1 is the radiograph of a 21-year-old college lacrosse player who has a 2-year history of
progressive left groin pain that is exacerbated by activity. Pain is preventing him from
participating with his team. Examination reveals a fit man without tenderness to palpation around
the hip. No clicking or popping occurs with hip range of motion. Strength of all muscles about the
hip is normal, but there is some mild pain with resisted hip flexion and hip adduction. While lying
supine, progressive hip flexion with internal rotation and adduction reproduces his groin pain.

Further workup confirms an anterosuperior tear of the acetabular labrum and prominence of the
acetabulum. What is the most likely location of a chondral injury associated with these findings?

a. Posterosuperior acetabulum
b. Posteroinferior acetabulum
c. Femoral head above the fovea
d. Femoral head below the fovea

Preffered Answer : 2
This clinical scenario describes a patient with FAI attributable to pincer (acetabular) deformity.
This form of FAI, which involves prominence of the anterosuperior acetabular lip, may be more
common among women. Decreased range of motion and pain occur secondary to the abutment
of the femoral head against the acetabular labrum and rim. Hip flexion, combined with adduction
and internal rotation, recreates this contact and causes pain, but CAM or pincer etiology remains
unknown.

The differential diagnosis of hip pain in a young athlete includes femoral neck stress
reaction/fracture, sacroiliac arthritis, intra-articular loose body, trochanteric bursitis, osteitis pubis,
and hernia. No information presented in this scenario suggests any of these causes. Diagnosis of
FAI is best performed via MR imaging, with an arthrogram increasing the sensitivity and
specificity for labral pathology. Ultrasonography may be useful in the diagnosis of dysplasia or for
dynamic assessment of a snapping hip, but ultrasonography is not commonly used to diagnose
labral pathology.

Although concomitant chondral lesions of the femoral head are uncommon, the forced leverage
of the anterosuperior femoral neck upon the anterior acetabulum may result in a ―contra-coup‖
chondral injury on the posteroinferior acetabulum. This is the most common location of chondral
lesions in this scenario. Without bony resection to prevent further impingement, this patient will
continue to experience symptoms. Because there is no evidence of femoral neck prominence

UI // UNAIR // UNPAD // UNHAS // UNS // UGM // UB // UNUD // USU


Sports Medicine Scored and Recorded Self-Assessment Examination 2019

(CAM lesion), there is no indication for osteoplasty of the femoral neck; resection of the pincer
lesion is necessary. This will often require take-down of the labrum in this location. If possible,
iatrogenic or traumatic labral tears should subsequently be repaired after pincer debridement
because the labrum has important functions for hip stability and maintenance of the suction seal
of the joint.

84. Figure 1 is the radiograph of a 21-year-old college lacrosse player who has a 2-year history of
progressive left groin pain that is exacerbated by activity. Pain is preventing him from
participating with his team. Examination reveals a fit man without tenderness to palpation around
the hip. No clicking or popping occurs with hip range of motion. Strength of all muscles about the
hip is normal, but there is some mild pain with resisted hip flexion and hip adduction. While lying
supine, progressive hip flexion with internal rotation and adduction reproduces his groin pain.

The patient experiences little improvement with activity modification and more physical therapy.
An intra-articular corticosteroid injection provides excellent relief, but relief only lasts for 1 month.
The player requests further treatment for his hip and is counseled regarding surgical intervention.
Hip arthroscopy is performed. Intraoperatively, a capsulolabral separation is observed with an
underlying pincer lesion. No articular cartilage injury is seen. Which treatment is most appropriate
considering these findings?

Figure 1

a. Debridement of the labral tear plus bony resection of the pincer lesion
b. Debridement of the labral tear and no bony resection of the pincer lesion
c. Femoral neck osteoplasty plus labral repair using suture anchor
d. Resection of the bony pincer lesion plus labral repair using suture anchor

Preffered Answer: 4
This clinical scenario describes a patient with FAI attributable to pincer (acetabular) deformity.
This form of FAI, which involves prominence of the anterosuperior acetabular lip, may be more
common among women. Decreased range of motion and pain occur secondary to the abutment
of the femoral head against the acetabular labrum and rim. Hip flexion, combined with adduction
and internal rotation, recreates this contact and causes pain, but CAM or pincer etiology remains
unknown.

The differential diagnosis of hip pain in a young athlete includes femoral neck stress
reaction/fracture, sacroiliac arthritis, intra-articular loose body, trochanteric bursitis, osteitis pubis,
and hernia. No information presented in this scenario suggests any of these causes. Diagnosis of
FAI is best performed via MR imaging, with an arthrogram increasing the sensitivity and

UI // UNAIR // UNPAD // UNHAS // UNS // UGM // UB // UNUD // USU


Sports Medicine Scored and Recorded Self-Assessment Examination 2019

specificity for labral pathology. Ultrasonography may be useful in the diagnosis of dysplasia or for
dynamic assessment of a snapping hip, but ultrasonography is not commonly used to diagnose
labral pathology.

Although concomitant chondral lesions of the femoral head are uncommon, the forced leverage
of the anterosuperior femoral neck upon the anterior acetabulum may result in a ―contra-coup‖
chondral injury on the posteroinferior acetabulum. This is the most common location of chondral
lesions in this scenario. Without bony resection to prevent further impingement, this patient will
continue to experience symptoms. Because there is no evidence of femoral neck prominence
(CAM lesion), there is no indication for osteoplasty of the femoral neck; resection of the pincer
lesion is necessary. This will often require take-down of the labrum in this location. If possible,
iatrogenic or traumatic labral tears should subsequently be repaired after pincer debridement
because the labrum has important functions for hip stability and maintenance of the suction seal
of the joint.

85. Figure 1 is the MRI scan of a 35-year-old female soccer player who injured her knee during a
game. Given the findings of the scan, physical examination is most likely to reveal

a. grade 2 pivot shift.


b. positive Thessaly test.
c. positive quadriceps active test.
d. positive dial test at 30°.

Preffered Answer : 1
The MRI scan clearly reveals bone bruises in the mid lateral femoral condyle and posterior tibial
plateau. These MRI findings are commonly associated with acute anterior cruciate ligament
injuries. Therefore, the preferred answer would be a positive pivot shift examination. A positive
posterior drawer and positive quad active test are associated with posterior cruciate ligament
injuries. A positive dial test would be suggestive of a posterolateral instability of the knee.

86. A 28-year-old woman undergoes a closing-wedge high tibial osteotomy (HTO) for medial
compartment overload after medial meniscectomy. Postsurgically, she reports improvement in
her medial pain and resumes normal activities. About 9 months after her surgery, however, she
reports burning pain in the front of her knee with running. Her examination reveals no joint line
tenderness, mild pain with patellar compression, and limited patellar glides. What is the most
likely cause of her symptoms?
a. Patella infera (baja)

UI // UNAIR // UNPAD // UNHAS // UNS // UGM // UB // UNUD // USU


Sports Medicine Scored and Recorded Self-Assessment Examination 2019

b. Patella alta
c. Recurrence of medial joint overload
d. Nonunion of the osteotomy

Preffered Answer : 1
After HTO, particularly in patients who have been immobilized after a closing-wedge osteotomy,
patella baja is a common finding. This can precipitate anterior knee pain or patellofemoral pain
syndrome. Recurrence of medial joint overload is incorrect because the patient has no medial
joint complaints. Nonunion is less likely with a closing-wedge osteotomy and likely will not result
in anterior knee pain.

87. Figures 1 and 2 are the MR arthrogram images of a 20-year-old right-hand dominant collegiate
basketball player who sustained an initial shoulder dislocation 1 year ago. In the month prior to
presentation, he dislocated his shoulder two more times. Each time it occurred when going up for
a rebound and an opponent grabbed the ball from behind him, hyperextending his shoulder.
Physical examination demonstrates full range of motion, absence of atrophy, a positive
apprehension sign and relocation test, and a positive Kim test. What is the best next step?

Figure 1 Figure 2

a. Anterior labral repair


b. Anterior labral repair and remplissage
c. Posterior labral repair and rotator interval closure
d. Anterior and posterior labral repair

Preffered Answer : D
The mechanism of injury/dislocation is most consistent with anterior glenohumeral joint instability.
The axial cuts of the MR arthrogram reveals an anteroinferior labral tear, as well as a posterior
labral tear. A Hill-Sachs lesion is also consistent with anterior glenohumeral joint instability. At the
time of examination under anesthesia, this patient exhibited 2+ anterior and 2+ posterior
glenohumeral joint instability. Patients with pan-labral tears and 270° tears can be challenging to
diagnose, because patients can report anterior or posterior shoulder instability alone. The
physical examination and advanced imaging in these patients are crucial in directing appropriate
treatment.

UI // UNAIR // UNPAD // UNHAS // UNS // UGM // UB // UNUD // USU


Sports Medicine Scored and Recorded Self-Assessment Examination 2019

88. Figure 1 is the MRI scan of a 15-year-old boy who has had knee pain with running for 5 months.
Radiographs show an osteochondritis dissecans (OCD) lesion of the medial femoral condyle.
What is the most appropriate treatment?

a. Arthroscopic or open reduction and internal fixation


b. Arthroscopic loose body removal
c. Activity restriction for up to 9 months
d. Subchondral drilling

Preffered Answer : 1
OCD is an acquired lesion of the subchondral bone. Patients with OCD initially report nonspecific
pain and variable amounts of swelling. Initial radiographs help identify the lesion and establish
the status of the physes. An MRI scan is useful for assessing the potential for the lesion to heal
with nonsurgical treatment. Nonsurgical treatment is appropriate for small, stable lesions in
patients with open physes and focuses on activity restriction for 3 to 9 months. Surgical treatment
is necessary for unstable or detached lesions. Stable lesions with intact articular cartilage can be
treated with subchondral drilling to stimulate vascular ingrowth, with radiographic healing at an
average of 4.4 months. Fixation is indicated for unstable or hinged lesions, and stabilization of
the fragment can be achieved using a variety of implants through an arthroscopic or open
approach. The fragment should be salvaged and the normal articular surface restored whenever
possible.

89. A 17-year-old high school football linebacker sustains an injury while making a tackle. His initial
symptoms are right shoulder pain, bilateral biceps weakness, and right arm numbness. The
symptoms only last a few minutes, and he continues to play in the game. He tells his parents
after the game, and they bring him to your office for evaluation the next day. He no longer has

UI // UNAIR // UNPAD // UNHAS // UNS // UGM // UB // UNUD // USU


Sports Medicine Scored and Recorded Self-Assessment Examination 2019

any symptoms, and his examination findings and cervical spine radiographs are normal. What is
the best next step?

a. Allow him to continue playing football


b. Order an EMG
c. Observe and if symptoms are negative for one week, then a return to football
d. Order a cervical MRI scan

Preffered Answer : 4
The football player had bilateral weakness indicating that the injury was more significant than a
stinger. Stingers present with unilateral symptoms, and if they resolve, an athlete can return to
sports. However, bilateral symptoms indicate cervical spine pathology, such as spinal stenosis,
and warrant an MRI scan of the cervical spine. Despite the patient being able to continue playing
in the game and having symptom resolution, a cervical MRI scan should be performed prior to
return to sports.

90. Figures 1 and 2 are the AP and lateral radiographs of a 32-year-old man 10 years after anterior
cruciate ligament (ACL) reconstruction. The patient now has worsening medial knee pain and a
failed ACL with instability. What is the best surgical option?

Figure 1 Figure 2

a. Revision ACL with bone-patellar tendon-bone (BTB) allograft and meniscal transplant
b. Distal femoral osteotomy
c. Pure sagittal osteotomy
d. Closing wedge and slope neutralizing high-tibial osteotomy

Preffered Answer : 4
Lateral closing wedge (LCW) and medial opening wedge (MOW) high-tibial osteotomies (HTOs)
can both correct varus knee alignment and stabilize the ACL–deficient knee. Increasingly, HTO is
being proposed as a singular or concomitant procedure with ACL reconstruction for restoring
knee stability and as a more reliable slope correction. LCW HTO demonstrates more reliable
slope correction than does MOW HTO. Revision ACL with BTB allograft and meniscal transplant
will not help early arthritis in varus alignment. Distal femoral osteotomy is usually considered for
valgus knee. Pure sagittal osteotomy would not help the patient’s medial knee pain, arthritis, and
malalignment.

UI // UNAIR // UNPAD // UNHAS // UNS // UGM // UB // UNUD // USU


Sports Medicine Scored and Recorded Self-Assessment Examination 2019

91. Figure 1 is the T2 coronal MRI scan of a 52-year-old woman with a 6-month history of shoulder
pain. She does not recall a history of trauma. Physical therapy is recommended. What is the
most significant predictor of failure of nonoperative treatment?

Figure 1

a. Tear size
b. Pain scale score
c. Strength deficit
d. Patient expectations

Preffered Answer : 4
The MRI reveals a large full thickness supraspinatus tear. A large, prospective study showed that
physical therapy can be effective in the treatment of atraumatic full-thickness rotator cuff tears.
Patient expectations regarding the role of rehabilitation were the strongest predictor of surgery.
Other factors associated with surgery were higher activity level and not smoking. Anatomic
features of the rotator cuff tear and the severity of patient’s reported pain did not predict failure of
nonoperative treatment. Patients who have low expectations regarding the effectiveness of
physical therapy are more likely to fail nonoperative treatment.

92. Figures 1 through 5 are the MR images of a 12-year-old boy with left lateral-sided knee pain
following a football injury. He has a several-year history of recurrent knee pain that improves with
rest. An examination reveals a moderate effusion. Range of motion is 0° to 90° and is limited by
pain in deep flexion. He has tenderness to palpation along the lateral joint line, and no instability
is noted.

What is the underlying cause of the pathology noted in the figures?

UI // UNAIR // UNPAD // UNHAS // UNS // UGM // UB // UNUD // USU


Sports Medicine Scored and Recorded Self-Assessment Examination 2019

a. Genetic mutation
b. Recurrent trauma
c. Shallow intercondylar notch
d. Congenital abnormality

Preffered Answer : 4
The MR images show a tear through a discoid lateral meniscus. A discoid meniscus is caused by
a failure of apoptosis during development in utero and is considered a congenital abnormality.

UI // UNAIR // UNPAD // UNHAS // UNS // UGM // UB // UNUD // USU


Sports Medicine Scored and Recorded Self-Assessment Examination 2019

Discoid menisci are prone to tearing and can be caused by minimal trauma. There is no known
genetic cause for this condition. Radiographic findings that may be present in the setting of a
discoid meniscus include lateral joint space widening, squaring of the lateral femoral condyle,
and cupping of the lateral tibial plateau. Contralateral discoid menisci are noted in 20% of
patients. There are no other known associated conditions. Treatment of a symptomatic discoid
meniscus should include partial meniscectomy and saucerization or repair.

93. Figures 1 through 5 are the MR images of a 12-year-old boy with left lateral-sided knee pain
following a football injury. He has a several-year history of recurrent knee pain that improves with
rest. An examination reveals a moderate effusion. Range of motion is 0° to 90° and is limited by
pain in deep flexion. He has tenderness to palpation along the lateral joint line, and no instability
is noted.

Based on the pathology noted, which finding may be found on plain knee radiographs?

UI // UNAIR // UNPAD // UNHAS // UNS // UGM // UB // UNUD // USU


Sports Medicine Scored and Recorded Self-Assessment Examination 2019

a. Shallow trochlear groove


b. Squaring of the lateral femoral condyle
c. Deepening of the sulcus terminalis
d. Medial joint space narrowing

Preffered Answer : 2
The MR images show a tear through a discoid lateral meniscus. A discoid meniscus is caused by a
failure of apoptosis during development in utero and is considered a congenital abnormality.
Discoid menisci are prone to tearing and can be caused by minimal trauma. There is no known
genetic cause for this condition. Radiographic findings that may be present in the setting of a
discoid meniscus include lateral joint space widening, squaring of the lateral femoral condyle, and
cupping of the lateral tibial plateau. Contralateral discoid menisci are noted in 20% of patients.
There are no other known associated conditions. Treatment of a symptomatic discoid meniscus
should include partial meniscectomy and saucerization or repair.

94. Figures 1 through 5 are the MR images of a 12-year-old boy with left lateral-sided knee pain
following a football injury. He has a several-year history of recurrent knee pain that improves with
rest. An examination reveals a moderate effusion. Range of motion is 0° to 90° and is limited by
pain in deep flexion. He has tenderness to palpation along the lateral joint line, and no instability is
noted.

What other finding may be noted in patients with this diagnosis?

UI // UNAIR // UNPAD // UNHAS // UNS // UGM // UB // UNUD // USU


Sports Medicine Scored and Recorded Self-Assessment Examination 2019

a. Symmetric knee pathology


b. Excessive joint laxity
c. Recurrent patella instability
d. Extra-articular manifestations

Preffered Answer : 1
The MR images show a tear through a discoid lateral meniscus. A discoid meniscus is caused by
a failure of apoptosis during development in utero and is considered a congenital abnormality.

UI // UNAIR // UNPAD // UNHAS // UNS // UGM // UB // UNUD // USU


Sports Medicine Scored and Recorded Self-Assessment Examination 2019

Discoid menisci are prone to tearing and can be caused by minimal trauma. There is no known
genetic cause for this condition. Radiographic findings that may be present in the setting of a
discoid meniscus include lateral joint space widening, squaring of the lateral femoral condyle,
and cupping of the lateral tibial plateau. Contralateral discoid menisci are noted in 20% of
patients. There are no other known associated conditions. Treatment of a symptomatic discoid
meniscus should include partial meniscectomy and saucerization or repair.

95. Figures 1 through 5 are the MR images of a 12-year-old boy with left lateral-sided knee pain
following a football injury. He has a several-year history of recurrent knee pain that improves with
rest. An examination reveals a moderate effusion. Range of motion is 0° to 90° and is limited by
pain in deep flexion. He has tenderness to palpation along the lateral joint line, and no instability
is noted.

Treatment should include

UI // UNAIR // UNPAD // UNHAS // UNS // UGM // UB // UNUD // USU


Sports Medicine Scored and Recorded Self-Assessment Examination 2019

a. anterior cruciate ligament reconstruction with lateral meniscus repair.


b. partial lateral meniscectomy with saucerization.
c. lateral meniscus transplant.
d. protected weight bearing with referral for genetic testing.

Preffered Answer : 2
The MR images show a tear through a discoid lateral meniscus. A discoid meniscus is caused by
a failure of apoptosis during development in utero and is considered a congenital abnormality.
Discoid menisci are prone to tearing and can be caused by minimal trauma. There is no known
genetic cause for this condition. Radiographic findings that may be present in the setting of a
discoid meniscus include lateral joint space widening, squaring of the lateral femoral condyle,
and cupping of the lateral tibial plateau. Contralateral discoid menisci are noted in 20% of
patients. There are no other known associated conditions. Treatment of a symptomatic discoid
meniscus should include partial meniscectomy and saucerization or repair.

96. Figures 1 and 2 are the right shoulder MRI scans of a 22-year-old right-handed professional male
volleyball player with 4 months of right shoulder pain. The pain began insidiously and is
exacerbated by overhead activities and hitting during games. He has maintained a daily program
of shoulder stretching and strengthening exercises but has experienced a steady decline in
function to the point of not being able to participate in volleyball. Examination reveals some mild
atrophy at the posterior shoulder, full forward elevation, mild weakness of external rotation on the
right shoulder, negative empty-can testing, positive O’Brien’s and negative apprehension.
Surgical intervention would aim to resolve pathology related to which nerve?

UI // UNAIR // UNPAD // UNHAS // UNS // UGM // UB // UNUD // USU


Sports Medicine Scored and Recorded Self-Assessment Examination 2019

a. Lower subscapular nerve


b. Suprascapular nerve at the spinoglenoid notch
c. Suprascapular nerve at the suprascapular notch
d. Axillary nerve

Preffered Answer : 2
This athlete has a symptomatic posterior-superior labral tear, spinoglenoid notch cysts, and
subsequent suprascapular nerve compression, as evidenced by the atrophy of the infraspinatus
muscle on sagittal T1 MRI. The cyst is located at the spinoglenoid notch and is compressing the
suprascapular nerve after it has innervated the supraspinatus but before innervation of the
infraspinatus; hence, the atrophy of infraspinatus on examination and imaging. Compression of
the suprascapular nerve at the suprascapular notch would lead to weakness and atrophy of both
the supraspinatus and infraspinatus. The lower subscapular nerve innervates the teres major, as
well as, with the upper subscapular nerve, the subscapularis. The teres minor is innervated by
the axillary nerve.

97. During anatomic medial patellofemoral ligament (MPFL) reconstruction, the surgeon notes that
the graft is becoming too tight with greater knee flexion. What is the most likely cause?

a. Femoral attachment placed too distal


b. Femoral attachment placed too proximal
c. Patellar attachment placed too distal
d. Patellar attachment placed too proximal

Preffered Answer :B
If the graft becomes tighter with knee flexion, the femoral attachment is too proximal. This error is
referred to as ―high and tight,‖ meaning that a high or proximal femoral attachment produces a
graft that is too tight with knee flexion. If graft tension increases with increasing knee flexion, the
result is loss of knee flexion or graft failure, increased contact forces resulting in patella femoral
chondrosis, and possibly medial subluxation.

UI // UNAIR // UNPAD // UNHAS // UNS // UGM // UB // UNUD // USU


Sports Medicine Scored and Recorded Self-Assessment Examination 2019

98. Figure 1 is the radiograph of a 50-year old woman with lateral-sided left knee pain. She noticed
the pain over the last few months and has had no new injury. She had a microfracture performed
of her lateral femoral condyle 5 years ago. What is the likely cause of the finding noted on her
radiograph?

a. Uncontained cartilage lesion


b. Removal of the subchondral plate
c. Removal of the calcified cartilage layer
d. Failure to remove the calcified cartilage layer

Preffered Answer : 2
The radiograph reveals bony overgrowth of the microfracture site on the lateral femoral condyle.
This occurs from violation of the subchondral plate during aggressive removal of the calcified
cartilage layer during the microfracture. It is important during a microfracture to attempt to have a
contained lesion and remove the calcified cartilage layer down to the subchondral plate, but
avoid aggressively penetrating the plate.

99. A 29-year-old recreational basketball player has developed pain to the distal aspect of her patella
that occurs during warm-ups and returns toward the end of the game. She reports no history of
trauma, effusions, instability, and no mechanical symptoms. On examination, she is point tender
at the inferior pole of the patella, lacks patella apprehension, and has a Q-angle of 15°. She has
no ligamentous laxity. Radiographs are unremarkable. What is the best next step?

a. Therapy with an emphasis on eccentric exercises


b. Steroid injection
c. Platelet-rich plasma
d. Extracorporeal shock therapy
Preffered Answer : 1
Patellar tendinopathy is a relatively common condition in athletes for which repetitive jumping is
the norm, especially volleyball and basketball athletes. The prevalence has been reported to be
up to 32% in professional basketball players. Initial management is nonoperative in nature with
eccentric exercises providing the most reliable clinical results. The other selections have not
demonstrated consistent long-term results.

UI // UNAIR // UNPAD // UNHAS // UNS // UGM // UB // UNUD // USU


Sports Medicine Scored and Recorded Self-Assessment Examination 2019

100. Figure 1 is the radiograph of an 11-year-old baseball pitcher who has had right shoulder pain for
the past 3 months. He has full range of motion and normal strength in both external rotation and
abduction, although all tests cause him discomfort over the lateral and anterior shoulder. What is
the most likely basis for his injury?

a. Increased external rotation with an associated decrease in internal rotation


b. Excessive pitch counts
c. Use of breaking pitches such as sliders and curve balls
d. Congenital humeral cyst

Preffered Answer : 2
The radiograph reveals a widened lateral physis at the proximal humerus, consistent with a
physeal stress fracture or ―Little Leaguer’s shoulder.‖ Numerous studies have established that
children and adolescents are particularly prone to such overuse injuries. With regard to baseball
participation, a major contributor is over-pitching, i.e., excessive numbers of pitches, excessive
innings pitched, and insufficient rest days. Altered range of rotational motion, a gradual
adaptation to the increased stresses of throwing, can predispose to long-term injury, especially
internal impingement and labral pathology. A unicameral or aneurysmal bone cyst can often
occur in the proximal humerus, but has a distinct radiographic appearance and would predispose
to fracture. There is evidence that breaking pitches place increased stresses on the elbow and
shoulder, but it remains controversial whether such throws should be avoided at certain ages.

UI // UNAIR // UNPAD // UNHAS // UNS // UGM // UB // UNUD // USU


Adult Reconstructive Surgery of the Hip & Knee Self-Assessment Examination 2019

2019
SELF
ASSESSMENT
EXAMINATION
ADULT RECONSTRUCTION SURGERY OF THE

HIP AND KNEE


CONTRIBUTED BY:
CANDIDATES OF
INDONESIAN ORTHOPAEDIC & COMPILED BY
TRAUMATOLOGY SURGEONS
PERIOD OF JUNE 2019

UI // UNAIR // UNPAD // UNHAS // UNS // UGM // UB // UNUD // USU


Adult Reconstructive Surgery of the Hip & Knee Self-Assessment Examination 2019
1. A 58-year-old woman underwent a left total knee arthroplasty 6 years ago. She initially did well after surgery
but sustained a fall 2 months ago while at work. She now describes left knee pain and instability and an
inability to straighten her knee since the fall. She has been using a hinged knee brace, which provides
partial support. On examination, she has passive range of motion of 0° to 115° and active range of motion
of 80° to -115°. Her radiographs are shown in Figures 1 through 3. What is the best option for the
restoration of her function?

Figure 1 Figure 2 Figure 3

1. Revision total knee arthroplasty with placement of a hinge constrained device

2. Patellar tendon repair with nonabsorbable suture and patellar resurfacing

3. Hinged knee brace with drop lock design to restore stability during ambulation

4. Extensor mechanism reconstruction using synthetic mesh or allograft


Preffered Answer: 4

The patient has an extensor mechanism disruption with patellar tendon rupture. This injury is treated with
extensor mechanism reconstruction in the setting of previous total knee arthroplasty. There is a reported
high failure rate with attempted repair. Revision to hinge knee arthroplasty would provide implant stability but
would not restore the extensor mechanism. The patient is relatively young and is working, so reconstruction
would offer better long-term function than a drop lock brace, which can be better used in low-functioning
patients with this type of injury. Extensor mechanism reconstruction historically has been accomplished with
allograft material, but a novel technique using synthetic mesh also has proved successful in treating this
difficult problem.

2. In patients with Crowe types III and IV developmental dysplasia of the hip with high hip centers, acetabular
reconstruction often requires lowering the acetabular component into the native acetabulum. In doing so,
considerable risk for limb lengthening beyond 4 cm exists, making the hip difficult to reduce and raising the
risk for nerve injury. Which technique is used to overcome this problem?

1. Subtrochanteric osteotomy with femoral shortening

2. An offset femoral component

3. A lateralized liner

UI // UNAIR // UNPAD // UNHAS // UNS // UGM // UB // UNUD // USU


Adult Reconstructive Surgery of the Hip & Knee Self-Assessment Examination 2019
4. Extended trochanteric osteotomy
Preffered Answer: 1

When substantial lengthening of a dysplastic hip will occur because a high dislocation is relocated into a
considerably lower acetabulum, a femoral shortening may be necessary to reduce the hip and avoid a
stretch injury to the sciatic nerve. No other choice specifically addresses the need for femoral shortening,
and high offset stems and lateralized liners may exacerbate the problem if used alone and without femoral
shortening.

3. A 58-year-old man with insulin-dependent diabetes mellitus underwent primary total knee arthroplasty (TKA).
A full-thickness skin slough measuring 3 cm by 4 cm developed, with postsurgical exposure of the patellar
tendon. No change is observed in the appearance of the wound after 2 weeks of wet-to-dry dressing
changes. What is the best next treatment step for the soft-tissue defect?

1. Continued dressing changes

2. Split-thickness skin graft

3. Full-thickness skin graft

4. Local rotational
flap Preffered Answer:
4

If wound healing does not occur and deep soft tissues such as the patellar tendon are exposed following
TKA, local rotational flap is the procedure of choice. The procedure should be performed relatively early after
the recognition of a soft-tissue wound-healing problem. In the setting of TKA, the gastrocnemius muscle is an
excellent source of flaps for wound coverage of the proximal tibia.

4. Figure 1 shows the radiograph obtained from a 76-year-old woman who has sharp pain in her groin, thigh,
and buttocks that worsens with activity. She has been dealing with this pain for more than a year but is
otherwise healthy. Recently, she has begun to notice night pain. The pain no longer responds to NSAIDs.
She would like to be able to dance at her daughter's wedding in 4 months and wonders how best to proceed.
What is the best next step?

Figure 1

1. Radiograph-guided steroid injection followed by total hip arthroplasty 6 weeks later

UI // UNAIR // UNPAD // UNHAS // UNS // UGM // UB // UNUD // USU


Adult Reconstructive Surgery of the Hip & Knee Self-Assessment Examination 2019
2. Total hip arthroplasty
3. Physical therapy
4. Referral back to her spine surgeon
Preffered Answer: 2

The next best course of action is total hip arthroplasty. The patient is an otherwise healthy woman requesting
pain relief and expresses a desire to be dancing in 4 months. She has had more than 6 months of symptoms
that are classic hip osteoarthritis symptoms, with pain in the groin and thigh. Severe osteoarthritis is seen in the
radiograph as well. NSAIDs are no longer working. Given the objective findings, the subjective reports, and the
duration of symptoms, this patient merits surgery. Consideration for steroid injection is reasonable, but given
her desire to be dancing in 4 months, an injection would increase her risk of infection if total hip arthroplasty
were to be performed within 3 months of the injection.

5. A 68-year-old woman undergoes an uncomplicated total knee replacement through a midline incision that is
extended distally to join a previous incision from a high-tibial osteotomy done 12 years previously. Despite
relief of pain and appropriate knee mobility at 2 weeks, drainage continues from the distal part of the wound.
What are the most appropriate next step(s) in treatment?

1. Oral cephalexin while the wound heals

2. Vacuum suction drain applied over the draining part

3. Intravenous antibiotics and reassess the knee in 24 hours

4. Urgent open debridement of the knee, cultures, and evaluation of inflammatory laboratory data
Preffered Answer: 4

Drainage from a knee after arthroplasty can be ominous; here, a previous incision was made for an
osteotomy, possibly compromising wound healing. At 2 weeks, persistent wound drainage is not expected
and warrants urgent attention with surgical debridement, cultures, and a baseline check of inflammatory
laboratory data. Intravenous antibiotics can be started while awaiting cultures, but oral or intravenous
antibiotics alone are not sufficient and a vacuum drain is not indicated in this situation.

6. Figure 1 shows the radiograph and Figure 2 shows the MRI scan obtained from a 37-year-old woman with a
2-month history of left hip pain.Which combination of a single symptom and examination finding is most
likely in this scenario?

UI // UNAIR // UNPAD // UNHAS // UNS // UGM // UB // UNUD // USU


Adult Reconstructive Surgery of the Hip & Knee Self-Assessment Examination 2019
7.

Figure 1 Figure 2

1. Pain during sitting; flexion abduction and external rotation of the hip

2. Groin pain; pain with internal rotation and adduction while supine with the hip and knee flexed 90°

3. Clicking; abductor lurch

4. Buttock pain; pain with hip extension, adduction, and external rotation while prone
Preffered Answer: 2

MRI reveals an anterior labral tear, and the radiograph shows minimal arthritis with possible dysplasia. The
most common location of pain in patients with a labral tear is the groin, and the most common physical
finding is a positive impingement test result. Pain during sitting, clicking, and buttock pain are frequently
described by patients with a labral tear, but these symptoms are less common than groin pain. A positive
posterior impingement test finding is more common in patients with a posterior labral tear.

Although age over 40 years and a body mass index higher than 30 can adversely affect clinical outcomes
after joint preservation procedures such as PAO, hip arthroscopy, and femoral acetabular impingement
surgery, the presence of hip arthritis on presurgical radiographs is the most commonly mentioned cause of
failed hip joint preservation surgery. Tönnis grade is a radiographic measure of hip arthritis. A higher
Outerbridge score is associated with more frequent poor outcomes after hip arthroscopy; however, the
Outerbridge cartilage score is determined by direct visualization at the time of surgery. The Outerbridge
score cannot be determined presurgically.

8. Figure 1 shows the radiograph and Figure 2 shows the MRI scan obtained from a 37-year-old woman with a
2-month history of left hip pain.Which presurgical factor is most commonly associated with a poor outcome
after a hip joint salvage procedure?

UI // UNAIR // UNPAD // UNHAS // UNS // UGM // UB // UNUD // USU


Adult Reconstructive Surgery of the Hip & Knee Self-Assessment Examination 2019
9.

Figure 1 Figure 2

1. Age older than 40 years

2. Body mass index higher than 30

3. Tönnis grade of 2 or higher

4. Outer bridge grade of III or IV


Preffered Answer: 3

MRI reveals an anterior labral tear, and the radiograph shows minimal arthritis with possible dysplasia. The
most common location of pain in patients with a labral tear is the groin, and the most common physical
finding is a positive impingement test result. Pain during sitting, clicking, and buttock pain are frequently
described by patients with a labral tear, but these symptoms are less common than groin pain. A positive
posterior impingement test finding is more common in patients with a posterior labral tear.

Although age over 40 years and a body mass index higher than 30 can adversely affect clinical outcomes
after joint preservation procedures such as PAO, hip arthroscopy, and femoral acetabular impingement
surgery, the presence of hip arthritis on presurgical radiographs is the most commonly mentioned cause of
failed hip joint preservation surgery. Tönnis grade is a radiographic measure of hip arthritis. A higher
Outerbridge score is associated with more frequent poor outcomes after hip arthroscopy; however, the
Outerbridge cartilage score is determined by direct visualization at the time of surgery. The Outerbridge
score cannot be determined presurgically.

10. During a posterior cruciate ligament (PCL)-retaining total knee arthroplasty, a critical principle to remember
is to

1. accurately tension the PCL.


2. use bony resection to adjust the joint line.
3. maintain a small amount of residual deformity.
4. use intraoperative fluoroscopy to ensure femoral roll back.
Preffered Answer: 1

UI // UNAIR // UNPAD // UNHAS // UNS // UGM // UB // UNUD // USU


Adult Reconstructive Surgery of the Hip & Knee Self-Assessment Examination 2019
Maintenance of the joint line and accurately tensioning the PCL are critical in the proper execution of a PCL-
retaining total knee arthroplasty. Appropriate tension helps ensure femoral rollback and avoid stiffness or
instability. Raising the joint line to help ensure full extension should be avoided in cruciate-retaining knees,
because doing so creates an unfavorable kinematic environment. The three important principles of surgical
technique needed to maintain appropriate tensioning of the PCL include 1) choosing the proper femur size to
reproduce the native femoral anterior/posterior dimension, 2) reproducing the joint line by resecting as much
tibia from the healthy side as will be replaced by the smallest thickness of the tibial component and, 3)
ensuring that full extension is achieved by soft-tissue releases and not by taking additional distal femur, as
may be done in a posterior stabilized approach. Another important principle is to re-create the natural degree
of the patient’s posterior tibial slope to avoid tightness in flexion.

11. When comparing arthroscopic lavage and knee debridement with placebo in patients with chronic
symptomatic osteoarthritis, what outcome has been demonstrated?

1. Reliable and durable pain relief

2. No significant benefit for chronic osteoarthritis

3. Up to 75% pain relief for 2 months, then variable response

4. Three-month measurable pain relief, followed by recurrence

Preffered Answer: 2

Excluding a diagnosis of meniscal tear, loose body, or mechanical derangement, treating knee osteoarthritis
of indeterminate cause with arthroscopic lavage and debridement has been found to provide no discernable
benefit to offset the risk of surgery. The effects of arthroscopy have not been clinically significant in the vast
majority of patient-oriented outcomes measures for pain and function at multiple times between 1 week and
2 years after surgery.

12. Figures 1 through 3 show the radiographs obtained from a 75-year-old woman who underwent right total hip
arthroplasty in 2009. She did well until last month, when a right posterior hip dislocation occurred after she
fell from her bed to the floor. Successful closed reduction was performed. She sustained two more posterior
dislocations requiring closed reduction under anesthesia. The surgical report from the index arthroplasty
notes a 54-mm monoblock acetabular component with a 28-mm inner diameter compression molded
polyethylene and a high offset neck cementless stem with a +8-mm length, 28-mm head. What is the best
next step?

UI // UNAIR // UNPAD // UNHAS // UNS // UGM // UB // UNUD // USU


Adult Reconstructive Surgery of the Hip & Knee Self-Assessment Examination 2019
Figure 1 Figure 2 Figure 3

1. Hip spica cast placement


2. Acetabular revision arthroplasty
3. Resection arthroplasty
4. Femoral head revision to a 28-mm diameter, +10-mm length head

Preffered Answer: 2
This patient has demonstrated recurrent instability, and her current implants lack the modularity to upsize
and improve the head-neck ratio and range to impingement. Given the monoblock acetabular component
and a +7-mm neck length, the best option is revision to a large-diameter femoral head or dual-mobility
component. Placement of a hip spica cast and resection arthroplasty are unreasonable. Revision to a
longer ball length likely would not solve this recurrent instability pattern.

13. Figure 1 shows the radiograph obtained from a 73-year-old woman who returns status post total hip
arthroplasty 14 years earlier. She denies pain and has no discomfort on examination. She then undergoes
revision total hip arthroplasty with head and liner exchange and bone grafting. After a physical therapy
session two days after surgical intervention, she develops inability to dorsiflex the foot while she is sitting in
a chair. The initial treatment should consist of:

Figure 1

1. lying completely supine in bed.

2. remaining seated and placing the postsurgical leg on a stool.

3. transferring back to bed with the head of the bed no lower than 60°.

4. transferring back to bed with the head of the bed level and the surgical knee flexed.
Preffered Answer: 4

Gamma irradiation produces free radicals. Although these free radicals can form cross-links with other
polyethylene chains, the free radicals can also form a bond with oxygen, resulting in early oxidation.
Gamma irradiation in air produces the highest risk for oxidized polyethylene, resulting in the highest risk for
wear, delamination, and subsequent osteolysis. This patient demonstrates severe periarticular osteolysis.
When she is asymptomatic, this suggests the acetabular and femoral components remain well fixed to the
bone. Consequently, she can be treated by removing the wear generator (polyethylene exchange), along
with bone grafting of the osteolytic defect. Considering the extensive amount of osteolysis indefinite
observation would not be appropriate. A foot drop develops 2 days after surgery. As a result, it can be
assumed that the nerve was not injured directly during the surgical procedure. Although MRI or CT may be

UI // UNAIR // UNPAD // UNHAS // UNS // UGM // UB // UNUD // USU


Adult Reconstructive Surgery of the Hip & Knee Self-Assessment Examination 2019
indicated to identify an evolving hematoma, the immediate concern is to minimize pressure on the sciatic
nerve. Tension on the nerve can be reduced by flexing the surgical knee and positioning the bed flat.

14. A 45-year-old woman has severe anterior knee pain. Her radiographs indicate end-stage patellofemoral
compartment osteoarthritis. The tibiofemoral compartments are preserved. Extensive nonsurgical treatment
has failed to provide relief, and she is offered patellofemoral arthroplasty (PFA). What is the most common
long-term mode of failure for PFA using an implant with an onlay prosthesis design?

1. Infection

2. Patellar instability

3. Aseptic loosening

4. Progression of tibiofemoral

arthritis Preffered Answer: 4

Contemporary onlay-design trochlear prostheses in PFA replace the entire anterior trochlear surface.
Previous inlay designs were inset within the native trochlea and carried a higher risk of catching and patellar
instability, particularly in patients with trochlear dysplasia; they also generally have higher failure rates. The
current most common mode of failure is progression of arthritis throughout the knee, in some series as high
as 25% at 15 years. Aseptic loosening, particularly of cemented implants, is less common. Infection is an
uncommon long-term complication. Patients considering PFA should be advised of the risk of arthritis
progression. Many authors routinely obtain a preoperative MRI to assess the status of the tibiofemoral
compartments.

15. Figures 1 and 2 show the radiographs, and Figure 3 shows the CT obtained from a 58-year-old woman who
underwent cementless left total hip arthroplasty. Nine months after surgery, she continued to have groin pain
when actively flexing her hip. She has trouble walking up stairs and getting out of her car.

Figure 1 Figure 2 Figure 3

1. Trochanteric bursitis

UI // UNAIR // UNPAD // UNHAS // UNS // UGM // UB // UNUD // USU


Adult Reconstructive Surgery of the Hip & Knee Self-Assessment Examination 2019
2. Femoral component loosening

3. Iliopsoas tendonitis

4. Acetabular component loosening

Preffered Answer: 3

Groin pain after total hip replacement has a number of possible causes, and an exact diagnosis may remain
elusive in some patients. Infection should be ruled out with laboratory studies and, if indicated, diagnostic
aspiration of the hip joint. Implant loosening should be evaluated by plain radiograph and bone scan, if
indicated. Synovitis resulting from wear debris should be considered in patients with polyethylene liners who
experience late-onset symptoms or in any patient with a metal-on-metal bearing. This patient's symptoms
are classic for iliopsoas tendonitis. Physical examination usually reveals pain and weakness with resisted
hip flexion. A cross-table lateral radiograph and CT show that the anterior edge of the acetabulum protrudes
beyond the anterior wall, thereby acting as a source of iliopsoas tendon irritation. In such cases, acetabular
component revision and repositioning are indicated. Fluoroscope-guided iliopsoas cortisone injection can
help to establish the diagnosis and relieve groin pain. If the acetabular component is well positioned, then
iliopsoas tenotomy should be considered.

16. Figures 1 and 2 show the radiographs, and Figure 3 shows the CT obtained from a 58-year-old woman who
underwent cementless left total hip arthroplasty. Nine months after surgery, she continued to have groin pain
when actively flexing her hip. She has trouble walking up stairs and getting out of her car.A course of
appropriate nonsurgical treatment failed. What is the next step in definitive treatment?

1. Acetabular component revision


2. Femoral component revision
3. Acetabular liner exchange
4. Trochanteric bursectomy
Preffered Answer: 1

Groin pain after total hip replacement has a number of possible causes, and an exact diagnosis may
remain elusive in some patients. Infection should be ruled out with laboratory studies and, if indicated,
diagnostic aspiration of the hip joint. Implant loosening should be evaluated by plain radiograph and bone
scan, if indicated. Synovitis resulting from wear debris should be considered in patients with polyethylene
liners who experience late-onset symptoms or in any patient with a metal-on-metal bearing. This patient's
symptoms are classic for iliopsoas tendonitis. Physical examination usually reveals pain and weakness
with resisted hip flexion. A cross-table lateral radiograph and CT show that the anterior edge of the

UI // UNAIR // UNPAD // UNHAS // UNS // UGM // UB // UNUD // USU


Adult Reconstructive Surgery of the Hip & Knee Self-Assessment Examination 2019
acetabulum protrudes beyond the anterior wall, thereby acting as a source of iliopsoas tendon irritation. In
such cases, acetabular component revision and repositioning are indicated. Fluoroscope-guided iliopsoas
cortisone injection can help to establish the diagnosis and relieve groin pain. If the acetabular component
is well positioned, then iliopsoas tenotomy should be considered.

17. Figures 1 through 3 show the radiograph and MR arthrograms obtained from a 25-year-old woman who has
had right groin pain since joining the military 4 years ago. She has undergone treatment with NSAIDs,
physical therapy, and activity modification. Examination reveals positive flexion abduction and external
rotation, a positive external log roll, and increased range of motion. What is the most appropriate treatment?

Figure 1 Figure 2 Figure 3

1. Viscosupplementation of the right hip

2. Hip arthroscopy with labral repair

3. Periacetabular osteotomy

4. Total hip arthroplasty


Preffered Answer: 3

This patient has symptomatic hip dysplasia that has been recalcitrant to nonsurgical management.
Radiographs reveal an upsloping sourcil (acetabular index of 18) and a lateral center edge angle of 14, with
posterior uncovering. The MR arthrogram shows no definitive evidence of a labral tear. Appropriate surgical
management would include periacetabular osteotomy.

Viscosupplementation in the hip is controversial in the treatment of osteoarthritis and plays no role in the
treatment of dysplasia. Hip arthroscopy with labral repair is controversial in mild hip dysplasia, with studies
demonstrating between 60% and 77% good and excellent results, inferior to the results for hip arthroscopy in
a femoroacetabular impingement cohort. In moderate to severe dysplasia, hip arthroscopy is not
recommended. Because the acetabular cartilage is well maintained, total hip arthroplasty would not be
recommended in this young and active patient.

18. A patient has pain 2 years after undergoing a metal-on-metal (MOM) left total hip arthroplasty (THA). Which
test(s) best correlate with a prognosis if this patient is having a reaction to metal debris?

1. Erythrocyte sedimentation rate, C-reactive protein, and white blood cell count

UI // UNAIR // UNPAD // UNHAS // UNS // UGM // UB // UNUD // USU


Adult Reconstructive Surgery of the Hip & Knee Self-Assessment Examination 2019
2. Serum cobalt and chromium ion levels

3. MRI with metal artifact reduction sequence (MARS)

4. CT of pelvis

Preffered Answer: 3

Painful MOM THA and taper corrosion can cause substantial damage to a patient's hip if left untreated. In this
case, the workup for a painful MOM THA starts the same as a workup for a painful metal-on-polyethylene
bearing couple. Infection must be ruled out in every case with a set of inflammatory markers. If these markers
are remotely elevated, this is an indication for joint aspiration. In patients with metal debris, the pathology
report often indicates too many cells to count or cellular debris. Metal ion levels do not seem to correlate with
prognosis. There are well-functioning patients with high ion levels and poor-functioning patients with low ion
levels. Advanced imaging with MARS MRI to evaluate for peritrochanteric fluid collection, a soft-tissue mass,
or synovial/capsular hypertrophy will reveal signs of a metal reaction that indicate the need for a revision
discussion. A CT scan can show more advanced bony destruction as an indicator of poor prognosis. These
films can be used to determine the need for a structural graft or augments for reconstruction of bone loss
attributable to metal debris.

19. Figures 1 and 2 demonstrate the radiographs obtained from a 56-year-old man with a 3-year history of right
groin pain. A comprehensive nonsurgical program has failed, and the patient would like to proceed with total
hip arthroplasty. He is seen by a pain management specialist and is currently taking 40 mg of sustained-
release morphine twice daily with oxycodone 10 mg 2 to 3 times a day for severe pain. What is the
recommended course of action regarding his chronic narcotic use?

1. Increase his current opioid medication regimen prior to and after surgery as needed to control his pain.

2. Decrease his preoperative opioid use, and work with his pain management physician to decrease his
postoperative opioid requirement.

3. Avoid using narcotics in the perioperative period to prevent overdose, and use acetaminophen only for
pain control.

4. Stop all his opioids 5 days before surgery, and place the patient on a morphine pain control pump
postoperatively with a basal rate.

Preffered Answer: 2

UI // UNAIR // UNPAD // UNHAS // UNS // UGM // UB // UNUD // USU


Adult Reconstructive Surgery of the Hip & Knee Self-Assessment Examination 2019
Chronic opioid consumption prior to total joint arthroplasty has been associated with increased pain after
surgery, increased opioid requirements, a slower recovery and longer hospital stay, and higher 90-day
postoperative complications compared with patients not on chronic opioids preoperatively.

Based on this information, Nguyen and associates performed a study in three patient groups that included
1) chronic opioid users who underwent no preoperative intervention, 2) chronic opioid users who were
weaned down to 50% of their prior opioid regimen, and 3) patients who were not chronic opioid users. The
authors found that the reduction of preoperative opioid use improved postoperative function, pain, and
recovery and that the weaned group performed more like the opioid naive group than the chronic opioid
user group.

Increasing opioid use prior to surgery in this patient would make it more difficult to control pain after surgery.
Stopping all of his opioids just prior to surgery would place the patient at substantial risk for opioid
withdrawal and is not recommended. Avoiding the use of all narcotics and using only acetaminophen
postoperatively is very unlikely to provide appropriate pain relief in a chronic opioid user. The
recommendation based on the provided literature is to decrease the patient's narcotic use prior to surgery.

20. When do most symptomatic thromboembolic events occur after total joint arthroplasty?

1. On the day of surgery

2. Within the first week after surgery

3. Between 1 week and 6 weeks after surgery

4. More than 3 months after surgery

Preffered Answer: 3

Most clinical venous thromboembolism events occur between the second and sixth weeks after surgery. It
is estimated that 10% of patients are readmitted to the hospital within the first 3 months after total hip or
knee arthroplasties. Most pulmonary events on the day of surgery are related to fat embolism or cardiac
events.

21. Figures 1 through 3 represent the radiographs obtained from a 37-year-old man with severe right knee pain.
He has a history of prior tibial osteotomy for adolescent tibia vara but notes residual bowing of his legs. On
examination, he is 5'8" tall and weighs 322 pounds. He has a waddling gait with a bilateral varus thrust and
20° varus deformity of both legs. His right knee range of motion is 0° to 120° with a fixed varus deformity.
What is the best next step?

UI // UNAIR // UNPAD // UNHAS // UNS // UGM // UB // UNUD // USU


Adult Reconstructive Surgery of the Hip & Knee Self-Assessment Examination 2019

1. Total knee arthroplasty with standard components

2. Correction of tibial deformity with osteotomy and nonsurgical management of the osteoarthritis

3. Arthrodesis with a long antegrade nail

4. Total knee arthroplasty with a constrained device


Preffered Answer: 4

This patient has severe, uncorrectable varus deformity and pain from end-stage osteoarthritis secondary to
prior adolescent tibia vara. Although he is young to consider arthroplasty, this option is likely to give him the
most functional limb, compared with arthrodesis with a long antegrade nail. During arthroplasty surgery, his
knee will likely require extensive medial release to achieve anatomic limb alignment. Standard components
in total knee arthroplasty likely would result in lateral instability, so this option is not the best answer. The
best choice is total knee arthroplasty with a constrained device, which adds constraint to the knee to provide
balance.

22.A A 70-year-old woman has a 3-year history of gradually increasing diffuse and global
right knee pain. Her main issues are difficulty with stairs, stiffness with prolonged sitting,
and swelling. She has taken NSAIDs and has received intra-articular steroid injections, all
with decreasing efficacy. Her right knee examination reveals a range of motion of 15° to 80°
with a fixed deformity to varus and valgus stress. Her symptoms are no longer
manageable nonsurgically. Radiographs reveal a 30-degree mechanical axis
deformity.

The deformity shown in Figure 1 is predominantly associated with

1. a hypoplastic lateral femoral condyle.

2. a contracted medial collateral ligament.

3. an excessive proximal tibial slope.

4. trochlear dysplasia.

Preffered Answer: 1

In the setting of valgus deformities, TKA poses different challenges than those encountered when varus
deformities are present. Most valgus alignment is attributable to a deformity of the distal femur rather than
of the proximal tibia, as seen in varus knees. One of the major anatomical differences is a hypoplastic
lateral femoral condyle which, when not recognized and used as a rotational reference point, can lead to
internal rotation of the femoral component. This malrotation in turn leads to patellofemoral maltracking or
instability, which is a common complication associated with primary TKA.

UI // UNAIR // UNPAD // UNHAS // UNS // UGM // UB // UNUD // USU


Adult Reconstructive Surgery of the Hip & Knee Self-Assessment Examination 2019
23. A 70-year-old woman has a 3-year history of gradually increasing diffuse and global right knee pain. Her
main issues are difficulty with stairs, stiffness with prolonged sitting, and swelling. She has taken NSAIDs
and has received intra-articular steroid injections, all with decreasing efficacy. Her right knee examination
reveals a range of motion of 15° to 80° with a fixed deformity to varus and valgus stress. Her symptoms are
no longer manageable nonsurgically. Radiographs reveal a 30-degree mechanical axis deformity.When
using the measured resection technique during total knee arthroplasty (TKA), the best way to avoid femoral
malrotation is to reference the:

1. anteroposterior axis.

2. tibial intramedullary axis.

3. posterior condylar axis.

4. femoral intramedullary

axis. Preffered answer: 1

In the setting of valgus deformities, TKA poses different challenges than those encountered when varus
deformities are present. Most valgus alignment is attributable to a deformity of the distal femur rather than
of the proximal tibia, as seen in varus knees. One of the major anatomical differences is a hypoplastic
lateral femoral condyle which, when not recognized and used as a rotational reference point, can lead to
internal rotation of the femoral component. This malrotation in turn leads to patellofemoral maltracking or
instability, which is a common complication associated with primary TKA.

24. When balancing gaps in the coronal plane, what structure preferentially impacts the flexion space more than
the extension space?
1. Iliotibial band

2. Popliteus tendon

3. Lateral collateral ligament

4. Lateral head of the gastrocnemius

Preffered Answer: 2

In the setting of valgus deformities, TKA poses different challenges than those encountered when varus
deformities are present. Most valgus alignment is attributable to a deformity of the distal femur rather than
of the proximal tibia, as seen in varus knees. One of the major anatomical differences is a hypoplastic
lateral femoral condyle which, when not recognized and used as a rotational reference point, can lead to
internal rotation of the femoral component. This malrotation in turn leads to patellofemoral maltracking or
instability, which is a common complication associated with primary TKA.

25. A 70-year-old woman with a body mass index (BMI) of 34 and a history of hypercholesterolemia has elected
to undergo total hip arthroplasty. Her son recently learned he has factor V Leiden following an episode of
pulmonary embolism. What are this patient's risk factors for thromboembolic disease?

1. Type of surgery, age, and BMI

2. Type of surgery, hypercholesterolemia, and age

UI // UNAIR // UNPAD // UNHAS // UNS // UGM // UB // UNUD // USU


Adult Reconstructive Surgery of the Hip & Knee Self-Assessment Examination 2019
3. Age, BMI, and hypercholesterolemia

4. BMI, type of surgery, and hypercholesterolemia

Preffered Answer: 1

Risk stratification is one of the most critical clinical evaluations to undertake before performing total joint
arthroplasty. Many factors have been identified that increase the risk for venous thromboembolism (VTE).
The major factors include previous VTE, obesity, type of surgery (such as total joint arthroplasty),
hypercoagulable states, myocardial infarction, congestive heart failure, family history of VTE, and
hormone replacement therapy. Hypercholesterolemia is not a risk factor for thromboembolic disease.

26. An otherwise healthy 76-year-old woman has pain 2 years after total hip arthroplasty. The clinical
photograph in Figure 1 demonstrates her skin envelope, and Figure 2 shows her associated radiograph. Her
C-reactive protein level is normal, and her erythrocyte sedimentation rate is mildly elevated. The white blood
cell count is normal. Hip aspiration attempted under fluoroscopy generates no fluid. What is the best
definitive treatment?

Figure 1 Figure 2

1. Repeat left hip aspiration

2. Initiation of a wound care consult and oral antibiotics

3. Irrigation and debridement with closure of the dehisced wound, performance of a liner exchange, and
administration of intravenous antibiotics

4. Debridement of the wound, explant of the total hip, placement of a spacer, and administration of
intravenous antibiotics

Preffered Answer: 4

This patient clearly has a chronically infected total hip arthroplasty, indicated by the open, draining sinus, as
well as trochanteric bone resorption on radiographs, and two years of pain. Recently, specific guidelines
have been published to better help the clinician define infection. Repeating the hip aspiration is
unnecessary, because infection is already evident. Initiating a wound care consult would not address the
underlying infection. The determination whether to retain the components or perform a two-stage exchange
is based more on the acuity of infection. In this particular case, the patient is chronically infected. Irrigation
and debridement with a liner exchange and retention of the components are reserved for the acute setting.

UI // UNAIR // UNPAD // UNHAS // UNS // UGM // UB // UNUD // USU


Adult Reconstructive Surgery of the Hip & Knee Self-Assessment Examination 2019
27. Figures 1 and 2 show the radiograph and CT obtained from a 78-year-old woman who underwent right total
hip replacement in 1995. She initially did well with no pain. She was last seen 7 years ago and was having
mild hip pain at that time. She was found to have a supra-acetabular cyst on radiographs. She has had
severe right hip pain for the past 9 months while using a walker for ambulation. The initial blood work reveals
an estimated erythrocyte sedimentation rate of 32 mm/hr, a C-reactive protein level of 0.5 mg/L, a serum
cobalt level of 0.4 µg/L, and a serum chromium level of 0.6 µg/L. Right hip aspiration is performed, revealing
a white blood cell count of 139, 52% neutrophils, and a negative leukocyte esterase test. What is the best
next step?

Figure 1 Figure 2

1. Acetabular revision, with placement of a custom triflange acetabular component and femoral head
exchange

2. Removal of the femoral and acetabular components and placement of an antibiotic spacer, with 6
weeks of intravenous antibiotics

3. Head and liner exchange and retention of the femoral and acetabular implants with acetabular bone
grafting

4. Nonsurgical management with the initiation of bisphosphonates and referral to pain management
Preffered Answer: 1

The hip replacement was performed in 1995, during the period when the previous generation of
polyethylene was utilized. This polyethylene was subjected to irradiation in air, with subsequent oxidation
and consequent osteolysis after implantation. The mechanism of osteolysis begins with the uptake of
polyethylene particles by macrophages, which then initiate an inflammatory cascade and the release of
osteolytic factors. This cycle continues, with eventual implant loosening and failure. The imaging shows
significant osteolysis and raises concern for pelvic discontinuity and acetabular implant failure. The
surgical treatment consists of acetabular reconstruction. In this patient, concern exists for discontinuity
based on the substantial amount of bone loss and nonsupportive anterior and posterior columns. This
scenario requires complex acetabular revision using a custom triflange device, distraction with a jumbo
acetabular component, or placement of a porous metal cup/cage construct with augmentation. The
laboratory values are not consistent with infection or failure due to metal debris.

28. Which modality has the broadest application for the reduction of postsurgical transfusion?
1. Regional anesthesia

UI // UNAIR // UNPAD // UNHAS // UNS // UGM // UB // UNUD // USU


Adult Reconstructive Surgery of the Hip & Knee Self-Assessment Examination 2019
2. Tranexamic acid (TXA) administration

3. Reduced transfusion trigger

4. Hypotensive

anesthesia Preffered

Answer: 2

TXA is easy to administer, inexpensive, and safe for virtually all patients. Multiple studies have
demonstrated transfusion rates lower than 3% for total knee arthroplasty and lower than 10% for total hip
arthroplasty. Regional and hypotensive anesthesia effectively reduce transfusion; however, they cannot be
used in as wide a range of patients as can TXA. A reduced transfusion trigger must be considered along
with patient symptoms when determining the need for transfusion.

29.A A 45-year-old man has a draining sinus and recurrent infection of his right total knee arthroplasty. He
has had two prior revision surgeries after the primary procedure and three other surgeries before his
initial replacement, including a proximal tibial osteotomy and subsequent hardware removal. On
clinical examination, he has a draining sinus in the mid portion of his surgical scar and a range of motion
of 5° to 85°. AP and lateral radiographs of the right knee are shown in Figures 1 and 2. During surgery, the
femoral component is found to be grossly loose, but the tibial component is well fixed. What is the most
appropriate extensile approach that would provide adequate exposure and aid in tibial component
extraction?

1. Extended medial parapatellar approach

2. Quadriceps snip

3. Extended tibial tubercle osteotomy

4. Medial epicondyle osteotomy


Preffered Answer: 3

Extended tibial tubercle osteotomy is an extensile approach to revision total knee arthroplasty that affords
excellent exposure and can facilitate removal of tibial sleeves and cones. This patient has had multiple
surgeries, including a proximal tibial osteotomy, as well as poor range of motion, patella baja, and a well-
fixed metaphyseal sleeve component. Classically, an extended tibial tubercle osteotomy provides
outstanding exposure for component removal in the setting of prior high tibial osteotomy and patella baja.
For this patient, it is important to recognize the patella baja on the radiographs, as well as the tibial sleeve. In
many of these cases the osteotomy provides access to the sleeve to help with extraction, because the stem

UI // UNAIR // UNPAD // UNHAS // UNS // UGM // UB // UNUD // USU


Adult Reconstructive Surgery of the Hip & Knee Self-Assessment Examination 2019
will not pull through the sleeve or detach from the tray to allow visualization of the sleeve. The extended

UI // UNAIR // UNPAD // UNHAS // UNS // UGM // UB // UNUD // USU


Adult Reconstructive Surgery of the Hip & Knee Self-Assessment Examination 2019
medial parapatellar approach is just a long medial approach that typically yields good exposure but would
not help with the patella baja or extraction of the tibial sleeve. The quadriceps snip would give good
exposure to the knee but would not aid in tibial component removal. Lastly, the medial epicondyle osteotomy
could help with exposure and tensioning of the medial complex of the knee but would not help with tibial
component extraction.

30. A 68-year-old woman underwent an uncemented medial/lateral tapered femoral placement during a total hip
arthroplasty. The orthopaedic surgeon noticed a nondisplaced vertical fracture in the calcar region of the
femoral neck during final implant insertion. What is the most appropriate treatment?

1. Removal of the press-fit implant and cementing of the same femoral stem

2. Removal of the uncemented femoral component and placement of a revision modular taper-fluted
femoral stem

3. Removal of the implant, placement of a cerclage wire around the femoral neck above the lesser
trochanter, and reinsertion of the implant

4. Final seating of the uncemented femoral component without additional measures

Preffered answer: 3

The recognized treatment for a proximal periprosthetic fracture is to first identify the extent and then
optimize the correction of the fracture. Several studies indicate that proximal cerclage wiring is adequate to
create &quot;barrel hoop&quot; stability of the proximal femur. Braided cables offer superior stability
compared with twisted wires or Luque wires. Finally, the appropriate postoperative treatment is protected
weight bearing for 6 weeks, with periodic radiographs taken at 2-week intervals. Other options such as
cementing the femoral stem and using a revision arthroplasty device are indicated for unstable fractures.

31. Figures 1 through 4 show the radiographs, and Figures 5 through 8 show the MRIs obtained from a 32-year-
old man with worsening left knee pain. A 3-foot hip-to-ankle radiograph shows a 13-degree varus knee
deformity. The patient sustained a major left knee injury 5 years ago and a confirmed complete anterior
cruciate ligament (ACL) tear. He managed this injury nonsurgically with a functional brace but experienced
worsening pain. He was seen by an orthopaedic surgeon 18 months ago, and a medial meniscus tear was
diagnosed; the tear was treated with an arthroscopic partial medial meniscectomy. Since then, his knee has
been giving way more often, and he no longer feels safe working on a pitched roof. The patient received 6
months of formal physical therapy and was fitted for a new functional ACL brace, but he still has pain and
instability. He believes he has exhausted his nonsurgical options and would like to undergo surgery. What is
the most appropriate treatment at this time?

UI // UNAIR // UNPAD // UNHAS // UNS // UGM // UB // UNUD // USU


Adult Reconstructive Surgery of the Hip & Knee Self-Assessment Examination 2019

1 2 3

5 6

7 8 9

1. ACL reconstruction and subsequent proximal tibial osteotomy


2. ACL reconstruction alone
3. Distal femoral osteotomy with simultaneous ACL reconstruction
4. Proximal tibial osteotomy with subsequent ACL reconstruction

Preffered Answer: 4
Proximal tibial osteotomy is the most appropriate intervention to correct varus malalignment and to reduce
stress on the ACL. In some cases, proximal tibial osteotomy alone may address both pain and instability, but
if instability persists, particularly in the setting in which instability can be dangerous, subsequent ACL
reconstruction can further stabilize the knee with less stress on the graft after the correction of malalignment.
Varus alignment places increased stress on the native or reconstructed ACL. ACL reconstruction should be
performed only at the same time as or following proximal tibial osteotomy to correct alignment in the setting
of varus malalignment. It is not appropriate to perform ACL reconstruction prior to proximal tibial osteotomy

UI // UNAIR // UNPAD // UNHAS // UNS // UGM // UB // UNUD // USU


Adult Reconstructive Surgery of the Hip & Knee Self-Assessment Examination 2019
in this setting. Distal femoral osteotomy is not indicated to correct varus malalignment. Varus alignment
places increased stress on the native or reconstructed ACL, and ACL reconstruction alone is not indicated
for this patient.

32. What is the most important preoperative factor predicting conversion to total hip arthroplasty after
arthroscopic surgery of the hip?
1. Age over 60 years

2. Morbid obesity

3. Diagnosis of osteoarthritis

4. Tobacco use

Preffered Answer: 2

The authors cited in the references examined large databases to determine the risk factors for conversion
to total hip arthroplasty after arthroscopic surgery of the hip. In the study by Kester and associates, obesity
had an odds ratio (OR) of 5.6 for conversion to hip arthroplasty, whereas age over 60 years had an OR of
3.4, osteoarthritis had an OR of 2.4, and tobacco use had an OR of 1.9.

33. Hip pain of 1-month duration has developed in a 72-year-old man with a previous total hip arthroplasty. He
underwent dental work 6 weeks ago. Aspiration shows a white blood cell count of more than 6,000 cells/μL
(reference range 4,500 to 11,000 cells/μL) and the presence of gram-positive cocci in clusters on Gram
stain. The orthopaedic surgeon recommends urgent debridement and irrigation. Fixation of the components
is judged to be stable, and the surgeon elects to retain the implants.What is this patient's prognosis for
infection resolution?

1. Good because it is a gram-positive organism

2. Good because it is an acute infection

3. Poor because it is a gram-positive organism

4. Poor because it is a late infection


Preffered Answer: 4

The patient has a late infection of at least 4 weeks symptomatic duration that most likely is hematogenous
in etiology. This infection is not an acute hematogenous infection that can successfully be treated with
irrigation and debridement. Retention of the implants with debridement and irrigation alone has been
associated with a poor prognosis. In a recent study, the success rate was only 44% in a series of 104
patients at a mean 5.7-year follow-up. In one study of 50 infections attributable to MRSA or methicillin-
resistant Staphylococcus epidermidis organisms treated with a two-stage protocol, the failure rate was 21%.
Patients who experienced successful infection treatment had lower functional outcome measures using the
Western Ontario and McMaster Universities Osteoarthritis Index, the University of California Los Angeles
Activity Score, and the 12-item Oxford Knee Score, however.

34. Hip pain of 1-month duration has developed in a 72-year-old man with a previous total hip arthroplasty. He
underwent dental work 6 weeks ago. Aspiration shows a white blood cell count of more than 6,000 cells/μL
(reference range 4,500 to 11,000 cells/μL) and the presence of gram-positive cocci in clusters on Gram
stain. The orthopaedic surgeon recommends urgent debridement and irrigation. Fixation of the components

UI // UNAIR // UNPAD // UNHAS // UNS // UGM // UB // UNUD // USU


Adult Reconstructive Surgery of the Hip & Knee Self-Assessment Examination 2019
is judged to be stable, and the surgeon elects to retain the implants.The patient has a final culture that
reveals methicillin-resistant Staphylococcus aureus (MRSA). If the attending physician recommends the two-
stage protocol, including the use of an antibiotic-cement spacer, what is the most likely prognosis for this
patient?

1. Better functional outcome than that associated with infections from sensitive organisms

2. Same functional outcome as that associated with infections from sensitive organisms

3. Same prognosis for eradication of infection as that associated with infections from sensitive organisms

4. Poorer prognosis for eradication of infection than that associated with infection from sensitive
organisms

Preffered Answer: 4

The patient has a late infection of at least 4 weeks symptomatic duration that most likely is hematogenous in
etiology. This infection is not an acute hematogenous infection that can successfully be treated with irrigation
and debridement. Retention of the implants with debridement and irrigation alone has been associated with
a poor prognosis. In a recent study, the success rate was only 44% in a series of 104 patients at a mean 5.7-
year follow-up. In one study of 50 infections attributable to MRSA or methicillin-resistant Staphylococcus
epidermidis organisms treated with a two-stage protocol, the failure rate was 21%. Patients who experienced
successful infection treatment had lower functional outcome measures using the Western Ontario and
McMaster Universities Osteoarthritis Index, the University of California Los Angeles Activity Score, and the
12-item Oxford Knee Score, however.

35. A 55-year-old woman is referred for evaluation of a painful knee replacement. She underwent total knee
arthroplasty (TKA) more than 1 year ago without perioperative complications but has had consistent pain
since the surgery. The patient’s preoperative radiographs are shown in Figures 1 and 2, and her and
postoperative radiographs are shown in Figures 3 and 4. Examination reveals medial laxity during valgus
stress testing and range of motion of 0° to 70°. Her erythrocyte sedimentation rate and C-reactive protein
level are normal. What is the best next step?

1 3

UI // UNAIR // UNPAD // UNHAS // UNS // UGM // UB // UNUD // USU


42 2
Adult Reconstructive Surgery of the Hip & Knee Self-Assessment Examination 2019

1. Unloader brace
2. Distal femoral osteotomy
3. Open arthrofibrosis debridement with lateral ligament balancing and polyethylene exchange
4. Revision TKA of both the femoral and tibial components

Preffered Answer: 4

The radiographs show substantial valgus malalignment of the femoral component, with lateral mechanical
axis deviation. Clinically, by examination she displays instability and stiffness as a result. Revision knee
replacement is appropriate and should consist of total revision to stemmed femoral and tibial components
with a varus-valgus constrained insert, given the likely attenuation of the medial collateral ligament. Open
debridement with ligament balancing and polyethylene exchange do not address the underlying cause and
are inappropriate. Distal femoral osteotomy is not useful in the setting of previous total knee replacement.
Nonsurgical treatment with an unloader brace would be ineffective in correcting the alignment.

36. What factor is considered one of the early changes in osteoarthritic cartilage?

1. Decreased water content

2. Increased proteoglycan content

3. Decreased loading of the solid matrix

4. Increased cartilage tissue permeability

Preffered Answer: 4

The normal regulation of a cartilage surface is a delicate balance of degradation and synthesis. When this
normal regulation of the cartilage is disturbed, a proinflammatory state tips the cellular pathway in the
direction of degradation. The proinflammatory state upregulates the production of cytokines and
proteolytic enzymes, specifically matrix metalloproteinases. These enzymes attack the proteoglycan
content of the cartilage, leading to an overall reduction in the proteoglycan content. This reduction in
content leads to increased permeability of the cartilage substrate. With increased permeability, water is
able to move into the cartilage itself, thereby increasing the overall water content within the cartilage in an
arthritic state. Finally, because of the increased permeability and increased water content, the overall load

UI // UNAIR // UNPAD // UNHAS // UNS // UGM // UB // UNUD // USU


Adult Reconstructive Surgery of the Hip & Knee Self-Assessment Examination 2019
or pressure placed on the underlying solid matrix is increased. Increased water content, decreased
proteoglycan content, and an increased load on the solid matrix are typical of an osteoarthritic process
within normal cartilage. Therefore, the only correct option is that the cartilage has an increased amount of
permeability in osteoarthritis.

37.A A 47-year-old obese man with a body mass index of 42 comes into the office with left knee pain 1 year
after undergoing an uncomplicated left medial unicompartmental knee arthroplasty (UKA). Radiographs
show a loose tibial component in varus. What is the most appropriate next step to treat this failed construct?

1. Aspiration of joint fluid to obtain a cell count

2. Revision of the UKA using primary total knee arthroplasty (TKA) components

3. Revision of the UKA using a revision TKA with augments

4. Procurement of the erythrocyte sedimentation rate (ESR) and C-reactive protein (CRP) level

Preffered Answer: 4

This patient likely is experiencing failure of the UKA secondary to poor patient selection. In this young,
heavy man, the component likely loosened due to the ongoing varus alignment of the knee and his
elevated weight. Despite this likely scenario, the next step is determining whether an infection is the cause
of his pain. Prior to obtaining an aspiration, the surgeon can order ESR and CRP studies to determine
whether aspiration is warranted. If the laboratory studies are unremarkable, the surgeon likely can forgo
the aspiration and proceed to a revision TKA with possible augments on standby.

38. A 70-year-old man undergoes removal of an infected total hip arthroplasty (THA) and insertion of an
articulating antibiotic-loaded spacer to treat a deep periprosthetic hip infection. While in a nursing home
receiving intravenous antibiotics 3 weeks after surgery, the patient trips and falls. Examination reveals
swelling in the mid and distal thigh, intact skin and neurovascular structures, and severe pain with knee or
hip movement. Radiographs of the femur are shown in Figures 1 through 4. What is the most appropriate
treatment for the fracture below the implant?

Figure 1 Figure 2 Figure 3 Figure 4

1. Balanced traction to address concern for persistent

infection with reoperation

UI // UNAIR // UNPAD // UNHAS // UNS // UGM // UB // UNUD // USU


Adult Reconstructive Surgery of the Hip & Knee Self-Assessment Examination 2019
2. Open reduction and internal fixation of the fracture with a lateral plate and screws

3. Removal of the articulating spacer and revision to a longer-stem antibiotic-loaded articulating spacer

4. Removal of the articulating spacer and reimplantation using a long-stem fluted uncemented hip
replacement

Preffered Asnwer: 2

This patient has a type C periprosthetic femoral fracture. The articulating spacer is not involved in the
fracture, which is well distal to the implant. The most appropriate treatment is open reduction and internal
fixation of the fracture. Traction is not appropriate for this fracture because the injury can be treated
surgically despite the history of previous hip infection. Traction would also be needed for at least 5 weeks
and would delay the surgical treatment of the periprosthetic fracture until the time of second-stage revision
THA. The fracture is fairly distal, and revision to a longer antibiotic-loaded implant or uncemented stem is not
suitable for this fracture pattern, because it extends well past the isthmus. A femoral stem in the distal
fragment would provide little stability for the fracture. Removal of the articulating spacer and reimplantation
using a long-stem fluted uncemented hip replacement is not appropriate, because it would be premature to
reimplant the man's hip while he is still receiving treatment for a deep hip infection.

39. Early postoperative infections following primary total hip arthroplasty are most likely caused by which
organism?

1. Staphylococcus epidermidis

2. Streptococcus viridans

3. Propionibacterium acnes

4. Staphylococcus aureus

Preffered Answer: 4

S aureus is the most common organism cultured in early (fewer than 4 weeks postoperative) periprosthetic
infections. Methicillin-resistant S aureus is becoming a more common pathogen in certain patient
populations. B hemolytic Streptococcus and some gram-negative infections can also be found in early
postoperative infections. S epidermidis, S viridans, and P acnes are more commonly found in late (more
than 4 weeks postoperative) infections.

40. Figures 1 through 5 show the radiographs, MRI, and MR arthrogram obtained from a 25-year-old collegiate
soccer player who has new-onset left groin pain. He played competitive soccer from a young age and has
competed or practiced 5 to 6 times per week since the age of 10. He denies any specific hip injury that
necessitated treatment, but his trainer contends that he had a groin pull. He reports groin pain with passive
flexion and internal rotation of the left hip, and his hip has less internal rotation than his asymptomatic right
hip. He is otherwise healthy.
Approximately what percentage of asymptomatic athletes have cam deformities of the hip?

UI // UNAIR // UNPAD // UNHAS // UNS // UGM // UB // UNUD // USU


Adult Reconstructive Surgery of the Hip & Knee Self-Assessment Examination 2019

Figure 1

Figure 2 Figure 3

Figure 4 Figure 5

1. 5%

2. 10%

3. 25%

4. At least 50%
Preffered Answer: 4

Multiple studies have confirmed that cam or pincer anatomy is commonly present in asymptomatic hips.
According to a large systematic review, cam deformities are present in approximately one-third of
asymptomatic hips in young adults, and the proportion is higher than 50% in the subgroup of athletes.

Ganz and associates proposed that femoral acetabular impingement is the root cause of osteoarthritis in
most nontraumatic, nondysplastic hips, and functional improvement with surgical correction of the deformity
has been demonstrated. Despite the link between cam deformity and hip osteoarthritis, a corresponding link
between the correction of the deformity and prevention of osteoarthritis has never been proven.

The results of cam deformity correction, typically including repair of the degenerative labral tear, are much
poorer when substantial joint space loss is present. A typical joint space cutoff of 2 mm or less is used to
recommend against hip preservation surgery.

UI // UNAIR // UNPAD // UNHAS // UNS // UGM // UB // UNUD // USU


Adult Reconstructive Surgery of the Hip & Knee Self-Assessment Examination 2019
41. Figures 1 through 5 show the radiographs, MRI, and MR arthrogram obtained from a 25-year-old collegiate
soccer player who has new-onset left groin pain. He played competitive soccer from a young age and has
competed or practiced 5 to 6 times per week since the age of 10. He denies any specific hip injury that
necessitated treatment, but his trainer contends that he had a groin pull. He reports groin pain with passive
flexion and internal rotation of the left hip, and his hip has less internal rotation than his asymptomatic right
hip. He is otherwise healthy.What is the primary cause of a cam deformity?

Figure 3

Figure 1 Figure 2 Figure 3

UI // UNAIR // UNPAD // UNHAS // UNS // UGM // UB // UNUD // USU


Adult Reconstructive Surgery of the Hip & Knee Self-Assessment Examination 2019

Figure 5

Figure 4

1. A genetic problem

2. Repetitive activities involving an open proximal femoral physis

3. Early closure of the proximal femoral physis

4. Hip dysplasia
Preffered Answer: 2

Multiple studies have confirmed that cam or pincer anatomy is commonly present in asymptomatic hips.
According to a large systematic review, cam deformities are present in approximately one-third of
asymptomatic hips in young adults, and the proportion is higher than 50% in the subgroup of athletes.

Ganz and associates proposed that femoral acetabular impingement is the root cause of osteoarthritis in
most nontraumatic, nondysplastic hips, and functional improvement with surgical correction of the deformity
has been demonstrated. Despite the link between cam deformity and hip osteoarthritis, a corresponding link
between the correction of the deformity and prevention of osteoarthritis has never been proven.

The results of cam deformity correction, typically including repair of the degenerative labral tear, are much
poorer when substantial joint space loss is present. A typical joint space cutoff of 2 mm or less is used to
recommend against hip preservation surgery.

42. Figures 1 through 5 show the radiographs, MRI, and MR arthrogram obtained from a 25-year-old collegiate
soccer player who has new-onset left groin pain. He played competitive soccer from a young age and has
competed or practiced 5 to 6 times per week since the age of 10. He denies any specific hip injury that
necessitated treatment, but his trainer contends that he had a groin pull. He reports groin pain with passive
flexion and internal rotation of the left hip, and his hip has less internal rotation than his asymptomatic right
hip. He is otherwise healthy.When counseling patients who have a cam deformity, the orthopaedic surgeon
should note that

UI // UNAIR // UNPAD // UNHAS // UNS // UGM // UB // UNUD // USU


Adult Reconstructive Surgery of the Hip & Knee Self-Assessment Examination 2019

Figure 3

Figure 1

Figure 2

Figure 4 Figure 5

1. osteoarthritis of the hip is likely to occur later in life.

2. correction prevents later development of osteoarthritis.

3. most acetabular tears are symptomatic, and surgical treatment will be necessary.

4. this is an inherited deformity.


Preffered Answer: 1

Multiple studies have confirmed that cam or pincer anatomy is commonly present in asymptomatic hips.
According to a large systematic review, cam deformities are present in approximately one-third of
asymptomatic hips in young adults, and the proportion is higher than 50% in the subgroup of athletes.

Ganz and associates proposed that femoral acetabular impingement is the root cause of osteoarthritis in
most nontraumatic, nondysplastic hips, and functional improvement with surgical correction of the deformity
has been demonstrated. Despite the link between cam deformity and hip osteoarthritis, a corresponding link
between the correction of the deformity and prevention of osteoarthritis has never been proven.

UI // UNAIR // UNPAD // UNHAS // UNS // UGM // UB // UNUD // USU


Adult Reconstructive Surgery of the Hip & Knee Self-Assessment Examination 2019
The results of cam deformity correction, typically including repair of the degenerative labral tear, are much
poorer when substantial joint space loss is present. A typical joint space cutoff of 2 mm or less is used to
recommend against hip preservation surgery.

44. A 77-year-old man who underwent right total knee replacement surgery 2 and a half years ago has had knee
pain since surgery. The pain is diffuse, constant, and made worse with activity. He notes warmth and
swelling in his knee. Examination shows a well-healed incision, no erythema, moderate warmth, synovitis,
and an effusion. The knee is stable, and has an arc of flexion between 3° and 120°. Radiographs show well-
fixed and well-aligned implants. What is the most appropriate initial treatment?

1. Knee aspiration for culture

2. CT of the knee to assess implant rotation

3. Indium-111 leukocyte/technetium-99m sulfur colloid scan of the knee

4. Erythrocyte sedimentation rate (ESR) and C-reactive protein (CRP) laboratory studies
Preffered answer: 4

This patient's history and physical findings are concerning for deep infection. Inflammatory markers,
including ESR and CRP, should be obtained first. If the levels are elevated, knee aspiration should be
performed for the synovial cell count and culture. A bone scan is not indicated in an initial investigation for
deep infection; it is rarely helpful and is not cost effective. CT to assess implant rotation is an appropriate
investigation for knee pain when the clinical scenario is not suspicious for a deep infection and when
infection has been excluded.

45. During total hip arthroplasty, what characteristic of irradiated (10 Mrad) and subsequently melted highly
cross-linked polyethylene should provide a more wear-resistant construct than traditional gamma-irradiated
(2.5-4 Mrad)-in-air polyethylene mated with the same head?

1. Resistance to adhesive wear

2. Resistance to abrasive wear

3. Resistance to fatigue wear

4. Resistance to creep

Preffered Answer: 1

Highly cross-linked polyethylene makes material resistant to adhesive wear. Abrasive wear from third
bodies does not decrease wear. The fatigue strength of such material is inferior to that of traditional
polyethylene, and its resistance to creep is the same, if not lower, than that of traditional polyethylene.

46.A A 63-year-old woman had a primary total hip arthroplasty 7 years ago that included a proximally
coated titanium stem, a cobalt alloy femoral head, a titanium hemispherical acetabular component,
and a polyethylene liner. She did well for 4 years but has now had two dislocations and reports pain and
weakness around the left hip. She denies any fevers, chills, or constitutional symptoms. On examination,
the patient

UI // UNAIR // UNPAD // UNHAS // UNS // UGM // UB // UNUD // USU


Adult Reconstructive Surgery of the Hip & Knee Self-Assessment Examination 2019
walks well without any signs of an antalgic or Trendelenburg gait. Her abductor mechanism demonstrates
good strength. Her erythrocyte sedimentation rate and C-reactive protein level are normal. On radiographs,
all components appear well fixed and in good alignment. What is the most appropriate treatment at this
time?

1. Physical therapy to improve hip stability

2. Use of an abduction brace to limit the patient’s range of motion

3. Conversion to a constrained acetabular liner

4. Cobalt and chromium serum metal ion level testing


Preffered Answer: 4

Trunnionosis is a recently recognized complication following total hip arthroplasty and can occur when a
cobalt alloy femoral head is used on a titanium alloy or cobalt alloy femoral stem. Patients often present with
pain or swelling around the hip but at times can present with instability. Certain femoral stem designs have
been associated with increased reports of trunnionosis. In a patient with a cobalt alloy femoral head who
presents with instability, swelling, and weakness around the hip, the potential for trunnionosis and an
adverse local tissue reaction should be considered.

47.A A surgeon prepares a medial gastrocnemius rotational flap to cover a medial proximal tibia defect at the
time of revision knee replacement surgery. To optimize coverage, the surgeon must optimally mobilize
which artery?

1. Profunda femoris

2. Middle genicular

3. Medial sural

4. Inferior medial

genicular Preffered Answer:

The medial sural arteries vascularize the gastrocnemius, plantaris, and soleus muscles proximally. These
arteries arise from the popliteal artery. If this artery is not adequately mobilized, a gastrocnemius soleus flap
can be devascularized.

48. A 60-year-old man who underwent left partial knee arthroplasty 6 months earlier was doing well until he
experienced left knee pain and swelling for 4 weeks following a dental procedure. The left knee aspirate was
bloody, with a white blood cell count of 8,000 and 70% neutrophils. Culture grew group B Streptococcus
(Granulicatella adiacens), and serologies were elevated, with an erythrocyte sedimentation rate of 55 mm/h
(reference range: 0 to 20 mm/h) and a C-reactive protein level of 24 mg/L (reference range: 0.08 to 3.1
mg/L). What is the best next step?

1. Arthroscopic debridement

2. Two-stage total knee revision arthroplasty

3. Resection arthroplasty without an antibiotic impregnated cement spacer

UI // UNAIR // UNPAD // UNHAS // UNS // UGM // UB // UNUD // USU


Adult Reconstructive Surgery of the Hip & Knee Self-Assessment Examination 2019
4. Knee fusion

Preffered Answer: 2

This complication is best addressed with either a single-stage or two-stage total knee arthroplasty. A recent
report suggests that a single-stage arthroplasty can be effective, although many surgeons would perform a
two-stage procedure with an articulating or static spacer. Arthroscopic would be non-effective, especially
given 4 weeks of symptoms. Resection arthroplasty without a spacer would leave an unstable and poorly
functioning extremity. Knee fusion should be used as a salvage procedure.

49. According to Musculoskeletal Infection Society (MSIS) guidelines, which set of patient laboratory study
results fits the definition of chronic prosthetic joint infection?

1. Erythrocyte sedimentation rate (ESR) 50 mm/hr, C-reactive protein (CRP) 8 mg/L, joint aspiration white
blood cell (WBC) count 542, 62% neutrophils, and positive leukocyte esterase

2. ESR 42 mm/hr, CRP 12 mg/L, joint aspiration WBC count 3,540, 72% neutrophils, and positive
leukocyte esterase

3. ESR 20 mm/hr, CRP 15 mg/L, joint aspiration WBC count 4,135, 54% neutrophils, and negative
leukocyte esterase

4. ESR 25 mm/hr, CRP 7 mg/L, joint aspiration WBC count 252, 82% neutrophils, and negative leukocyte
esterase

Preffered Answer: 2

The MSIS definition of periprosthetic joint infection was updated in 2014 with two major and six minor
criteria. The presence of one major criterion or three minor criteria is diagnostic for infection. The major
criteria are two positive cultures with the same organism or a draining sinus tract. The current MSIS minor
criteria are 1) an elevated ESR (more than 30 mm/hr) and CRP level (more than 10 mg/L), 2) an elevated
synovial WBC count (more than 3,000 cells per/microliter), 3) an elevated synovial fluid polymorphonuclear
count (more than 80%), 4) a positive histological analysis of periprosthetic tissue, and 5) a single positive
culture.

50. Figure 1 shows the radiograph obtained from a 68-year-old man who fell 3 weeks after undergoing a
successful left primary total hip arthroplasty. He is experiencing a substantial increase in pain and an
inability to bear weight. What is an appropriate treatment plan?

UI // UNAIR // UNPAD // UNHAS // UNS // UGM // UB // UNUD // USU


Adult Reconstructive Surgery of the Hip & Knee Self-Assessment Examination 2019

Open
1. reduction and internal
fixation (ORIF) of the
fracture

2. Removal of the current stem,


femur ORIF, and insertion of
a longer revision stem

3. Femur ORIF with cables and


strut graft, leaving the
current stem in situ

4. Femur ORIF combined


with reimplantation of the

Preffered Answer: 2

The fracture has occurred around the stem, representing a Vancouver type B fracture, and the stem is
clearly loose, making it a type B2 fracture. The appropriate treatment is removal of the loose in situ stem;
ORIF of the femur using cerclage wires, cables, or a plate; and insertion of a longer revision stem such as a
tapered fluted modular titanium or fully porous coated cylindrical stem to bypass the fracture. All of the other
options are incorrect, because they represent inappropriate treatment options for a Vancouver type B2
fracture.

51. What factor is associated with a higher risk of dislocation after total hip arthroplasty?

1. Male gender

2. Previous hip surgery

3. A direct lateral surgical approach

4. Metal-on-metal bearing surfaces

Preffered Answer: 2

Dislocation after total hip arthroplasty is a multifactorial problem. Numerous risk factors may act
independently or cumulatively to increase the risk of this complication. Previous hip surgery of any kind is
associated with a twofold increased risk for dislocation. Other risk factors include female gender, impaired
mental status, inflammatory arthritis, and older age. Numerous studies have shown a lower dislocation rate
with a direct lateral approach, although surgical techniques such as capsular repair have significantly
lowered the incidence of dislocation after using the posterior approach. Metal-on-metal bearings have been
associated with other complications such as adverse tissue reactions but are often used with larger-
diameter bearings, which pose a lower risk of dislocation.

52. The direct anterior (Smith-Peterson) approach to hip arthroplasty is most commonly associated with injury to
what

UI // UNAIR // UNPAD // UNHAS // UNS // UGM // UB // UNUD // USU


Adult Reconstructive Surgery of the Hip & Knee Self-Assessment Examination 2019
1. Lateral femoral cutaneous

2. Sciatic

3. Pudendal

4. Superior gluteal

Preffered Answer: 1

Some authors have reported the incidence of lateral femoral cutaneous nerve neuropraxia following hip
arthroplasty with the direct anterior approach to be near 80%, but resolution of the sensory deficits has
been observed in most patients over time. Femoral nerve palsy has been reported to occur in .64% to
2.3% direct lateral (Hardinge) and anterolateral (Watson-Jones) approaches, and the superior gluteal
nerve may be injured with proximal extension of the abductor muscular dissection. The posterior approach
has been reported to be associated with sciatic nerve injury, especially in cases of dysplasia. Pudendal
nerve injury has not been reported with the anterior, anterolateral, direct lateral, or posterior approaches to
hip arthroplasty. It has been reported following hip arthroscopy and the use of a traction table, however.

53.A A healthy, active 72-year-old man trips and falls, landing on his left hip 10 weeks after an uncomplicated
left primary uncemented total hip replacement. A radiograph taken 6 weeks after surgery and before the
fall is shown in Figure 1. A radiograph taken after the fall is shown in Figure 2. He is unable to bear weight
and is brought to the emergency department. Examination reveals a slightly shortened left lower
extremity and some mild ecchymosis just distal to the left greater trochanteric region, but his skin is
intact, without abrasions or lacerations. What is the most appropriate treatment?

Figure 1 Figure 2

1 Open reduction and cerclage fixation of the fracture

2 Open reduction and revision of the femoral implant to a long cemented stem

3 Open reduction and revision of the femoral implant to a long fluted and tapered uncemented stem

4 Application of balanced traction followed by surgery after the ecchymosis has resolved

UI // UNAIR // UNPAD // UNHAS // UNS // UGM // UB // UNUD // USU


Adult Reconstructive Surgery of the Hip & Knee Self-Assessment Examination 2019
Preffered Answer: 3

This patient has a periprosthetic femoral fracture with a loose femoral stem and normal femoral bone stock,
representing a Vancouver type B2 fracture. The most appropriate treatment is fixation of the fracture, along
with revision of the stem. Considering his age, bone quality, and activity level, a longer uncemented stem is
most predictable. Although a cylindrical stem may also be used, the fluted stem option is the only
uncemented choice listed and is the most appropriate option. A cemented stem is a poorer choice because it
is difficult to keep the cement out of the fracture site, which would pose a risk for nonunion at the fracture.
Also, overall poorer results have been associated with long cemented stems in healthy, active people.
Surgery does not need to be delayed to allow the ecchymosis to resolve, and simple open reduction and
fixation does not address the loose stem.

54. An 80-year-old African American woman who lives in a large city is scheduled for total hip arthroplasty to
address primary osteoarthritis. Part of the presurgical protocol includes nasal swab screening to assess for
methicillin-resistant Staphylococcus aureus (MRSA) colonization. Which demographic factor places this
patient at highest risk for a positive result?

1. Gender
2. Age
3. Race
4. Environment

Preffered Answer:3

Demographic factors are associated with increased risk for MRSA colonization, so it is important to identify
vulnerable patients. Female gender and advanced age reduce the risk for colonization, whereas African
American race increases this risk. Urban environments do not influence MRSA colonization.

55. Figure 1 shows the standing AP radiograph obtained from a 55-year-old man who has a 5-year history of
daily left knee medial joint line pain with weight-bearing activities. He denies night pain or symptoms of
instability. On examination, his range of motion is 0° to 140°. He has a mild, fully correctable varus deformity
and a negative Lachman test result. Nonsurgical treatment has failed.Unicompartmental knee arthroplasty
(UKA) is discussed with the patient. The most appropriate next radiographic evaluation should be:

Figure 1

1. MRI of the left knee to evaluate the lateral compartment.

UI // UNAIR // UNPAD // UNHAS // UNS // UGM // UB // UNUD // USU


Adult Reconstructive Surgery of the Hip & Knee Self-Assessment Examination 2019
2. a CT arthrogram to evaluate the status of the medial and lateral meniscus.

3. a stress radiograph to evaluate correction of the varus deformity.

4. a sunrise view to determine the status of the patellofemoral joint.


Preffered Answer: 3

A patient with medial compartment arthritis and a correctable varus deformity with no clinical or examination
findings of knee instability most likely has an intact anterior cruciate ligament (ACL). The pattern of medial
compartment osteoarthritis most commonly associated with an intact ACL is that of anteromedial
osteoarthritis. An incompetent ACL is commonly associated with a fixed varus deformity and radiographic
signs of posteromedial wear. An incompetent ACL is a relative contraindication to a mobile-bearing UKA.
When evaluating patients for a mobile-bearing UKA, a stress radiograph aids the orthopaedic surgeon in
determining the correction of the varus deformity and assessing the lateral compartment. An inability to fully
correct the deformity or narrowing of the lateral compartment with valgus stress should influence the
surgeon against UKA. Joint registries across the world have shown decreased survivorship associated with
TKA and UKA in men compared with other age groups, but survivorship is lower for UKA than for TKA. No
studies to date have shown any differences in survivorship between fixed-bearing and mobile-bearing UKAs.
The complication that is unique to mobile-bearing UKA is bearing spinout, which occurs in less than 1% of
mobile-bearing UKA procedures. In vivo and in vitro polyethylene wear in mobile-bearing UKA are low.
Arthritis may progress faster in patients with mobile-bearing UKAs than in those with fixed-bearing UKAs.

56. Figure 1 shows the standing AP radiograph obtained from a 55-year-old man who has a 5-year history of
daily left knee medial joint line pain with weight-bearing activities. He denies night pain or symptoms of
instability. On examination, his range of motion is 0° to 140°. He has a mild, fully correctable varus deformity
and a negative Lachman test result. Nonsurgical treatment has failed.
What is the UKA survivorship for a 55-year-old patient, compared with the survivorship for total knee
arthroplasty?

Figure 1

1. Equal at 10 years

2. Lower at 10 years

3. Higher at 10 years

4. Not known when using a mobile-bearing UKA

UI // UNAIR // UNPAD // UNHAS // UNS // UGM // UB // UNUD // USU


Adult Reconstructive Surgery of the Hip & Knee Self-Assessment Examination 2019
Preffered Answer: 2

A patient with medial compartment arthritis and a correctable varus deformity with no clinical or examination
findings of knee instability most likely has an intact anterior cruciate ligament (ACL). The pattern of medial
compartment osteoarthritis most commonly associated with an intact ACL is that of anteromedial
osteoarthritis. An incompetent ACL is commonly associated with a fixed varus deformity and radiographic
signs of posteromedial wear. An incompetent ACL is a relative contraindication to a mobile-bearing UKA.
When evaluating patients for a mobile-bearing UKA, a stress radiograph aids the orthopaedic surgeon in
determining the correction of the varus deformity and assessing the lateral compartment. An inability to fully
correct the deformity or narrowing of the lateral compartment with valgus stress should influence the
surgeon against UKA. Joint registries across the world have shown decreased survivorship associated with
TKA and UKA in men compared with other age groups, but survivorship is lower for UKA than for TKA. No
studies to date have shown any differences in survivorship between fixed-bearing and mobile-bearing UKAs.
The complication that is unique to mobile-bearing UKA is bearing spinout, which occurs in less than 1% of
mobile-bearing UKA procedures. In vivo and in vitro polyethylene wear in mobile-bearing UKA are low.
Arthritis may progress faster in patients with mobile-bearing UKAs than in those with fixed-bearing UKAs.

57. Figures 1 and 2 show the radiographs obtained from a 68-year-old morbidly obese man who underwent left
total hip replacement 7 years ago and did well, with no symptoms prior to the current presentation. He
recently rose from a seated position and felt a pop in the hip, with immediate pain and inability to bear weight.
Any pressure on the left foot now produces a painful, grinding sensation with loss of left hip stability. What is
the best next step?

Figure 1 Figure 2

1. Revision of the acetabular implant to a constrained bearing with modular exchange of the femoral head
and neck

2. Revision of the acetabular and femoral implants

3. Retention of the acetabular implant with modular exchange of the femoral head and neck

4. Revision of the femoral component alone with a new ceramic head


Preffered Answer: 2

UI // UNAIR // UNPAD // UNHAS // UNS // UGM // UB // UNUD // USU


Adult Reconstructive Surgery of the Hip & Knee Self-Assessment Examination 2019
The modular femoral stem has fractured. Changing the liner to a constrained design is not warranted at this
time based on the information provided. Revision of the acetabular implant is appropriate because of the
potential for damage to the existing cup from metal debris and femoral implant contact and to convert from a
metal-on-metal articulation. Nonsurgical management would not provide pain relief or improvement; revision
of the total hip arthroplasty is recommended. The implant failed in a short time, and retention of the femoral
stem is not recommended because of the concern for failure with only a neck exchange. A dual-mobility
bearing may be a good option if the surgeon plans to retain the acetabular component. Extended
trochanteric osteotomy is a useful technique for the removal of a well-fixed femoral implant. In this patient,
femoral stem removal without osteotomy would be difficult due to the fracture of the implant’s femoral neck
and the inability to gain purchase for extraction.

58. A 70-year-old man reports symptomatic medial knee pain that has become progressively worse during the
past year. MRI reveals a complex, posterior horn medial meniscus tear with associated medial lateral and
patellofemoral cartilage defects. Radiographs reveal medial joint space narrowing and osteophytes in the
other compartments. What treatment is most likely to provide long-term, durable relief of symptoms?

1. High tibial osteotomy

2. Total knee replacement

3. Unicondylar knee replacement

4. Arthroscopic partial meniscectomy

Preffered Answer: 2

Total knee replacement is a well-established surgery for diffuse, symptomatic osteoarthritis of the knee
joint, and its efficacy has been shown in many studies. According to the 2008 AAOS Clinical Practice
Guideline, Treatment of Osteoarthritis of the Knee, arthroscopy in the setting of existing osteoarthritis is
efficacious for relieving the signs and symptoms of a torn meniscus but not for osteoarthritis. Likewise, in
young and active patients, clinical outcomes show improvement after realignment osteotomy for single-
compartment osteoarthritis. Unicondylar knee replacement is not indicated for tricompartmental disease of
the knee.

59. Figures 1 through 3 depict the radiographs obtained from a 76-year-old woman with a painful total knee
arthroplasty. She describes an uneventful recovery with no wound-healing issues and was pain free for the
first 10 years. Although reporting no trauma or inciting event, she now describes pain in the entire knee that
is most severe with her first few steps. She has begun to notice night pain and, more recently, constant
swelling. What is the most appropriate work-up at this time?

UI // UNAIR // UNPAD // UNHAS // UNS // UGM // UB // UNUD // USU


Adult Reconstructive Surgery of the Hip & Knee Self-Assessment Examination 2019

Figure 1 Figure 2 Figure 3

1. Knee aspiration with cell count/cultures, C-reactive protein (CRP) level, erythrocyte sedimentation rate
(ESR), CT

2. Knee aspiration with cell count/cultures, CRP, ESR

3. Fresh-frozen specimen at the time of revision knee arthroplasty only

4. Technetium-99m bone scan, knee aspiration with cell count/cultures


Preffered Answer: 2

An evaluation of the painful total knee must be supported by an understanding of the potential etiologies of
pain. They may include, aseptic loosening, infection, osteolysis, gap imbalance, referred pain, stiffness, and
complex regional pain syndrome. In this case, the patient demonstrates start-up pain and had no prior
history of infections. Her radiographs show subsidence of the tibia, indicating a loose prosthesis. Knowing
that the prosthesis is already loose precludes the need for a bone scan. It is, however, important to rule out
infection in this case; therefore, CRP and ESR testing is essential. Aspiration is also recommended when
going into knee arthroplasty, and infection is a concern.

60. At the time of revision knee arthroplasty, a surgeon performs a rectus snip to gain exposure to the knee.
When compared with a standard parapatellar approach, what is the expected outcome?

1. Improvement in range of motion

2. Reduction in range of motion

3. Increase in extensor mechanism lag

4. No differences in motion and strength


Preffered Answer: 4

Rectus snip during total knee arthroplasty has no effect on motion or strength at long-term follow-up. It has
not been associated with extensor mechanism lag.

UI // UNAIR // UNPAD // UNHAS // UNS // UGM // UB // UNUD // USU


Adult Reconstructive Surgery of the Hip & Knee Self-Assessment Examination 2019

61. Compared with retention of the native patella in primary total knee arthroplasty, routine patellar resurfacing is
associated with

1. no patellar complications.

2. an increased occurrence of anterior knee pain.

3. a reduced patellar fracture rate.

4. a reduced risk for revision surgery.


Preffered Answer: 4

Despite concerns regarding fracture, osteonecrosis, and patellar clunk, the routine retention of the native
patella during primary total knee replacement is associated with a 20% to 30% increased revision risk in
large joint registries. In addition, the retention of the native patella results in a 5.7% revision surgery rate in
patients with anterior knee pain.

62. Figure 1 shows the radiograph obtained from a 67-year-old man recently diagnosed with osteoarthritis, 8
years after receiving a left metal-on-metal total hip arthroplasty (THA). The acetabular component has a
modular cobalt alloy acetabular liner. The patient states that he did very well postoperatively, but for the last
6 months has noted worsening pain and swelling in his left hip. Serum metal ion testing reveals a chromium
level of 12.4 ng/mL, compared with a normal level of less than 0.3 ng/mL, and a cobalt level of 11.8 ng/mL,
compared with a normal level less than 0.7 ng/mL. An MRI with metal artefact reduction sequence (MARS)
was performed and is shown in Figure 2. What is the most appropriate management at this time?

Figure 1 Figure 2

1. Annual monitoring of serum metal ion levels

2. Repeated MRI with MARS in 6 months

3. Conversion of the THA to a cobalt alloy femoral head and polyethylene bearing

4. Conversion of the THA to a ceramic femoral head with an inner titanium sleeve and polyethylene
bearing

Preffered Answer: 4

UI // UNAIR // UNPAD // UNHAS // UNS // UGM // UB // UNUD // USU


Adult Reconstructive Surgery of the Hip & Knee Self-Assessment Examination 2019
Metal-on-metal THA was initially introduced in the 1990s, with the proposed advantages of decreased wear
and improved stability. However, catastrophic adverse local tissue reactions associated with their use has
raised numerous concerns. The work-up of a patient with a prior metal-on-metal total hip arthroplasty
involves a thorough history and physical examination; blood analysis, including the erythrocyte
sedimentation rate, C-reactive protein, and metal ion levels; and secondary imaging, including
ultrasonography, CT, and MRI. In a patient with clinical symptoms, elevated metal ion levels, and a large
fluid collection seen on MRI, the most appropriate treatment would be removal of the metal-on-metal
bearing. Given the presence of an adverse reaction involving cobalt and chromium, a revision ceramic head
may be most appropriate to avoid the potential of trunnion-associated corrosion.

63. Figure 1 shows the abdominal radiograph obtained from a 70-year-old woman who experiences nausea and
abdominal tightness 48 hours following left total knee arthroplasty performed under general anesthesia. She
received 24 hours of cefazolin antibiotic prophylaxis and a patient-controlled analgesia narcotic pump for
pain management. She has been receiving warfarin for thromboembolic prophylaxis. Her severe abdominal
distension and markedly decreased bowel sounds are most likely secondary to the administration of

Figure 1

1. general anesthesia.

2. antibiotics.

3. warfarin.

4. narcotics.
Preffered Answer: 4

The radiograph reveals severe intestinal dilatation, which has occurred as the result of acute colonic pseudo-
obstruction and is associated with excessive narcotic administration following total joint arthroplasty.
Anesthetic type, antibiotic administration, and warfarin have not been associated with this obstruction.
Electrolyte imbalances such as hypokalemia have been associated with postsurgical acute colonic pseudo-
obstruction.

64. What factor is associated with a high risk of developing pseudotumors after metal-on-metal hip resurfacing?

UI // UNAIR // UNPAD // UNHAS // UNS // UGM // UB // UNUD // USU


Adult Reconstructive Surgery of the Hip & Knee Self-Assessment Examination 2019

1. Large-diameter components

2. Age 40 or older for men

3. Age 40 or younger for women

4. Diagnosis of primary osteoarthritis


Preffered Answer: 3

The recent experience of a large clinical cohort revealed the most likely risk factors as being female gender,
age younger than 40, small components, and a diagnosis of hip dysplasia causing osteoarthritis. Failure was
least likely among men and after procedures involving larger components. These data have prompted some
authors to caution against using metal-on-metal hip resurfacing in women and to primarily target candidates
who are men younger than age 50. Small components may be more prone to failure because
ofmalpositioning and edge loading, which have been noted to be more common in dysplasia cases.

65. Figures 1 through 3 depict the radiographs obtained from a 60-year-old man with instability and pain 1 year
after primary right total knee arthroplasty. He states that he had surgery on two occasions for a tendon
rupture that was repaired with sutures but that his knee popped again, and now the leg is unable to hold his
weight. On examination, he is in no acute distress. His height is 6'3", and he weighs 240 pounds. He is
ambulatory with crutches. Range of motion of the right knee is 50° to 120° actively and 0° to 120° passively.
More than 10° of varus/valgus laxity and more than 5 mm of anteroposterior drawer are present. A palpable
defect is observed in the tissue just proximal to the patella. The incision is well healed. The erythrocyte
sedimentation rate is 46 mm/h (reference range 0 to 20 mm/h) and the C-reactive protein level is 2.04 mg/L
(reference range 0.08 to 3.1 mg/L). Aspiration of the right knee reveals hazy yellow fluid with a white blood
cell count of 120 and 1% neutrophils. No growth of organisms is seen on routine culture. What is the best
next step?

Figure 1

Figure 2 Figure 3

1. Revision total knee arthroplasty with extensor mechanism allograft

2. Revision total knee arthroplasty with liner change and primary quadriceps repair

3. Resection knee arthroplasty and arthrodesis with antegrade nail

4. Two-stage revision total knee arthroplasty with extensor mechanism allograft

UI // UNAIR // UNPAD // UNHAS // UNS // UGM // UB // UNUD // USU


Adult Reconstructive Surgery of the Hip & Knee Self-Assessment Examination 2019
Preffered Answer: 1

This patient has a chronic quadriceps tendon rupture after total knee arthroplasty. Two previous primary
repair attempts have failed, which is not surprising based on the poor results of primary repair reported in the
literature. The patient also has an unstable knee and will require revision of some or all of the prosthesis to
achieve a stable knee. Revision total knee arthroplasty with extensor mechanism allograft allows an allograft
reconstruction of the ruptured quadriceps tendon. The other option is to utilize a synthetic mesh extensor
mechanism reconstruction. These are likely to have the best result in this situation. Revision total knee
arthroplasty with liner change and primary quadriceps repair is not the best form of management, because it
involves a third attempt at primary tendon repair, which will likely fail again. Resection knee arthroplasty and
arthrodesis with antegrade nail is a possible option but is not the best, because it would likely make driving
and other daily activities difficult. Two-stage revision total knee arthroplasty with extensor mechanism
allograft is not the best option because the laboratory results show no signs of infection, so a single-stage
procedure is preferred.

66. Figures 1 and 2 show the radiographs obtained from a 90-year-old woman who is seen in the emergency
department after a fall from a height. She has right hip and thigh pain and is unable to bear weight. Based on
this patient's history and imaging, what is the best next step?

Figure 1 Figure 2

1. Hip revision and implantation of a proximal femoral replacement

2. Hip revision and implantation of a tapered fluted stem

3. Open reduction and internal fixation with a locked plate and allograft struts

4. Erythrocyte sedimentation rate and C-reactive protein laboratory studies


Preffered Answer: 4

Periprosthetic fracture is the third most common reason (after loosening and infection) for revision surgery
after total hip arthroplasty (THA). Late periprosthetic fracture risk is 0.4% to 1.1% after primary THA and
2.1% to 4% after revision THA. Risk factors for periprosthetic fracture include age over 70 years,
decreasing bone mass, and loosening of implants and osteolysis. The risk of concomitant infection in the
presence of a periprosthetic fracture is 11%, according to Chevillotte and associates. Obtaining presurgical
aspiration or intrasurgical tissue for culture is recommended if concomitant infection is suspected.

UI // UNAIR // UNPAD // UNHAS // UNS // UGM // UB // UNUD // USU


Adult Reconstructive Surgery of the Hip & Knee Self-Assessment Examination 2019
67. Figures 1 and 2 show the clinical photograph and radiograph obtained from a 62-year-old man who has
deformity and pain 1 year after primary total hip arthroplasty. What is the reason for the observed deformity?

Figure 1

Figure 2

1. A Vancouver type B1 fracture

2. Residual leg-length discrepancy

3. Loosening and subsidence of the femoral stem into anteversion

4. Loosening and subsidence of the femoral stem into retroversion


Preffered Answer : 4

Figure 1 reveals an external rotation deformity of the right lower extremity. This deformity can have
numerous causes, including extra-articular deformity. Figure 2 reveals a loose, subsided femoral
component. Femoral stems typically subside into retroversion due to proximal femoral biomechanics, which
cause a compensatory external rotation deformity. The combined findings from both images suggest an
external rotation deformity most likely related to subsidence into retroversion.

68.A A 40-year-old man with a history of Legg-Calve-Perthes disease underwent a right hip resurfacing 3
years ago with no perioperative complications. Hip pain has developed gradually during the last 4
months. Radiographs show no evidence of fixation loosening or any adverse changes at the femoral
neck. No periarticular osteolysis is evident.What is the most appropriate management of this condition?

1. Continue to observe with repeat radiographs in 6 months

2. Fluoroscopic-guided iliopsoas tendon cortisone injection

3. Hip aspiration

4. Serum cobalt and chromium levels and metal-reduction MRI scan


Preffered Answer: 4

UI // UNAIR // UNPAD // UNHAS // UNS // UGM // UB // UNUD // USU


Adult Reconstructive Surgery of the Hip & Knee Self-Assessment Examination 2019
Controversy persists over what exactly is the best approach to managing patients with metal-on-metal
(MOM) hip arthroplasties. All patients with painful MOM hip arthroplasties should be examined for fixation
loosening, wear/osteolysis, and infection—no differently than patients without MOM hip arthroplasties. It is
recommended to obtain serum trace element levels. If the levels are high, cross-sectional imaging should be
obtained to determine whether any pseudotumor or tissue necrosis is present around the hip arthroplasty.
Hip aspiration should be considered if concern for infection exists. Adverse tissue reaction has been
identified to occur around MOM hip arthroplasties. The predominant histologic feature is tissue necrosis with
infiltration of lymphocytes and plasma cells.

69.A A 40-year-old man with a history of Legg-Calve-Perthes disease underwent a right hip resurfacing 3
years ago with no perioperative complications. Hip pain has developed gradually during the last 4
months. Radiographs show no evidence of fixation loosening or any adverse changes at the femoral
neck. No periarticular osteolysis is evident.
A large intra-articular and intrapelvic pseudotumor has developed. What predominant histological feature(s)
is/are present in such a lesion?

1. Polymorphonuclear leukocytes

2. Extracellular metal-wear debris

3. Cement particles within the macrophages

4. Lymphocytes and plasma cells


Preffered Answer: 4

Controversy persists over what exactly is the best approach to managing patients with metal-on-metal (MOM)
hip arthroplasties. All patients with painful MOM hip arthroplasties should be examined for fixation loosening,
wear/osteolysis, and infection—no differently than patients without MOM hip arthroplasties. It is
recommended to obtain serum trace element levels. If the levels are high, cross-sectional imaging should be
obtained to determine whether any pseudotumor or tissue necrosis is present around the hip arthroplasty. Hip
aspiration should be considered if concern for infection exists. Adverse tissue reaction has been identified to
occur around MOM hip arthroplasties. The predominant histologic feature is tissue necrosis with infiltration of
lymphocytes and plasma cells.

70.A A 70-year old woman undergoes revision total knee arthroplasty for tibial component aseptic loosening.
She is concerned about recurrent loosening, and tibial stem fixation options during revision are reviewed.
Figure 1 displays a radiograph of the revision technique used for this patient. What is the incidence of
intraoperative tibial shaft fracture that is associated with this type of revision surgery?

UI // UNAIR // UNPAD // UNHAS // UNS // UGM // UB // UNUD // USU


Adult Reconstructive Surgery of the Hip & Knee Self-Assessment Examination 2019

Figure 1

1. 0% to 1% with press-fit tibial stems

2. 3% to 5% with press-fit tibial stems

3. 3% to 5% with cemented tibial stems

4. More than 5% with press-fit tibial stems

Preffered Answer: 2

Using press-fit tibial stems during a hybrid revision total knee arthroplasty is associated with a 3% to 5%
incidence of intraoperative tibial shaft fracture. Diaphyseal fixation of press-fit stems has the advantage of
setting component alignment, dispersing forces on the proximal tibia, and offers excellent clinical results. The
disadvantages include proximal and distal tibia anatomic mismatch and tibial shaft fracture. Cipriano and
associates reported a tibial shaft fracture incidence of 4.9% in a series of 420 consecutive knee revisions. All
fractures healed with nonsurgical management, and none led to implant loosening. In this patient, it is
important to recognize on the radiograph that this technique is a hybrid method of revision total knee
arthroplasty, with cementation along the tibial tray and metaphysis and with press-fit fixation of the diaphyseal
engaging stem. Then, it is important to know the risk and management of intraoperative diaphyseal tibial
fractures. Cemented tibial stems are associated with a low rate of intraoperative fracture, because the implant
is typically undersized to allow for an appropriate cement mantle. Option C is incorrect, because this revision
is not cemented. Option A underestimates the incidence of fracture, whereas D overestimates the rate of
fracture.

71. A 61-year-old man with a body mass index of 31 had a 6-month gradual onset of right medial knee pain.
Examination revealed a small effusion, stable ligaments, a normally tracking patella, and mild medial joint line
tenderness. Standing radiographs show mild medial joint space narrowing. Effective treatment at this stage of
early medial compartmental osteoarthritis includes

1. glucosamine 1,500 mg/day and chondroitin sulfate 800 mg/day.

2. weight loss through dietary management and low-impact aerobic exercises.

3. arthroscopic debridement and lavage.

4. a valgus-directing brace.

Preffered Answer: 2

According to the 2008 AAOS Clinical Practice Guideline, Treatment of Osteoarthritis of the Knee
(Nonarthroplasty), level 1 evidence confirms that weight loss and exercise benefit patients with knee
osteoarthritis. The other responses have either inclusive evidence (a valgus-directing brace) or no evidence
to support their use (glucosamine 1,500 mg/day and chondroitin sulfate 800 mg/day as well as arthroscopic
debridement and lavage).

UI // UNAIR // UNPAD // UNHAS // UNS // UGM // UB // UNUD // USU


Adult Reconstructive Surgery of the Hip & Knee Self-Assessment Examination 2019

72. Figures 1 and 2 depict the radiographs obtained from a 53-year-old man who has had swelling in his right
knee for 2 years, with minimal pain. He did not note an injury to the knee but has been unable to ambulate
without crutches during this period. His past history is unremarkable, and he denies a history of diabetes or
back problems. The social history reveals that he emigrated from China, and he works at a desk job. Physical
examination shows a healthy man in no acute distress. Range of motion of the right knee is 5° to 120°
actively and 0° to 120° passively, without pain. Sensation is decreased on the bottom of both feet, but
otherwise the neurologic examination is unremarkable. Laboratory testing reveals a positive rapid plasma
reagin (RPR) test. What is the best next step?

Figure 1 Figure 2

1. Open reduction and internal fixation

2. Hinged total knee arthroplasty

3. Arthrodesis using an intramedullary nail

4. Irrigation and debridement with spacer placement


Preffered Answer: 2

This patient has a neuropathic knee caused by neurosyphilis, as shown by the joint destruction on the
radiographs, with a lack of pain and a positive RPR test. He has a low-demand job and would be best
treated with a hinged knee arthroplasty to provide stability for his knee.

73. Figure 1 shows the radiograph obtained from a 78-year-old woman who has a recent history of increasing
thigh pain 12 years after undergoing total hip arthroplasty. Figure 2 depicts the radiograph obtained after she
fell and was unable to ambulate. What is the most appropriate treatment?

UI // UNAIR // UNPAD // UNHAS // UNS // UGM // UB // UNUD // USU


Adult Reconstructive Surgery of the Hip & Knee Self-Assessment Examination 2019

Figure 1 Figure 2

1. Application of a femoral cable plate

2. Application of cerclage-wired double allograft femoral struts

3. Femoral revision with an uncemented long stem

4. Femoral revision with a cemented long-stem prosthesis


Preffered Answer: 3

The surgical treatment of periprosthetic fractures of total hip replacement with a loose implant and
progressive bone loss is associated with a high complication rate. The recent literature would favor the use
of long "Wagner-type" stems, which have a long distal taper that may optimally engage the remaining
femoral shaft isthmus. Plating options are problematic, because the intramedullary stem limits the ability to
use screws with the plate. Using long distally fixed stems circumvents this problem by enhancing fracture
healing and creating a long-term prosthetic solution in these most difficult cases.

74.A A 62-year-old active man returns for routine follow-up 16 years after hip replacement. He has no hip
pain. Radiographs reveal a well-circumscribed osteolytic lesion around a single acetabular screw.
All hip components are perfectly positioned. Comparison radiographs obtained 6 months later show an
increase in the size of the osteolytic lesion. CT depicts a well-described lesion that is 3 cm at its largest
diameter and is localized around one screw hole with an eccentric femoral head. What treatment is
appropriate, assuming that well-fixed cementless total hip components are in place?

1. Revision of the polyethylene liner, removal of the screw, and debridement of the osteolytic lesion with or
without bone grafting

2. Revision of the acetabular component to a newer design without screws

3. Removal of the screw, revision of the polyethylene liner, and stem cell injection into the lytic lesion

4. Removal of the offending screw from the metal socket and placement of a new polyethylene liner in the
existing socket

Preffered Answer: 1

Given a well-fixed acetabular metal shell and a localized osteolytic lesion, good outcomes can be expected
from liner revision in this clinical scenario with retention of the metal socket, assuming no damage to the
components or other unexpected findings arise during revision surgery. Here, complete cup revision is not
warranted, considering the appropriate implant position. Beaulé and associates reviewed 83 consecutive
patients (90 hips) in whom a well-fixed acetabular component was retained in a clinical scenario such as the
one described. No hip showed recurrence or expansion of periacetabular osteolytic lesions. If the metal cup
is unstable, acetabular component revision may be indicated.

UI // UNAIR // UNPAD // UNHAS // UNS // UGM // UB // UNUD // USU


Adult Reconstructive Surgery of the Hip & Knee Self-Assessment Examination 2019

75. Figures 1 and 2 show the radiographs obtained from a 79-year-old woman who has been experiencing
increasing tibial pain 10 years after undergoing revision total knee arthroplasty. No evidence of infection is
seen. What is the most appropriate treatment?

Figure 2

Figure 1

1. Retain the components, and implant a tibial strut allograft.

2. Revise the tibial component with a metaphyseal cone and metaphyseal uncemented stem.

3. Revise the tibial component with a metaphyseal cone and a press-fit diaphyseal-engaging stem.

4. Revise the tibial component with a long cemented diaphyseal-engaging stem.


Preffered Answer: 3

Stems are available for cemented and press-fit implantation. To be effective, press-fit stems should engage
the diaphysis, as shown in Figures 3 and 4. They also assist in obtaining correct limb alignment. Short
metaphyseal-engaging stems are associated with failure rates that range between 16% and 29%. Cemented
stems may be shorter than press-fit stems, because they do not have to engage the diaphysis. Short, fully
cemented stems offer the advantage of metaphyseal fixation. Hybrid stem fixation makes use of the
metaphysis for cement fixation with metaphyseal cones or sleeves and diaphyseal-engaging press-fit stems.

76. Figures 1 and 2 show the AP and lateral radiographs obtained from a 54-year-old woman who has
worsening groin pain 18 months after a primary left total hip arthroplasty. The pain is worst when climbing
stairs, when rising from a seated position, and during resisted hip flexion. Her pain improved early after
surgery but did not completely resolve. Her C-reactive protein and erythrocyte sedimentation rate results of
less than 1 mg/dL and 10 mm/hr, respectively, were obtained in the office. What is the best next step?

UI // UNAIR // UNPAD // UNHAS // UNS // UGM // UB // UNUD // USU


Adult Reconstructive Surgery of the Hip & Knee Self-Assessment Examination 2019

Figure 1 Figure 2

1. MRI with MARS of the left hip

2. Revision of the left acetabular component

3. Intra-articular ultrasound-guided left hip injection

4. Physical therapy for the left hip


Preffered Answer: 2

Iliopsoas impingement is a potential cause of persistent groin pain after a total hip arthroplasty. This patient’s
history gives groin pain with resisted hip flexion and during activities that require this level of function. The
radiographs depict an acetabular component with substantial retroversion. Typical options for the
management of iliopsoas tendon impingement include injections, tenotomy, and acetabular revision.
Recently, Chalmers and associates reported more predictable groin pain resolution with 8 mm or more of
anterior acetabular component when overhang was revised. The radiographs clearly show more
retroversion, with a cup prominence of more than 8 mm anteriorly. MRI with MARS could potentially help in
the diagnosis of this impingement but would not help in management (option A). An ultrasound-guided
injection would need to be administered into the iliopsoas tendon sheath to be of help and, in this case,
would likely be performed for diagnostic purposes due to the extreme anterior overhang (option C). Option D
would be useful for mild cases of iliopsoas impingement but likely would not help much in this more extreme
case.

77. Figure 1 shows a cross-table lateral radiograph obtained from a healthy 56-year-old woman with recurrent hip
dislocations 6 months after total hip arthroplasty performed through a posterolateral approach. Each
dislocation occurred when she was bending over to put her shoes on or pick something up. She has
dislocated four times and has had no pain between dislocations. Abductor strength is 5 out of 5. The infection
work-up is negative. What is the best next step?

UI // UNAIR // UNPAD // UNHAS // UNS // UGM // UB // UNUD // USU


Adult Reconstructive Surgery of the Hip & Knee Self-Assessment Examination 2019

Figure 1

1. Revision of the acetabulum and evaluation of the femoral stem

2. Conversion to a constrained liner

3. Gluteus medius repair and application of a hip abductor brace

4. Revision to an elevated acetabular polyethylene liner


Preffered Answer: 1

The cross-table lateral radiograph shows that the patient has decreased acetabular anteversion. She is likely
impinging on her cup in flexion and levering the femoral component posteriorly. Given a well-fixed and well-
aligned femoral component and a negative infection work-up, the preferred treatment is to revise the
acetabulum with a goal of increasing acetabular anteversion to avoid prosthetic impingement. Conversion to
a constrained or elevated rim liner is suboptimal in this setting, because the problem is impingement.
Indications for a constrained liner are neuromuscular compromise, abductor deficiency, or instability despite
well-fixed and well-placed components. Given her 5 of 5 abductor strength, gluteus medius repair is not
indicated.

78. When compared with patients having a body mass index (BMI) lower than 35, patients with a BMI above 40
who undergo primary total hip arthroplasty (THA) and total knee arthroplasty (TKA) are likely to have

1. smaller incisions.

2. more wound complications.

3. fewer 30-day and 90-day readmissions.

4. lower rates of patient satisfaction.

Preffered Answer: 2

The obesity epidemic is increasing, and the number of patients with a BMI higher than 35 undergoing THA
and TKA also is growing. Controversy exists over the optimal BMI cutoff and the ability to perform joint
replacements safely in patients who are morbidly obese. Several clinical series and national database
analyses have shown that morbidly obese patients undergoing THA or TKA are at increased risk for wound
complications as well as 30-day and 90-day readmissions. These patients’ incisions are typically larger
because of the size of the soft-tissue envelope. Although the clinical scores following successful THA or
TKA often are lower than the scores of controls, the overall changes in clinical function and satisfaction are
equivalent in nonobese and obese patients.

79. In total knee arthroplasty, in vitro testing has shown that cross-linking can diminish the rate of polyethylene
wear by 30% to 80%. What other change in material properties is possible when polyethylene is highly cross-
linked?

UI // UNAIR // UNPAD // UNHAS // UNS // UGM // UB // UNUD // USU


Adult Reconstructive Surgery of the Hip & Knee Self-Assessment Examination 2019
1. Increased ductility

2. Increased wettability

3. Diminished fatigue strength

4. Decreased resistance to abrasive wear


Preffered Answer: 3

The most important concern regarding highly cross-linked polyethylene relates to decreased mechanical
properties. Cross-linking results in reduced ductility, tensile strength, and fatigue crack propagation
resistance. These problems have not been shown to cause implant failure in the most recent clinical trials,
but they remain the most important mechanical issues associated with current material processing methods.

80. Figure 1 depicts the radiograph obtained from a 52-year-old woman who has leg-length inequality and
chronic, activity-related buttock discomfort. This problem has been lifelong, but it is getting worse and
increasingly causing back pain. What is the best current technique for total hip arthroplasty?

Figure 1

1. High hip
center
2. Anatomic hip center with trochanteric osteotomy and progressive femoral shortening

3. Anatomic hip center with subtrochanteric shortening osteotomy

4. Iliofemoral lengthening followed by an anatomic hip center


Preffered Answer: 3

A high hip center is not recommended for Crowe type IV hips because of the lack of acetabular bone and
altered hip biomechanics. An anatomic center is a better option but necessitates a technique to address the
tight soft-tissue envelope. A trochanteric osteotomy with progressive femoral shortening has been described
but can be prone to trochanter nonunion. Iliofemoral lengthening prior to surgery has been described but
may not be tolerated by all patients. A shortening subtrochanteric osteotomy avoids trochanter nonunion and
allows adjustment of femoral anteversion. Fixation of the osteotomy can include a stem with distal rotational
control, plate fixation, a step versus oblique cut, or strut grafts.

UI // UNAIR // UNPAD // UNHAS // UNS // UGM // UB // UNUD // USU


Adult Reconstructive Surgery of the Hip & Knee Self-Assessment Examination 2019
81. A 70-year-old woman who underwent total knee replacement 18 months ago has had 3 weeks of moderate
drainage from a previously healed wound. What is the most appropriate treatment?

1. Vacuum-assisted wound closure dressing

2. Intravenous antibiotics for 6 weeks, followed by long-term oral antibiotic administration

3. Irrigation and debridement, followed by polyethylene exchange

4. Two-stage debridement and reconstruction

Preffered Answer: 4

This situation represents a definitively and chronically infected knee replacement. Antibiotic therapy alone
might suppress the infection but would not eradicate it. Debridement and polyethylene exchange would be
appropriate treatment for an early postoperative infection. The treatment of choice is to perform a two-
stage debridement and reconstruction. Although not among the listed choices, an aspiration or culture
could be done presurgically and might help clinicians identify the best antibiotics to treat the condition.
Antibiotic selection would not affect the need for the two-stage reconstruction, however.

82. Figures 1 and 2 demonstrate the radiographs obtained from a 35-year-old woman with end-stage debilitating
osteoarthritis of the right hip. She is contemplating total hip arthroplasty (THA). She has a history of right hip
dysplasia and underwent hip osteotomy as an adolescent. Over the years, nonsurgical treatment, including
weight loss, activity modifications, and intra-articular injections, has failed. Her infection work-up reveals
laboratory findings within defined limits.
Which bearing surface is contraindicated for this patient?

Figure 1 Figure 2

1. Ceramic-on-ceramic

2. Ceramic-on-highly cross-linked polyethylene (HXPE)

3. Metal-on-HXPE

4. Metal-on-metal
Preffered Answer: 4

THA has proven durable and reliable for pain relief and improving function for patients with end-stage
arthritis. Appropriate bearing selection is critical to minimize wear and hip complications. A metal-on-metal

UI // UNAIR // UNPAD // UNHAS // UNS // UGM // UB // UNUD // USU


Adult Reconstructive Surgery of the Hip & Knee Self-Assessment Examination 2019
articulation is associated with excellent wear rates in vitro. With its capacity to offer a low wear rate with
large femoral heads, it is an attractive bearing choice for THA. However, local soft-tissue reactions,
pseudotumors, and potential systemic reactions including renal failure, cardiomyopathy, carcinogenesis,
and potential teratogenesis with potential transfer of metal ions across the placental barrier make metal-on-
metal bearings less desirable and relatively contraindicated for younger women of child-bearing age.

The workup of a painful metal-on-metal hip arthroplasty necessitates a systematic approach. Several
algorithms have been proposed. Routine laboratory studies including sedimentation rate, CRP, and serum
cobalt and chromium ion levels should be obtained for all patients with pain. Advanced imaging including
MARS MRI should be performed to evaluate for the presence of fluid collections, pseudotumors, and
abductor mechanism destruction. Infection can coexist with metal-on-metal reactions, so, when indicated (if
the CRP level is elevated), a hip arthrocentesis should be obtained. However, in this setting, a manual cell
count and differential should be obtained because an automated cell counter may provide falsely elevated
cell counts.

The results of revision surgery for a failed metal-on-metal hip prosthesis can be variable. The amount of
local tissue destruction and the integrity of the hip abductor mechanism can greatly influence outcomes.
Instability is the most common complication following revision of failed metal-on-metal hip replacements.

83. Figures 1 and 2 demonstrate the radiographs obtained from a 35-year-old woman with end-stage
debilitating osteoarthritis of the right hip. She is contemplating total hip arthroplasty (THA). She has a
history of right hip dysplasia and underwent hip osteotomy as an adolescent. Over the years, nonsurgical
treatment, including weight loss, activity modifications, and intra-articular injections, has failed. Her infection
work-up reveals laboratory findings within defined limits.The patient undergoes successful primary THA with
a metal-on-metal bearing. At 1-year follow-up, she reports no pain and is highly satisfied with the procedure.
However, 3 years after the index procedure, she reports atraumatic right hip pain that worsens with
activities. Radiographs reveal the implants in good position with no sign of loosening or lysis. An initial
laboratory evaluation reveals a normal sedimentation rate and C-reactive protein (CRP) level. The most
appropriate next diagnostic step is

UI // UNAIR // UNPAD // UNHAS // UNS // UGM // UB // UNUD // USU


Adult Reconstructive Surgery of the Hip & Knee Self-Assessment Examination 2019

Figure 1 Figure 2

1. MRI with metal artifact reduction sequence (MARS) only.

2. serum cobalt only.

3. serum cobalt and chromium levels.

4. serum cobalt and chromium levels and MRI with MARS.


Preffered Answer: 4

THA has proven to be durable and reliable for pain relief and improvement of function in patients with end-
stage arthritis. Appropriate bearing selection is critical to minimize wear and hip complications. A metal-on-
metal articulation is associated with excellent wear rates in vitro. Because it offers a low wear rate with large
femoral heads, it is an attractive bearing choice for THA. However, local soft-tissue reactions,
pseudotumors, and potential systemic reactions—including renal failure, cardiomyopathy, carcinogenesis,
and potential teratogenesis after the possible transfer of metal ions across the placental barrier—make metal-
on-metal bearings less desirable and relatively contraindicated for younger women of child-bearing age.

The work-up of a painful metal-on-metal hip arthroplasty necessitates a systematic approach. Several
algorithms have been proposed. Routine laboratory studies including the erythrocyte sedimentation rate, C-
reactive protein (CRP) level, and serum cobalt and chromium ion levels should be obtained for all patients
with pain. Advanced imaging, including MRI with MARS, should be performed to evaluate for the presence
of fluid collections, pseudotumors, and abductor mechanism destruction. Infection can coexist with metal-on-
metal reactions, so when indicated (if the CRP level is elevated), a hip arthrocentesis should be obtained.
However, in this setting, a manual cell count and differential should be obtained because an automated cell
counter may provide falsely elevated cell counts.

The results of revision surgery for a failed metal-on-metal hip prosthesis can be variable. The amount of
local tissue destruction and the integrity of the hip abductor mechanism can greatly influence outcomes.
Instability is the most common complication following a revision of failed metal-on-metal hip replacements.

84. Figures 1 and 2 demonstrate the radiographs obtained from a 35-year-old woman with end-stage debilitating
osteoarthritis of the right hip. She is contemplating total hip arthroplasty (THA). She has a history of right hip
dysplasia and underwent hip osteotomy as an adolescent. Over the years, nonsurgical treatment, including
weight loss, activity modifications, and intra-articular injections, has failed. Her infection work-up reveals
laboratory findings within defined limits.
A further work-up reveals elevations in serum cobalt and chromium levels and fluid collections surrounding
the hip on MRI with MARS. Revision THA is recommended. The most common complication following
revision of a failed metal-on-metal hip arthroplasty is

UI // UNAIR // UNPAD // UNHAS // UNS // UGM // UB // UNUD // USU


Adult Reconstructive Surgery of the Hip & Knee Self-Assessment Examination 2019

Figure 1 Figure 2

1. infection.

2. instability.

3. loosening.

4. periprosthetic fracture.
Preffered Answer: 2

THA has proven durable and reliable for pain relief and improving function for patients with end-stage
arthritis. Appropriate bearing selection is critical to minimize wear and hip complications. A metal-on-metal
articulation is associated with excellent wear rates in vitro. With its capacity to offer a low wear rate with
large femoral heads, it is an attractive bearing choice for THA. However, local soft-tissue reactions,
pseudotumors, and potential systemic reactions including renal failure, cardiomyopathy, carcinogenesis, and
potential teratogenesis with potential transfer of metal ions across the placental barrier make metal-on-metal
bearings less desirable and relatively contraindicated for younger women of child-bearing age.

The workup of a painful metal-on-metal hip arthroplasty necessitates a systematic approach. Several
algorithms have been proposed. Routine laboratory studies including sedimentation rate, CRP, and serum
cobalt and chromium ion levels should be obtained for all patients with pain. Advanced imaging including
MARS MRI should be performed to evaluate for the presence of fluid collections, pseudotumors, and
abductor mechanism destruction. Infection can coexist with metal-on-metal reactions, so, when indicated (if
the CRP level is elevated), a hip arthrocentesis should be obtained. However, in this setting, a manual cell
count and differential should be obtained because an automated cell counter may provide falsely elevated
cell counts.

The results of revision surgery for a failed metal-on-metal hip prosthesis can be variable. The amount of
local tissue destruction and the integrity of the hip abductor mechanism can greatly influence outcomes.
Instability is the most common complication following revision of failed metal-on-metal hip replacements.

85.A A 55-year-old man is about to undergo right total hip arthroplasty. A preoperative AP pelvis radiograph is
shown in Figure 1. The final acetabular component and polyethylene liner are implanted. With the broach in
place, the surgeon trials a standard offset neck and neutral length femoral head. The leg lengths are
approximately equal, but the hip is unstable. What is the
best next step?

UI // UNAIR // UNPAD // UNHAS // UNS // UGM // UB // UNUD // USU


Adult Reconstructive Surgery of the Hip & Knee Self-Assessment Examination 2019

Figure 1

1. Choosing a longer femoral head and accepting a resulting leg-length discrepancy

2. Trialing a lateralized femoral neck component

3. Removing the acetabular liner and implanting an offset liner instead

4. Performing a trochanteric osteotomy with advancement


Preffered Answer: 2

The radiograph shows that this patient has a high offset varus femoral morphology of both hips. Preoperative
templating would identify this, and the surgeon should choose an implant system that has extended offset
options to help match the native anatomy and biomechanics and minimize the risk of instability. Trialing a
high offset neck, rather than a standard offset neck, is the next most appropriate step. Depending on the
design of the implant system, this step can be accomplished by direct medialization of the femoral head,
which would not affect leg length, or by lowering the neck angle, which would affect the leg length and would
require a longer femoral head, because the leg lengths had previously been equal. Placement of a longer
femoral head would likely improve hip stability but would also make the leg length uneven, which is a
common cause of dissatisfaction after total hip arthroplasty. An offset acetabular liner also increases the leg
length and does not correct the issue, which is on the femoral side. Trochanteric advancement is sometimes
used as a treatment for instability but would be inappropriate as the next step in this setting.

86. A 72-year-old woman is scheduled to undergo right total hip arthroplasty. Her preoperative radiograph is
shown in Figure 1. To avoid increasing this patient’s combined offset while maintaining her leg length, what is
the most appropriate surgical plan?

1.Lateralize the acetabular component, use a low offset femoral component, and make a shorter neck cut

UI // UNAIR // UNPAD // UNHAS // UNS // UGM // UB // UNUD // USU


Adult Reconstructive Surgery of the Hip & Knee Self-Assessment Examination 2019
2.Medialize the acetabular component, use a low offset femoral component, and make a longer neck cut

3.Lateralize the acetabular component, use a high offset femoral component, and make a shorter neck cut

4.Medialize the acetabular component, use a high offset femoral component, and make a longer neck cut
Preffered Answer: 2

The management of patients with proximal femoral deformity can be difficult. Appropriate implant selection
and preoperative templating are critical. In this patient, it would be difficult to avoid increasing the combined
offset by too much, which could contribute to the overtensioning of the soft tissues and trochanteric pain. By
medializing the acetabular component (decreasing the combined offset), using a low offset femoral
component or a cemented component placed more valgus (decreasing the combined offset), and making a
longer neck cut (to avoid shortening of the lower extremity), restoration of the patient’s native offset and leg
length can be achieved.

87. In patients undergoing elective hip or knee arthroplasty who are not at elevated risk (beyond the risk
associated with the surgery) for venous thromboembolism or bleeding, using pharmacologic agents and/or
mechanical compressive devices for the prevention of venous thromboembolism was assigned what grade of
recommendation by the 2011 AAOS Clinical Practice Guideline, Preventing Venous Thromboembolic
Disease in Patients Undergoing Elective Hip and Knee Arthroplasty?

1. Strong

2. Moderate

3. Limited

4. Inconclusive

Preffered Answer: 2

Using pharmacologic agents and/or mechanical compressive devices for the prevention of venous
thromboembolism in patients undergoing elective hip or knee arthroplasty who are not at elevated risk
beyond that of the surgery itself for venous thromboembolism or bleeding was given a moderate grade of
recommendation in the 2011 AAOS Clinical Practice Guideline referenced above.

88. Figure 1 shows the 6-week postoperative AP hip radiograph, and Figure 2 shows the current radiograph
obtained from a 54-year-old avid hiker who returns for routine follow-up 3 years after an uncomplicated,
uncemented modular metal-on-metal hip replacement. He reports mild, diffuse, activity-related aching around
the right hip region but does not feel restricted in his activities. Examination reveals no local tenderness, a
well-healed incision, and mild discomfort at the extremes of rotation. An erythrocyte sedimentation rate (ESR)
and C-reactive protein (CRP) level are obtained and show 9 mm/h (reference range 0 to 20 mm/h) and 2.0
mg/L (reference range 0.08 to 3.1 mg/L), respectively. What is the etiology of the radiographic finding?

UI // UNAIR // UNPAD // UNHAS // UNS // UGM // UB // UNUD // USU


Adult Reconstructive Surgery of the Hip & Knee Self-Assessment Examination 2019

Figure 1 Figure 2

1. Osteolysis secondary to metal particle wear, an adverse reaction to metal debris

2. Osteolysis secondary to loosening of the femoral implant

3. Metastatic lesion to the proximal femur

4. Deep periprosthetic joint


infection Preffered Answer: A

The radiograph shows a large area of osteolysis involving the proximal femur. The implants appear solidly
fixed on radiographs, and in the absence of symptoms, it is unlikely the implants are loose or infected.
Although infection or metastatic disease is a possibility, the normal ESR and CRP values make the
diagnosis of deep infection unlikely. The most likely cause for the radiographic findings is wear from metal
particles, resulting in an adverse local tissue response and osteolysis.

89. Figures 1 and 2 show the radiographs obtained from a 19-year-old woman with a 3-year history of
progressive hip pain in the left groin with activity, which is unresponsive to activity modification and physical
therapy. Examination reveals normal range of motion, with pain on anterior impingement testing. What
treatment is associated with the best long-term results?

UI // UNAIR // UNPAD // UNHAS // UNS // UGM // UB // UNUD // USU


Adult Reconstructive Surgery of the Hip & Knee Self-Assessment Examination 2019

Figure 1 Figure 2

1. Hip arthroscopy with labral repair

2. Reverse periacetabular osteotomy

3. Varus rotational osteotomy

4. Open surgical dislocation with rim trimming


Preffered Answer: 2

This patient has symptomatic femoroacetabular impingement as well as clinical and radiographic signs of
acetabular retroversion, including a cross-over sign, ischial spine sign, and posterior wall sign bilaterally.
Good midterm to long-term outcomes have been reported with reverse (anteverting) Bernese periacetabular
osteotomy (PAO). In patients with less retroversion, open or arthroscopic rim trimming with labral refixation
have shown good short-term results, but longer-term results have yet to be fully delineated.

Isolated hip arthroscopy and labral repair would not be indicated without addressing the retroversion
deformity. Femoral varus rotational osteotomy plays no role in the treatment of this pathology. Open surgical
dislocation with rim trimming could be considered in patients with less deformity, but some studies have
shown inferior long-term results compared with reverse PAO.

90. Injury to the popliteal artery during total knee arthroplasty (TKA) is most likely to occur when placing a sharp
retractor

1. directly posterior to the posterior cruciate ligament (PCL).

2. posteromedial to the PCL.

3. posterolateral to the PCL.

4. in the posteromedial corner of the knee.

Preffered Answer: 3

Vascular complications during TKA are rare but do occur. Traditionally, it was taught that the popliteal artery
was situated posterior to the PCL; however, more recent anatomic dissections have demonstrated that this
artery is usually located posterolateral to the PCL.

91. Figures 1 through 3 show the radiographs obtained from an 86-year-old-woman who has had chronic left hip
pain for several years. She now uses a walker and a wheelchair for ambulation. She is medically healthy.
What is the most appropriate surgical intervention?

UI // UNAIR // UNPAD // UNHAS // UNS // UGM // UB // UNUD // USU


Adult Reconstructive Surgery of the Hip & Knee Self-Assessment Examination 2019

1. Cemented left total hip arthroplasty (THA)

2. Cementless left THA with a proximally porous coated femoral stem

3. Hybrid left THA

4. Cementless left THA with a diaphyseal engaging conical femoral stem


Preffered Answer: 3

This 86-year-old woman has poor bone quality and osteoarthritis of the left hip. Her lateral radiograph
confirms Dorr type C bone quality. A hybrid left THA with a cemented femoral stem would be the treatment of
choice.

92. Figures 1 and 2 show the radiographs obtained from a 68-year old man with progressively worsening right
side hip pain over the last 8 months. He is 6 feet tall, with a BMI of 51 kg/m2 and reports that his index total
hip arthroplasty was performed 8 years ago. The preoperative work-up includes negative infectious laboratory
results. What is the most appropriate surgical plan for revision of the femoral component in this patient?

Figure 1 Figure 2

1. Superior approach with trochanteric slide

2. Direct anterior approach with a chevron modification of the standard greater trochanteric osteotomy

3. Lateral approach with a partial greater trochanter osteotomy

4. Posterolateral approach with an extended trochanteric osteotomy


Preffered Answer: 4

The patient’s radiographs demonstrate varus femoral remodeling around a broken cylindrical, distally fixed
femoral stem. Proximal femoral remodeling around loose or fractured stems occurs in 21% to 42% of
femoral revisions, based on the definitions outlined by Foran and associates. In definition 1, varus femoral

UI // UNAIR // UNPAD // UNHAS // UNS // UGM // UB // UNUD // USU


Adult Reconstructive Surgery of the Hip & Knee Self-Assessment Examination 2019
remodeling occurs when the template falls within 2 mm of the endosteal cortex of the metaphysis on
templating with a diaphyseal engaging stem. In definition 2, varus femoral remodeling = when the template
crosses the lateral femoral cortex proximally. Based on the templating or drawing a line from the isthmus
proximally along the lateral cortex, implantation of a straight stem would perforate the cortex proximally,
indicating varus femoral remodeling. An extended trochanteric osteotomy would aid in the removal of the
well-fixed distal segment and enable the safe insertion of the new femoral component. The approach is not
the concern in this case, because extended trochanteric osteotomies have been described from the
posterior and direct lateral approaches with excellent outcomes and union rates. The key is that the
extended osteotomy is necessary and not a trochanteric slide or standard (shorter or incomplete
trochanteric) osteotomy. These types would not provide access to the well-fixed distal stem, nor would they
afford a straight tube in which to insert a new femoral component.

93.A A 57-year-old woman experiences pain 1 year after total knee arthroplasty (TKA). She reports sharp
anterior pain and a painful catching sensation that is aggravated by rising from a chair or climbing stairs.
Physical examination reveals a mild effusion and a range of motion of 2° to 130°, with patellar crepitus. The
symptoms are reproduced by resisted knee extension. Radiographs show a well-aligned posterior-
stabilized TKA without evidence of component loosening.

What is the most likely cause of this patient's pain?

1. Patellar clunk syndrome

2. Flexion gap instability

3. Polyethylene wear

4. Femoral component malrotation

Preffered Answer: 1

Patellar clunk syndrome is caused by the development of a fibrous nodule on the posterior aspect of the
quadriceps tendon at its insertion into the patella. It causes a painful catching sensation when the extensor
mechanism traverses over the trochlear notch as the knee extends from 45° of flexion to 30° from full
extension. It characteristically occurs in posterior stabilized total knee arthroplasties and appears to be
related to femoral component design. The syndrome can usually be prevented by excising the residual
synovial fold just proximal to the patella. Flexion gap instability can also cause a painful total knee
arthroplasty but is less common in posterior stabilized implants. Femoral component malrotation can cause
pain attributable to a flexion gap imbalance or patellar tracking problems. Polyethylene wear would be
unlikely after just 1 year. Patellar clunk syndrome can usually be addressed successfully with arthroscopic
synovectomy. Recurrence is uncommon. Physical therapy may help to strengthen the quadriceps following
synovectomy but would not resolve the clunk syndrome symptoms. Femoral or tibial insert revision is not
indicated if patellar clunk syndrome is the only problem resulting in a painful total knee arthroplasty.

94. A 57-year-old woman experiences pain 1 year after total knee arthroplasty (TKA). She reports sharp anterior
pain and a painful catching sensation that is aggravated by rising from a chair or climbing stairs. Physical
examination reveals a mild effusion and a range of motion of 2° to 130°, with patellar crepitus. The symptoms
are reproduced by resisted knee extension. Radiographs show a well-aligned posterior-stabilized TKA
without evidence of component loosening.
What is the recommended treatment for this patient?

UI // UNAIR // UNPAD // UNHAS // UNS // UGM // UB // UNUD // USU


Adult Reconstructive Surgery of the Hip & Knee Self-Assessment Examination 2019
1. Physical therapy

2. Arthroscopic synovectomy

3. Tibial insert revision

4. Femoral component revision

Preffered Answer: 2

Patellar clunk syndrome is caused by the development of a fibrous nodule on the posterior aspect of the
quadriceps tendon at its insertion into the patella. It causes a painful catching sensation when the extensor
mechanism traverses over the trochlear notch as the knee extends from 45° of flexion to 30° from full
extension. It characteristically occurs in posterior stabilized total knee arthroplasties and appears to be
related to femoral component design. The syndrome can usually be prevented by excising the residual
synovial fold just proximal to the patella. Flexion gap instability can also cause a painful total knee
arthroplasty but is less common in posterior stabilized implants. Femoral component malrotation can cause
pain attributable to a flexion gap imbalance or patellar tracking problems. Polyethylene wear would be
unlikely after just 1 year. Patellar clunk syndrome can usually be addressed successfully with arthroscopic
synovectomy. Recurrence is uncommon. Physical therapy may help to strengthen the quadriceps following
synovectomy but would not resolve the clunk syndrome symptoms. Femoral or tibial insert revision is not
indicated if patellar clunk syndrome is the only problem resulting in a painful total knee arthroplasty.

95. Figures 1 and 2 depict the radiographs obtained from a 76-year-old woman who comes to the emergency
department after experiencing a fall. She is an unassisted community ambulator with a history of right hip
pain. What is the most appropriate surgical treatment for this fracture?

UI // UNAIR // UNPAD // UNHAS // UNS // UGM // UB // UNUD // USU


Adult Reconstructive Surgery of the Hip & Knee Self-Assessment Examination 2019

Figure 2

Figure 1

1. Cemented unipolar hemiarthroplasty

2. Cemented bipolar hemiarthroplasty

3. Total hip replacement

4. Open reduction and internal fixation


Preffered Answer: 3

This patient has pre-existing right hip osteoarthritis. The most correct option for the treatment of this active
patient is a right total hip arthroplasty. Hemiarthroplasty would not address the patient's pain from
osteoarthritis, and open reduction and internal fixation would not fix the femoral head issue or the
osteoarthritis.

96. Figures 1 through 3 depict the AP and lateral radiographs obtained from a 64-year-old man with long-
standing right knee osteoarthritis and pain that is unresponsive to nonsurgical treatment. The patient
undergoes navigated cruciate-retaining right total knee arthroplasty.
After surgery, this patient continues to experience pain and swelling of the knee with recurrent effusions.
He returns to the office reporting continued pain 2 years after surgery. He describes instability, particularly
when descending stairs. On examination, range of motion of 0° to 120° is observed, with no extensor lag.
Slope of the tibial component is 7°. The knee is stable to varus and valgus stress in extension, but flexion
instability is present in both the anterior-posterior direction and the varus-valgus direction. Bracing leads to
a slight decrease in symptoms but is not well tolerated. Isokinetic testing demonstrates decreased knee
extension velocity at mid push. Radiographs demonstrate well-aligned and fixed knee implants. An
infection work-up is negative. What is the most appropriate surgical intervention at this time?

UI // UNAIR // UNPAD // UNHAS // UNS // UGM // UB // UNUD // USU


Adult Reconstructive Surgery of the Hip & Knee Self-Assessment Examination 2019

Figure 1 Figure 2 Figure 3

1. Tibial polyethylene exchange

2. Revision of the femoral and tibial components and conversion to a posterior stabilized insert

3. Revision of the femoral and tibial components to a constrained rotating hinge prosthesis

4. Isolated femoral component revision and upsizing of the femoral implant with a new posterior cruciate
ligament (PCL)-retaining polyethylene insert

Preffered Answer: 2

The patient’s symptoms at follow-up—pain, swelling, and difficulty descending stairs—suggest knee flexion
instability. Considering his history, an incompetent PCL must be considered. Revision of the knee to a
posterior stabilized or nonlinked constrained condylar implant (depending on the condition of the ligaments)
likely is needed to address his symptoms. The difference in extension stability and flexion stability makes
polyethylene exchange a poor option. A constrained rotating hinge design is not necessary. Repeat use of a
PCL-retaining insert is not recommended. Tibial and femoral revision both are required. Correction of
excessive slope will be attained with tibial revision, femoral component revision is required to convert to a
PCL-substituting design. There is also an opportunity to increase posterior condylar offset if needed.

97.A A 59-year-old active woman undergoes elective total hip replacement in which a posterior approach is
used. She has minimal pain and is discharged to home 2 days after surgery. Four weeks later, she
dislocates her hip while shaving her legs. She undergoes a closed reduction in the emergency department.
Postreduction radiographs show a reduced hip with well-fixed components in satisfactory alignment.
What is the most appropriate management of this condition from this point forward?

1. Observation and patient education regarding hip dislocation precautions

2. Revision to a larger-diameter femoral head

3. Revision to a constrained acetabular component

4. Application of a hip orthosis for 3 months

Preffered Answer: 1

UI // UNAIR // UNPAD // UNHAS // UNS // UGM // UB // UNUD // USU


Adult Reconstructive Surgery of the Hip & Knee Self-Assessment Examination 2019
First-time early dislocations are often treated successfully without revision surgery, especially when no
component malalignment is present. In this clinical scenario, it appears the patient would benefit from better
education about dislocation precautions. Hip orthoses are of questionable benefit unless the patient is
cognitively impaired. Revision surgery can be successful but is usually reserved for patients with recurrent
dislocations.

98. Venous thromboembolism may occur after total joint arthroplasty. The risk of this complication is elevated in
patients with

1. a BMI lower than 30.

2. diabetes mellitus, with a hemoglobin A1c test result less than 7.

3. tranexamic acid use.

4. metabolic syndrome.

Preffered Answer: 4

Obesity, a prior history of venous thromboembolism, and metabolic syndrome have all been associated with
an increased risk of thromboembolism. A recent meta-analysis showed that diabetes had no significant
relationship with venous thromboembolism following hip or knee arthroplasty. Tranexamic acid is an
antifibrinolytic agent that has been shown to reduce blood loss substantially following hip and knee
arthroplasty. It has also been shown to be safe in patients with severe medial comorbidities and a prior
history of venous thromboembolism.

99. Figures 1 and 2 demonstrate the radiographs obtained from a 63-year-old man who had right total hip
arthroplasty (THA) 4 months ago. Progressive stiffness began 2 months after surgery, and he now reports
pain only after prolonged physical activity. His examination reveals a normal gait and painless range of
motion with flexion of 70°, extension of 0°, internal rotation of 20°, external rotation of 20°, abduction of 10°,
and adduction of 10°. His erythrocyte sedimentation rate and C-reactive protein level are within defined limits.
Physical therapy has produced no benefit. What is the most appropriate next step?

Figure 1 Figure 2

1. 25 mg of indomethacin 3 times daily for 6 weeks

2. 1 dose of irradiation at 800 Gy

UI // UNAIR // UNPAD // UNHAS // UNS // UGM // UB // UNUD // USU


Adult Reconstructive Surgery of the Hip & Knee Self-Assessment Examination 2019
3. Surgical excision of heterotopic ossification (HO)

4. Reevaluation in 6 months
Preffered Answer: 4

This patient presents with HO 4 months after undergoing THA. Symptomatic HO may complicate nearly 7%
of primary THA cases. Improvement in pain is expected within 6 months, and most patients will not need
surgical treatment. Surgical excision may be warranted for symptomatic patients after full maturation of the
HO, usually 6 to 18 months after the surgery. Patients can be followed with repeated serum alkaline
phosphatase levels, which are elevated initially and should return to normal upon maturation of the HO.
Alternatively, a bone scan can show decreased activity after the HO has matured. Twenty-five milligrams of
indomethacin 3 times daily for 6 weeks or 1 dose of irradiation at 700 to 800 Gy is effective in the prevention
of HO but not for the treatment of established HO.

100. An 85-year-old obese woman has left knee pain. She had surgery 5 years ago for a patellar nonunion
after total knee arthroplasty that was complicated by infection, which was treated with implant removal and
patellectomy. She has not been ambulatory since then. She states she is no longer on antibiotics. She has
moderate pain, but her primary problem is instability of the knee. She has a 40° extensor lag. Darkening of
the skin is present distal to the incision consistent with venous stasis changes. The erythrocyte sedimentation
rate is 12 mm/h (reference range 0 to 20 mm/h) and her C-reactive protein level is 1.0 mg/L (reference range
0.08 to 3.1 mg/L). Left knee aspiration shows a white blood cell count of 800 and 20% neutrophils. What is
the best next step?

1. Revision total knee arthroplasty with primary quadriceps tendon repair

2. Hinged knee arthroplasty with full extensor mechanism allograft

3. Arthrotomy with debridement and antegrade knee arthrodesis nailing

4. Two-stage revision knee arthroplasty and quadriceps repair with Achilles allograft

Preffered Answer: 3

This patient is elderly, obese, and nonambulatory and has a chronic quadriceps tendon rupture after infected
total knee arthroplasty. Her potential for ambulation after revision total knee arthroplasty is very low. Primary
repair of the tendon is unlikely to be successful, even with augmentation, so revision total knee arthroplasty
with primary quadriceps tendon repair and two-stage revision knee arthroplasty and quadricep repair with
Achilles allograft are not the best management techniques. Extensor mechanism allograft could be done but
would have a high failure rate in a patient of this size. No sign of infection is seen, based on laboratory
studies, so a two-stage procedure is not necessary. The best management although not optimal, would be
treatment in a drop-lock brace. Arthrodesis is also an option, but would have a high complication rate, and in
a patient that is nonambulatory, a fused knee would be increasingly difficult with activities of daily living and
mobility.

101. A 59-year-old woman with a history of osteoporosis is involved in a high-speed motor vehicle accident,
resulting in left hip pain and deformity. The initial radiograph from the trauma bay is shown in Figure 1.
Postreduction CT is shown in Figures 2 through 4. What is the most appropriate definitive surgical treatment?

UI // UNAIR // UNPAD // UNHAS // UNS // UGM // UB // UNUD // USU


Adult Reconstructive Surgery of the Hip & Knee Self-Assessment Examination 2019

1. Open reduction and internal fixation (ORIF) of the acetabular fracture with concomitant acute total hip
arthroplasty

2. ORIF of the acetabular fracture and ORIF of the femoral head fracture fragments

3. ORIF of the acetabular fracture and hemiarthroplasty

4. Skeletal traction with delayed total hip arthroplasty after the acetabular fracture has healed
Preffered Answer: 1

The radiograph shows a posterior wall acetabular fracture-dislocation. Postreduction CT indicates a large
comminuted posterior wall fracture with marginal impaction of the articular surface. A comminuted femoral
head fracture also is seen extending to the superior weight-bearing surface. Given the marginal impaction of
the acetabulum and the considerable comminution of the femoral head (which is likely unreconstructible),
this injury would have a high risk of causing posttraumatic arthritis if treated with ORIF of the fractures alone.
Considering this fact and the patient’s age, ORIF of the posterior wall and acute total hip arthroplasty would
likely have the best functional result and the lowest risk of reoperation. Hemiarthroplasty is inappropriate for
this injury considering the acetabular fracture. Skeletal traction currently plays a limited role in the definitive
treatment of acetabular fractures.

102. A 72-year-old patient fell 3 weeks after undergoing a total hip arthroplasty using cementless fixation of
the femoral component. She sustained a comminuted Vancouver type B-2 fracture with displacement of the
calcar fragment. What is the best treatment option?

UI // UNAIR // UNPAD // UNHAS // UNS // UGM // UB // UNUD // USU


Adult Reconstructive Surgery of the Hip & Knee Self-Assessment Examination 2019
1. Revision using a proximal femoral replacement prosthesis

2. Revision using a diaphyseal engaging femoral prosthesis along with cerclage fixation

3. Open reduction internal fixation using a locking plate with strut graft

4. Protected weight bearing with abduction bracing

Preffered Answer: 2

The patient has an acute postoperative fracture of the proximal femur with subsidence. It is also common that
the stem retroverts relative to the femur. It is most often seen in proximally porous coated stems within 90
days of surgery, one paper found it to occur 0.7% of the time in modern implants. There is always a debate
whether this is a missed intraoperative fracture, or a new fracture that has resulted from an event of
increased hoop stresses. Removal of the primary stem, placement of a diaphyseal engaging stem (most
frequently a tapered-fluted stem), and cabling of the fracture is the most successful treatment.

103. Figure 1 depicts the radiograph obtained from a 30-year-old woman who began having more right than left
hip pain during a recent pregnancy. Physical examination reveals increased range of motion with positive
flexion abduction and external rotation and flexion adduction and internal rotation as well as pain with
external logroll. Assessment of Figure 1 reveals

Figure 1

1. classic dysplasia with volume deficient acetabula.

2. acetabular retroversion with positive crossover signs and ischial spine signs.

3. no substantial dysplasia, with normal acetabular volume and anteversion.

4. inadequate radiographic evidence to assess for hip dysplasia.


Preffered Answer: 4

Studies have demonstrated that pelvic inclination can dramatically affect the interpretation of radiographs in
the dysplastic hip, with 9° of increased pelvic inclination leading to the presence of crossover signs and
posterior wall signs. A distance of 30 mm to 50 mm from the sacrococcygeal junction to the pubis is often
used to assess the adequacy of pelvic inclination on radiographs, although Siebenrock and associates
determined the mean difference to be 32 mm in men and 47 mm in women. In this patient, the pelvic
inclination is dramatically increased, leading to overestimation of acetabular retroversion.

UI // UNAIR // UNPAD // UNHAS // UNS // UGM // UB // UNUD // USU


Hand and Wrist Self-Assessment Examination 2019

2019
SELF
ASSESSMENT
EXAMINATION

HAND AND WRIST


CONTRIBUTED BY:
CANDIDATES OF
INDONESIAN ORTHOPAEDIC &
TRAUMATOLOGY SURGEONS COMPILED BY
PERIOD OF JUNE 2019

UI // UNAIR // UNPAD // UNHAS // UNS // UGM // UB // UNUD // USU


Hand and Wrist Self-Assessment Examination 2019

1. The pathology of the lesion shown in Figures 1 and 2 reveal what cellular pattern?

Figure 1 Figure 2

1. Uniform distribution of stromal cells and giant cells


2. Mixture of mature fat cells and spindle cells
3. Mucin-filled space with occasional spindled fibroblasts
4. Lobular pattern of vascular proliferation with inflammation
Preferred Answer: 3

The MRIs display the classic appearance of a ganglion cyst, which is a benign fluid-filled sac
most commonly presenting at the wrist level. The signal characteristics of a ganglion on MRI are
low signal on the T1-weighted image and high signal on the T2-weighted image, as revealed in
Figures 1 and 2. The pathology of a ganglion cyst is characterized by a mucin-filled synovial lined
sac with spindled fibroblasts.
A uniform distribution of stromal cells and giant cells would fit the pathologic description of a giant
cell tumor of the tendon sheath, and numerous giant cells are the hallmark of this lesion. This
type of tumor is benign, usually presenting in the region of the digital flexor sheath. Recurrence
after excision is common. MRI of giant cell tumors shows an extra-articular soft-tissue mass, and
on both T1-weighted and T2-weighted images, some portions of the tumor show decreased
signal intensity.
A mixture of mature fat cells and spindle cells would fit the pathologic description of a spindle cell
lipoma. This benign tumor of mature fat usually is found in a superficial/subcutaneous location.
The pathology is characterized by a mixture of mature fat cells and spindle cells. MRI would
show high signal intensity on both T1-weighted and T2-weighted images.
A lobular pattern of vascular proliferation with inflammation would fit the pathologic description of
a pyogenic granuloma. These skin growths are small, round, and usually blood red in color. They
frequently bleed, because they contain a large number of blood vessels. The pathology is
characterized by a lobular pattern of vascular proliferation with inflammation. MRI findings show
isointensity on the T2-weighted image. These MRI findings are consistent with hypervascularity.

2. Figures 1 and 2 are the radiographs of a 35-year-old right-hand-dominant man who has had
progressive right wrist pain for 1 year. There is no history of trauma, and he has had no
treatment to date. He reports some pain at rest with limited motion and substantial pain with use.
He is currently out of work on short-term disability because of this wrist problem. An examination
reveals mild dorsal wrist swelling, decrease wrist range of motion, and decreased grip strength.
Contralateral wrist examination findings are normal. What is the most appropriate course of
treatment?

UI // UNAIR // UNPAD // UNHAS // UNS // UGM // UB // UNUD // USU


Hand and Wrist Self-Assessment Examination 2019

Figure 1 Figure 2

1. Proximal row carpectomy


2. Radial shortening osteotomy and vascularized bone grafting
3. Scaphoid excision and midcarpal arthrodesis
4. Capitate hamate fusion
Preferred answer: 1

This patient has late-stage Kienböck disease. According to the Lichtman classification for
Kienböck disease, this would represent stage IIIB, with lunate collapse/fragmentation, loss of
carpal height secondary to proximal capitate migration, and a flexed scaphoid. The lateral
radiograph reveals a radioscaphoid angle exceeding 60 degrees, so disease stage is IIIB.
According to Condit and associates, when the presurgical radioscaphoid angle exceeds 60
degrees, results are poor when an attempt to maintain the lunate is made. As a result, the
procedure with the most predictable outcome is a proximal row carpectomy. A radial-shortening
osteotomy could be performed because the ulnar variance is negative. Considering the marked
lunate fragmentation and collapse, a vascularized bone graft likely is contraindicated and
associated with less predictable results than a proximal row carpectomy. There is no role for
supervised hand therapy and splinting in the setting of advanced Kienböck disease. Similarly,
there is no role for maintenance of the lunate in the setting of advanced collapse and
fragmentation.

3. Assuming that the fracture shown in this radiograph (Figure 1) is aligned on the anteroposterior
radiograph and heals in this position, secondary to fracture malalignment, there will be loss of
active

Figure 1

1. metacarpophalangeal (MP) joint extension


2. proximal interphalangeal (PIP) joint extension.

UI // UNAIR // UNPAD // UNHAS // UNS // UGM // UB // UNUD // USU


Hand and Wrist Self-Assessment Examination 2019

3. MP flexion.
4. PIP joint flexion.
Preferred Answer: 2

This is a transverse proximal phalanx fracture with apex volar angulation. The fracture displaces
into an apex volar angulated position under the pull of the central slip on the distal fragment and
the interossei insertions at the base of proximal phalanx. Although it is possible to lose motion in
flexion or extension of the MP or PIP joints, the biomechanics will not allow full extension of the
PIP joint. If allowed to heal in apex palmar malunion, the predicted corresponding extensor lags
are for a 10-degree lag at 16 degrees of angular deformity, a 24-degree lag at 27 degrees of
deformity, and a 66-degree lag at 46 degrees of deformity. These fractures usually can be
treated with closed reduction with or without percutaneous pinning. With surgical treatment, there
may be loss of motion both at the MP and PIP joints.

4. Figures 1 through 3 are the radiographs of a 27-year-old man who has had wrist pain since
falling 1 day ago. Which treatment offers the best prognosis for prevention of carpal collapse and
progressive arthritis?

Figure 1 Figure 2

Figure 3

1. Long-arm cast
2. Percutaneous screw fixation
3. Open reduction and internal fixation (ORIF) with bone graft
4. Proximal row carpectomy

Preferred Answer: 3

UI // UNAIR // UNPAD // UNHAS // UNS // UGM // UB // UNUD // USU


Hand and Wrist Self-Assessment Examination 2019

Although this patient’s history includes a recent fall, the radiographs show evidence of a
scaphoid nonunion with carpal collapse but no arthritis. Obtaining union of the scaphoid is
important to prevent progressive carpal collapse and arthritic changes. ORIF with bone graft is
most appropriate to obtain union and correct the collapse deformity. Screw fixation with volar
wedge graft often is performed to realign a scaphoid humpback deformity, although cancellous
bone graft also is a reasonable option. Vascularized bone graft is considered for a nonunion of
long duration, avascular necrosis of the proximal pole, and failed prior surgery. Cast
immobilization will not lead to union of the scaphoid. Percutaneous screw fixation is not indicated
for the treatment of a displaced nonunion. A proximal row carpectomy is a salvage procedure
and is not indicated for this patient because there are no arthritic changes.

5. Figures 1 and 2 show the clinical photograph and ultrasonography image obtained from an 8-
month-old boy who has a 2-month history of a well-circumscribed mass in the palm, just proximal
to the palmar digital crease of the index finger. The mass has not changed in size and does not
seem to cause pain. What is the best next step in treatment?

Figure 1 Figure 2

1. rasonography-guided aspiration
2. Excisional biopsy
3. Observation
4. MRI of the hand for further characterization of the mass
Preferred answer: 3
The history, clinical photograph, and ultrasonography evaluation point to a diagnosis of ganglion
cyst. The likelihood of resolution of ganglion cysts without intervention in pediatric patients
ranges from 66-79%. Therefore the most appropriate treatment at this time is observation.
Because the diagnosis is confirmed with the information provided, excisional biopsy and further
imaging are not necessary.

6. What sign or symptom may occur with cubital tunnel syndrome that does not occur with Guyon
neuropathy?

1. Abnormal sensation of the dorsal ulnar hand


2. A positive Froment sign
3. Abnormal sensation in the volar ring and small fingers
4. Weakness of the interosseous muscles

UI // UNAIR // UNPAD // UNHAS // UNS // UGM // UB // UNUD // USU


Hand and Wrist Self-Assessment Examination 2019

Preferred Answer:3

Ulnar neuropathy at the elbow is termed cubital tunnel syndrome, whereas ulnar nerve
compression at the wrist is considered Guyon neuropathy. Patients with cubital tunnel syndrome
have numbness on the dorsal ulnar aspect of the hand due to involvement of the dorsal ulnar
sensory nerve branch (DUSN). Ulnar neuropathy at both the elbow and the wrist may manifest
with abnormal sensation about the volar ring and small fingers and with weakness of the
interosseous muscles, which can lead to a positive Froment sign. The Froment sign is
considered positive when flexion of the thumb interphalangeal joint occurs to compensate for a
lack of adductor function. Patients with a Guyon neuropathy do not have symptoms of numbness
in the dorsal ulnar distribution, because the DUSN branch arises more proximally in the forearm
and is not compressed in the ulnar tunnel at the wrist.

7. Figure 1 shows the radiograph obtained from a 54-year-old woman with rheumatoid arthritis who
has thumb pain and dysfunction. Nonsurgical treatment, including splinting, oral NSAIDs, activity
modification, and steroid injections, has failed. What is the most appropriate surgical
intervention?

Figure 1

1. Thumb carpometacarpal (CMC) arthroplasty with ligament suspensionplasty


2. Thumb CMC and thumb metacarpophalangeal (MCP) joint fusion
3. Thumb CMC arthroplasty with ligament suspensionplasty and thumb MCP joint stabilization
4. Trapezial resection and distraction arthroplasty

PreferredAnswer: 3

Various options exist to treat thumb CMC arthritis: trapezial resection alone, trapezial resection
with ligament suspensionplasty or tendon interposition, trapezial resection with both ligament
suspensionplasty and tendon interposition, CMC fusion, and CMC replacement.

MCP hyperextension can develop in long-standing CMC arthritis, contributing to CMC instability
as well as thumb pain and weakness. In patients with concomitant MCP hyperextension that
exceeds 30°, correction of the deformity of the MCP joint must also be addressed and can be

UI // UNAIR // UNPAD // UNHAS // UNS // UGM // UB // UNUD // USU


Hand and Wrist Self-Assessment Examination 2019

done with MCP capsulodesis, extensor pollicis brevis tendon transfer, or MCP fusion. Fusion of
both the thumb CMC and MP joints is not recommended as this would result in marked stiffness
and dysfunction.

8. Video 1 demonstrates a unilateral "piano key" sign, which indicates

1. distal radioulnar joint (DRUJ) instability.


2. interosseous membrane disruption.
3. midcarpal instability.
4. physiologic motion of hypermobility syndrome.
Preferred answer: 1

The piano key sign is a demonstration of instability at the DRUJ, typically seen after healing from
a distal radius fracture with an associated ulnar styloid fracture (as in this case) or other wrist
injury. The hand is pushed down against a table top, and the distal radius translates dorsally
(with the distal ulna apparently moving volarly). In fact, the distal radius is the mobile segment,
while the distal ulna is fixed in space. Treatment involves repair or reconstruction of the foveal
insertion of the triangular fibrocartilage complex (TFCC) and distal radioulnar ligaments.

This type of instability is also common in malunions of the distal radius or distal one-third of the
radial shaft (Galeazzi-type fractures). In malunions, DRUJ instability can be treated with a
corrective osteotomy of the distal radius to restore the anatomic relationship between the distal
ulna and the distal radius at the DRUJ.

Radiocarpal and midcarpal instability do not involve the DRUJ. Disruption of the interosseous
membrane (in isolation, with intact distal radioulnar ligaments and an intact TFCC) does not lead
to translational instability of the DRUJ. Although hypermobility syndrome may lead to
ligamentous laxity, it does not lead to unilateral DRUJ instability.

9. Figures 1 and 2 are the radiographs of a 36-year-old man who has had left wrist pain for the past
6 months following a fall onto his outstretched arm. Examination reveals a positive ballottement
test, dorsal and ulnar carpal tenderness, and a painful snap with ulnar deviation, pronation, and
axial compression. Injury to what ligament is the cause of this patient's pain?

UI // UNAIR // UNPAD // UNHAS // UNS // UGM // UB // UNUD // USU


Hand and Wrist Self-Assessment Examination 2019

Figure 1 Figure 2

1. Short radiolunate
2. Dorsal scapholunate interosseous
3. Volar lunotriquetral interosseous
4. Radioscaphocapitate
Answer: Volar lunotriquetral interosseous

The radiographs reveal a volarly tilted lunate on the lateral view (Figure 1) and an incongruous
lunotriquetral articulation on the AP gripped view (Figure 2). The patient has what appears to
be radiographic findings of volar intercalated segmental instability (VISI), a type of carpal
instability, dissociative. An injury to the volar lunotriquetral ligament is the most important
contributor to this type of instability. An injury to the dorsal scapholunate ligament typically
leads to a dorsal intercalated segmental instability. Isolated injuries to the dorsal radiocarpal
ligament and the radioscaphocapitate ligament do not lead to VISI, although combined injuries
may lead to instability between the radius and the proximal row.

10. A 25-year-old man has an isolated flexor digitorum profundus laceration just proximal to the
distal interphalangeal (DIP) flexion crease of his ring finger. The tendon ends are trimmed,
removing 10 mm from each end (secondary to fraying) and the tendon repaired. Four months
later, he reports limited finger motion of the long, ring, and small fingers. He cannot fully extend
his wrist and all joints of the 3 fingers simultaneously. He has full passive flexion but cannot
actively completely close his fingers into a fist. What is the most likely cause?

1. Quadrigia
2. Intrinsic tightness
3. Lumbrical plus deformity
4. Disruption of the tendon repair
PreferredAnswer: 1

If a single flexor digitorum profundus (FDP) tendon is debrided more than 1 cm prior to repair,
the tendon is advanced too far distally, essentially shortening the musculotendon unit. The
finger will likely develop a flexion posture. Because of the common muscle belly and
interconnections of the profundi, the long and small fingers adjacent to the injured finger will be
affected because of loss of some of their normal proximal excursion. The result is an inability of

UI // UNAIR // UNPAD // UNHAS // UNS // UGM // UB // UNUD // USU


Hand and Wrist Self-Assessment Examination 2019

the adjacent fingers to completely flex. This condition, known as quadrigia, is named after the
Roman chariot driver who held control of the reins of 4 horses, forcing them to move as 1.
Quadrigia occurs when the FDP tendon is advanced too far distally, when a tendon graft is too
short, or when the profundus is sutured over the end of an amputated digit.

Intrinsic muscles of the hand flex the metacarpophalangeal (MP) joints and extend the PIP
joint. Intrinsic tightness causes decreased PIP flexion when the MP joint is in extension. The
lumbrical muscle modulates tension on the flexor profundus tendon. When a tendon graft to
repair the profundus tendon is too long, a lumbrical plus deformity occurs. This is a paradoxical
PIP extension as the finger is flexed. Disruption of the tendon repair causes limited flexion of
the injured finger.

11. Figure 1 shows the radiograph obtained from a 67-year-old woman who has progressive wrist
pain. She undergoes a salvage motion-sparing surgery that relies on the intact cartilage of the
capitate head. It is necessary to preserve what structure during this procedure?

Figure 1

1. Long radiolunate ligament


2. Radioscaphocapitate ligament
3. Dorsal radiocarpal ligament
4. Dorsal intercarpal ligament
Preferred Answer: 2

The radioscaphocapitate ligament must be preserved in cases of proximal row carpectomy


or scaphoidectomy with four-corner fusion. Failure to do so can result in ulnar translocation of the
carpus. The attachment of the long radiolunate ligament to the lunate is compromised in proximal
row carpectomy, although it is left intact in scaphoidectomy with four-corner fusion. Preservation
of the dorsal radiocarpal ligament through a limited arthrotomy is advocated by Ozyurekoglu and
Turker as a method of preserving the blood and nerve supply of the carpus, but this technique
has not been proven to be required. The authors did cut through the dorsal intercarpal ligament
in their exposure.

UI // UNAIR // UNPAD // UNHAS // UNS // UGM // UB // UNUD // USU


Hand and Wrist Self-Assessment Examination 2019

12. At which joint do degenerative changes occur first in a patient with chronic, untreated
scapholunate dissociation?

1. Radioscaphoid
2. Radiolunate
3. Scapholunate
4. Capitolunate

Preferred answer: 1

Stage I of scapholunate advanced collapse (SLAC) is characterized by the presence of


radioscaphoid arthritis. A predictable pattern exists of the progression of degenerative changes
for SLAC wrist, including stage I (radial styloid involvement at the scaphoid fossa), stage II
(scaphoid and entire scaphoid facet involvement), stage III (degeneration between the capitate
and lunate), and stage IV (pancarpal involvement). The radiolunate joint is often spared.

13. Figures 1 through 3 are the radiographs of a 55-year-old woman who underwent a volar plating
of an extra-articular distal radius fracture 2 weeks ago. She is experiencing weakness with
flexion of the interphalangeal (IP) thumb joint. IP joint flexion was normal before surgery. What is
the best next step?

Figure 1 Figure 2 Figure 3

1. Observation
2. Electromyogram/nerve conduction study (EMG/NCS)
3. CT scan
4. Immediate exploration

Preferred Answer: 1

Prevalence of flexor tendon rupture after distal radius fracture is between 2% and 12%. The
FPL tendon is the most common flexor tendon rupture associated with volar plating. It is usually
seen with plates that are distal to the watershed line (W) and with plates extending volar to the
critical line (C) (Figure 4). The watershed line (W) is the location of the origin of the volar carpal
ligaments and the bone prominence at which flexor tendons are most closely opposed to the
distal radius (Figure 4). In this scenario, the lateral radiograph shows that the plate is not distal
to the watershed line (W) and is between the critical line (C) and the line parallel to the volar
cortex of the radius (R). This is the optimal position for the plate. Placement of a volar locking
plate distal to the watershed line of the distal radius and excessive plate prominence has been
associated with FPL tendon rupture.

UI // UNAIR // UNPAD // UNHAS // UNS // UGM // UB // UNUD // USU


Hand and Wrist Self-Assessment Examination 2019

This patient is only 2 weeks past surgery and there is some FPL function. FPL weakness after
volar distal radius plating is common and has been seen in as many as 50% of patients. This
usually recovers spontaneously by 2 months, and no treatment is needed. A nerve conduction
study would be indicated if an anterior interosseous nerve compression were considered, but it
is too early for this test. A CT scan could be obtained to judge the alignment of the fracture
fragment and position of the screws, but it is not indicated in this case. Exploration could be
performed if an FPL rupture were considered, but, because it is only 2 weeks after surgery,
there is some FPL function, the plate is proximal to the watershed line, and immediate
exploration is not indicated. If this does not improve after 2 to 3 months, further investigation
with ultrasound or MRI would be indicated.

14. The decision to perform fasciotomy of the fingers for a hand compartment syndrome is most
appropriately made using

1. clinical examination.
2. invasive pressure measurement.
3. arterial Doppler study.
4. MRI.

Preferred answer: 1

Compartment syndrome of the hand can result from a variety of factors, including a traumatic
event such as crush injury, fracture, vascular insult, a high-pressure injection injury, or an insect
or spider bite. The treatment involves decompressive fasciotomy of the involved compartments.
The diagnosis of hand compartment syndrome is determined by history, examination, and
objective testing. Patients experience pain out of proportion to the injury, along with swelling and
tense skin. Pain may occur with passive motion of the metacarpophalangeal joints as the intrinsic
muscles are stretched. Invasive intracompartmental pressures can be measured in the
compartments of the hand but not in the fingers. Arterial Doppler studies assess arterial blood
flow, and an abnormality would be a late finding. MRI would show edema of the hand and
fingers, but the decision to perform surgical release is less likely made from the findings. The
most appropriate method of determining the need for finger fasciotomy is the history and physical
examination.

15. Figure 1 is the radiograph of an 18-year-old, right hand-dominant man who has right side thumb
pain after a tackle during a rugby game. Examination shows ecchymosis and swelling of the right
thumb along with tenderness to palpation about the thumb CMC joint and metacarpal base. What
ligament is holding the small fracture fragment in anatomical location to the trapezium?

UI // UNAIR // UNPAD // UNHAS // UNS // UGM // UB // UNUD // USU


Hand and Wrist Self-Assessment Examination 2019

Figure 1

1. Posterior oblique
2. Dorsal radial
3. Anterior oblique
4. Dorsal trapeziometacarpal
Preferred Answer: 3

Bennett fractures are defined as intra-articular thumb metacarpal base fractures. The fracture is
often caused by axial loading, and concomitant injuries to the thumb MCP joint and trapezium
are common.
The palmar ulnar aspect of the base of the metacarpal stays in place through its attachment to
the trapezium by way of the anterior oblique ligament. The metacarpal shaft is displaced
dorsally, proximally, and radially by the pull of the abductor pollicis longus, extensor pollicis
brevis, extensor pollicis longus, and adductor pollicis brevis. These fractures are often
considered unstable and are treated surgically.

16. A 45-year-old woman has a distal radius fracture, which is treated with open reduction and
internal fixation. The surgery was uncomplicated, and the patient is discharged to home. At the
first follow-up appointment, the patient demonstrates signs that are concerning for complex
regional pain syndrome (CRPS). What factor is included in the International Association for the
Study of Pain (IASP) criteria (Budapest criteria) for the diagnosis of CRPS?

1. Hypoesthesia
2. Elevated white blood cell count
3. Elevated C-reactive protein level
4. Pain disproportionate to the inciting event
Preferred answer: 4

The diagnosis of CRPS is complex. The IASP has approved diagnostic criteria to standardize
both the diagnosis and the study of CRPS. The criteria are:
1. Continuing pain disproportionate to any inciting event
2. At least one symptom in three of the following four categories
1. Sensory: reports of hyperesthesia and/or allodynia
2. Vasomotor: reports of temperature asymmetry and/or skin color changes and/or
skin color asymmetry

UI // UNAIR // UNPAD // UNHAS // UNS // UGM // UB // UNUD // USU


Hand and Wrist Self-Assessment Examination 2019

3. Sudomotor/edema: reports of edema and/or sweating changes and/or sweating


asymmetry
4. Motor/trophic: reports of decreased range of motion (ROM) and/or motor
dysfunction (weakness, tremor, dystonia) and/or trophic changes (hair, skin,
nails)
3. At least one sign at the time of evaluation in two (for clinical diagnosis) or three (for
inclusion in scientific studies) of the following categories
1. Sensory: evidence of hyperesthesia (to pinprick) and/or allodynia (to light touch
and/or deep somatic pressure and/or joint movement)
2. Vasomotor: evidence of temperature asymmetry and/or skin color changes
and/or skin color asymmetry
3. Sudomotor/edema: evidence of edema and/or sweating changes and/or
sweating asymmetry
4. Motor/trophic: evidence of decreased ROM and/or motor dysfunction
(weakness, tremor, dystonia) and/or trophic changes (hair, skin, nails)
4. No other diagnosis better explains the signs and symptoms.

Decreased sensation and focal numbness are not consistent with CRPS. Laboratory and
imaging studies can be helpful in evaluating for the exclusion of differential diagnoses for CRPS,
including infection, rheumatic disease, fracture, nonunion, tenosynovitis, or osteomyelitis.

17. According to clinical and biomechanical studies, the most appropriate position for a headless
scaphoid compression screw for repair of a scaphoid waist fracture is

1. retrograde to protect the dorsal blood supply to the scaphoid.


2. retrograde eccentrically in the dorsal scaphoid to avoidtrapezium impingement.
3. deep and centrally placed, respecting the articular surface.
4. anterograde to protect the volar blood supply to the scaphoid.

Preferred Answer: 3

The position of a scaphoid screw for scaphoid fracture repair (Figure 1) is as critical as the
position of a sliding hip screw for intertrochanteric fracture repair. Positioning the screw deep in
the center of the densest portion of cancellous bone is beneficial for both of these fracture types.

Trumble and associates have shown time to union for scaphoid nonunions to be decreased for
centrally placed scaphoid screws. McCallister and associates documented improved
biomechanical stability for scaphoid waist fractures repaired with a centrally placed screw vs an
eccentrically placed screw. Dodds and associates demonstrated significantly improved
biomechanical stability with centrally placed long screws vs centrally placed short screws.

The screw may be placed retrograde or antegrade. Although the screw position may be relatively
parallel to the radial inclination as the shape of the scaphoid body follows the contour of the
radial styloid, screw position should be assessed relative to the scaphoid’s own architecture.
Placing the screw in a retrograde fashion can force the treating surgeon to start the screw
eccentrically in an attempt to avoid the interference of the trapezium lying over the distal pole of
the scaphoid. This may result in noncentral screw placement.

18. Figures 1 and 2 show the intraoperative photographs obtained during surgical treatment for de
Quervain tendonitis. For orientation purposes, dorsal is at the top. Figure 1 is obtained just after
the initial first extensor compartment release, and Figure 2 shows the floor of the first extensor
compartment. If the structure marked by the black dot is not addressed, the most common
postoperative problem would be

UI // UNAIR // UNPAD // UNHAS // UNS // UGM // UB // UNUD // USU


Hand and Wrist Self-Assessment Examination 2019

Figure 1 Figure 2

1. persistent pain.
2. tendon subluxation.
3. altered sensation.
4. tendon rupture.
PreferredAnswer: 1

The black dot identifies an accessory compartment of the extensor pollicis brevis (EPB) tendon.
The incidence of accessory EPB compartment in patients undergoing surgical treatment for de
Quervain syndrome ranges from 46% to 60%. Failure to release this compartment at the time of
initial surgery can cause persistent postoperative pain.
The patient would not experience altered sensation if this compartment were not released.
Altered sensation would most commonly occur following injury to the dorsal radial sensory nerve
branch during surgery. EPB tendon subluxation also would not occur should the accessory
compartment not be released. For EPB tendon subluxation to occur, its own compartment would
need to be released first. Finally, EPB tendon rupture would be an extremely uncommon
complication of failure to release the accessory compartment.

19. Video 1 depicts a 20-year-old right-hand-dominant man with a 6-month history of left wrist pain
and popping that has failed nonsurgical measures. No other positive findings upon examination
are noted. What is the most appropriate course of treatment?

UI // UNAIR // UNPAD // UNHAS // UNS // UGM // UB // UNUD // USU


Hand and Wrist Self-Assessment Examination 2019

1. Triangular fibrocartilage complex (TFCC) repair


2. Lunotriquetral fusion
3. Distal radioulnar joint (DRUJ) tenodesis
4. Extensor carpi ulnaris (ECU) tendon sheath reconstruction
PreferredAnswer: 4

Upon examination, this patient is exhibiting dislocation of the ECU tendon because of a
disrupted sheath. He has failed nonsurgical measures, so surgery that would involve either
direct repair or reconstruction of the tendon sheath is indicated. An option for reconstruction is to
use a portion of the extensor retinaculum as a sheath substitute. Deepening of the ECU tendon
groove at the distal ulna with direct repair of the sheath is another option, although a 2016 paper
by Ghatan and associates did not find depth of the groove as a risk factor for subluxation. TFCC
repair, lunotriquetral fusion, and DRUJ tenodesis are not appropriate because the examination
clearly shows ECU tendon dislocation. TFCC and lunotriquetral ligament tears can occur along
with ECU tendon dislocation, but no other examination findings suggest these conditions for this
patient.

20. Figures 1 and 2 are of a 51-year-old man who underwent open reduction and internal fixation of
a right proximal humerus fracture with concomitant rotator cuff repair. Within 1 year, he
develops heterotopic ossification, for which he undergoes excision and hardware removal.
Postoperatively, he was noted to have progressive atrophy in the shoulder and anterior
humeral head subluxation with attempted shoulder abduction. What nerve was damaged during
the most recent procedure?

Figure 1 Figure 2

1. Suprascapular
2. Radial
3. Anterior branch of axillary
4. Spinal accessory (cranial nerve XI)
Preferred Answer: 3

This patient has a deficiency of the anterior deltoid muscle, resulting in inferior subluxation of
the humerus with associated glenohumeral instability. Axillary nerve injury during shoulder
surgery accounts for 6% to 10% of brachial plexus injuries. In the posterior scapular region, the
axillary nerve terminates by dividing into two main branches: the posterior terminal branch,
which provides motor innervation to the teres minor and posterior deltoid muscles, and the

UI // UNAIR // UNPAD // UNHAS // UNS // UGM // UB // UNUD // USU


Hand and Wrist Self-Assessment Examination 2019

anterior terminal branch, which provides motor innervation to the anterior and middle portions
of the deltoid muscle. The deltoid determines the silhouette of the shoulder and is a stabilizer of
the humeral head.

21. A 65-year-old right–hand-dominant woman has been experiencing thenar and wrist pain for 18
months. She has no history of trauma. The pain worsens during the opening of jars, grasping,
writing, and repetitive thumb use. Examination reveals tenderness to palpation over the volar
thenar eminence, just distal to the scaphoid tubercle, and along the flexor carpi radialis sheath.
A Watson scaphoid shift test produces pain but no instability or clunk. Radiographs reveal
isolated scaphotrapeziotrapezoidal (STT) arthritis with mild dorsal intercalated segment
instability (DISI) deformity. She has worn a splint on and off for the past year, has had multiple
cortisone injections, and has modified her activity, all of which helped initially. She wants to
move forward with surgical intervention. STT arthrodesis is chosen over distal pole scaphoid
excision. What factor in her evaluation indicates that arthrodesis would be preferred over distal
pole excision?

1. Failure of pain relief from steroid injection and NSAID use


2. Tenderness that is distal to the scaphoid tubercle
3. Isolated STT arthritis on radiograph
4. Mild DISI deformity on radiograph
Preferred Answer:4

Isolated STT arthritis is common and can cause substantial patient disability and pain. After
nonsurgical treatment has failed, surgical intervention is warranted. Surgical options include
distal pole scaphoid excision, STT arthrodesis, or carpometacarpal (CMC) arthroplasty, if
concomitant thumb CMC degenerative changes are present. Distal pole scaphoid excision is
less commonly employed for the treatment of STT arthritis, because of the potential
development of intercalated segmental instability. However, distal pole scaphoid excision is less
technically demanding, engenders fewer surgical complications, and promotes a faster return to
previous activity levels. In any patient with preoperative carpal malalignment, removing the distal
pole of the scaphoid would exacerbate that deformity and could lead to symptoms from the DISI
deformity. Thus, in this patient with mild DISI deformity seen on preoperative radiographs, STT
arthrodesis is the most appropriate treatment option.

22. Nerve conduction velocity is slowed by

1. increased skin temperature.


2. increased perineural blood flow.
3. external compression.
4. hand dominance.
Preferred Answer: 3

A number of factors affect nerve conduction velocity; for example, increased body temperature
increases nerve conduction velocity. Nerve conduction velocity is slowed by advancing age,
compression, decreased blood flow, and fibrosis (from large imprecise sutures used for nerve
repair). There is no association between hand dominance and nerve conduction velocity.

UI // UNAIR // UNPAD // UNHAS // UNS // UGM // UB // UNUD // USU


Hand and Wrist Self-Assessment Examination 2019

23. Figures 1 through 3 demonstrate the MRIs obtained from a 36-year-old man with an injury to the
elbow. The expected result of nonsurgical treatment would be weakness of

1. finger flexion.
2. elbow extension.
3. finger extension.
4. forearm supination.
PreferredAnswer: 4

The axial T2-weighted MRIs demonstrate a distal biceps rupture. The increased signal is
noted superficial to the brachialis muscle and adjacent to the biceps tuberosity. The distal
biceps tendon is not seen in the distal cuts and has retracted proximally. The physical
examination of patients with these injuries show abnormal contour of the arm and tenderness
in the antecubital fossa. The hook test is a provocative maneuver that documents biceps
integrity. When performing the maneuver, the examiner attempts to hook a finger around the
distal biceps tendon while the patient actively supinates with the elbow held in flexion.
Nonsurgical treatment has been documented to result in an average loss of 40% of supination
strength and 30% of elbow flexion strength. Repair is optimal within several weeks of injury.
The alternative options would not occur with a distal biceps rupture.

24. A 44-year-old man sustains the injury shown in Figures 1 through 3. What is the most
appropriate treatment?

UI // UNAIR // UNPAD // UNHAS // UNS // UGM // UB // UNUD // USU


Hand and Wrist Self-Assessment Examination 2019

Figure 1 Figure 2 Figure 3

1. Reduction and internal fixation


2. Closed reduction and splinting alone
3. Carpometacarpal arthrodesis
4. Carpometacarpal (CMC) joint suspension arthroplasty

Answer: Reduction and internal fixation


Reduction, either open or closed, with internal fixation (pinning) is the recommended treatment
for the majority of these injuries. Closed reduction with pinning is most often performed for
acute injuries. Open reduction with pinning is performed for those injuries that cannot be
reduced by closed means or those with a delayed presentation
.
Four cases of successful closed reduction and splinting, all performed upon presentation in the
emergency department, have been described by Storken and associates, but the authors note
that their review of three prior reports uncovered cases of secondary dislocation, which
required surgical stabilization. One of the dislocations occurred 4 months after the reduction.
They assert that an indication for primary ORIF is a CMC dislocation associated with major
fractures.

Primary arthrodesis can be considered in cases with severe intra-articular comminution, but this
procedure substantially limits the ability of the hand to increase and decrease the transverse
metacarpal arch, which is an important functional movement. It can also lead to osteoarthritis of
the triquetrohamate joint.

Suspension arthroplasty has been described for old fracture-dislocations of the fifth CMC joint,
using a partial slip of the extensor carpi ulnaris.

25. What is the most common complication after distal biceps tendon repair at the elbow?

1. Lateral antebrachial cutaneous neuritis


2. Radial sensory neuritis
3. Symptomatic heterotopic ossification

UI // UNAIR // UNPAD // UNHAS // UNS // UGM // UB // UNUD // USU


Hand and Wrist Self-Assessment Examination 2019

4. Rupture of the repair


Preferred Answer: 1

Cain and associates retrospectively reviewed 198 consecutive surgical repairs of the biceps
and noted a 36% overall complication rate, including 26% paresthesia of the lateral
antebrachial cutaneous nerve, 6% paresthesia of the sensory branch of the radial nerve, 2%
superficial infection, 4% injury to the posterior interosseous nerve, 3% symptomatic heterotopic
ossification, and 2% rerupture.

Grewal and associates randomized 47 patients to have single-incision repair with two suture
anchors and 44 patients to have two-incision transosseous suture repair. They reported that the
single-incision technique had a higher rate of transient neurapraxia to the lateral antebrachial
cutaneous nerve, affecting 19 of 47 in the suture anchor group and three of 43 in the
transosseous group. Also, four reruptures occurred, which appeared to be independent of the
repair technique.

Recordon and associates found only three complications in their series of 46 patients, 19
having endobutton repair and 27 with transosseous suture repair. They reported two injuries to
the lateral antebrachial cutaneous nerve and one case of heterotopic ossification.

The review by Banerjee and associates showed one rupture of repair (disengagement of the
cortical button), two persistent lesions of the superficial branch of the radial nerve, and one
symptomatic heterotopic ossification.

Cohen remarked that rerupture of the tendon after repair is uncommon in both one-incision and
two-incision techniques.
Van den Bogaerde and Shin presented a case of posterior interosseous nerve incarceration
with endobutton repair.

26. A 67-year-old woman has a painful, arthritic proximal interphalangeal (PIP) joint, and
nonsurgical measures have failed to improve the pain. What implant and joint replacement
approach combination has been demonstrated to have the lowest rate of revision surgery?

1. Silicone replacement arthroplasty through a volar approach


2. Surface replacement arthroplasty through a volar approach
3. Silicone replacement arthroplasty through a dorsal approach
4. Surface replacement arthroplasty through a dorsal approach
PreferredAnswer: 1

A recent systematic review compared silicone replacement, pyrocarbon replacement, and


surface replacement arthroplasty for PIP arthritis. Silicone arthroplasty through a volar
approach showed the greatest gains in arc of motion and had the lowest rate of revision
surgeries. The rates of revision surgeries from low to high for each type of arthroplasty were
6% for silicone volar, 10% for silicone lateral, 11% for silicone dorsal, 18% for surface
replacement dorsal, and 37% for surface replacement volar. Revision surgeries include implant
replacement (to silicone or maintaining the surface replacement), arthrodesis, explantation,
amputation, and other procedures. Surface replacement arthroplasty through a volar approach
showed the highest revision rate, the worst gain in arc of motion, and the greatest extension
lag. However, substantial pain relief and higher satisfaction still were reported after surface
replacement arthroplasty, regardless of the complications.

UI // UNAIR // UNPAD // UNHAS // UNS // UGM // UB // UNUD // USU


Hand and Wrist Self-Assessment Examination 2019

27. Figures 1 through 4 are the radiographs and MR images of a healthy 21-year-old woman who
has had persistent dorsal wrist pain despite immobilization and no history of trauma. The
surgical procedure associated with the best prognosis in this scenario is

Figure 1 Figure 2

Figure 3 Figure 4

1. capitate excision with interposition arthroplasty.


2. capitate proximal pole excision and drilling.
3. proximal row carpectomy (PRC).
4. vascularized bone graft.

Answer: vascularized bone graft.


This patient has osteonecrosis of the capitate. The MR images show evidence of osteonecrosis
with decreased signal on the T1-weighted image. The radiographs are unremarkable, with the
exception of lunotriquetral coalition, which does not necessitate treatment. The etiology of

UI // UNAIR // UNPAD // UNHAS // UNS // UGM // UB // UNUD // USU


Hand and Wrist Self-Assessment Examination 2019

osteonecrosis of the capitate may be related to trauma, abnormal Interosseous vascular


supply, and hypermobility. Surgery is an option for patients with persistent symptoms despite
immobilization. Vascularized bone graft should be considered in this scenario because there is
no evidence of capitate collapse or arthritic change about the wrist. Free and local vascularized
bone grafts have produced satisfactory results. Capitate excision with interposition arthroplasty
is indicated for patients with proximal pole capitate collapse. Total wrist fusion is a salvage
procedure and would be considered if there were evidence of collapse and arthritic change.
PRC would leave the capitate articulating with the radius and is not indicated.

28. What vitamin supplement has been shown in some studies to reduce the risk of complex
regional pain syndrome following a distal radius fracture?

1. A
2. B
3. C
4. D

Preferred answer: 3

Two studies have shown that supplemental vitamin C reduces the risk of developing complex
regional pain syndrome following a distal radius fracture. The recommended dose is 500 mg
daily for 50 days. Supplemental vitamin C is a recommendation of the AAOS evidence-based
Clinical Practice Guidelines and has moderate evidence. The vitamin supplements listed as
alternative options have not been shown to prevent disproportionate pain following a distal
radius fracture.

29. Figures 1 through 3 demonstrate the radiographs obtained from a 25-year-old man who injured
his right, by punching a wall 3 weeks earlier. He notes pain and deformity about the ulnar aspect
of his hand. The best treatment option is

Figure 1 Figure 2 Figure 3

UI // UNAIR // UNPAD // UNHAS // UNS // UGM // UB // UNUD // USU


Hand and Wrist Self-Assessment Examination 2019

1. closed reduction and cast immobilization.


2. open reduction and internal fixation (ORIF).
3. arthrodesis.
4. resection arthroplasty.

Preferred answer: 2
The initial radiographs reveal a fourth and fifth carpometacarpal (CMC) joint fracture
dislocation. The injury is associated with a shear fracture of the dorsal rim of the hamate.
Further assessment with CT might be helpful in fully evaluating the extent of injury. Extensor
carpi ulnaris is a deforming force at the base of the fifth metacarpal. This unstable fracture
dislocation could be treated with closed reduction and pinning if the patient presented within a
few days of injury. However, because he presented in a delayed fashion (3 weeks after injury),
open reduction with internal fixation was required (Figures 4 and 5). In the series by Zhang and
associates, patients with fourth and fifth CMC fracture dislocations presenting in a delayed
fashion and treated nonsurgically had suboptimal results. Therefore, closed reduction and
casting are not appropriate. An arthrodesis and resection arthroplasty are salvage procedures
considered for a painful arthritic joint and would less likely should not be considered for this
acute injury.

30. Figures 1 through 3 show the clinical photographs obtained from a 45-year-old woman who is
right-hand dominant. She has pain in the left ring proximal interphalangeal (PIP) joint that gets
worse during lifting or gripping activities. On examination, she has PIP range of motion of 15° to
50° with laxity of the radial collateral ligament and tenderness around the joint. The flexor and
extensor tendons are intact. She has rotational malalignment when making a composite fist.
Radiographs reveal end-stage arthritis at the PIP joint. She elects to move forward with surgery
and undergoes arthroplasty. What component of the examination is essential to determine
which implant arthroplasty—silicone or surface replacement—is best?

Figure 1 Figure 2 Figure 3

1. Preoperative range of motion


2. Flexor tendon integrity
3. Rotational malalignment
4. Collateral ligament stability

UI // UNAIR // UNPAD // UNHAS // UNS // UGM // UB // UNUD // USU


Hand and Wrist Self-Assessment Examination 2019

Answer: Collateral ligament stability


This patient has end-stage arthritis in conjunction with ligament insufficiency. The treatment for
arthritis is arthroplasty or fusion. Given that her ring finger is affected, arthroplasty is
recommended to preserve motion and grip. Two types of arthroplasties are available: silicone
and surface replacement. The prerequisites are the same for both and include good bone
stock, good sensibility of the joint, adequate soft-tissue coverage, and normally functioning
tendons. Adequate collateral ligaments are required for surface replacement arthroplasty. This
patient has a deficiency of the radial collateral ligament, evidenced by her clinical examination.
Thus, silicone arthroplasty is the recommended option for joint replacement in this patient.

31. Figures 1 through 4 are the wrist MR images of a 43-year-old right-hand-dominant bricklayer
who reports gradually progressive left hand weakness for 4 months. He describes difficulty
gripping objects, tying his shoes, and holding utensils. He denies any numbness, paresthesias,
or a previous injury. An examination reveals intact sensation in a median, radial, and ulnar
nerve distribution. He has atrophy of hand interossei and a positive Froment sign finding. He
has no Tinel sign finding at the wrist or elbow and no exacerbation of symptoms with elbow
hyperflexion. Electromyography shows signs of denervation in an ulnar nerve distribution distal
to the wrist. What is the best next step?

Figure 1 Figure 2

Figure 3 Figure 4
1. Cubital tunnel release
2. Guyon's canal release
3. Hook-of-hamate excision
4. Excision of the ganglion cyst
Preferred Answer: 4

The MR images show a lesion consistent with a ganglion cyst located near the hook of the
hamate. The ulnar nerve divides into motor and sensory branches just proximal to this lesion. In
this case, the ganglion cyst compresses the ulnar nerve motor branch but not the sensory
branch, resulting in motor dysfunction but no sensory disturbance. Excision of the ganglion cyst
should alleviate his symptoms.

UI // UNAIR // UNPAD // UNHAS // UNS // UGM // UB // UNUD // USU


Hand and Wrist Self-Assessment Examination 2019

Compression of the ulnar nerve proximal to the motor branch take-off (in either the cubital
tunnel or proximal Guyon’s canal) would cause both sensory and motor dysfunction. Although
chronic nonunion of the hook of the hamate can cause ulnar nerve symptoms, the hook of the
hamate appears intact on the MR image. The MR image shows a lesion that is well
circumscribed with high intensity on T1 and T2 images, consistent with a benign ganglion cyst,
and ganglion cysts are relatively common lesions in this area.

32. Figures 1 and 2 are the radiographs of a 55-year-old woman homemaker with a 1-year history
of insidious onset left wrist pain. She has failed conservative treatment and desires surgery.
Her medical history is complicated by a smoking history of 1.5 packs of cigarettes per day. At
the time of surgery her capitate articular surface is normal in appearance. The best procedure
for her would be

Figure 1 Figure 2

1. radial shortening osteotomy.


2. capitate shortening osteotomy.
3. scaphoid excision and four-corner fusion.
4. proximal row carpectomy.
Preferred Answer: 4

This patient has Lichtman stage 3B Kienbock disease. She is 55 years old and is a "low-
demand" patient; however, she is a heavy smoker. Based on her condition and her current
smoking status, salvage treatment that does not require bone healing such as a proximal row
carpectomy is likely the best treatment option. A radial shortening osteotomy and a capitate
shortening osteotomy may be helpful in offloading the lunate, but both procedures require bone
healing and are better options in earlier stages of Kienbock disease. A scaphoid excision and
four-corner fusion is typically performed for scapholunate advanced collapse or scaphoid
nonunion advanced collapse wrist arthritis and would not be recommended in this scenario, as
the lunate is avascular.

33. Figure 1 shows the clinical photograph obtained from a child with a congenital difference of the
hand. What clinical feature(s) is/are characteristic of this condition?

UI // UNAIR // UNPAD // UNHAS // UNS // UGM // UB // UNUD // USU


Hand and Wrist Self-Assessment Examination 2019

Figure 1

1. Cardiac anomalies
2. Radial deviation of the thumb
3. Acrosyndactyly with proximal sinus tracts
4. Absence of the ulna

Preferred Answer: 3
The clinical photograph reveals a child with amniotic band syndrome or constriction band
syndrome. If a band causes an autofusion of the digits without amputation, acrosyndactyly can
occur, as demonstrated in the clinical photograph. Typically, a proximal sinus tract with a distal
syndactyly is present.

Radial deviation of the thumb can be seen most frequently in Apert syndrome. Cardiac
anomalies are associated with many congenital upper extremity differences but are not
characteristic of amniotic band syndrome. Ulnar longitudinal deficiency is characterized by
hypoplasia or complete absence of the ulna.

34. Botulinum toxin is used to treat vasospastic disorders of the hand such as the Raynaud
phenomenon to improve digital perfusion and reduce pain. Botulinum toxin enables which
transmitter to be unopposed, resulting in vasodilation?

1. Substance P
2. Glutamate
3. Rho kinase
4. Nitric oxide
Preferred answer: 4

Nitric oxide is the only transmitter listed that is not inhibited by botulinum toxin. Substance P
and glutamate are inhibited by botulinum toxin from release by pain nociceptors, thus reducing
pain. Fonseca and associates have postulated that botulinum toxin inhibits the RhoA kinase
pathway by blocking reactive oxygen species, which in turn does not allow actin/myosin to
activate, thus preventing vasoconstriction of smooth muscle. Blocking the RhoA kinase

UI // UNAIR // UNPAD // UNHAS // UNS // UGM // UB // UNUD // USU


Hand and Wrist Self-Assessment Examination 2019

pathway allows the action of nitric oxide to be unopposed, causing vasodilation. Nitric oxide is a
potent vasodilator. Thus, botulinum toxin promotes nitric oxide activity to increase vasodilation.

35. Figures 1 and 2 depict the postoperative radiographs obtained from a 22-year-old man who
was involved in a motor vehicle accident. The most likely limitation in motion arising from this
treatment is

Figure 1 Figure 2

1. loss of wrist flexion.

2. loss of wrist extension.

3. loss of elbow extension.

4. loss of pronation.
Preferred Answer: 4

This patient sustained fractures of his radius and ulna; both were treated with plate and screw
fixation. The plate used on the radius was straight, resulting in loss of the radial bow, which is
critical for enabling the radius to curve around the ulna during pronation. This patient is unable
to pronate beyond 20°.

Schemitsch and Richards correlated a good functional outcome, defined as more than 80% of
normal rotation of the forearm, with restoration of the normal amount and location of the radial
bow. Additionally, they related the restoration of grip strength with appropriate restoration of the
radial bow. Matthews and associates reported little significant loss of rotation with 10° of
angulation; however, 20° of angulation resulted in a statistically and clinically significant loss of
forearm rotation.

36. Figure 1 is the ultrasound of a 23-year-old patient who has had a volar radial 1.5-cm tender and
painful wrist mass for 6 months. The additional workup prior to surgery should consist of

UI // UNAIR // UNPAD // UNHAS // UNS // UGM // UB // UNUD // USU


Hand and Wrist Self-Assessment Examination 2019

Figure 1

1. serum and urine protein electrophoresis.


2. a chest CT scan.
3. MRI with intravenous contrast.
4. age-appropriate presurgical laboratory studies.
Preferred Answer: 4
The ultrasound shows a homogeneous anechoic mass consistent with a ganglion cyst. As a
benign lesion, no further workup or biopsy is required prior to a marginal surgical excision other
than age-appropriate laboratory studies. An MRI study with contrast would provide no
diagnostic benefit.

37. A 65-year-old woman has severe pain and numbness in her hand. She notes frequent awakenings at
nighttime and difficulty with fine tasks. She also has a history of cervical radiculopathy and notes
intermittent pain in her upper arm and periscapular region. An examination reveals a positive Tinel
sign over the midforearm and carpal tunnel. Electrodiagnostic testing shows a median nerve sensory
distal latency of 3.8 ms (normal latency is 3.5 ms). Which intervention or test would best predict if
carpal tunnel release would be successful in relieving this patient's symptoms?

1. Trigger point injections with lidocaine


2. Carpal tunnel corticosteroid injection
3. Ultrasound of the wrist
4. Carpal tunnel view radiograph
Preferred answer: 2

This patient demonstrates several upper extremity issues including possible carpal tunnel syndrome,
cervical radiculopathy, and pronator syndrome. The electrodiagnostic testing is equivocal, and a
corticosteroid carpal tunnel injection should be performed prior to surgical intervention to assess its
effectiveness in eliminating the patient's symptoms. Positive response (meaning improvement in
symptoms), after corticosteroid injection at the carpal tunnel correlates well with symptom relief
following surgery. Trigger-point injections are not indicated for carpal tunnel syndrome. Ultrasound
and carpal tunnel view radiograph can provide diagnostic information but would not be helpful in
determining treatment in this specific case.

UI // UNAIR // UNPAD // UNHAS // UNS // UGM // UB // UNUD // USU


Hand and Wrist Self-Assessment Examination 2019

38. Figures 1 and 2 show the intraoperative photographs obtained from a man who is undergoing
open reduction and internal fixation of a fifth carpometacarpal joint fracture dislocation. If the
structure marked with an arrow in Figure 2 is cut, the patient can expect to experience

1. the inability to extend the small finger.


2. weakness of small finger abduction.
3. sensory loss of the dorsal ulnar hand.
4. clawing of the small and ring fingers.
Preferred Answer: 3
The arrow in Figure 2 marks the dorsal sensory branch of the ulnar nerve. Injury to this nerve
results in sensory loss of the dorsal ulnar palm and the dorsal small and ring finger digits. The
dorsal sensory branch of the ulnar nerve exits the main ulnar nerve at an average distance of 8.3
cm from the proximal border of the pisiform. It becomes subcutaneous on the ulnar aspect of the
forearm at an average distance of 5 cm from the proximal edge of the pisiform. It then travels
dorsal to the extensor carpi ulnaris tendon to innervate the dorsal ulnar hand and the dorsal ring
and small digits. Injuries to this nerve can occur from open and arthroscopic procedures (such as
triangular fibrocartilage complex repair) as well as from procedures requiring percutaneous
pinning. Care must be taken to identify and protect this nerve to avoid the complications of
numbness and possible neuroma formation.

The inability to extend the small finger would be caused by an injury to the extensor tendon(s) in
this area, and the inability to abduct the small finger would require an injury to the abductor digiti
minimi muscle/tendon unit or the ulnar nerve motor branch, which is located on the volar aspect
of the proximal palm. Clawing of the small and ring fingers would be caused by absent intrinsic
function due to an injury to the ulnar motor nerve branch located on the volar proximal palm.

39. Figures 1 through 5 show the radiographs obtained from a 37-year-old man who has a 10-year
history of right, ulnar-sided wrist pain and a volar ulnar prominence with wrist supination.
Approximately 20 years ago, he had a forearm injury that was definitively treated in a long arm
cast. What surgical treatment option is most likely to improve his symptoms and maintain
pronosupination?

UI // UNAIR // UNPAD // UNHAS // UNS // UGM // UB // UNUD // USU


Hand and Wrist Self-Assessment Examination 2019

Figure 1 Figure 2 Figure 3

Figure 1 Figure 2

1. Distal radial ulnar joint (DRUJ) ligament reconstruction


2. Ulnar head implant arthroplasty
3. Radial shaft osteotomy
4. One-bone forearm procedure
Preferred Answer: 3
The patient sustained a radial shaft fracture with subsequent apex volar malunion. As a result,
his distal ulna subluxates volarly with wrist supination. Radiographs of the wrist reveal minimal
arthritic changes. The most appropriate treatment option is to surgically correct his radial shaft
malunion, which would indirectly address his DRUJ instability. A DRUJ ligament reconstruction
or triangular fibrocartilage complex repair could be used to augment DRUJ stability; however,

UI // UNAIR // UNPAD // UNHAS // UNS // UGM // UB // UNUD // USU


Hand and Wrist Self-Assessment Examination 2019

they might be unnecessary after correction of the radial shaft malunion. A DRUJ ligament
reconstruction alone would not achieve stability of the DRUJ joint and maintain full wrist
pronosupination. An ulnar head implant arthroplasty would not be reliable in eliminating the
instability or the pain. Similarly, a one-bone forearm procedure might improve the patient's pain
and instability but at the cost of abnormal wrist and forearm mechanics and kinematics.

40. Figures 1 and 2 are the radiographs of an 18-year-old man who had surgery 6 months ago at
an outside institution. He is being referred now because he has persistent pain. He is tender
over the scaphoid at the snuffbox. What is the most appropriate next imaging step in his pain
workup?

Figure 1 Figure 2

1. MR imaging with contrast


2. MR imaging without contrast
3. CT scan along the scaphoid axis
4. Axial-cut CT scans with reformats

PreferredAnswer: 3
Scaphoid nonunions are difficult to diagnose on plain radiographs, which offer poor reliability
when attempting to determine if there is bridging trabeculae crossing the fractures site. CT scans
are more useful for diagnosing scaphoid nonunion. When scanned using conventional axial cuts,
the slices mayskip through the fracture nonunion site, thereby missing the defect, even with
reformats. MR imaging is useful in diagnosing acute scaphoid fractures and has a high sensitivity
and diagnostic value for excluding scaphoid fractures as well. Contrast does not enhance the
utility of MR imaging in fracture diagnosis.

41. A 25-year-old man sustains a left brachial plexus injury from a fall while rock climbing. Examination
reveals poor intrinsic function of the hand, ptosis, and miosis. He is able to abduct and forward flex his
shoulder with full strength. This combination of physical findings is most suggestive of what pattern of
nerve injury?

1. C5-C6 postganglionic injury


2. C8-T1 preganglionic injury
3. C5 through C7 preganglionic injury
4. C8-T1 postganglionic injury

UI // UNAIR // UNPAD // UNHAS // UNS // UGM // UB // UNUD // USU


Hand and Wrist Self-Assessment Examination 2019

PreferredAnswer: 4

A preganglionic lesion occurs proximal to the spinal foramen, whereas a postganglionic lesion occurs
distal to the spinal foramen in the root, trunk, division, cord, or branches of the brachial plexus. The
Horner sign, which is characterized by miosis, ptosis, anhydrosis, and enophthalmos, results from an
injury to the sympathetic ganglion, which lies in close proximity to the T1 root level. The presence of a
Horner sign is highly suggestive of a T1 preganglionic injury. Other physical examination indicators of
a preganglionic injury include atrophy of the parascapular muscles (injury to the dorsal rami of the
cervical spinal nerve roots), winged scapula (injury to the long thoracic nerve) and hemidiaphragmatic
paralysis (phrenic nerve injury).

The lack of intrinsic hand function in this patient is also suggestive of an injury at the level of C8-T1.
Preservation of shoulder abduction and forward flexion would not typically be seen with an injury to
the C5-C6 roots or the upper trunk.

42. Based on the best available evidence, what is the maximum number of days at which a
successful manipulation can be performed following collagenase injection?

1. 7
2. 5
3. 3
4. 1
PreferredAnswer: 1

In the original clinical trial, the authors performed manipulation at 24 to 72 hours following the
collagenase injection for Dupuytren disease. Evidence now suggests that up to 7 days is a safe and
well-tolerated interval for manipulation following collagenase injection.

43. Figure 1 is the radiograph of a 22-year-old man who underwent an open reduction and pinning of
a perilunate dislocation 10 weeks ago. The hardware has been removed. What is the best next
step?

Figure 1

o Observation
o Vascularized bone grafting to the lunate
o Core decompression of the radius and ulna

UI // UNAIR // UNPAD // UNHAS // UNS // UGM // UB // UNUD // USU


Hand and Wrist Self-Assessment Examination 2019

o Immobilization

Preferred Answer: 1
Lunate or perilunate dislocations are usually treated with open reduction and internal fixation
through a dorsal or combined dorsal and volar approach. A high index of suspicion is
necessary when treating patients who sustain multiple trauma because as many as 25% of
lunate or perilunate dislocations are missed initially.

The radio dense appearance of the lunate seen in Figure 1 is an example of transient ischemia
of the lunate that can occur following treatment of lunate and perilunate dislocations. It has
been reported in up to 12.5% of cases. This usually is seen between 1 and 4 months post injury
with a relative radio density of the lunate. This appearance of the lunate should not be over
treated and usually is a benign self-limiting event. Surgery is not indicated at this time; the
incorrect responses are treatment options for Kienbock disease.

Treatment of the lunate or perilunate dislocation involves initial gentle closed reduction followed
by open reduction, ligamentous and bone repair, and internal fixation. Median nerve
dysfunction is common, and a simultaneous carpal tunnel release is often performed. Early
treatment seems to produce better results, but good results have been reported when
treatment is delayed for up to 6 months. The many questions regarding treatment of this
problem involve the use of capsulodesis to supplement intercarpal ligament repair,
repair/stabilization of the lunotriquetral interval vs no treatment of that articulation, and
intercarpal fixation techniques. In delayed cases, proximal row carpectomy when the head of
the capitate is intact and total wrist fusion if there are degenerative changes have been used.
Chondral injuries are common, may not be recognized on radiographs, and may negatively
affect long-term outcomes. Even when treatment is optimal, this injury is associated with a
guarded prognosis and possible permanent partial loss of wrist motion and grip strength. At 10-
year follow-up, radiographs will often demonstrate degenerative changes, but these changes
do not always substantially negatively affect hand function.

44. A 64-year-old woman with rheumatoid arthritis cannot fully extend her fingers actively at the
metacarpophalangeal (MCP) level. Full passive extension is possible, but she cannot actively
maintain that extension when her fingers are released. The MCP joints do extend when her wrist is
passively flexed. What is the most likely cause of this problem?

1. Extensor tendon ruptures at the wrist


2. Subluxation of the extensor mechanisms at the MCP joint
3. Caput ulnae syndrome
4. Posterior interosseous nerve palsy

PreferredAnswer: 4

Loss of MCP extension is common in the setting of rheumatoid arthritis, but potential causes are
varied. An understanding of various etiologies is needed to address the multiple facets of
inflammatory arthropathy, for which causes include extensor tendon rupture, MCP joint flexion
contracture/dislocation, subluxation of the extensor tendons at the MCP joint (sagittal band rupture),
and posterior interosseous nerve (PIN) palsy. These causes are differentiated during an examination.

Extensor tendon ruptures, as seen with caput ulnae syndrome (Vaughan-Jackson syndrome) allow for
passive MCP extension, not active extension or the ability to maintain extension actively, and would
not involve finger extension with passive flexion tenodesis of the wrist. Flexion contractures of the
MCP joints, as may be seen in dislocations, would not allow active or passive motion.

Subluxation of the extensor tendons at the MCP joints, as seen in attenuation of the sagittal bands,

UI // UNAIR // UNPAD // UNHAS // UNS // UGM // UB // UNUD // USU


Hand and Wrist Self-Assessment Examination 2019

will usually preclude the initiation of active extension, but patients will be able to maintain extension of
the MCP joints once they are placed in extension by the examiner. With a PIN palsy, no active
extension or active hold is possible, but passive extension and tenodesis with wrist flexion are
maintained. Synovitis of the elbow is the most likely cause of the PIN palsy. An extensor tenosynovitis
or caput ulnae syndrome would not prohibit MCP extension unless the condition progressed to the
point of causing other pathology

45. The development of complex regional pain syndrome (CRPS) following distal radius fracture is
associated with what factor?

1. Diabetes
2. Fibromyalgia
3. Nonsurgical fracture management
4. Male gender
PreferredAnswer: 2

CRPS is an uncommon complication following distal radius fractures; its incidence is reported to
range between 1% and 37%. Two recent studies have evaluated for risk factors in the development of
CRPS following distal radius fractures. Female gender, concomitant fracture of the distal ulna, and
surgical treatment were all associated with an increased likelihood of CRPS, as was fibromyalgia.
Older age was identified as conferring both an increased and a decreased risk for CRPS in the two
studies.

46. A 20-year-old woman with spastic hemiplegia is evaluated for function and hygiene issues with her
right wrist. Her wrist has a resting posture of 90° of flexion and can be passively extended to 65° of
flexion. Her fingers are flexed into her palm but can be passively extended with the wrist at 95°. What
treatment is likely to provide the most durable result for improved hygiene, function, and cosmesis?

1. Flexor carpi ulnaris to extensor carpi radialis brevis transfer


2. Fractional lengthening of the wrist and finger flexor tendons
3. Wrist arthrodesis with proximal row carpectomy
4. Botulinum toxin injection

PreferredAnswer:3

The patient has a static deformity of the wrist with a fixed flexion deformity of more than 45°. Soft-
tissue procedures such as those referenced in options A and B would not be sufficient to address the
degree of contracture. Additionally, Botulinum toxin injections would not provide relief for capsular
contractures. Wrist arthrodesis combined with a proximal row carpectomy has been shown to provide
a functional and cosmetic alternative for patients with severe wrist flexion contractures.

47. Figures 1 and 2 are the radiographs of a 36-year-old right-hand-dominant man who has had
persistent wrist pain for 6 months after a motor vehicle collision. The initial treatment was splint
immobilization. What is the best next step?

UI // UNAIR // UNPAD // UNHAS // UNS // UGM // UB // UNUD // USU


Hand and Wrist Self-Assessment Examination 2019

Figure 1 Figure 2
1. Therapy/rehabilitation

2. Open reduction and internal fixation (ORIF)


3. Proximal row carpectomy
4. Wrist arthrodesis
PreferredAnswer: 3

This patient has a chronic untreated volar lunate dislocation. Lunate dislocations are usually the result
of a high-energy injury. Recommended treatment for an acute lunate dislocation is ORIF with repair of
injured structures (ligament and bone). If the patient has paresthesias in a median nerve distribution,
carpal tunnel release is recommended in the same setting as ORIF. Six months after injury, the
prognosis for successful ORIF is poor and proximal row carpectomy is recommended. Among
perilunate/lunate dislocations, 25% are initially missed. If a patient arrives for treatment and there is
evidence of radiocarpal and midcarpal arthrosis, wrist arthrodesis is recommended.

48. Figures 1 through 3 show the MRI images and a radiograph obtained from a 31-year-old woman
who has a 1-year history of diffuse right wrist pain that is gradually worsening. She denies fever
or chills and also denies a history of injury. Her examination reveals no swelling, no erythema, an
80 degree arc of active wrist flexion and extension, and dorsal wrist tenderness. The most likely
diagnosis is

Figure 1 Figure 2 Figure 3

1. scapholunate advanced collapse (SLAC) wrist with cystic capitate changes.


2. idiopathic avascular necrosis (AVN) of the capitate.
3. capitate osteomyelitis.

UI // UNAIR // UNPAD // UNHAS // UNS // UGM // UB // UNUD // USU


Hand and Wrist Self-Assessment Examination 2019

4. aneurysmal bone cyst in the capitate.


Preferred Answer: 2
This patient's history, examination, and imaging suggest a rare condition known as capitate AVN.
SLAC arthritis would show osteoarthritic changes in the radioscaphoid joint. Capitate osteomyelitis
would likely have more diffuse edematous changes on MRI and less range of motion secondary to
associated septic arthritis. An aneurysmal bone cyst in the capitate would likely show fluid-fluid levels
on the MRI.

The treatment of capitate AVN is usually surgical and varies on the extent of the area of necrosis, the
degree of fragmentation, the degree of collapse, and the absence or presence of arthritic changes.
Common treatment options include vascularized bone grafts, resection of the affected area with
interposition tendon graft, intercarpal fusion, and complete wrist fusion or replacement.

49. When treating a closed long finger central slip tendon rupture conservatively, what is the most
appropriate plan of care?

1. Splint the proximal interphalangeal (PIP) joint in flexion with early motion of the distal
interphalangeal (DIP) joint
2. Allow early motion of the PIP joint with DIP extension joint splinting
3. Splint both the PIP and DIP joints in full extension
4. Splint the PIP joint in extension with early motion of the DIP joint
Preferred Answer: 4

Closed central slip injuries treated nonsurgically require extension splinting of the PIP joint. DIP joint
active range of motion is allowed during this time period. This allows the connections between the
lateral bands and the central slip to pull the central slip distally with DIP joint active motion, minimizing
the gap across the central tendon injury and keeping the DIP joint from getting stiff as well.

50. Figure 1 is the clinical photograph of a 64-year-old man who crashed while riding his motorcycle.
An examination reveals his long-finger metacarpophalangeal (MP) joint is stuck in extension. He
cannot passively or actively flex at the MP joint. A hand radiograph is seen in Figure 2. Which
interposed structure is preventing reduction?

Figure 1 Figure 2

1. Flexor tendons
2. Lateral band

UI // UNAIR // UNPAD // UNHAS // UNS // UGM // UB // UNUD // USU


Hand and Wrist Self-Assessment Examination 2019

3. Lumbrical
4. Volar plate

Preferred Answer:4
This patient has a dorsally dislocated MP joint. In these cases, the volar plate can be displaced
dorsal to the metacarpal head, preventing reduction. Although early publications described a
―noose effect‖ of the lumbrical and flexor tendons, the primary block to reduction is the volar
plate. Simple MP dislocations can be reduced closed by flexing the wrist and then gently sliding
the base of the proximal phalanx over the end of the metacarpal. Longitudinal traction on the
finger will only incarcerate the volar plate further and should be avoided. Patients with complex
dislocations that fail closed reduction require open reduction.

51. Which method of flexor tendon repair that necessitates excursion through the A2 pulley allows for the
most thorough assessment of tendon gliding?

1. 4-strand repair with 6-0 epitendinous suture with Bier block anesthesia
2. 4-strand repair with 6-0 epitendinous suture under local anesthesia only
3. 6-strand repair with regional anesthesia
4. Repair of the flexor tendon with incision of the remaining A2 pulley
PreferredAnswer: 2
Wide-awake repair under only local anesthesia, regardless of the technique, allows direct inspection
of the tendon repair and active excursion. Regional anesthesia and Bier block anesthesia do not
allow active motion (Bier block necessitates continued use of a tourniquet, which limits muscle
function). The A2 pulley should be preserved, especially the distal 50%, to maintain tendon function.
All of the listed techniques for suture repair are acceptable options.

52. Figure 1 is the clinical photograph of a 42-year-old woman who has a lesion that has failed prior silver
nitrate applications. She experiences frequent bleeding from this lesion. A tissue biopsy performed by
a dermatologist revealed capillary hypertrophy with lobular arrangement. Which treatment is most
appropriate to minimize recurrence?

1. Sclerotherapy
2. Shave excision with cautery
3. Cryotherapy
4. Wide surgical excision

UI // UNAIR // UNPAD // UNHAS // UNS // UGM // UB // UNUD // USU


Hand and Wrist Self-Assessment Examination 2019

Preferred Answer: 4
This lesion is a pyogenic granuloma, which is a common benign vascular lesion that can occur
on skin or mucosa. The etiology is unclear, although this lesion tends to occur in areas of
physical trauma. Initial treatment with silver nitrate with an average of 1.6 applications has a
success rate of 85%. This patient, however, has failed silver nitrate applications. Wide surgical
excision (Figures 2 and 3) is associated with the lowest recurrence rate and offers the benefit of
a single procedure. Other options often necessitate repeated procedures to completely eradicate
this lesion.

53. In the injury shown in Figures 1 and 2, what ligament remains intact?

Figure 1 Figure 2

1. Short radiolunate
2. Scapholunate
3. Radioscaphocapitate
4. Dorsal radiocarpal
Preferred Answer:1
Perilunate dislocations result from high-energy injuries to the extended wrist. The injury shown
is a lunate dislocation. Two classification systems have been described, the Mayfield system
and the Herzberg system. Mayfield described the four stages of progressive ligamentous
instability following injury. In stage I, the radioscaphocapitate and scapholunate ligaments fail.
Stage II involves dislocation of the lunocapitate joint, usually a dorsal dislocation of the capitate.
In stage III, the lunotriquetral ligament fails. In stage IV, the dorsal radiocarpal ligament is torn,
and the lunate dislocates volarly. The short radiolunate ligament is the only ligament that
remains intact, resulting in rotation of the lunate volarly. Herzberg and associates further
classified perilunate dislocations as stage I injuries and lunate dislocations as stage II injuries.
Lunate dislocations were further classified into stage IIA, in which the lunate exhibits rotation
less than 90°, and stage IIB, in which the lunate exhibits rotation greater than 90°. The
radiographs represent a Mayfield stage IV, Herzberg stage IIA injury.

54. A 50-year-old patient underwent multiple débridements for an open radial shaft fracture with bone
loss. The bed currently shows no evidence of infection but has a 14-cm diaphyseal bone defect. The
most appropriate treatment includes open reduction and internal fixation along with

1. free vascularized fibula.


2. calcium sulfate pellets.

UI // UNAIR // UNPAD // UNHAS // UNS // UGM // UB // UNUD // USU


Hand and Wrist Self-Assessment Examination 2019

3. corticocancellous autograft.
4. demineralized bone matrix.
PreferredAnswer: 1
The patient developed a large bone defect after undergoing multiple débridements for an open
fracture. The most appropriate graft in this setting is a vascularized bone graft. Considering the length
of the defect, a free vascularized fibular graft would be a suitable graft. The indications for a
vascularized bone graft include infection, inadequate vascularity of the surrounding tissues, bone
defects larger than 6 centimeters, and previous failed bone grafts. The osteocytes survive in the
vascularized graft, allowing primary bone healing and thereby limiting a loss of graft strength. In
contrast, nonvascularized bone graft heals by creeping substitution with a loss of its initial strength.
Calcium sulfate pellets and calcium phosphate cement are synthetic bone substitutes with
osteoconductive properties. Their role in fracture healing is limited. Demineralized bone matrix is the
matrix remaining after allograft undergoes decalcification processing. These products alone would not
be optimal in the treatment of such a large bone defect.

55. A 45-year-old man underwent a fingertip amputation through the distal phalanx after his ring finger
was caught in a garage door. He was treated in the emergency department with a revision amputation
by advancement of the flexor digitorum profundus (FDP) tendon to the extensor mechanism. Three
months following the injury, he is able to fully flex his injured ring finger to touch his palm, but he
reports that it is difficult for him to make a tight fist due to decreased flexion of his other fingers. What
is this complication called?

1. Lumbrical plus deformity


2. Intrinsic tightness
3. Quadrigia effect
4. Proximal interphalangeal joint contracture

PreferredAnswer: 3

The quadrigia effect can occur due to over-advancement of the FDP tendon during repair (usually
greater than 1 cm), development of FDP tendon adhesions, and (as in this case) "over the top" repair
of the FDP tendon to the extensor tendon after amputation at the distal phalanx level. All of these
conditions result in a functionally shortened FDP tendon of the injured digit. Because the FDP
tendons of the long, ring, and small digits share a common muscle belly, excursion of the combined
tendons is equal to the shortest tendon. Therefore, the uninjured digits will not have full excursion of
their respective FDP tendons and will not be able to close into a full fisting position. Treatment of this
condition is most commonly release of the injured FDP tendon.

A lumbrical plus deformity can occur in amputations distal to the flexor digitorum superficialis insertion
through the middle phalanx. The FDP tendon retracts and increases tension on the lumbrical muscle,
which leads to paradoxical interphalangeal (IP) joint extension with attempted flexion. Intrinsic
tightness and interphalangeal joint contractures can be caused by hand trauma but would not lead to
the clinical condition this patient has.

56. When performing a Green transfer for cerebral palsy—flexor carpi ulnaris (FCU) to extensor carpi
radialis brevis (ECRB)—in addition to improving wrist extension, what other motion may be improved
if the FCU is routed around the ulna instead of through the interosseous membrane?

 Thumb extension
 Forearm supination

UI // UNAIR // UNPAD // UNHAS // UNS // UGM // UB // UNUD // USU


Hand and Wrist Self-Assessment Examination 2019

 Finger extension
 Forearm pronation
Preferred Answer: 2

The typical upper extremity deformity in spastic hemiplegic cerebral palsy consists of shoulder internal
rotation, elbow flexion, forearm pronation and wrist flexion, and ulnar deviation. The pronation position
of the forearm can make bimanual activities more challenging for the child. The wrist flexion and ulnar
deviation deformity interferes with finger function and therefore with grasp and release patterns. By
transferring the FCU tendon to the ECRB, the deforming force is released, and central wrist extension
is augmented. This transfer can lead to a supination moment when it is routed around the ulna to the
ECRB insertion on the dorsum of the wrist.

Thumb and finger extension are not affected by an FCU-to-ECRB tendon transfer. Forearm
supination, not pronation, is potentially improved with this tendon transfer.

57. Figure 1 shows a radiograph obtained from an active 30-year-old man who sustained an injury to
his ring finger 1 week earlier. The most appropriate treatment is

1. open reduction and internal fixation (ORIF).


2. a mallet splint.
3. repair of the terminal tendon.
4. arthrodesis.

PreferredAnswer: 1

Figure 1 reveals evidence of an intra-articular distal phalanx fracture with a distal interphalangeal
(DIP) joint dorsal subluxation. This injury is unstable and requires surgical management for an active
individual. Volar distal phalanx fractures are often associated with flexor digitorum profundus avulsion
injuries, which are addressed concomitantly. This injury was treated with ORIF of the intra-articular
fracture, pinning of the DIP joint, and repair of an avulsed flexor digitorum profundus tendon with a
button on the dorsal nail plate, as shown in Figure 2.

Splint immobilization would not maintain a reduction of this unstable injury. The terminal tendon is not
injured in this patient but is often injured in a dorsal distal phalanx fracture with a volar dislocation.
Arthrodesis of the DIP is a salvage procedure and would not be considered acutely.

UI // UNAIR // UNPAD // UNHAS // UNS // UGM // UB // UNUD // USU


Hand and Wrist Self-Assessment Examination 2019

58. A 55-year-old man was injured when a large piece of sheet metal lacerated his medial elbow
while working at a factory. He underwent primary repair of the lacerated structures shown in
Figures 1 and 2 on the day of injury. In addition to this surgical treatment, what nerve transfer
procedure should be considered during this primary operative intervention to improve his
functional recovery?

Figure 1 Figure 2

1. Flexor digitorum superficialis (FDS) branch transfer to the extensor carpi radialis brevis
(ECRB) branch
2. Third web space median fascicle transfer to the ulnar sensory fascicle
3. Flexor carpi ulnaris fascicle (FCU) transfer to the biceps branch
4. Terminal anterior interosseous nerve (AIN) transfer to the deep ulnar motor fascicle
Preferred Answer: 4

In adults, the repair of high ulnar nerve injuries typically yields incomplete motor recovery and
disappointing functional results despite early surgical intervention and careful surgical
technique. Early transfer of the terminal branch of the AIN to the deep ulnar motor fascicle can
rapidly reinnervate distal targets and potentially preserve motor end plate function in the
intrinsic musculature of the hand because of the proximity of the nerve transfer to the target
muscle.

Sensory deficits due to an ulnar nerve injury can be restored through a transfer of median
sensory fascicles to the distal ulna sensory fascicles. This procedure typically would not be
considered at the time of the original surgery, because sensory recovery is more likely than
motor recovery in the setting of a high ulnar nerve injury.

For radial nerve injuries, wrist extension can be restored through an FDS branch of the median
nerve transfer to the ECRB branch of the radial nerve. The FCU fascicle of the ulnar nerve can
be transferred to the biceps branch of the musculocutaneous nerve to restore elbow flexion and
supination.

59. What cardiac condition causes most upper extremity emboli?

1. Atrial fibrillation

2. Viral cardiomyopathy

3. Valvular disease

UI // UNAIR // UNPAD // UNHAS // UNS // UGM // UB // UNUD // USU


Hand and Wrist Self-Assessment Examination 2019

4. Atrial septal defect


PreferredAnswer: 1

Atrial fibrillation is responsible for approximately 80% of all upper extremity emboli. All other cardiac
conditions listed can cause upper extremity emboli; however, atrial fibrillation is the most common
cause.

Patients with an upper extremity embolic event should undergo prompt evaluation, with a careful
history and physical examination as well as focused laboratory tests for hypercoagulability. Arterial
Doppler studies or angiography is/are warranted. Electrocardiogram and echocardiogram are also
used to evaluate for potential cardiac abnormalities. Consultation with vascular, radiology, and
cardiology personnel is often necessary when patients present with upper extremity emboli.
Treatment usually involves anticoagulation, embolectomy if necessary, and treatment for any
recognized cardiac abnormality.

60. Figures 1 through 4 are the radiographs, sagittal-cut CT scan, and coronal T1 MR image of a 16-
year-old boy who has wrist stiffness and pain after sustaining an injury 2 years ago. There is no
bleeding from the proximal pole during surgery. Which procedure will most likely result in
restoration of alignment and healing?

Figure 1 Figure 2

Figure 3 Figure 4
1. 1,2 intercompartmental supraretinacular artery (ICSRA) graft

UI // UNAIR // UNPAD // UNHAS // UNS // UGM // UB // UNUD // USU


Hand and Wrist Self-Assessment Examination 2019

2. Free-vascularized medial femoral condyle graft


3. Iliac crest corticocancellous graft
4. 4+5 extensor compartmental artery (ECA) vascularized bone graft
Preferred Answer: 2

The imaging studies show an established scaphoid waist nonunion with a humpback deformity
(significant flexion through the nonunion site) and carpal collapse. In addition, the proximal pole
appears sclerotic on the plain radiographs and appears poorly perfused on the MR image. Correction
of alignment of this scaphoid nonunion would require a volar approach with a structural bone graft.
Additionally, the graft would need to provide a vascular supply to the bone.

Both the 1,2 ICSRA (the Zaidenberg graft) and the 4+5 ECA grafts are vascularized grafts from the
dorsal distal radius. Neither of these grafts would correct the humpback deformity, and the 4+5 ECA
graft pedicle is not long enough to reach the scaphoid. An iliac crest bone graft could be used to
correct the deformity, but would not provide an adequate blood supply. A free-vascularized medial
femoral condyle graft provides both adequate bone graft to correct the deformity and revascularization
of the scaphoid.

61. Figure 1 is the clinical photograph of a 65-year-old right-hand dominant man who has finger
contracture and stiffness. He experiences minimal pain but has severe functional limitations and
elects for treatment with injectable collagenase Clostridium histolyticum. What types of collagen
will be affected by this injection?

o Types I and II
o Types II and III
o Types I and III
o Types III and IV

Preferred Answer: 3

Type II collagen is the predominant type found in articular cartilage. Type IV collagen is the
predominant type found in the basement membranes of neurovascular structures.
Collagenase Clostridium histolyticum injection targets type I and type III collagen.

62. After performing an uneventful partial palmar fasciectomy for Dupuytren contracture of the palm and
ring finger, a general postsurgical pain medication prescription should include how many narcotic
pills?

UI // UNAIR // UNPAD // UNHAS // UNS // UGM // UB // UNUD // USU


Hand and Wrist Self-Assessment Examination 2019

1. 0
2. 10
3. 20
4. 30

PreferredAnswer: 2

After the designation of pain as the fifth vital sign, opioid analgesic use has steadily increased. Many
surgeons routinely prescribe 30 or more pills after elective hand surgery. However, studies show that
patients generally use fewer than 30 pills. Patients who underwent bone procedures used 14 pills,
and those undergoing soft-tissue procedures used 9 pills. Education and decision aids may help
physicians size prescriptions appropriately to avoid overmedication. Patients undergoing small soft-
tissue surgeries such as trigger releases should not need narcotics. Those undergoing small-joint
surgeries, carpal tunnel releases, and Dupuytren fasciectomy may benefit from a prescription of 10
pills. More extensive surgery, such as open fracture treatment, may justify more pills, but prescriptions
should not exceed 40 tablets under typical circumstances.

63. Figures 1 and 2 show the postreduction radiographs obtained from a 32-year-old man who fell
from a ladder onto his outstretched right arm. He reports right wrist pain and dense numbness in
his radial digits. What is the most appropriate treatment option?

Figure 1 Figure 2

1. Emergent surgery, including open carpal tunnel release, open reduction of the perilunate
dislocation, repair of the scapholunate ligament, and intercarpal pinning
2. Emergent surgery, including open carpal tunnel release, closed reduction of the perilunate
dislocation, and casting
3. Elective outpatient surgery, including open carpal tunnel release, open reduction of the
perilunate dislocation, repair of the scapholunate ligament, and intercarpal pinning
4. Emergent surgery, including open reduction of the perilunate dislocation, repair of the
scapholunate ligament, and intercarpal pinning

Preferred Answer: 1

This patient sustained a lesser-arc perilunate dislocation. As a result of the injury, he also
developed acute carpal tunnel syndrome. The closed reduction attempt was unsuccessful;
therefore, this injury is best managed with emergent surgery, an open carpal tunnel release, an

UI // UNAIR // UNPAD // UNHAS // UNS // UGM // UB // UNUD // USU


Hand and Wrist Self-Assessment Examination 2019

open reduction of the perilunate dislocation, scapholunate ligament repair, and intercarpal
pinning. Outpatient surgery in a delayed fashion is not advised because of the acuity and
severity of the carpal tunnel syndrome. Closed reduction and casting is not advised, because it
commonly leads to continued carpal instability with subsequent dorsal intercalated segment
instability deformity and scaphoid lunate advanced collapse wrist arthritis.

Emergent surgery, including open carpal tunnel release, open reduction of the perilunate
dislocation, repair of the scapholunate ligament, and intercarpal pinning

64. While attempting to recreate the inclination of the distal radius during volar fixation of an intra-
articular sagittal split fracture, use of intraoperative fluoroscopic imaging in the position shown
in Figure 1 would be helpful in showing

1. intra-articular screw penetration.


2. alignment of the joint surface.
3. alignment of the sigmoid notch.
4.dorsal screw penetration.
Preferred Answer: 1

The image demonstrates a rotational fluoroscopic view of the lateral distal radius while
attempting to recreate the inclination of the distal radius. This view is most useful to ensure
against intra-articular screw penetration. The overall alignment of the joint surface is best
viewed with a posteroanterior tilt of 11 degrees. The alignment of the sigmoid notch is not seen
well on lateral images. Carpal alignment is seen well on lateral images. Dorsal screw
penetration is best viewed dynamically with a flexed wrist tangential view.

65. Figures 1 and 2 show the radiographs obtained from a 56-year-old man who has been
experiencing progressive wrist pain since he felt a pop while throwing a 25-pound bag over his
shoulder 6 months ago. Failure to address the injury surgically might lead to progressive arthritic
changes in what order?

UI // UNAIR // UNPAD // UNHAS // UNS // UGM // UB // UNUD // USU


Hand and Wrist Self-Assessment Examination 2019

Figure 1 Figure 2

1. Lunocapitate, radioscaphoid, radial styloid, radiolunate


2. Radioscaphoid, radial styloid, lunocapitate, radiolunate
3. Radial styloid, radioscaphoid, lunocapitate, radiolunate
4. Radial styloid, radioscaphoid, radiolunate, lunocapitate

Preferred Answer: 3

This patient demonstrates scapholunate dissociation with an associated dorsal intercalated


segment instability deformity. Chronic scapholunate ligament tears lead to scapholunate
advanced collapse (SLAC) wrist. Watson and Ballet describe SLAC wrist as having a
predictable progression of arthritic changes, starting at the radial styloid, progressing to the
radioscaphoid joint, and advancing to the lunocapitate joint. Some authors have described the
radiolunate joint as being affected in very late-stage SLAC wrist.

66. Figures 1 through 3 show the radiographs obtained from a 40-year-old woman who injured her right
index finger in a bicycle collision. Failure to restore sagittal plane alignment would likely result in

1. overlapping of the digits.


2. index finger proximal interphalangeal (PIP) joint arthritis.

UI // UNAIR // UNPAD // UNHAS // UNS // UGM // UB // UNUD // USU


Hand and Wrist Self-Assessment Examination 2019

3. extensor lag at the PIP joint.


4. hyperextension at the PIP joint.
PreferredAnswer: 3

The radiographs reveal an extra-articular proximal phalanx fracture of the index finger. The fracture is
comminuted with dorsal angulation of the distal fragment. The question specifically asks about the
restoration of sagittal alignment. The fracture is comminuted with dorsal angulation of the distal
fragment. The other options are incorrect, because overlapping of the digits occurs with rotational
malalignment, the development of arthritis may occur with intra-articular fractures, and hyperextension
would not occur with this type of deformity.

67. Figure 1 depicts an intraoperative photograph obtained following proximal row carpectomy. The
black dot denotes the capitate. The top of the figure is radial and the bottom of the figure is ulnar.
Surgical disruption of the structure identified by the forceps would result in

1. loss of active thumb interphalangeal (IP) flexion.


2. distal radioulnar joint instability.
3. avascular necrosis of the capitate.
4. ulnar carpal translocation.

PreferredAnswer: 4

The structure identified by the forceps is the radioscaphocapitate ligament. During a proximal row
carpectomy, it is very important to identify and protect this ligament. Compromise of the ligament
would result in ulnar translocation of the carpus and early failure of the proximal row carpectomy
procedure. If the ligament is injured during surgery, immediate repair should be performed. Green and
associates discuss the importance of the radioscaphocapitate ligament in stabilizing the carpus after
this procedure is performed.

Nakamura and associates compared 3-mm, 6-mm, and 10-mm radial styloidectomies, and only the 3-
mm styloidectomy subsequently preserved carpal stability. Compromise of the radioscaphocapitate
ligament occurred when larger portions of the radial styloid were excised.

Distal radioulnar joint instability would result only from the disruption of the distal radioulnar joint
stabilizers. Avascular necrosis would not occur, because the capitate receives its blood supply mainly

UI // UNAIR // UNPAD // UNHAS // UNS // UGM // UB // UNUD // USU


Hand and Wrist Self-Assessment Examination 2019

from the palmar vessels. Finally, loss of active thumb IP flexion would not occur, because the flexor
pollicis longus tendon would remain intact even if ligament compromise were to occur.

68. A 23-year-old man cut the dorsal and ulnar aspects of his long finger on a table saw. The dorsal and
ulnar skin over the middle phalanx is missing, with a 2-cm x 2-cm area of loss. There is a 50% loss of
the extensor tendon (ulnar), and the remaining tendon has no tenosynovium. The physician should
recommend irrigation and debridement and

1. tendon repair, and thenar flap coverage.


2. full-thickness skin graft.
3. reversed cross-finger flap from the ring finger.
4. cross-finger flap coverage from the ring finger.
PreferredAnswer: 3

The patient has exposed bone and tendon and a partial tendon injury. The remaining radial tendon is
satisfactory and no tendon repair is required. The exposed bone and tendon necessitate vascularized
tissue coverage. A reversed cross-finger flap from the ring finger is suitable for coverage of the dorsal
surface of an adjacent digit.

69. A 35-year-old man sustained a traumatic low ulnar nerve palsy 18 months ago. The extent of the
clawing and intrinsic atrophy as well as the active radial deviation are seen in Figures 1 through 3. No
hyperextensibility of any of the proximal interphalangeal (PIP) joints is observed. Preoperatively, the
patient is not able to fully extend the PIP joints with the wrist in neutral position and the examiner
holding the metacarpophalangeal (MCP) joints flexed. Figure 4 shows the intraoperative photograph
obtained during the intrinsic reconstruction procedure that is performed. The tendon grafts were
inserted distally into the

UI // UNAIR // UNPAD // UNHAS // UNS // UGM // UB // UNUD // USU


Hand and Wrist Self-Assessment Examination 2019

1. proximal phalanx.
2. radial lateral bands.
3. first annular pulley.
4. second annular pulley.
PreferredAnswer: 2

Originally, Burkhalter and Strait recommended bony insertion into the proximal phalanx through a drill
hole. This procedure does require more surgical dissection and flexes only the MCP joints; thus it
cannot extend the PIP joints directly. It does improve clawing in the fingers if the PIPs can extend with
preoperative MCP flexion. The ability to extend the PIP joints is evaluated preoperatively using the
Bouvier test. With the wrist in neutral position, the examiner holds the MCPs flexed and looks for the
ability in that position to actively extend the PIPs. If the patient is able to do so, then the test is
considered positive, and this describes "simple" clawing. In such cases, procedures that flex only the
MCPs are appropriate. The insertion sites for these procedures include the proximal phalanx, the first
annular pulley, and the second annular pulley.

If the Bouvier test is negative, then it is best to insert the tendon grafts distally into the lateral bands.
This technique has a low chance of leading to hyperextension of the PIP joints, particularly when
performed with a wrist extensor motor (which leaves the flexor digitorum superficialis undisturbed)
and with no preoperative hyperextensibility of the PIPs.

70. Figure 1 points to the "tear drop" of the wrist. This radiographic landmark represents which anatomic
portion of the wrist?

Figure 1

UI // UNAIR // UNPAD // UNHAS // UNS // UGM // UB // UNUD // USU


Hand and Wrist Self-Assessment Examination 2019

o Ulnar head
o Volar ulnar corner
o Radial styloid
o Lister tubercle

PreferredAnswer: 2

Medoff described the radiographic teardrop of the distal radius. This radiographic landmark matches
the critical volar ulnar corner of the distal radius. A malreduction of the volar ulnar corner of the distal
radius in an intra-articular distal radius fracture leads to volar subluxation of the lunate and the rapid
development of posttraumatic arthritis at the distal radioulnar and radiolunate joints. Knowledge of the
specific shape and appearance of this radiographic landmark helps the surgeon when he or she is
critically analyzing postreduction imaging.

The volar portion of the ulnar head may be mistaken for this teardrop sign and should be separately
identified as distinct from the distal radius. The radial styloid and Lister tubercle are not part of the
volar aspect of the lunate facet.

71. Figures 1 through 3 demonstrate the radiographs obtained from a 45-year-old construction
worker who has wrist pain, loss of motion, and loss of strength. Nonsurgical measures have
failed, and the patient requests surgery. What is the best surgical option for this patient?

Figure 1 Figure 2 Figure 3

1. Proximal row carpectomy


2. Excision of the proximal pole of the scaphoid
3. Bone grafting with fixation of the scaphoid
4. Scaphoid excision with four-corner fusion
Preferred Answer: 4
This patient has a physically demanding occupation. His radiographs show a long-standing nonunion
with avascular necrosis and collapse of the proximal pole. This condition is referred to as a scaphoid
nonunion advanced collapse wrist (SNAC). He has advanced arthritis of the capitolunate joint, but the
radiolunate joint is preserved. The best option for him would be scaphoid excision with four-corner
fusion.

UI // UNAIR // UNPAD // UNHAS // UNS // UGM // UB // UNUD // USU


Hand and Wrist Self-Assessment Examination 2019

Proximal row carpectomy, would be incorrect because of the advanced arthritis of the capitolunate
joint. This procedure relies on normal cartilage/joint articulation of the capitate as well as of the lunate
fossa of the distal radius. Proximal pole scaphoid excision alone would not address the advanced
arthritis of the capitolunate joint. Scaphoid fixation with bone grafting would not address the midcarpal
arthritis and advanced proximal pole collapse.

72. A 54-year-old laborer has a 6-month history of lateral elbow pain. An elbow examination reveals full
range of motion, tenderness over the lateral epicondyle, and pain with resisted wrist extension with
the elbow in extension. Elbow radiograph findings are normal. You perform a steroid injection and the
patient's symptoms are decreased 6 weeks later. One year after receiving the injection, this patient—
when compared to a patient who did not have a steroid injection—is likely to

1. have no difference in elbow pain.


2. no longer have elbow pain.
3. need surgery.
4. experience tendon rupture.

Preferred Answer: 1

This patient has signs and symptoms of lateral epicondylitis. Treatments include various forms of
physical therapy, iontophoresis, corticosteroid injection, nitroglycerin patch treatment, blood injections,
prolotherapy, and surgical intervention. No single treatment is superior to other treatments for this
common problem. Several studies have demonstrated a short-term decrease in symptoms following
steroid injection (6 weeks) but an increased likelihood of persistent symptoms 1 year after treatment.
Steroid injection at this site has not been associated with increased risk for tendon rupture or need for
surgical intervention.

73. A 17-year-old boy with left spastic hemiplegia secondary to cerebral palsy is being evaluated for
persistent swan neck deformities of the affected hand. Splinting has been tried with some
improvement, but the patient does not want to wear the splints any more. On physical examination, he
demonstrates full extension of the metacarpophalangeal (MCP) joints, 30° of hyperextension of the
proximal interphalangeal (PIP) joints, and flexion of the distal interphalangeal (DIP) joints when he
attempts to actively extend his digits. He is able to initiate flexion at the PIP joints with his MCP joints
held in neutral extension. He has equal PIP flexion when the MCP joints are extended and flexed.
What is the most appropriate surgical treatment to address his swan neck deformity?

1. Central slip tenotomy


2. Terminal tendon release
3. Dorsal rerouting of the lateral bands
4. Intrinsic lengthening

PreferredAnswer: 1

This patient demonstrates full extension of the MCP joints when he actively extends his fingers,
indicative of overpull of the extrinsic finger extensors. This clinical scenario can be corrected by a
central slip tenotomy.

A terminal tendon release is used to address a Boutonnière deformity. The patient does not
demonstrate intrinsic tightness (equal PIP flexion while the MCP flexed and extended), therefore his
swan neck would be unlikely to respond to intrinsic lengthening. Dorsal rerouting of the lateral bands

UI // UNAIR // UNPAD // UNHAS // UNS // UGM // UB // UNUD // USU


Hand and Wrist Self-Assessment Examination 2019

is performed for a Boutonnière deformity. A central slip tenotomy would balance the extension forces
between the PIP and DIP joints.

74. You are counseling a 55-year-old woman for a right carpal tunnel release. What can you tell her about
the treatment benefit (grip strength and paresthesia relief) 1 year after surgery compared with
continued splinting, NSAID use, physical therapy, and a single steroid injection?

1. No change in paresthesias and grip strength


2. Increase in grip strength and decrease in paresthesias
3. Decrease in grip strength and increase in paresthesias
4. Increase in grip strength and paresthesias

PreferredAnswer: 2

Gerritsen and associates, Hui and associates, and Jarvik and associates compared the effectiveness
of surgical versus nonsurgical treatment for the relief of carpal tunnel symptoms. All three studies
showed that surgery was superior for the relief of paresthesias and the improvement of grip strength.
According to the American Academy of Orthopaedic Surgeons Clinical Guidelines on the Treatment of
Carpal Tunnel Syndrome, strong evidence supports the assertion that surgical treatment of carpal
tunnel syndrome should have a greater treatment benefit at 6 and 12 months than splinting, NSAIDs,
physical therapy, and a single steroid injection.

The other choices, including no change in grip strength and paresthesias, decrease in grip strength
and increase in paresthesias, and increase in grip strength and paresthesias, are not supported by
the evidence.

75. Figures 1 and 2 display the radiographs obtained from a woman who had volar plating of the distal
radius 8 months earlier. Two days ago, she noticed she could not actively extend her thumb. What is
the most appropriate treatment that would restore active thumb extension?

1. Removal of hardware with tendon transfer


2. Repair of the extensor pollicis longus (EPL) tendon primarily
3. Thumb interphalangeal (IP) arthrodesis
4. Nonsurgical treatment with cast placement keeping the thumb in a fully extended position for 4
weeks

PreferredAnswer: 1

Although the fracture is aligned in anatomic position, prominence of a least one of the distal screws is
evident on the lateral radiographic view. The prominent screw is the most likely cause of the EPL
tendon rupture. If the patient chooses surgical treatment, the best option would be removal of the
offending hardware combined with extensor indicis proprius to EPL tendon transfer. Intercalary

UI // UNAIR // UNPAD // UNHAS // UNS // UGM // UB // UNUD // USU


Hand and Wrist Self-Assessment Examination 2019

grafting would also be an acceptable option. If the tendon transfer were to be performed alone, the
prominent screw(s) could rupture the transferred tendon as well. Also, it is rarely possible to repair the
EPL tendon primarily in such cases, because this rupture is an attrition type. Casting would obviously
not provide any benefit in this situation, and IP arthrodesis would not be the first surgical treatment
option.

This problem can be avoided by using shorter screws or not placing screws in plate holes that direct
screws into the third dorsal extensor compartment. Intraoperative fluoroscopy and special views, such
as the carpal shoot-through view, are useful for avoiding this complication.

76. When performing a radioscapholunate (RSL) fusion for posttraumatic radiocarpal arthritis, excision of
the distal pole of the scaphoid will cause a decrease in

1. the nonunion rate.


2. wrist extension.
3. carpal height.
4. avascular necrosis.

PreferredAnswer: 1

RSL arthrodesis is a motion-sparing option for posttraumatic radioscaphoid or radiolunate arthritis


when the midcarpal joint is preserved. Preserving the midcarpal joint allows the dart-thrower motion to
remain. Mühldorfer-Fodor and associates reported that the rates of nonunion for RSL fusion were
reduced by excision of the distal pole of the scaphoid. Multiple studies have shown increased radial
and ulnar deviation with excision of the distal pole of the scaphoid; excision of the triquetrum further
increases the radial-ulnar arc of motion. Bain and associates and Pervaiz and associates reported
increased wrist flexion-extension arcs with distal scaphoid and triquetrum excisions in cadaveric
models; other authors have reported no difference.

77. Figures 1 and 2 are the radiographs of a 17-year-old man who injured his wrist 6 months ago. He is
experiencing pain and limited motion. What is the most effective treatment option?

Figure 1 Figure 2

1. Long-arm thumb spica casting


2. Bracing and bone stimulation

UI // UNAIR // UNPAD // UNHAS // UNS // UGM // UB // UNUD // USU


Hand and Wrist Self-Assessment Examination 2019

3. Scaphoid excision with intercarpal fusion


4. Bone grafting with screw placement
A: Bone grafting with screw placement

Figures 1 and 2 show a scaphoid nonunion with substantial bone resorption at the nonunion site. Cast
immobilization and bracing with bone stimulator use would not be successful treatments at this point
because the fracture is 6 months old and there is considerable bone resorption at the fracture site.
Scaphoid excision with intercarpal fusion is an option to use only after bone-grafting procedures have
failed or arthritis is present.

Bone-grafting procedures using both vascularized and nonvascularized graft sources are associated
with a high success rate that decreases with avascular necrosis of the proximal pole. If left untreated,
scaphoid nonunions can progress to carpal collapse and degenerative arthritis.

78. A 32-year-old man sustained an injury to the right thumb metacarpophalangeal (MP) joint ulnar
collateral ligament (UCL) and is undergoing surgical repair (Figure 1). What structure in the clinical
photograph is blocking reduction of the ulnar collateral ligament?

1. Extensor pollicis longus (EPL) tendon


2. Adductor aponeurosis
3. EPB and dorsal capsule
4. Ulnar sesamoid bone and volar plate

Preferred Answer: 2

When the thumb MP UCL is torn from the proximal phalanx, the distal stump can be displaced
superficial to the adductor aponeurosis, known as a Stener lesion. The adductor aponeurosis
effectively blocks reduction of the ligament to the normal attachment site. The EPB and EPL tendons
are dorsal to the UCL, and the ulnar sesamoid bone/volar plate are in a volar position in relation to the
UCL. The dorsal capsule would also not block reduction of the UCL due to it's anatomic location. The
other responses do not block the UCL with this type of injury.

UI // UNAIR // UNPAD // UNHAS // UNS // UGM // UB // UNUD // USU


Hand and Wrist Self-Assessment Examination 2019

79. At a minimum 2-year follow-up and compared with the metacarpophalangeal (MCP) joint,
pyrolytic carbon resurfacing arthroplasties of the proximal interphalangeal (PIP) joint

1. produce less squeaking or clicking.


2. result in more dislocations.
3. provide superior pain relief.
4. result in better motion compared with the preoperative status.

Preferred answer: 2

Wall and Stern published a report on MCP joint pyrolytic carbon arthroplasty for osteoarthritis and
another on PIP joint pyrolytic carbon resurfacing arthroplasty for osteoarthritis. They found different
outcomes, and MCP joint implants outperformed PIP joint implants. Of eleven MCP joint
arthroplasties, two produced asymptomatic squeaking and clicking, whereas eleven of 31 PIP joint
implants produced this problem. No dislocations were reported among the MCP joint implants, but five
PIP joint dislocations were observed.

Outcomes were measured by the Michigan Hand Outcomes Questionnaire in both studies and were
satisfactory for the MCP joint implants, with an average score of 80. The PIP implants did not fare as
well, showing a higher degree of pain along with an average score of 53. The authors noted that, in
the 15 patients in the PIP study who had unilateral surgery, the uninvolved, nonsurgical hand motion
was actually statistically significantly (P<0.01) better than the surgical hand. MCP joint motion
increased from 62º before surgery to 76º after surgery, whereas PIP joint motion got worse after
surgery, with the average motion decreasing from 57º to 31º.

80. Figure 1 is the right hand of a 65-year-old man with a history of hypertension and rheumatoid
arthritis. He is taking immunosuppressive disease-modifying antirheumatic drugs (DMARDs)
and is seen in the emergency department with rapid progression of erythema from his right
thumb to his right arm during the last 12 hours. He is confused, lethargic, and has these vital
signs: blood pressure 92/40, respiratory rate 45, temperature 39.7°C, pulse 135, and oxygen
saturation 90% on 4 liters of oxygen by face mask. An examination of his right upper extremity
reveals black bulla extending from the metacarpophalangeal down to the tip and no capillary
refill at the pulp. Immediate treatment should consist of

1. intravenous (IV) antibiotics and admission to a medical intensive care unit.


2. emergent radical debridement including thumb amputation.
3. emergent revascularization of the thumb with a vein graft.

UI // UNAIR // UNPAD // UNHAS // UNS // UGM // UB // UNUD // USU


Hand and Wrist Self-Assessment Examination 2019

4. urgent irrigation of the thumb flexor tendon sheath

Preferred Answer : 2

This patient has multiple criteria for necrotizing soft-tissue infection (NSTI, also known as
necrotizing fasciitis) including rapidly progressive infection, black bulla, hypotension and
hypoxia, and a history of immune compromise. Aggressive emergent debridement including the
removal of all necrotic tissue and IV antibiotics can decrease morbidity and mortality.

Not all patients will have such obvious NSTI findings. In less clear cases, a scoring system
using laboratory values (the Laboratory Risk Indicator for Necrotizing Fasciitis) can help clarify
the diagnosis. IV antibiotics are key to treatment as well, but any delay in surgical treatment
can increase morbidity and mortality. The black bulla and necrotic-appearing thumb indicate
that this infection is not confined to the flexor sheath, therefore irrigation of the tendon sheath
alone would be insufficient treatment. Although the thumb is dysvascular, this is because of an
infection, and revascularization is not indicated.

81. A 72-year-old woman with diabetes mellitus has right hand numbness. Provocative test findings are
consistent with carpal tunnel syndrome, and electrodiagnostic study (EDS) findings show prolonged
median motor and sensory distal latencies with low-amplitude thenar compound muscle action
potential. Poor prognosis is most associated with which factor?

5. Diabetes
6. Older age
7. Female gender
8. Severity of EDS findings

Preferred Answer: 4
Although carpal tunnel syndrome remains a clinical diagnosis, EDS findings have become important
tools. Moderate disease is defined as abnormal median sensory distal latency and prolonged median
motor distal latency. Severe disease is defined as prolonged median motor and sensory distal
latencies witheither absent sensory nerve action potential or mixed nerve action potential or low-
amplitude or absent thenar compound muscle action potential. Although female gender and diabetes
are considered risk factors, there is little association with postsurgical outcome. Severe disease as
confirmed by EDS is the factor most strongly associated with poor recovery after surgery.

82. A 51-year-old male 2-pack per day smoker presents with a hyperkeratotic light brown plaque on the
dorsum of his left ring finger that has been present for 7 years. It measures 14 mm by 13 mm. Initially,
it responded to topical wart treatments, but has failed to do so recently so he sought evaluation by a
dermatologist who biopsied the lesion. The results revealed squamous cell carcinoma (SCC) in situ,
and he was referred for further surgical management. He has no other skin lesions, no history of SCC
and no axillary lymphadenopathy. What is the next step in management?

9. Continued observation and re-evaluation in 6 months


10. Sentinel lymph node biopsy
11. Wide excision
12. Primary ray resection

Preferred Answer: 3

UI // UNAIR // UNPAD // UNHAS // UNS // UGM // UB // UNUD // USU


Hand and Wrist Self-Assessment Examination 2019

SCC in situ is a low-grade malignancy that typically presents as painless lesions on areas of high sun
exposure such as the dorsum of the hand and fingers. The recommended treatment for lesions
smaller than 100 mm is wide excision with 4 mm margins to a depth 1 layer below the tumor, along
with any adjacent area of induration. Sentinel lymph node biopsy is typically not indicated in the
setting of a low-grade tumor such as this one and in the absence of axillary lymphadenopathy.

83. At the first postoperative visit after mini-open carpal tunnel release, a patient reports hand weakness.
Poor index finger interphalangeal joint extension and metacarpophalangeal joint flexion are present.
This finding is most consistent with

1. unrecognized injury to the recurrent motor branch.


2. neuropraxia of the proper palmar digital nerve.
3. new-onset stenosing flexor tenosynovitis.
4. injury to the flexor digitorum profundus to the index finger.
PreferredAnswer:2

Complications after carpal tunnel release are relatively uncommon. The clinical picture described
above is most consistent with lumbrical muscle weakness secondary to neuropraxia of the proper
palmar digital nerve to the index finger supplying motor innervation to that muscle. The recurrent
motor branch of the median nerve innervates the thenar musculature and would not present as index
finger weakness. A new onset of trigger finger may result from a loss of the pulley effect of the
transverse carpal ligament, postoperative tendon inflammation, or previously unrecognized flexor
tendon triggering. Flexor digitorum profundus to the index finger lies deep within the carpal tunnel,
making its injury unlikely. If it were injured, the result would not be weakness of interphalangeal joint
extension.

84. A 35-year-old man who is left-hand dominant has pain and swelling around his left index metacarpal
phalangeal (MCP) joint following a motor vehicle accident 2 months ago. Radiographs reveal no
fractures. He has point tenderness over the radial side of the MCP joint and increased laxity with
ulnarly applied stress. He has failed conservative treatment including 5 weeks of immobilization. If the
patient elects to live with this condition and not have surgery, what would be the most common
outcome?

1. Development of a trigger finger


2. Presence of intrinsic tightness
3. Weakness of pinch strength
4. Subluxation of the extensor tendon with MCP joint motion
PreferredAnswer: 3

The thumb MCP ulnar collateral ligament (UCL) and index MCP radial collateral ligament (RCL) are
the primary stabilizers to pinch. Secondary stabilizers include the volar plate, dorsal capsule, and
accessory collateral ligaments. With insufficiency of either the thumb MCP UCL or the index MCP
RCL, pinch strength will be severely weakened. The other options listed are unlikely to occur with this
particular injury.

85. Figure 1 shows an injury sustained by a 60-year-old man 4 weeks ago. Since that time he has
had substantial pain and catching of his finger during attempts at range of motion. What is the
most appropriate treatment at this point?

UI // UNAIR // UNPAD // UNHAS // UNS // UGM // UB // UNUD // USU


Hand and Wrist Self-Assessment Examination 2019

1. Tendon debridement
2. Release of the A2 pulley
3. Tendon repair with core sutures
4. Tendon repair with epitendinous sutures

PreferredAnswer: 4
Approximately 70% laceration of the flexor digitorum profundus tendon with active locking is best
treated with epitendinous sutures. Performing this procedure under local anesthetic allows for better
assessment of whether the triggering has been resolved. Cyclic loading has been shown to increase
with high-grade partial lacerations. Use of core sutures adds little strength to a partial laceration.
Debridement alone is reserved for injuries involving less than 60% of the tendon diameter. Release of
the A2 wouldcompromise pulley function.

86. Figures 1 through 3 are the radiographs of a 65-year-old man who sustained a fracture from a fall.
The patient elects open reduction and internal fixation of the distal radius. After plating the distal
radius, the distal radioulnar joint (DRUJ) is examined and found to be unstable in both pronation and
supination. What is the best next step?

Figure 1 Figure 2 Figure 3


1. Early range of motion (ROM) program with a removable short-arm splint

2. Long-arm casting in pronation for 4 weeks

UI // UNAIR // UNPAD // UNHAS // UNS // UGM // UB // UNUD // USU


Hand and Wrist Self-Assessment Examination 2019

3. Pin fixation of the DRUJ


4. Fixation of the ulnar styloid fracture
Preferred Answer: 4

The initial radiographs show a comminuted displaced distal radius fracture, along with a displaced
fracture of the base of the ulnar styloid. The displacement is best seen on the oblique view. After
reduction and fixation of the radius, DRUJ stability should be assessed. The majority of scenarios that
involve this injury pattern will not be unstable because of the oblique band of the interosseous
ligament. When DRUJ instability is present after fixation of the radius, reduction and fixation of the
ulnar styloid fracture is the best option to provide stability of the distal radioulnar joint (DRUJ) (Figure
4). A study by Lawton and associates revealed that all distal radius fractures complicated by DRUJ
instability were accompanied by an ulnar styloid fracture. A fracture at the ulnar styloid’s base and
substantial displacement of an ulnar styloid fracture were found to increase risk for DRUJ instability.
An ulnar styloid base fracture involves the insertion of the radioulnar ligaments and can cause DRUJ
instability if displaced. If persistent instability is present after fixation of the ulnar styloid, DRUJ pinning
is a reasonable option. Early ROM with splinting would not allow reduction or healing of the ulnar
styloid and would result in persistent instability. Short-arm casting also would not allow stability of the
DRUJ and would be a less reliable method with which to achieve healing of the ulnar styloid.

87. A 45-year-old man feels a pop in the anterior aspect of his elbow while lifting furniture. He denies any
antecedent pain or injury. Which examination method is best for diagnosing a distal biceps rupture?

1. The examiner brings a finger from medial to lateral across the antecubital fossa, feeling for a cord-
like structure.

2. The examiner brings a finger from lateral to medial across the antecubital fossa, feeling for a cord-
like structure.

3. With the elbow flexed to 90°and the forearm pronated, the examiner resists patient supination,
evaluating for pain at the bicipital groove.

4. With the patient’s arm elevated to 90° of forward flexion, the elbow extended, and the forearm
supinated, the examiner resists elevation distal to the elbow, evaluating for pain at the bicipital
groove.
Preferred answer: 2

Rupture of the distal biceps tendon is predictably identified by the hook test, which is performed by
bringing a finger from lateral to medial across the antecubital fossa of a flexed elbow, feeling for a
cord-like structure on which the examiner can "hook" a finger. Bringing the finger from medial to
lateral can cause a false-negative result, hooking the lacertus fibrosus, which can remain intact even
with a ruptured distal biceps tendon. The Yergason test (option 3) and the Speed test (option 4) are
used to assist in diagnosing proximal, not distal, biceps and labral pathology. Even if the distal biceps
tendon is ruptured, the supinator remains intact. Although supination weakness may be present, an
inability to supinate should not be observed.

88. A 75-year-old man has persistent radial sided hand and wrist pain. Radiographs demonstrate severe
scaphotrapezial trapezoidal arthritis. His basal joint is unaffected. His pain has failed to improve with
bracing, activity modification, and image-guided corticosteroid injection. He has elected surgical
treatment. What long-term complication can arise from a distal scaphoid resection?

1. Avascular necrosis of the proximal pole of the scaphoid

UI // UNAIR // UNPAD // UNHAS // UNS // UGM // UB // UNUD // USU


Hand and Wrist Self-Assessment Examination 2019

2. Dorsal intercalated segment instability (DISI)


3. Volar intercalated segment instability
4. Thumb metacarpophalangeal joint hyperextension
Preferred answer: 2

Resection of the distal pole of the scaphoid eliminates the arthritic contact at the
scaphotrapeziotrapezoid joint; however, it functionally shortens the scaphoid. Theoretically, the lunate
is at equilibrium between the extension moment of the capitate and the triquetrum and the flexion
moment of the scaphoid. Shortening the scaphoid allows the extension moment of the triquetrum to
predominate, pulling the lunate into extension and creating a DISI deformity. Concomitant
capsulodesis or interposition is recommended by some authors to prevent this complication.

89. Figures 1 through 3 are the clinical photograph and radiographs of a 25-year-old, left-hand-
dominant man who injured his left index finger. Which treatment option will most effectively allow
satisfactory fracture alignment and maximize motion?

Figure 1 Figure 2 Figure 3

1. Buddy-taping to the long finger with an early range of motion (ROM) program
2. Closed reduction and static external fixation in extension
3. Open reduction and internal fixation (ORIF) with an early ROM program
4. Digital splinting for 4 weeks followed by a ROM program

Preferred Answer: 3

This patient has an oblique index proximal phalanx fracture with malrotation. Buddy-taping and
digital splinting would not predictably maintain fracture reduction and would result in a malunion
with rotational deformity and possible shortening. Closed reduction and spanning external fixation
in extension would result in significant digital stiffness. ORIF (Figures 4 and 5) followed by an
early ROM program would allow anatomic fracture alignment and give this patient the best
chance to regain the majority of motion in the shortest amount of time.

90. Figures 1 and 2 are the clinical photographs of a 36-year-old woman who cannot fully extend the
metacarpophalangeal (MP) joints of her long and ring fingers 9 months after the removal of a
plate from the proximal radius via a dorsal approach. What is the most likely cause of this
problem?

UI // UNAIR // UNPAD // UNHAS // UNS // UGM // UB // UNUD // USU


Hand and Wrist Self-Assessment Examination 2019

Figure 1 Figure 2

1. Laceration of a branch of the posterior interosseous nerve (PIN)


2. Postsurgical tendon adhesion
3. Laceration of the (EDC) tendons to long and ring fingers
4. Neuropraxia of the PIN

Preferred Answer: 1

Plating of the proximal radius from a dorsal or Thompson approach (between the extensor carpi
radialis brevis and extensor digitorum communis) allows complete visualization of the PIN
through the supinator. Going through the same incision from proximal to distal produces a scar-
filled proximal approach, and it is not uncommon to drift ulnarly and injure an individual nerve to
the extensor digitorum communis muscle, resulting in the deformity seen in Figures 1 and 2.

Postsurgical tendon adhesions rarely involve only 2 tendons. Complete tendon lacerations are
rare at this level in the proximal forearm, as is a complete laceration of the PIN. Neuropraxias
are common but usually resolve after 3 months. The gossamer-thin branch of the PIN to the
EDC can be easily damaged in scar tissue, resulting in an inability to fully extend the MP joints
of the long and ring fingers. The proprius tendons allow the patient to fully extend the index and
little finger MP joints. Connecting the EDC of the long finger to the extensor indicis proprius and
the EDC of the ring finger to the extensor digiti quinti proprius can correct the deformity. To
avoid the problem, the surgeon should start the incision distally in normal anatomy, and the
interval between the mobile wad and the digital extensors will be more easily found.

91. Which characteristic of complex regional pain syndrome (CRPS) type 2 differentiates it from CRPS 1?

1. Positive bone scan result


2. Identified nerve injury
3. Pseudomotor changes
4. No identified nerve injury

Preferred Answer: 2

CRPS 2 develops after nerve injury, whereas CRPS 1 occurs without nerve injury. The diagnostic
criteria are otherwise the same for the 2 conditions. A 3-phase bone scan can be helpful; a pattern of
increased uptake in all 3 phases, and particularly diffuse periarticular uptake in and around the joints
of the affected extremity during the delayed phase, is considered typical of CRPS, especially during
the first 6 months; however, it is not specific enough to be used as a diagnostic criteria.

UI // UNAIR // UNPAD // UNHAS // UNS // UGM // UB // UNUD // USU


Hand and Wrist Self-Assessment Examination 2019

Unfortunately, the pathophysiological mechanisms underlying CRPS remain unclear, and, as such, no
standard diagnostic test for CRPS exists. There are 3 major sets of diagnostic criteria for CRPS: the
International Association for the Study of Pain (IASP) criteria, the Budapest Research Criteria, and
the Veldman criteria. According to IASP, CRPS type 1 involves an initiating noxious event or a cause
of immobilization; continuing pain/allodynia/hyperalgesia for which the pain is disproportionate to any
known inciting event; evidence of edema, changes in skin blood flow, or abnormal pseudomotor
activity in the region of pain; and exclusion of the diagnosis by the existence of other conditions that
would otherwise account for the degree of pain and dysfunction. CRPS type 2 is a syndrome that
develops after nerve injury, contends IASP. Spontaneous pain or allodynia/hyperalgesia occurs and is
not necessarily limited to the territory of the injured nerve; there is or has been evidence of edema, a
skin blood flow abnormality, or abnormal sudomotor activity in the region of pain since the inciting
event; and the diagnosis is excluded by the existence of conditions that would otherwise account for
the degree of pain and dysfunction.

92. A 35-year-old man has a brachial plexus injury affecting the lateral cord. He partially improves with
observation and now has complete return of median nerve function and pectoral muscle function.
What nerve transfer is most likely to restore the motor function he is lacking?

1. Median and ulnar fascicles to musculocutaneous nerve transfer


2. Medial triceps branch to axillary nerve transfer
3. Intercostal nerve to triceps branch of radial nerve transfer
4. Anterior interosseous nerve (AIN) to ulnar motor transfer

PreferredAnswer: 1

The lateral cord of the brachial plexus gives off the lateral pectoral nerve, the musculocutaneous
nerve, and then contributes to the median nerve. The patient has had recovery of function of these
components except for the musculocutaneous nerve. The musculocutaneous nerve innervates the
biceps and the brachialis, which provide elbow flexion. To restore motor function, a nerve transfer
would have to provide reinnervation of the biceps and brachialis.

93. Figures 1 and 2 show the MRI studies obtained from a 35-year-old manual laborer with persistent
wrist pain despite immobilization. At the time of surgery, collapse of the capitate and arthritic
changes of the midcarpal joint are noted. What is the most appropriate procedure for this
condition?

Figure 1 Figure 2

UI // UNAIR // UNPAD // UNHAS // UNS // UGM // UB // UNUD // USU


Hand and Wrist Self-Assessment Examination 2019

1. Local vascularized bone graft


2. Proximal row carpectomy
3. Midcarpal fusion
4. Total wrist arthroplasty

Preferred Answer: 3

The T1-weighted MRI reveals decreased signal that is consistent with avascular necrosis
(AVN) of the capitate. Figure 2 demonstrates increased signal of the capitate consistent with
edema. The etiology of AVN of the capitate may be related to trauma, abnormal interosseous
vascular supply, and hypermobility. Surgical treatment is considered for patients who have had
persistent symptoms despite immobilization. At the time of surgery, collapse of the capitate and
arthritic changes would be treated most appropriately with a salvage procedure. A midcarpal
fusion is a motion-preserving salvage procedure and is the most appropriate option given to
address the pain associated with the midcarpal arthritic changes. The alternative options are
not appropriate for this patient. Local vascularized bone grafts are considered for situations in
which no evidence of capitate collapse or arthritis is observed. A proximal row carpectomy
would lead to suboptimal results, because the collapsed proximal pole of the capitate would
articulate with the lunate facet of the radius. In addition, a high rate of failure has been seen in
young patients requiring full wrist fusion following proximal row carpectomy. A total wrist
arthroplasty is a salvage procedure but is not indicated in this young manual laborer. This
patient population experiences a high rate of failure due to mechanical prosthetic
complications.

94. The radiographs shown in Figures 1 and 2 reveal squamous cell carcinoma of the
thumb involving the distal phalanx. Following biopsy confirmation, what would be the most
appropriate course of management?

Figure 1 Figure 2

1. Curettage and bone grafting


2. External beam radiation
3. Ray amputation of the thumb
4. Interphalangeal (IP) joint disarticulation
Preferred Answer: 4
Squamous cell carcinoma of the fingertip/nail region is uncommon but remains the most common
malignancy in the hand. A high degree of suspicion is needed to diagnose this condition. Biopsy and
radiographs are necessary initially. The subsequent treatment depends on the extent of the lesion at
the time of presentation. Treatment can vary from Mohs micrographic surgery (MMS) to digital
amputation.

UI // UNAIR // UNPAD // UNHAS // UNS // UGM // UB // UNUD // USU


Hand and Wrist Self-Assessment Examination 2019

Amputation is recommended when bone involvement is present. In this patient, because the distal
phalanx tip is involved and no further bone involvement proximally was observed, an amputation at
the IP joint level is recommended. More proximal involvement would require a more proximal
amputation level.

Curettage and bone graft is not appropriate for this malignant lesion. External beam radiation therapy
is not a first-line treatment option for this condition. Metastatic spread is uncommon. MMS is
inappropriate when bone invasion has occurred.

95. Which examination finding points toward a brachial plexus injury rather than root avulsion?

1. Winging of the scapula


2. Intact rhomboid function
3. A biceps with 0/5 strength
4. An ipsilateral clavicle fracture
PreferredAnswer: 2

A brachial plexus injury distal to the root level should leave the rhomboid muscle with intact function.
Root avulsions of C5-6 will cause weakness of the rhomboids. The branching of the dorsal scapular
nerve is proximal and often spared with upper brachial plexus injuries. Winging and biceps weakness
may occur with either injury, and an ipsilateral fracture does not differentiate an avulsion from a
brachial plexus injury.

96. Figure 1 is the radiograph of an 18-year-old right-hand-dominant man who has pain and stiffness
3 months after sustaining an injury to his dominant ring finger while playing basketball. An
examination reveals significant proximal interphalangeal (PIP) joint swelling with active and
passive PIP joint motion of 15/40 degrees of flexion. What is the best next step?

1. Supervised hand therapy


2. Hemi-hamate autograft
3. Dynamic external fixation
4. Open reduction and internal fixation (ORIF
Answer: 2

This patient has a subacute PIP joint dorsal fracture dislocation with involvement of 50% to
60% of the palmar articular surface of the base of P2. A "V sign" (Figure 2) is evident, indicating
dorsal subluxation of the joint. In some cases, an ORIF is possible, but substantial comminution
often precludes proper restoration of the critical volar buttress. Therapy is not the answer
because the joint is dorsally subluxated and must be corrected. Dynamic external fixation on its

UI // UNAIR // UNPAD // UNHAS // UNS // UGM // UB // UNUD // USU


Hand and Wrist Self-Assessment Examination 2019

own would not result in a reduced joint. The hemi-hamate autograft has proven useful in this
type of scenario and serves to restore the volar buttress of P2 using an osteochondral autograft
harvested from the distal articular aspect of the hamate at its articulation with the fourth/fifth
metacarpal bases. Intraoperative clinical photographs and a postsurgical radiograph are shown
in Figures 3 through 5.

97. The examination finding shown in Video 1 is consistent with which defect?

1. Trigger finger
2. Flexor digitorum profundus (FDP) incompetence
3. Flexor digitorum sublimis (FDS) incompetence
4. Extensor digitorum communis (EDC) incompetence
PreferredAnswer: 2

The video shows the lack of tenodesis caused by the incompetence of the FDP tendon to the ring
finger, which can be attributable to a laceration, tendon rupture, or avulsion. Note how the ring finger
stays extended (compared to the other digits) when the extensor tendons are tightened during wrist
extension. The other fingers are pulled into flexion by the FDP tendons when the extensor tendons
are relaxed during wrist extension. With the wrist flexed, the extensor mechanism to all fingers
appears to be functioning normally. Findings indicating a trigger finger would be locking in flexion of
the proximal interphalangeal joint. FDS incompetence can only be detected by blocking FDP function
of the other fingers and actively flexing the examined finger.

98. Compared with percutaneous pinning with Kirschner wires (K-wires), the treatment of metacarpal
neck fractures with cannulated intramedullary screws is associated with

1. increased rates of soft-tissue infection.


2. greater initial construct stiffness and peak load until failure.
3. a slower return of digital range of motion.
4. an earlier time to bony union.

PreferredAnswer: 2

In a biomechanical study, headless compression screws showed superior load to failure, higher
three-point bending strength, and greater strength in axial loading compared with percutaneous
K-wire fixation for metacarpal neck fractures. Headless compression screws provide greater
initial stability to allow earlier motion in the postoperative period. No data comparing infection
rates between the two methods of fixation are available; however, it is assumed that K-wires

UI // UNAIR // UNPAD // UNHAS // UNS // UGM // UB // UNUD // USU


Hand and Wrist Self-Assessment Examination 2019

placed outside of the skin would have increased rates of infection. Neither fixation method would
increase the time to healing.

99. Figures 1 through 4 show the radiographs and MRI obtained from a 40-year-old man who has a
6-week history of ring finger pain, redness, and swelling after puncturing the finger with a
toothpick. Purulent drainage from the puncture wound site grew Eikenella corrodens. The patient
was initially treated with oral antibiotics for 10 days and then intravenous (IV) antibiotics for 3
weeks. What is the best next step in treatment?

Figure 1 Figure 2

Figure 3 Figure 4

1. Continued IV antibiotics for 4 weeks


2. Continued oral antibiotics for 6 weeks
3. Bone scan, biopsy, and metastatic work-up
4. Surgical débridement along with antibiotics
Preferred Answer: 1

This patient has a septic distal interphalangeal joint, which was treated with antibiotics alone.
As a result, the patient developed osteomyelitis with bone destruction and abscess. The best
way to treat this problem is to perform surgical débridement of bone and soft tissue, along with
abscess drainage and an appropriate antibiotic regimen. Antibiotic treatment without surgery
would not be successful in eliminating this particular infection. Bone scan with biopsy is not the

UI // UNAIR // UNPAD // UNHAS // UNS // UGM // UB // UNUD // USU


Hand and Wrist Self-Assessment Examination 2019

correct option, because this problem is an infection and not a tumor, and MRI already has
provided enough diagnostic information.

100. A 63-year-old woman is seen 10 weeks after sustaining a closed minimally displaced distal radius
fracture. She has been in a short-arm cast and reports minimal pain but notes that she is having
difficulty using her thumb. An extensor pollicis longus (EPL) tendon rupture is suspected. Which
examination finding would confirm lack of EPL function?

13. Positive froment sign with the ulnar palm flat on a table
14. Weak thumb abduction with the dorsal palm flat on a table
15. Inability to flex the thumb with the palm flat on a table
16. Inability to extend the thumb with the palm flat on a table

PreferredAnswer: 4

As many as to 5% of patients with a nondisplaced distal radius fracture experience EPL rupture. The
extensor pollicis brevis (EPB) tendon often attaches to the extensor hood and sometimes continues
more distally, providing weak metacarpophalangeal extension even in the setting of EPL disruption.
However, because of the vector of its pull, the EPB cannot extend the thumb dorsal to the plane of the
palm. A positive Froment sign is noted when flexion of the thumb interphalangeal joint with an
attempted key pinch is caused by adductor pollicis weakness from ulnar nerve dysfunction.
Compression of the median nerve in the carpal tunnel affects the recurrent motor branch of the
abductor pollicis brevis, leading to thenar atrophy. The flexor pollicis longus tendon (FPL) is intact so
the patient would not have difficulty flexing the thumb with the palm flat.

UI // UNAIR // UNPAD // UNHAS // UNS // UGM // UB // UNUD // USU


Shoulder and Elbow Self-Assessment Examination 2019

2019
SELF
ASSESSMENT
EXAMINATION

SHOULDER & ELBOW


CONTRIBUTED BY:
CANDIDATES OF
INDONESIAN ORTHOPAEDIC &
COMPILED BY
TRAUMATOLOGY SURGEONS
PERIOD OF JUNE 2019

UI // UNAIR // UNPAD // UNHAS // UNS // UGM // UB // UNUD // USU


Shoulder and Elbow Self-Assessment Examination 2019

1. Figure 1 is the radiograph of a 27-year-old man who is involved in a motorcycle collision and
sustains a right femoral and tibial shaft fracture, in addition to the injury shown in Figure 1. All
fractures are closed. In addition to intramedullary nailing of the tibia and femur, appropriate
treatment and weight-bearing status of the humeral shaft fracture should include :

Figure 1

1. Fracture bracing with full weight bearing.


2. Coaptation splinting with non-weight bearing.
3. Plate fixation with full weight bearing.
4. Plate fixation with non-weight bearing for 4 weeks, followed by full weight bearing.

Preferred Answer :3
Bell and associates and Tingstad and associates both showed that immediate, full weight bearing
through the upper extremity can be safely allowed for a humeral shaft fracture fixed using a plate
and screw construct. Tingstad and associates showed no difference in malunion or nonunion rate
following non-weight bearing or full weight bearing. Because he is a polytrauma patient, the
patient would benefit from operative fixation of his humerus to expedite recovery and facilitate
mobilization. A coaptation splint and a fracture brace would be appropriate treatment options for a
non-polytrauma patient, but in neither case would full weight bearing generally be allowed
immediately following the injury.

2. A 23-year-old left-hand dominant professional football player sustains a left shoulder injury after
being tackled and lands directly on his shoulder 1 month ago. The patient was diagnosed with a
Rockwood type 2 acromioclavicular separation. Following physical therapy, his symptoms have
improved. He has good scapular control and shoulder strength. What physical examination test
would help determine the contribution of the acromioclavicular joint injury to his residual
symptoms?

1. O'Brien test
2. Hawkins-Kennedy test
3. Dynamic labral shear test (DLST)
4. Upper cut test

Preferred Answer :1

UI // UNAIR // UNPAD // UNHAS // UNS // UGM // UB // UNUD // USU


Shoulder and Elbow Self-Assessment Examination 2019

The clinical scenario describes an athlete who is recovering from a type 2 acromioclavicular joint
separation. The goal of this question is to stress the importance of the physical examination to
guide treatment decisions, as well as recovery. It is important to recognize which factors can aid
in decision making especially with type type 2 acromioclavicular joint separation injuries as the
data are still not clear as to who would best be served with surgical versus nonsurgical
management. The active compression test as described by O’Brien and associates in 1998 was
equally as effective at assessing the acromioclavicular joint as it was for assessment of the
integrity of the superior labrum. The Hawkins-Kennedy test has demonstrated utility in the
diagnosis of rotator cuff impingement, wherein the greater tuberosity comes into contact with the
coracoacromial ligament. The DLST has been described for the diagnosis of superior labral
anterior-posterior (SLAP) tears, wherein the patient reports pain and a click felt with movement of
the shoulder through an arc of abduction with the shoulder externally rotated. The upper cut test
has been described in the setting of biceps tendinopathy and SLAP tears.

3. Figure 1 is the radiograph of a 54-year-old man who has increasing weakness and numbness in
his lateral arm. No prior surgery or injury is reported. What is the most appropriate next
diagnostic test?

Figure 1

1. MRI of the shoulder


2. MRI of the cervical spine
3. CT scan of the chest
4. Radiograph of the chest

Preferred Answer :2
The radiograph reveals a Charcot neuropathic shoulder. The atraumatic destruction of the humeral
head is concerning for a neuropathic etiology and warrants MR imaging of the cervical spine to
evaluate for the presence of a syrinx. Shoulder arthroplasty in the setting of a neuropathic joint is
challenging given the local bone and soft-tissue loss, in addition to the loss of protective sensation.
The scant literature on the use of shoulder arthroplasty in these challenging patients reports an
improvement in pain but only modest improvements in shoulder function.

4. A 32-year-old man sustains elbow trauma in a motor vehicle collision and has medial elbow pain,
swelling, and bruising. A 3D-CT scan with representative images are shown in Figures 1 and 2.
What function does the ligamentous structure attached to the ulnar fracture fragment provide?

UI // UNAIR // UNPAD // UNHAS // UNS // UGM // UB // UNUD // USU


Shoulder and Elbow Self-Assessment Examination 2019

Figure 1 Figure 2

P 1. Tertiary restraint to posterior subluxation of the ulna at the ulnohumeral joint


r 2. Primary restraint to valgus stress at the elbow
e 3. econdary restraint to posterior elbow subluxation
f 4. Secondary restraint to anterior subluxation of the radial head
e
rred Answer :2
The medial collateral ligament of the elbow inserts into the sublime tubercle, which is shown
as a fracture fragment on the 3D-CT scan. The medial collateral ligament is the primary
restraint to valgus stress of the elbow. Secondary restraints to elbow instability are the
radiohumeral articulation, the common flexor-pronator tendon, the common extensor tendon,
and the elbow capsule. The primary restraint to posterolateral rotatory instability is the lateral
collateral ligament that originates on the lateral epicondyle and inserts on the crista
supinatoris of the ulna. There are no described "tertiary" restraints to the ulnohumeral joint.
The anteromedial coronoid facet is part of the coronoid, which extends more lateral and
anterior than the anteromedial facet. The anteromedial facet represents the critical weight-
bearing portion of the ulnohumeral joint. Damage to this structure causes posteromedial
subluxation that often results in severe progressive arthritis. The coronoid is the larger
structure of which the anteromedial coronoid facet is a portion. The posteromedial coronoid
facet does not appear to be critical in weight bearing. The radial notch is not associated with
increased stress with weight bearing. The treatment of displaced fractures of this structure is
ORIF utilizing buttress plating. Closed treatment is acceptable only for nondisplaced fractures
with appropriate radiographic follow-up. Suture fixation is not advocated because of
inadequate strength.

5. A 32-year-old man sustains elbow trauma in a motor vehicle collision and has medial elbow pain,
swelling, and bruising. A 3D-CT scan with representative images are shown in Figures 1 and 2.
The critical weight-bearing portion of the elbow joint that is damaged in this fracture is the?

Figure 1 Figure 2

1. anteromedial coronoid facet.


2. osteromedial olecranon facet.
3. coronoid.
4. radial notch.
UI // UNAIR // UNPAD // UNHAS // UNS // UGM // UB // UNUD // USU
Shoulder and Elbow Self-Assessment Examination 2019

Preferred Answer :1
The medial collateral ligament of the elbow inserts into the sublime tubercle, which is shown as a
fracture fragment on the 3D-CT scan. The medial collateral ligament is the primary restraint to
valgus stress of the elbow. Secondary restraints to elbow instability are the radiohumeral
articulation, the common flexor-pronator tendon, the common extensor tendon, and the elbow
capsule. The primary restraint to posterolateral rotatory instability is the lateral collateral ligament
that originates on the lateral epicondyle and inserts on the crista supinatoris of the ulna. There
are no described "tertiary" restraints to the ulnohumeral joint.
The anteromedial coronoid facet is part of the coronoid, which extends more lateral and anterior
than the anteromedial facet. The anteromedial facet represents the critical weight-bearing portion
of the ulnohumeral joint. Damage to this structure causes posteromedial subluxation that often
results in severe progressive arthritis. The coronoid is the larger structure of which the
anteromedial coronoid facet is a portion. The posteromedial coronoid facet does not appear to be
critical in weight bearing. The radial notch is not associated with increased stress with weight
bearing. The treatment of displaced fractures of this structure is ORIF utilizing buttress plating.
Closed treatment is acceptable only for nondisplaced fractures with appropriate radiographic
follow-up. Suture fixation is not advocated because of inadequate strength.

6. A 32-year-old man sustains elbow trauma in a motor vehicle collision and has medial elbow pain,
swelling, and bruising. A 3D-CT scan with representative images are shown in Figures 1 and 2.
Definitive treatment of this fracture should consist of?

Figure 1 Figure 2

1. closed reduction, limited immobilization (1-2 weeks), and early functional


rehabilitation.
2. limited immobilization in a long-arm cast (4 weeks) and early functional rehabilitation.
3. open reduction and internal fixation (ORIF).
4. open reduction, capsular repair, and suture fixation of the bony fragment and ligament.
Preferred Answer :3
The medial collateral ligament of the elbow inserts into the sublime tubercle, which is shown as a
fracture fragment on the 3D-CT scan. The medial collateral ligament is the primary restraint to
valgus stress of the elbow. Secondary restraints to elbow instability are the radiohumeral
articulation, the common flexor-pronator tendon, the common extensor tendon, and the elbow
capsule. The primary restraint to posterolateral rotatory instability is the lateral collateral ligament
that originates on the lateral epicondyle and inserts on the crista supinatoris of the ulna. There
are no described "tertiary" restraints to the ulnohumeral joint.
The anteromedial coronoid facet is part of the coronoid, which extends more lateral and anterior
than the anteromedial facet. The anteromedial facet represents the critical weight-bearing portion
of the ulnohumeral joint. Damage to this structure causes posteromedial subluxation that often
results in severe progressive arthritis. The coronoid is the larger structure of which the
anteromedial coronoid facet is a portion. The posteromedial coronoid facet does not appear to be

UI // UNAIR // UNPAD // UNHAS // UNS // UGM // UB // UNUD // USU


Shoulder and Elbow Self-Assessment Examination 2019

critical in weight bearing. The radial notch is not associated with increased stress with weight
bearing. The treatment of displaced fractures of this structure is ORIF utilizing buttress plating.
Closed treatment is acceptable only for nondisplaced fractures with appropriate radiographic
follow-up. Suture fixation is not advocated because of inadequate strength.

7. A 45-year-old woman with diabetes has a 3-month history of atraumatic left shoulder pain and
motion loss. She previously underwent treatment with nonsteroidal anti-inflammatory medication
and a home stretching program, experiencing minimal relief of her symptoms. Examination
reveals loss of passive external rotation, abduction, and forward elevation without reduction in
strength. Radiographs are normal. What is the most appropriate next step?

1. MRI scan with and without contrast


2. Cortisone injection therapy with continued physical therapy (PT)
3. Closed manipulation under anesthesia
4. Arthroscopic release with manipulation under anesthesia

Preferred Answer :2
Based upon the duration of symptoms and clinical presentation, this patient would benefit from
cortisone injection therapy and continued PT. Adhesive capsulitis, most commonly an idiopathic
process that results in joint pain and loss of motion from capsular contracture, affects
approximately 2% to 5% of the general population. The process typically affects middle-age
women. There are secondary causes such as previous trauma and fractures, as well as
associated medical conditions such as diabetes, stroke, and cardiac and thyroid disease. Debate
remains as to whether a genetic predisposition for the development of adhesive capsulitis exists,
despite the increased frequency noted in twin studies. Although the underlying etiology and
pathophysiology are not well understood, the consensus is that synovial inflammation and
capsular fibrosis result in pain and joint volume loss. It is hypothesized that in patients with
diabetes, an increased rate of glycosylation and cross-linking of the shoulder capsule raises the
incidence of frozen shoulder. For this patient, history reveals a short duration of symptoms that
did not improve with nonsurgical modalities. Clinically, the patient has reduced passive range of
motion, particularly with external and internal rotation and forward elevation. Radiographs are
usually obtained to exclude other causes of shoulder pain such as glenohumeral arthrosis,
malignancy, calcific tendonitis, impingement, and acromioclavicular degeneration. If pain and
stiffness persist beyond 6 months, closed manipulation may be an option. Complications
associated with this modality may include humerus fracture, dislocation, hematoma, rotator cuff
and labral tears, and brachial plexus injury. Some surgeons advocate arthroscopic capsular
release to allow for examination of concomitant pathology and controlled release of capsular
tissue, with the potential for reduced required force when performing the manipulation portion of
the procedure. This modality may be appropriate after an initial treatment with PT. Controversy
remains as to whether posterior capsular release should be routinely performed because studies
have shown outcomes to be similar with anterior and combined approaches. Therapy should be
initiated early after intervention, with some surgeons advocating admission to the hospital with
inpatient therapy for pain management and compliance.

8. Figures 1 through 3 are the radiographs of a 55-year-old woman who fell on her outstretched
right arm, resulting in acute elbow pain and swelling. On examination, she has lateral elbow
bruising with mechanical block to supination and pronation. She has no medial tenderness. She
is unable to extend her elbow within 60° of full extension. During surgery utilizing a direct lateral
approach, the surgeon observes a completely bare lateral epicondyle. After surgical repair, a
stable and congruent joint is achieved. Initial postoperative rehabilitation should include ?

UI // UNAIR // UNPAD // UNHAS // UNS // UGM // UB // UNUD // USU


Shoulder and Elbow Self-Assessment Examination 2019

Figure 1 Figure 2 Figure 3

1. 3 weeks of cast immobilization.


2. Elbow extension exercises with the forearm supinated.
3. Elbow extension exercises with the forearm pronated.
4. Elbow extension exercises with the forearm in neutral rotation.

Preferred answer :3
Radial head fractures are thought to occur as a result of valgus posterolateral rotary load across
the elbow, although the mechanism can certainly vary. Minimally or nondisplaced fractures without
any clinical instability or block to motion can often be successfully managed non-surgically.
Fractures with >2 mm of displacement or fragments that block motion require surgical repair. A
critical aspect during surgery is identifying concomitant injury to the lateral collateral ligament
complex (LCL). When encountered, the LCL will be avulsed from its origin from the lateral
epicondyle, resulting in a bare area. After the radial head is either reduced and fixed or replaced
(Figures 4 and 5), the LCL should be repaired back to its anatomic origin. Postoperatively, the
surgeon must communicate to the therapist that elbow extension exercises should be performed
with the forearm in pronation as a result of the compromised LCL. Elbow extension exercises in
supination and neutral are recommended for compromised medial collateral ligament or combined
medial and lateral ligament injury, respectively. Without any medial elbow bruising, swelling, or
tenderness, it is unlikely that the patient has an injury to the medial collateral ligament.

Figure 4 Figure 5

9. MRI results are shown in Figure 1 for a 22-year-old, right-hand-dominant collegiate athlete who
reports a 6-month history of progressive weakness in his right arm. He denies any specific

UI // UNAIR // UNPAD // UNHAS // UNS // UGM // UB // UNUD // USU


Shoulder and Elbow Self-Assessment Examination 2019

traumatic event, has altered his weight-lifting activities, and has tried over-the-counter ibuprofen
without having a benefit. No appreciable deformity or atrophy is found on examination of the
upper extremities. He demonstrates full active shoulder range of motion, and there is no
weakness with abduction in the plane of the scapula. Belly press test findings are normal, but
weakness is seen in external rotation with the arm in adduction. He does not demonstrate
anterior apprehension, and there is no instability with load and shift testing. Radiographs are
unremarkable. What is the best surgical option?
Figure 1

1. Arthroscopic labral debridement and biceps tenodesis


2. Shoulder arthroscopy with undersurface cuff debridement and acromioplasty
3. Cyst decompression at the spinoglenoid notch with possible labral repair
4. Cyst decompression at the suprascapular notch with possible labral repair

Preferred Answer :3
This patient’s clinical and MRI findings are consistent with a posterior paralabral cyst with
compression of the suprascapular nerve, specifically at the spinoglenoid notch. Compression of
the suprascapular nerve can occur at either the suprascapular or spinoglenoid notch.
Compression of the nerve at the suprascapular notch affects innervation to both the
supraspinatus and infraspinatus muscles, resulting in weakness in both shoulder abduction and
external rotation. However, compression at the spinoglenoid notch only affects innervation to the
infraspinatus muscle, resulting in isolated weakness in external rotation.
Compression at the spinoglenoid notch often is seen in overhead athletes, and studies have
shown associated posterior labral tears (Piatt and associates). Several studies have addressed
nonsurgical and surgical treatment options. The treatment decision should focus on the
underlying cause (Martin and associates)—in this patient, the cyst. Nonsurgical treatment in the
presence of a known lesion has been associated with a higher failure rate than addressing the
lesion, which can result in functional improvement (Chen and associates, Cummins and
associates). The best response in this scenario is decompression of the cyst at the spinoglenoid
notch with possible labral repair.

10. A 17-year-old high school football player sustains a traumatic anterior shoulder dislocation,
resulting in a small bony Bankart lesion and small Hill-Sachs lesion. The patient undergoes an
arthroscopic Bankart repair with incorporation of the bone fragment and returns to play football
the following year. He has a recurrent dislocation at football practice but decides to finish the
football season before considering additional treatment. He sustains nine additional dislocations,
with the last dislocation occurring while sleeping.
What diagnostic test is most appropriate when planning revision surgery?

UI // UNAIR // UNPAD // UNHAS // UNS // UGM // UB // UNUD // USU


Shoulder and Elbow Self-Assessment Examination 2019

1. CT scan with 3D reconstructions


2. Ultrasonography
3. MRI scan
4. Fluoroscopically-guided arthrogram

Preferred Answer : A
A failed bony Bankart repair with multiple dislocations can further erode the anteroinferior glenoid,
changing the sagittal morphology of the glenoid into an “inverted pear.” Quantitative bone loss is
best evaluated by CT scan with 3-D reconstructions and subtraction of the humeral head. MRI
and ultrasonography can assist in evaluating soft-tissue injury, but they are not as helpful in
determining bone loss compared with a CT scan. An arthrogram alone is not sufficient to evaluate
bone loss. Bone loss >30% necessitates glenoid augmentation with either a Latarjet procedure or
iliac crest bone grafting. A revision arthroscopic or open Bankart repair with capsular shift or
remplissage do not address bone loss. The Latarjet procedure can effectively restore stability with
glenoid bone loss and after failed stabilizing procedures. Patients with pain before surgery are
more likely to have pain after surgery. Age and activity level are lesser influences on satisfaction.

11. A 17-year-old high school football player sustains a traumatic anterior shoulder dislocation,
resulting in a small bony Bankart lesion and small Hill-Sachs lesion. The patient undergoes an
arthroscopic Bankart repair with incorporation of the bone fragment and returns to play football
the following year. He has a recurrent dislocation at football practice but decides to finish the
football season before considering additional treatment. He sustains nine additional dislocations,
with the last dislocation occurring while sleeping.
The patient has eroded one-third of the inferior glenoid surface area. What is the most
appropriate surgical treatment?

1. Revision arthroscopic Bankart repair with capsular shift


2. Open Bankart repair with capsular shift
3. Repair of infraspinatus tendon into the Hill-Sachs defect (remplissage procedure)
4. Coracoid transfer to the glenoid (Latarjet procedure)

Preferred Answer : D
A failed bony Bankart repair with multiple dislocations can further erode the anteroinferior
glenoid, changing the sagittal morphology of the glenoid into an “inverted pear.” Quantitative
bone loss is best evaluated by CT scan with 3-D reconstructions and subtraction of the humeral
head. MRI and ultrasonography can assist in evaluating soft-tissue injury, but they are not as
helpful in determining bone loss compared with a CT scan. An arthrogram alone is not sufficient
to evaluate bone loss. Bone loss >30% necessitates glenoid augmentation with either a Latarjet
procedure or iliac crest bone grafting. A revision arthroscopic or open Bankart repair with
capsular shift or remplissage do not address bone loss. The Latarjet procedure can effectively
restore stability with glenoid bone loss and after failed stabilizing procedures. Patients with pain
before surgery are more likely to have pain after surgery. Age and activity level are lesser
influences on satisfaction.

12. Figures 1 and 2 are the radiographs of a 30-year old recreational polo player who sustained an
injury to his right shoulder following a fall from a horse. He denies any prior injuries to the
shoulder. He reports pain in the superior aspect of the shoulder and has an abrasion over the
lateral acromion. Which anatomic structure is most important for maintaining the anterior-
posterior stability of the injured joint?

UI // UNAIR // UNPAD // UNHAS // UNS // UGM // UB // UNUD // USU


Shoulder and Elbow Self-Assessment Examination 2019

Figure 1 Figure 2

1. Conoid ligament
2. Trapezoid ligament
3. Inferior acromioclavicular (AC) ligament
4. Posterosuperior AC ligament

Preferred Answer :4
The patient has radiographic evidence of a grade III AC joint separation. Treatment of grade III
AC separations remains controversial. In a grade III AC separation, both the AC joint capsule, as
well as the coracoclavicular ligaments (conoid and trapezoid) are disrupted. The coracoclavicular
ligaments primarily provide superior/inferior stability to the clavicle, and the AC joint
capsule/ligament complex provides anterior-posterior stability. Within this capsule complex,
biomechanical studies have shown that the posterosuperior portion is most crucial for maintaining
anterior-posterior stability of the AC joint and should be preserved during AC joint resection
procedures. The inferior AC capsule/ligament does not play a significant role in AC joint horizontal
stability.

13. A 55-year-old woman develops posttraumatic arthritis in the elbow following a distal humerus
fracture. What is the most likely mid-term (5-10 years after surgery) complication following
semiconstrained total elbow arthroplasty (TEA)?

1. Bushing wear
2. Infection
3. Aseptic component loosening
4. Omponent fracture

Preferred Answer :1
TEA has been described for posttraumatic arthritis of the elbow and typically involves a young
patient population with multiple previous operations on the affected elbow. Morrey and
Schneeberger found aseptic component loosening to be uncommon (<10% of patients) and
usually occurring >10 years after surgery. Prosthetic fracture, usually of the ulnar component, is
also a late-term finding. Infection is the most common mode of early failure but usually occurs
within the first 5 years and has an overall rate of approximately 5%. Bushing wear has been
reported as the most common cause of mechanical TEA failure in this population at intermediate-
term follow-up.

UI // UNAIR // UNPAD // UNHAS // UNS // UGM // UB // UNUD // USU


Shoulder and Elbow Self-Assessment Examination 2019

14. Figure 1 is the MRI scan of a 25-year-old left-hand dominant minor league pitcher who has elbow
pain during pitching that has gotten worse for the past several months. He fails nonoperative
treatment and undergoes surgery to address the problem. What is the most common
complication of this procedure?

Figure 1

1. Ulnar nerve neuropraxia


2. Flexor pronator mass avulsion
3. Posterolateral rotatory instability
4. Symptomatic hardware

Preferred Answer :1
The MRI scan shows evidence of a medial collateral ligament (MCL) injury. In a patient with a
chronic MCL injury that has failed non-operative treatment, MCL reconstruction would be
indicated. Initial MCL reconstruction technique involved routine transposition of the ulnar nerve
and detachment of the flexor-pronator mass. Subsequent modification of surgical techniques has
been made to attempt to minimize complications by avoiding routine ulnar nerve transposition
and performing a muscle-splitting approach. Regardless, transient ulnar nerve neuropraxia
remains the most common complication, and patients should be counseled about its occurrence.
Flexor pronator mass avulsion is more likely with a muscle-detaching approach but is not more
common than ulnar nerve neuropraxia. Posterolateral rotatory instability is a complication of
lateral collateral ligament repair or reconstruction, not medial collateral ligament reconstruction.
Symptomatic hardware is not a common complication.

15. Figure 1 is the radiograph of a right-hand-dominant 70-year-old woman who arrives at the
emergency department with acute left shoulder pain following a fall down a flight of stairs. She
expresses acute diffuse left shoulder pain and swelling. Prior to her injury, she had fully active,
painless shoulder range of motion.
Which radiographic parameter places this patient at the highest risk for osteonecrosis?

UI // UNAIR // UNPAD // UNHAS // UNS // UGM // UB // UNUD // USU


Shoulder and Elbow Self-Assessment Examination 2019

Figure 1

1. 3-part displaced valgus-type fracture pattern


2. 3-part displaced varus-type fracture pattern
3. anterior fracture extension disrupting the bicipital groove
4. Posteromedial metaphyseal head extension of <8 mm

Preferred Answer :4
Fractures of the proximal humerus are now the third most common fracture in patients >60 years
of age. This patient sustained a displaced, commonly described 3-part/4-part proximal humerus
fracture. The number of fracture fragments and angulation, as initially described by Codman and
then Neer, does not necessarily help to predict risk for subsequent AVN. Although the main
blood supply to the humeral head historically was believed to be a branch from the anterior
circumflex, adequate perfusion can remain through the posteromedial calcar following trauma.
Hertel and associates reported that the most accurate predictor of ischemia was whether the
length of the metaphyseal head extension for the calcar segment was <8 mm.
Locking plates have provided surgical alternatives to many unstable fracture patterns previously
considered ominous. Although much enthusiasm remains for this implant choice, a relatively high
level of complications has been reported with their use. A multicenter study reported a 14%
incidence of intra-articular screw perforation as the most common complication.
When patients are deemed poor candidates for head preservation treatment, both
hemiarthroplasty and rTSA can provide successful results when applied and performed
appropriately. An increasing body of evidence appears to support consideration of a reverse
prosthesis for older patients. A crucial aspect of hemiarthroplasty success is anatomic healing of
the tuberosities around the implant. If the greater tuberosity displaces or reabsorbs, patients
experience significant loss of active motion. The level of pain, however, is unpredictable.
Although beneficial in a reverse prosthesis, anatomic tuberosity position does not appear to be
as crucial. Depending on the definition of “complication,” some researchers have reported a
higher complication rate for the reverse prosthesis.
If posttraumatic osteonecrosis develops following head preservation treatment, conversion to an
anatomic shoulder arthroplasty can provide good success in motion and function, particularly
when the tuberosities heal in a relatively anatomic position. However, several researchers, most
recently Moineau and associates, report suboptimal results when the greater tuberosity is
positioned in a substantial amount of varus that necessitates osteotomy. In this scenario, rTSA
should be considered.

UI // UNAIR // UNPAD // UNHAS // UNS // UGM // UB // UNUD // USU


Shoulder and Elbow Self-Assessment Examination 2019

16. Figure 1 is the radiograph of a right-hand-dominant 70-year-old woman who arrives at the
emergency department with acute left shoulder pain following a fall down a flight of stairs. She
expresses acute diffuse left shoulder pain and swelling. Prior to her injury, she had fully active,
painless shoulder range of motion.
What is the most common complication following open reduction and locking plate
osteosynthesis of this injury?

Figure 1

1. Intra-articular screw penetration


2. Posttraumatic avascular necrosis (AVN)
3. Surgical neck nonunion
4. Subacromial plate impingement

Preferred Answer :1
Fractures of the proximal humerus are now the third most common fracture in patients >60 years
of age. This patient sustained a displaced, commonly described 3-part/4-part proximal humerus
fracture. The number of fracture fragments and angulation, as initially described by Codman and
then Neer, does not necessarily help to predict risk for subsequent AVN. Although the main
blood supply to the humeral head historically was believed to be a branch from the anterior
circumflex, adequate perfusion can remain through the posteromedial calcar following trauma.
Hertel and associates reported that the most accurate predictor of ischemia was whether the
length of the metaphyseal head extension for the calcar segment was <8 mm.
Locking plates have provided surgical alternatives to many unstable fracture patterns previously
considered ominous. Although much enthusiasm remains for this implant choice, a relatively high
level of complications has been reported with their use. A multicenter study reported a 14%
incidence of intra-articular screw perforation as the most common complication.
When patients are deemed poor candidates for head preservation treatment, both
hemiarthroplasty and rTSA can provide successful results when applied and performed
appropriately. An increasing body of evidence appears to support consideration of a reverse
prosthesis for older patients. A crucial aspect of hemiarthroplasty success is anatomic healing of
the tuberosities around the implant. If the greater tuberosity displaces or reabsorbs, patients
experience significant loss of active motion. The level of pain, however, is unpredictable.
Although beneficial in a reverse prosthesis, anatomic tuberosity position does not appear to be
as crucial. Depending on the definition of “complication,” some researchers have reported a
higher complication rate for the reverse prosthesis.
If posttraumatic osteonecrosis develops following head preservation treatment, conversion to an
anatomic shoulder arthroplasty can provide good success in motion and function, particularly
when the tuberosities heal in a relatively anatomic position. However, several researchers, most
recently Moineau and associates, report suboptimal results when the greater tuberosity is

UI // UNAIR // UNPAD // UNHAS // UNS // UGM // UB // UNUD // USU


Shoulder and Elbow Self-Assessment Examination 2019

positioned in a substantial amount of varus that necessitates osteotomy. In this scenario, rTSA
should be considered.
17. Figure 1 is the radiograph of a right-hand-dominant 70-year-old woman who arrives at the
emergency department with acute left shoulder pain following a fall down a flight of stairs. She
expresses acute diffuse left shoulder pain and swelling. Prior to her injury, she had fully active,
painless shoulder range of motion.
When considering arthroplasty options, which statement regarding the use of hemiarthroplasty or
reverse total shoulder arthroplasty (rTSA) is most accurate?

Figure 1

1. Midterm objective outcomes following hemiarthroplasty are superior.


2. Results following rTSA are less dependent on anatomic tuberosity healing.
3. Use of rTSA is associated with a lower complication rate.
4. Midterm visual analog pain scores following hemiarthroplasty are lower.

Preferred Answer :2
Fractures of the proximal humerus are now the third most common fracture in patients >60 years
of age. This patient sustained a displaced, commonly described 3-part/4-part proximal humerus
fracture. The number of fracture fragments and angulation, as initially described by Codman and
then Neer, does not necessarily help to predict risk for subsequent AVN. Although the main
blood supply to the humeral head historically was believed to be a branch from the anterior
circumflex, adequate perfusion can remain through the posteromedial calcar following trauma.
Hertel and associates reported that the most accurate predictor of ischemia was whether the
length of the metaphyseal head extension for the calcar segment was <8 mm.
Locking plates have provided surgical alternatives to many unstable fracture patterns previously
considered ominous. Although much enthusiasm remains for this implant choice, a relatively high
level of complications has been reported with their use. A multicenter study reported a 14%
incidence of intra-articular screw perforation as the most common complication.
When patients are deemed poor candidates for head preservation treatment, both
hemiarthroplasty and rTSA can provide successful results when applied and performed
appropriately. An increasing body of evidence appears to support consideration of a reverse
prosthesis for older patients. A crucial aspect of hemiarthroplasty success is anatomic healing of
the tuberosities around the implant. If the greater tuberosity displaces or reabsorbs, patients
experience significant loss of active motion. The level of pain, however, is unpredictable.
Although beneficial in a reverse prosthesis, anatomic tuberosity position does not appear to be
as crucial. Depending on the definition of “complication,” some researchers have reported a
higher complication rate for the reverse prosthesis.

UI // UNAIR // UNPAD // UNHAS // UNS // UGM // UB // UNUD // USU


Shoulder and Elbow Self-Assessment Examination 2019

If posttraumatic osteonecrosis develops following head preservation treatment, conversion to an


anatomic shoulder arthroplasty can provide good success in motion and function, particularly
when the tuberosities heal in a relatively anatomic position. However, several researchers, most
recently Moineau and associates, report suboptimal results when the greater tuberosity is
positioned in a substantial amount of varus that necessitates osteotomy. In this scenario, rTSA
should be considered.

18. Figures 1 through 4 are the radiographs and CT scans of a 78-year-old right-hand dominant man
with a recent-onset painful left shoulder and limited range of motion. He was reaching overhead
and felt a pop, which resulted in severe pain and dysfunction. The patient underwent an
anatomic total shoulder arthroplasty 5 years prior for glenohumeral osteoarthritis. A select axial
CT image from before the index surgery is seen in Figure 4. The patient has not had any fevers
or systemic symptoms of infection. ESR, CRP, and CBC levels remain normal. What
preoperative factors are most predictive of the complication experienced by this patient?

Figure 1 Figure 2 Figure 3 Figure 4

1. Patient age at the time of index surgical procedure


2. Presence of a small repairable full-thickness rotator cuff tear
3. Use of pegged all-polyethylene cemented glenoid component
4. Severe baseline glenoid erosion and posterior humeral subluxation

Preferred Answer :4
This patient has an anatomic total shoulder arthroplasty with a completely dislocated glenoid
component. The glenoid component can be seen in the posterior axillary pouch, as evidenced by
the radiographic marker seen best on the axial cut CT. Preoperative factors that influence the
outcome of an anatomic total shoulder arthroplasty resulting in the need for revision surgery for
failed glenoid component include preoperative fixed posterior humeral head subluxation and
moderate to severe eccentric glenoid erosion. Walch and associates describe three patterns of
glenoid component migration including superior tilting, subsidence, and posterior tilting. Superior
tilting was associated with implant position and rotator cuff integrity. Subsidence was associated
with aggressive reaming for correction of glenoid version, which does not maintain the
subchondral bone support. Posterior subluxation was associated with glenoid erosion and
posterior humeral head subluxation.

UI // UNAIR // UNPAD // UNHAS // UNS // UGM // UB // UNUD // USU


Shoulder and Elbow Self-Assessment Examination 2019

19. When a patient has recurrent anterior shoulder instability, a bony glenoid reconstructive
procedure should be considered in which clinical setting?

1. Associated humeral avulsion of the glenohumeral ligament (HAGL) lesion


2. Non-engaging Hill-Sachs lesion
3. Glenoid bone loss of at least 25%
4. Anterior labral periosteal sleeve avulsion (ALPSA)
Preferred Answer :3
HAGL lesions may initially be treated without surgery. Recurrent instability in the setting of a
HAGL lesion may be treated with a soft-tissue repair. A non-engaging or non-tracking Hill-Sachs
lesion may be treated with an anterior soft-tissue (Bankart) repair. A tracking or engaging lesion
may be treated with a bony glenoid procedure or a soft-tissue procedure plus remplissage. An
ALPSA lesion may be treated with a soft-tissue procedure unless it is associated with a glenoid
bony defect >25%. A glenoid bony defect >25% is associated with substantially higher
recurrence than defects <20%, and consideration for bony glenoid reconstruction is advised.
Consideration of bone augmentation procedures with less severe glenoid bone loss may be
considered in collision athletes.

20. Placement of the most distal interlocking screw seen in the radiographs provided in Figures 1 and
2 would most likely result in what clinical examination finding?

Figure 1 Figure 2

1. Weakness of elbow flexion


2. Inability to extend thumb interphalangeal (IP) joint
3. Decreased sensation over radial/volar forearm
4. Decreased sensation of small and ring fingers

Preferred Answer :3
Blunt dissection and soft-tissue protection is warranted with distal interlocking screw placement
following humeral intramedullary nailing. The most distal locking screw in this intramedullary nail
construct was placed from anterior to posterior, passing through the distal portion of the biceps
and brachialis muscle bellies. The musculocutaneous nerve which continues as the sensory lateral
antebrachial cutaneous (LABC) nerve is at risk, as it lies between these two muscles. Injury to the
LABC nerve results in decreased sensation over the radial volar aspect of the forearm. Malrotation
of the nail, producing a more anteromedial starting point for the anterior-to-posterior screw, can

UI // UNAIR // UNPAD // UNHAS // UNS // UGM // UB // UNUD // USU


Shoulder and Elbow Self-Assessment Examination 2019

lead to a path that intersects with the median nerve and brachial artery. More commonly, the
median nerve can be injured with overpenetration of the medial cortex with a lateral-to-medial
directed screw. Median nerve injury would affect innervations of the flexor
digitorumsuperficialisand profundus to the index finger (among other motors). Although the
dissection violates the muscle belly of these two elbow flexors, measurable weakness in elbow
flexion is not typically seen. The radial nerve has already provided function to triceps (elbow
extension) proximal to this level and lies sufficiently lateral to be more of a concern with a lateral-
to-medial screw placement (thumb IP extension). The ulnar nerve (decreased sensation of small
and ring fingers) is further medial at this level and would similarly be at risk with a lateral-to-medial
interlocking screw.

21. A patient sustains a displaced diaphysealhumerus fracture following a motor vehicle accident.
Open reduction internal fixation is indicated due to concomitant lower extremity trauma and is
planned through an anterior approach. Which intramuscular interval is exploited during the deep
dissection of the mid-humerus in this approach?

1. Lateral head of triceps (radial nerve) and brachialis (musculocutaneous nerve)


2. Lateral head of the triceps (radial nerve) and biceps brachii (musculocutaneous nerve)
3. Lateral brachialis (radial nerve) and medial brachialis (musculocutaneous nerve)
4. Brachialis (musculocutaneous nerve) and coracobrachialis (musculocutaneous nerve)
Preferred Answer :3
The anterior approach to the mid-humerus courses along the lateral margin of the biceps brachii.
This muscle is swept medially allowing exposure of the brachialis. The brachialis has a dual
innervation, with the lateral fibers innervated by the radial nerve and the medial fibers innervated
by the musculocutaneous nerve. The humerus is exposed by splitting this muscle in its midline.
The lateral head of the triceps resides in the posterior compartment of the arm and is not
involved in the anterior approach to the humerus. The interval between brachialis and
coracobrachialis is not an internervous plane, as both muscles are supplied by the
musculocutaneous nerve.

22. In rotator cuff tear arthropathy with pseudoparalysis, forward elevation of the humerus away from
the body is prohibited because of ?

1. Deltoid atony.
2. Loss of the glenoid concavity.
3. Loss of the humeral head depression of the biceps tendon.
4. Loss of compressive force on the humeral head.

Preferred Answer :4
The rotator cuff serves as a humeral head compressor that stabilizes the humeral head in the
glenoid concavity so that the deltoid can convert a vertical force into abduction and forward
elevation. The deltoid functions normally in patients with chronic rotator cuff arthropathy, so no
atony is present. Glenoid concavity can be lost over time, but this is not the primary mechanism
for failure of elevation. The biceps tendon does not serve as a humeral head compressor and
does not prevent proximal migration of the shoulder when it is present.

23. Figures 1 and 2 are the radiograph and axial CT scan of a 75-year-old woman with diffuse
superior shoulder pain 5 months after an uneventful reverse shoulder arthroplasty. She denies
trauma, but felt a "pop" when reaching overhead. She had initially done well postoperatively. On
physical examination, she has decreased active forward flexion with pain and diffuse superior

UI // UNAIR // UNPAD // UNHAS // UNS // UGM // UB // UNUD // USU


Shoulder and Elbow Self-Assessment Examination 2019

tenderness along the scapular spine and acromion. There are no signs or symptoms of infection.
What is the best next step in management?

Figure 1 Figure 2

1. Application of bone stimulator


2. Open reduction and internal fixation (ORIF)
3. Physical therapy for deltoid strengthening
4. Sling immobilization for 6 weeks

Preferred Answer :4
The radiograph shows a well-positioned reverse total shoulder and the axial CT image
demonstrates a minimally displaced fracture through the midacromion. Acromial fracture is a
concerning and not uncommon complication that can have devastating effects after reverse
shoulder arthroplasty, as function is highly dependent on the deltoid muscle. Fracture can occur
at the scapular spine, which may be related to placement of peripheral baseplate screws.
Acromial fracture has been reported to occur as early as 1 month postoperatively and as late as
8 years postoperatively. The reported incidence is <8%, and multiple authors note decreases in
shoulder elevation and shoulder outcome scores compared with that in patients with reverse
shoulder arthroplasty without a fracture. Patients with a fracture can complain of superior
shoulder pain that may radiate to the deltoid area and usually relate an acute onset of pain or
loss of function after initially good clinical progress. The diagnosis can be difficult to determine
and is missed on plain radiographs in up to 20% of cases. CT scans are helpful. Treatment is
typically nonoperative with sling immobilization for 6 weeks and then advancing activities as
tolerated. A bone stimulator and ORIF would not be indicated for a minimally displaced acute
acromion fracture. Continued physical therapy with strengthening would also be contraindicated.

24. A 23-year-old collegiate gymnast sustains a rupture of his medial collateral ligament of the elbow
when he falls off the parallel bars. On physical examination, he has instability to valgus stress
and tenderness along the medial elbow. Radiographs show no fracture. Which component of the
medial collateral ligament of the elbow is the dominant restraint to valgus stress?

1. Transverse ligament
2. Anterior band of the medial collateral ligament
3. Posterior band of the medial collateral ligament
4. Posterior capsule

UI // UNAIR // UNPAD // UNHAS // UNS // UGM // UB // UNUD // USU


Shoulder and Elbow Self-Assessment Examination 2019

Preferred Answer :2
The anterior bundle of the medial collateral ligament is the prime stabilizer against valgus stress.
The posterior bundle, which originates on the medial epicondyle and inserts broadly along the
medial edge of the trochlea from the sublime tubercle posteriorly, has stress only in elbow
flexion. The transverse band, which originates on the posteromedial olecranon and inserts on the
sublime tubercle, deepens the trochlea, but neither the band nor the posterior capsule provides
significant restraint. The lateral collateral ligament, which originates from the lateral epicondyle
and inserts on the crista supinatoris of the ulna, is the prime stabilizer of varus stress and
posterolateral rotatory subluxation.

25. A 65-year-old man who underwent an uncomplicated reverse total shoulder arthroplasty (rTSA)
to treat rotator cuff arthropathy 2 years ago has a routine follow-up visit in your clinic. A
radiograph at 2-year follow-up is shown in Figure 1. He denies shoulder pain and dysfunction
and constitutional symptoms, and his clinical examination findings are benign. Based upon the
present radiologic evaluation, what is the next most appropriate step?

Figure 1

1. Revision rTSA
2. Conversion to hemiarthroplasty
3. Continued observation
4. Infection work-up with screening labs and joint aspiration

Preferred Answer : C
Based upon the patient’s clinical examination and symptoms, continued observation is most
appropriate. The remaining options are not indicated. The radiograph reveals scapular notching,
one of the more common complications specific to rTSA. Notching is caused by repeated contact
between the humeral component and/or humerus and the inferior pillar of the scapular neck.
Generation of particulate debris from this interaction can result in osteolysis with the potential for
screw and base plate failure. The overall incidence of notching has been reported to be between
51% and 96%. This nearly ubiquitous finding has been attributed to implant positioning, altered
glenoid and humeral anatomy, and duration of implantation. Recent studies that indicate
increased lateral offset, increased glenosphere size, and inferior positioning of the base plate
may reduce the incidence of scapular notching.

26. Figure 1 is the MRI arthrogram of a 21-year-old professional baseball pitcher who complains of
right elbow pain after pitching a game 3 months ago. He had initially been treated with rest and

UI // UNAIR // UNPAD // UNHAS // UNS // UGM // UB // UNUD // USU


Shoulder and Elbow Self-Assessment Examination 2019

forearm strengthening. He now complains of persistent pain along his medial elbow during the
long toss portion of his throwing program. What is the most appropriate treatment at this time?
Figure 1

1. Arthroscopic debridement and capsular release


2. Common flexor pronator re-attachment
3. Medial ulnar collateral ligament (MUCL) reconstruction
4. Extensor carpi radialis brevis release

Preferred Answer :3
MUCL reconstruction, using either ipsilateral palmaris longus, hamstring autograft, or allograft
tendon is indicated for ulnar collateral ligament (UCL) injuries that fail nonsurgical management.
The MRI arthrogram shows a rupture of the UCL from its ulnar insertion, with the classic T sign of
contrast extravasation. The MRI study does not show any ulnohumeral or radiocapitellar
osteoarthritis to suggest radial head resection or debridement/capsular release as indicated
procedures; the patient’s symptoms are medial, precluding the need for a release of the extensor
carpi radialis brevis. Finally, the common origin of the flexor pronator mass appears intact on the
given arthrogram image.

27. The fracture seen in Figure 1 is most likely associated with injury to what ligamentous structure?

Figure 1

UI // UNAIR // UNPAD // UNHAS // UNS // UGM // UB // UNUD // USU


Shoulder and Elbow Self-Assessment Examination 2019

1. Inferior glenohumeral ligament


2. Acromioclavicular (AC) ligaments
3. Coracoclavicular ligaments
4. Coracoacromial ligament

Preferred Answer :3
The radiograph shows an extra-articular distal clavicle fracture lateral to the clavicular attachment
point of the coracoclavicular ligaments (conoid and trapezoid). However, unlike a scenario
featuring a typical Neer type I fracture, the interval between coracoid and clavicle is clearly
widened, and there is marked fracture displacement. It is clear that the coracoclavicular
ligaments must also be torn. The inferior glenohumeral ligament is important to glenohumeral
joint stability but has no effect on the relationship between clavicle and scapula. The AC
ligaments are thickenings of the AC joint capsule. They have been shown to be responsible for
90% of anteroposterior stability of the AC joint. The coracoclavicular ligaments are responsible
for 77% of stability for superior translation (as in this case). The coracoacromial ligament
connects two parts of the scapula (coracoid and acromion) and is part of the arch that supports
the rotator cuff.

28. Figure 1 is the radiograph of a 65-year-old active, right-hand-dominant woman with a 6-month
history of right shoulder pain, motion loss, and progressive weakness after undergoing a
hemiarthroplasty to address osteoarthritis 1 year ago. She denies recent trauma to her right
shoulder and denies constitutional symptoms. Her surgical wound site is benign. She can
actively forward flex to 90° degrees and abduct to 60°. Passive forward flexion and abduction are
150° and 90°, respectively.
What is the most likely cause of her symptoms?

Figure 1

1. Periprosthetic infection
2. Rotator cuff tear
3. Implant loosening
4. Implant instability

Preferred Answer :2
The radiograph reveals a rotator cuff dysfunction secondary to malpositioning of the humeral stem
and a nonanatomic humeral head. Glenohumeral kinematics have been altered, resulting in
damage to the rotator cuff, which in turn has led to impingement with the coracoacromial arch. This
single radiograph reveals excessive humeral head height, “overstuffing” of the joint, and severe

UI // UNAIR // UNPAD // UNHAS // UNS // UGM // UB // UNUD // USU


Shoulder and Elbow Self-Assessment Examination 2019

narrowing of the acromiohumeral interval. Osteolysis and implant loosening are not
radiographically apparent. An orthogonal view (axillary lateral) would be necessary to evaluate for
shoulder instability. A CT arthrogram is the most appropriate advanced imaging test in the setting
of a retained shoulder arthroplasty to evaluate the integrity of the rotator cuff. An MRI evaluation
would be obfuscated by artifact. Three-phase and indium-tagged WBC scans may be appropriate
in the setting of an occult infection evaluation but not as a test to evaluate rotator cuff injury.
In the absence of infection with rotator cuff compromise, the most appropriate procedure(s) during
revision would involve humeral component explantation and conversion to rTSA. Revision
anatomic hemiarthroplasty may provide pain relief, but function may not appreciably change
because of the unbalanced forced couples of the rotator cuff complex. Placement of a glenoid
component in the setting of an irreparable rotator cuff tear is contraindicated because rapid glenoid
loosening will occur due to eccentric loading during active shoulder motion. Resection arthroplasty
should be reserved for recalcitrant cases of infection, because this procedure does not provide
functional improvement.
In the event that frozen section analysis and positive Gram stain results indicate an infection, the
treating surgeon should remove all components, perform a thorough debridement and irrigation of
suspect tissue, implant an antibiotic spacer, and perform a second-stage reconstruction when
deemed appropriate (in light of laboratory studies, repeat shoulder aspiration, frozen section
analysis, and arthroscopic soft-tissue biopsy findings). Irrigation and debridement with primary
exchange/conversion of components remains inferior to 2-stage reconstruction for infection
eradication. Resection arthroplasty remains a salvage procedure for resistant cases that preclude
reimplantation and generally is performed for symptom control and sepsis prevention.

29. Figure 1 is the radiograph of a 65-year-old active, right-hand-dominant woman with a 6-month
history of right shoulder pain, motion loss, and progressive weakness after undergoing a
hemiarthroplasty to address osteoarthritis 1 year ago. She denies recent trauma to her right
shoulder and denies constitutional symptoms. Her surgical wound site is benign. She can actively
forward flex to 90° degrees and abduct to 60°. Passive forward flexion and abduction are 150°
and 90°, respectively.She completes the necessary testing and wishes to proceed with revision
surgery. The most appropriate surgical option in this scenario involves implant removal and ?

Figure 1

1. Unconstrained total shoulder arthroplasty (TSA).


2. Resection arthroplasty.
3. Reverse total shoulder arthroplasty (rtsa).
4. Hemiarthroplasty.

UI // UNAIR // UNPAD // UNHAS // UNS // UGM // UB // UNUD // USU


Shoulder and Elbow Self-Assessment Examination 2019

Preferred Answer :3
The radiograph reveals a rotator cuff dysfunction secondary to malpositioning of the humeral stem
and a nonanatomic humeral head. Glenohumeral kinematics have been altered, resulting in
damage to the rotator cuff, which in turn has led to impingement with the coracoacromial arch.
This single radiograph reveals excessive humeral head height, “overstuffing” of the joint, and
severe narrowing of the acromiohumeral interval. Osteolysis and implant loosening are not
radiographically apparent. An orthogonal view (axillary lateral) would be necessary to evaluate for
shoulder instability. A CT arthrogram is the most appropriate advanced imaging test in the setting
of a retained shoulder arthroplasty to evaluate the integrity of the rotator cuff. An MRI evaluation
would be obfuscated by artifact. Three-phase and indium-tagged WBC scans may be appropriate
in the setting of an occult infection evaluation but not as a test to evaluate rotator cuff injury.
In the absence of infection with rotator cuff compromise, the most appropriate procedure(s) during
revision would involve humeral component explantation and conversion to rTSA. Revision
anatomic hemiarthroplasty may provide pain relief, but function may not appreciably change
because of the unbalanced forced couples of the rotator cuff complex. Placement of a glenoid
component in the setting of an irreparable rotator cuff tear is contraindicated because rapid
glenoid loosening will occur due to eccentric loading during active shoulder motion. Resection
arthroplasty should be reserved for recalcitrant cases of infection, because this procedure does
not provide functional improvement.
In the event that frozen section analysis and positive Gram stain results indicate an infection, the
treating surgeon should remove all components, perform a thorough debridement and irrigation of
suspect tissue, implant an antibiotic spacer, and perform a second-stage reconstruction when
deemed appropriate (in light of laboratory studies, repeat shoulder aspiration, frozen section
analysis, and arthroscopic soft-tissue biopsy findings). Irrigation and debridement with primary
exchange/conversion of components remains inferior to 2-stage reconstruction for infection
eradication. Resection arthroplasty remains a salvage procedure for resistant cases that preclude
reimplantation and generally is performed for symptom control and sepsis prevention.
30. Figure 1 is the radiograph of a 65-year-old active, right-hand-dominant woman with a 6-month
history of right shoulder pain, motion loss, and progressive weakness after undergoing a
hemiarthroplasty to address osteoarthritis 1 year ago. She denies recent trauma to her right
shoulder and denies constitutional symptoms. Her surgical wound site is benign. She can actively
forward flex to 90° degrees and abduct to 60°. Passive forward flexion and abduction are 150°
and 90°, respectively.
Intraoperative frozen section analysis reveals 10 neutrophils per high-power field and a positive
Gram stain result. The most appropriate next step would consist of implant removal, irrigation and
debridement, and ?

Figure 1

UI // UNAIR // UNPAD // UNHAS // UNS // UGM // UB // UNUD // USU


Shoulder and Elbow Self-Assessment Examination 2019

1. Resection arthroplasty.
2. Placement of rtsa.
3. Revision hemiarthroplasty.
4. Placement of antibiotic cement spacer.

Preferred Answer :4
The radiograph reveals a rotator cuff dysfunction secondary to malpositioning of the humeral stem
and a nonanatomic humeral head. Glenohumeral kinematics have been altered, resulting in
damage to the rotator cuff, which in turn has led to impingement with the coracoacromial arch. This
single radiograph reveals excessive humeral head height, “overstuffing” of the joint, and severe
narrowing of the acromiohumeral interval. Osteolysis and implant loosening are not
radiographically apparent. An orthogonal view (axillary lateral) would be necessary to evaluate for
shoulder instability. A CT arthrogram is the most appropriate advanced imaging test in the setting
of a retained shoulder arthroplasty to evaluate the integrity of the rotator cuff. An MRI evaluation
would be obfuscated by artifact. Three-phase and indium-tagged WBC scans may be appropriate
in the setting of an occult infection evaluation but not as a test to evaluate rotator cuff injury.
In the absence of infection with rotator cuff compromise, the most appropriate procedure(s) during
revision would involve humeral component explantation and conversion to rTSA. Revision
anatomic hemiarthroplasty may provide pain relief, but function may not appreciably change
because of the unbalanced forced couples of the rotator cuff complex. Placement of a glenoid
component in the setting of an irreparable rotator cuff tear is contraindicated because rapid glenoid
loosening will occur due to eccentric loading during active shoulder motion. Resection arthroplasty
should be reserved for recalcitrant cases of infection, because this procedure does not provide
functional improvement.
In the event that frozen section analysis and positive Gram stain results indicate an infection, the
treating surgeon should remove all components, perform a thorough debridement and irrigation of
suspect tissue, implant an antibiotic spacer, and perform a second-stage reconstruction when
deemed appropriate (in light of laboratory studies, repeat shoulder aspiration, frozen section
analysis, and arthroscopic soft-tissue biopsy findings). Irrigation and debridement with primary
exchange/conversion of components remains inferior to 2-stage reconstruction for infection
eradication. Resection arthroplasty remains a salvage procedure for resistant cases that preclude
reimplantation and generally is performed for symptom control and sepsis prevention.

31. A 42-year-old woman sustains a closed posterior elbow dislocation. A closed reduction is
performed, and the elbow appears stable under fluoroscopic examination. Initial treatment should
consist of ?

1. Early mobilization only.


2. Surgical reconstruction of medial and lateral collateral ligaments.
3. Active motion in a hinged brace from 30° to 120°.
4. Application of hinged external fixator with early mobilization.

Preferred Answer :1
This is a simple (no associated fracture) elbow dislocation. Such dislocations can be treated with
closed reduction followed by mobilization after 5 to 7 days to avoid stiffness, provided the elbow
is stable through a full arc of motion at the time of reduction. If the elbow is unstable but has a
short arc of stability, then using a hinged brace in the stable arc may be considered. (Note: It
may be necessary to splint the elbow in pronation if the medial collateral ligament [MCL] is intact
and the lateral collateral ligament [LCL] is disrupted, or in supination if the LCL is intact but the

UI // UNAIR // UNPAD // UNHAS // UNS // UGM // UB // UNUD // USU


Shoulder and Elbow Self-Assessment Examination 2019

MCL disrupted.) Surgical reconstruction of the LCL and MCL may be required only if the elbow
does not have a stable arc at the time of reduction. If unstable after reconstruction, application of
a hinged external fixator may be considered.

32. A 65-year-old man undergoes an uneventful left total shoulder arthroplasty for primary
osteoarthritis using a lesser tuberosity osteotomy. At his 6-week postoperative visit, he is
progressing well with physical therapy. Two months after surgery, he slips and falls in his
driveway. His subsequent visits over the next 8 weeks with therapy are fraught with marked
deterioration of his active motion and the inability to reach his lower back. A subsequent
radiograph and axial CT view of his shoulder are shown in Figures 1 and 2. What is the best next
step in management?

Figure 1 Figure 2

1. Sling immobilization and continued formal therapy


2. Open excision of heterotopic bone formation
3. Open reduction internal fixation of lesser tuberosity
4. Conversion to reverse shoulder arthroplasty

Preferred Answer :4
Anatomic total shoulder arthroplasty remains the most utilized surgical modality for patients with
primary osteoarthritis of the shoulder. Recently, the lesser tuberosity osteotomy has generated
enthusiasm as a theoretical avenue to improve subscapularis function postoperatively. With
modern repair techniques, this approach has a reported high union rate, making it a comparable
alternative with the traditional subscapularis peel or tenotomy. However, loss of fixation and
subscapularis failure may still occur. The patient’s radiograph reveals a medially displaced lesser
tuberosity with extension of a fracture line to the lateral cortex of the proximal humerus. As a
result, the patient now has subscapularis insufficiency that has affected his active range of
motion, as evidence by his inability to reach his lower back. Owing to the timing following the
injury to presentation, conversion to a reverse shoulder arthroplasty would most reliably
maximize the patient’s outcome. In the acute setting, an attempt at operative repair of the failed
lesser tuberosity may be considered. Sling immobilization would not be recommended. One of
the benefits of an osteotomy is that plain radiographs are often more than adequate to confirm
the diagnosis of subscapularis failure secondary to loss of tuberosity fixation.

UI // UNAIR // UNPAD // UNHAS // UNS // UGM // UB // UNUD // USU


Shoulder and Elbow Self-Assessment Examination 2019

33. A 75-year-old man sustains an anterior dislocation of his reverse total shoulder arthroplasty.
What activity places the arm in the position most commonly associated with reverse total
shoulder dislocation?

1. Scratching the opposite shoulder


2. Pushing off ipsilateral chair armrest while standing up
3. Tying shoelaces on the contralateral foot
4. Reaching up to comb hair

Preferred Answer :4
Proper soft-tissue tension is critical to prevent instability of a reverse total shoulder implanted
with the deltopectoral approach; dislocation of the prosthesis is exceedingly rare if the superior
approach is employed. The arm position implicated in reverse total shoulder instability is
extension, adduction, and internal rotation, such as pushing out of a chair. The other positions
described do not involve extension of the shoulder.

34. A 13-year-old pitcher reports the immediate onset of medial elbow pain after throwing a pitch.
Upon examination, the patient is tender to palpation at the medial epicondyle and has pain and
instability with valgus testing of the elbow.
What would be the most appropriate initial diagnostic test for this patient?

1. MRI arthrogram
2. CT scan with 3-dimensional reconstructions
3. Plain radiographs of both elbows
4. Ultrasonography

Preferred Answer :4
The patient has an acute avulsion fracture of the medial epicondyle, which can occur in response
to the valgus load placed on the elbow while throwing. Diagnosis is confirmed by radiograph, with
comparison views of the uninjured elbow to evaluate for physeal closure versus injury. In older
pitchers, the UCL fails rather than the bone of the medial epicondyle. Advanced imaging may be
necessary to confirm the diagnosis of an UCL injury and/or bony injury.

35. A 13-year-old pitcher reports the immediate onset of medial elbow pain after throwing a pitch.
Upon examination, the patient is tender to palpation at the medial epicondyle and has pain and
instability with valgus testing of the elbow.
The diagnostic modality selected in the prior question would be most useful to evaluate for?

1. Apophyseal injury.
2. Osteochondral defect.
3. Presence of hematoma.
4. Valgus overload injury.

Preferred Answer :1
The patient has an acute avulsion fracture of the medial epicondyle, which can occur in response
to the valgus load placed on the elbow while throwing. Diagnosis is confirmed by radiograph, with
comparison views of the uninjured elbow to evaluate for physeal closure versus injury. In older
pitchers, the UCL fails rather than the bone of the medial epicondyle. Advanced imaging may be
necessary to confirm the diagnosis of an UCL injury and/or bony injury.

UI // UNAIR // UNPAD // UNHAS // UNS // UGM // UB // UNUD // USU


Shoulder and Elbow Self-Assessment Examination 2019

36. A 13-year-old pitcher reports the immediate onset of medial elbow pain after throwing a pitch.
Upon examination, the patient is tender to palpation at the medial epicondyle and has pain and
instability with valgus testing of the elbow.
If the patient were a college pitcher with a similar clinical presentation and physical examination,
what anatomic structure would most likely be injured?

1. Ulnar collateral ligament (UCL)


2. Pronator teres
3. Ligament of Struthers
4. Lateral collateral ligament

Preferred Answer :1
The patient has an acute avulsion fracture of the medial epicondyle, which can occur in response
to the valgus load placed on the elbow while throwing. Diagnosis is confirmed by radiograph, with
comparison views of the uninjured elbow to evaluate for physeal closure versus injury. In older
pitchers, the UCL fails rather than the bone of the medial epicondyle. Advanced imaging may be
necessary to confirm the diagnosis of an UCL injury and/or bony injury.

37. A 45-year-old construction worker has right elbow pain and swelling following a fall from a step
ladder onto his outstretched hand. Figures 1 and 2 are his radiograph and 3D-CT scan.
Examination under anesthesia reveals widening of the radiocapitellar joint space with varus
stress. Surgical intervention is undertaken for open reduction internal fixation (ORIF) of the
coronoid fragment. Following fixation, the elbow remains unstable with persistent varus laxity.
What is the best next step?

Figure 1 Figure 2

1. Application of a long-arm splint with 4 to 6 weeks of immobilization


2. Application of a dynamic external fixator
3. Transarticular pinning of the ulnohumeral joint for 4 to 6 weeks
4. Open repair of the lateral collateral ligament complex

Preferred Answer :4
Posteromedial rotatory instability of the elbow is typically associated with a fall backwards onto
an outstretched arm, resulting in varus stress across the elbow and progressive pronation of the
forearm. As the coronoid translates posterior to the trochlea, the anteromedial facet is often
fractured. The lateral ulnar collateral ligament is frequently compromised as a result of the varus
force. The degree of elbow instability relates to both the size of the coronoid fragment and the
associated soft-tissue injuries about the elbow. The images reveal a displaced O’Driscoll type 2
coronoid fracture that is amenable to ORIF. If this fails to restore stability to the elbow, the next
most appropriate intervention is to repair the lateral collateral ligament complex, an injury that is

UI // UNAIR // UNPAD // UNHAS // UNS // UGM // UB // UNUD // USU


Shoulder and Elbow Self-Assessment Examination 2019

suggested by the widened radiocapitellar joint space on preoperative evaluation. A dynamic


external fixator can be used in the setting of grossly unstable elbows, but all correctible instability
lesions should be addressed before determining if this is necessary. Immobilization or temporary
transarticular pinning would be unlikely to adequately restore stability to the elbow.

38. A right-hand-dominant 45-year-old man sustains an injury to the anterior aspect of his right elbow
while trying to lift a heavy load 3 days ago. He has ecchymosis in the anterior and medial elbow
regions and has difficulty with resisted forearm supination with the elbow in a flexed position. A
diagnosis of an acute distal biceps tendon rupture is made and surgical treatment is chosen. The
anatomic relationship of the distal biceps tendon to the median nerve and recurrent radial artery
Within The Antecubital Fossa Is Such That The Biceps Tendon Travels

1. Lateral (radial) to the median nerve and posterior (deep) to the recurrent radial artery.
2. Lateral (radial) to the median nerve and anterior (superficial) to the recurrent radial artery.
3. Medial (ulnar) to the median nerve and posterior (deep) to the recurrent radial artery.
4. Medial (ulnar) to the median nerve and anterior (superficial) to the recurrent radial artery.

Preferred Answer :1
During surgical repair of a distal biceps tendon rupture, regardless of the surgical approach or
technique, an understanding of the regional anatomy is important. The tendon passes distally
into the antecubital fossa. The antecubital fossa is defined by the brachioradialis radially and the
pronator teresulnarly. A sheath surrounds the biceps tendon as it passes through the antecubital
fossa toward its insertion on the radial tuberosity. The lateral antebrachial cutaneous nerve lies
superficially in the subcutaneous tissue of the antecubital fossa. The nerve parallels the
brachioradialis. While still superficial, the tendon is contiguous with the lacertusfibrosus that
becomes confluent medially with the fascia overlying the flexor-pronator mass. The brachial
artery lies just beneath the lacertusfibrosus at the level of the elbow flexion crease. The tendon
travels just lateral (radial) to the median nerve within the antecubital fossa and passes posterior
(deep) to the recurrent radial artery before it attaches to the radial tuberosity. Full forearm
supination allows visualization of the tendinous insertion on the radial tuberosity.

39. A 72-year-old active man has shoulder pain after undergoing an explantation of an anatomic
shoulder arthroplasty 6 months prior with an antibiotic cement spacer placed. The patient has 60°
of forward flexion, 40° of external rotation, and a positive belly press with limited internal rotation.
A recent work-up for continued infection is negative, and a follow-up MRI reveals grade 2 atrophy
of the supraspinatus and grade 3 atrophy of the subscapularis with tendon retraction to the
glenoid rim. What is the best next step in definitive management?

1. Revision anatomic total shoulder arthroplasty


2. Reverse total shoulder arthroplasty
3. Hemiarthroplasty with latissimus dorsi transfer
4. Resection arthroplasty

Preferred Answer :2
This patient has a previously failed total shoulder arthroplasty for which he underwent placement
of an antibiotic spacer, and now has continued shoulder pain. The recent MRI demonstrates a
likely irreparable subscapularis tendon, making revision with an anatomic shoulder arthroplasty
contraindicated. Use of a hemiarthroplasty is unlikely to restore function in this older patient with
underlying rotator cuff disease, though it may be helpful for pain relief. Furthermore, a latissimus
dorsi transfer is also contraindicated in the setting of a chronic subscapularis tear. A reverse

UI // UNAIR // UNPAD // UNHAS // UNS // UGM // UB // UNUD // USU


Shoulder and Elbow Self-Assessment Examination 2019

shoulder arthroplasty offers the most reliable clinical outcome. Given that the preoperative
infection work-up was negative, resection arthroplasty is not indicated for this otherwise active
patient.

40. Figures 1 and 2 are the radiographs of a 64-year-old woman with a history of rheumatoid arthritis
(RA) who complains of right elbow pain. She has been treated with tumor necrosis factor-alpha
inhibitors and oral corticosteroids for several years. What process is primarily responsible for the
radiographic joint destruction?

Figure 1 Figure 2

1. Traumatic insult resulting in complement activation


2. Mutation in the rheumatoid factor gene
3. Osteoblast paracrine signaling resulting in proteolytic collagen degradation
4. Inflammation resulting in a hyperplastic synovial joint lining

Preferred Answer :4
RA is a systemic inflammatory disorder marked by erosive arthritis in multiple joints. Elbow
involvement is common. The pathologic lesion in RA is pannus, a hyperplastic synovial
proliferation that ultimately results in proteoglycan and collagen digestion. Rheumatoid factor
mutations, traumatic insults resulting in complement activation, and osteoblast paracrine
signaling are not involved in the pathologic process. The Larsen classification assesses the
progression of rheumatoid changes in the elbow. Stage I is characterized by osteopenia without
joint space narrowing. Stage II indicates joint space narrowing but a normal joint contour. Stage
III is marked by joint space loss. This patient has stage IV disease, as seen by the advanced
erosive changes with trochlear groove deepening and resulting deformity. Stage V is ankylosis.

41. A healthy 65-year-old woman undergoes anatomic total shoulder arthroplasty to address
osteoarthritis (OA). The surgery is uncomplicated. What is the most common indication for future
revision?

1. Deep infection
2. Periprosthetic fracture
3. Glenoid component loosening
4. Rotator cuff tear

Preferred Answer :3

UI // UNAIR // UNPAD // UNHAS // UNS // UGM // UB // UNUD // USU


Shoulder and Elbow Self-Assessment Examination 2019

The most common reason for revision surgery following unconstrained shoulder arthroplasty for
glenohumeral OA is loosening of an implant. In most studies that distinguish glenoid from
humeral loosening, it appears the glenoid is the problem. Comprehensive systematic reviews
have found that radiographic glenoid loosening can comprise nearly 30% to 40% of all
complications following shoulder arthroplasty for non-inflammatory arthritis. Infections,
periprosthetic fractures, and rotator cuff tears are uncommon. In the population-based study by
Matsen and associates, 10% of the revisions were performed for loosening versus 7% for
infection and 7% for rotator cuff tearing.

42. A 25-year-old minor league baseball pitcher has posteromedial elbow pain during the
deceleration phase of his motion. Examination reveals a mild flexion contracture of the elbow,
moderate tenderness over the medial olecranon, mild tenderness over the radiocapitellar joint
and medial collateral ligament, and a grossly stable valgus stress test. Figure 1 is a coronal MR
arthrogram of his elbow. What is the most likely diagnosis?

Figure 1

1. Incomplete tear of the medial collateral ligament


2. Incomplete tear of the lateral collateral ligament
3. Valgus extension overload (VEO)
4. Osteochondritis dissecans of the capitellum

Preferred Answer :3
VEO is a condition most frequently seen in repetitive overhead throwing athletes. It represents a
combination of pathologies resultant from tensile stress on the medial collateral ligament and
common flexor pronator origin, lateral compressive force across the radiocapitellar joint, and
shear stress between the medial tip of the olecranon and olecranon fossa. The deceleration and
follow-through phases of the throwing motion are when contact between the olecranon tip and
fossa are at their highest, often resulting in osteophyte formation that can both block terminal
extension and cause local tenderness. While compromise of the medial collateral ligament (MCL)
and osteochondral injury of the radiocapitellar joint can occur in VEO, these findings are part of
the larger disease process. In this patient, the MR arthrogram reveals a grossly intact MCL and
no evidence of radiocapitellar degeneration. Compromise of the lateral ulnar collateral ligament is
not a component of VEO and is not suggested by the primary site of pain in the question.

UI // UNAIR // UNPAD // UNHAS // UNS // UGM // UB // UNUD // USU


Shoulder and Elbow Self-Assessment Examination 2019
1. Median nerve as it crosses the surgical field 6 cm proximal to the medial epicondyle.
2. Medial antebrachial cutaneous nerve as it crosses the field 3 cm distal to the medial
epicondyle.
3. Anterior antebrachial cutaneous nerve as it crosses the field at the medial epicondyle.
4. Posterior antebrachial cutaneous nerve that crosses the field 2 cm distal to the medial
epicondyle.

43. When performing an ulnar nerve decompression at the elbow, the surgeon must be aware of
the?
Preferred Answer :2
The medial antebrachial cutaneous and medial brachial cutaneous are nerves that can be injured
during ulnar nerve decompression at the elbow. The medial antebrachial cutaneous nerve
crosses the surgical field at an average of 3.1 cm distal to the medial epicondyle. The medial
brachial cutaneous nerve crosses the field 7 cm proximal to the medial epicondyle and arborizes
into two to three terminal branches. Because the surgical approach involves dissection on the
medial side, the posterior antebrachial cutaneous nerve is distant from the exposure. Although
the median nerve potentially can be located in the deep dissection of a submuscular
transposition, it is considered distant to an in situ decompression.

44. Figures 1 through 3 are the radiographs and MRI scan of a 13-year-old girl who has had right
lateral elbow pain and “popping” for 5 months. She has a history of competitive gymnastics for 5
years, which she stopped participating in 2 years ago. She has since been pitching in softball.
Her pain is getting worse, and she has not played any sports for the last 2 months. She has
undergone a course of physical therapy. On examination, she lacks 5° of elbow extension. What
is the next most appropriate step in management?

Figure 1 Figure 2 Figure 3

1. Application of dynamic hinged elbow brace


2. Arthroscopic debridement and capitellar microfracture
3. Lateral collateral ligament reconstruction
4. Debridement and release of extensor origin
Preferred Answer :2
The radiographs and MRI reveal a defect in the capitellum compatible with osteochondritis
dissecans (OCD). The initial treatment for an OCD lesion would be cessation of the offending
activity with gradual return to the activity over the course of 6 to12 weeks. In the setting of failed

UI // UNAIR // UNPAD // UNHAS // UNS // UGM // UB // UNUD // USU


Shoulder and Elbow Self-Assessment Examination 2019

nonsurgical treatment, surgical intervention may be considered. The patient has already
attempted cessation of sporting activities, as well as physical therapy; as such, a hinged elbow
brace would not be of benefit. For the smaller, unstable OCD lesions, arthroscopic debridement
and microfracture is the most appropriate treatment option. This would be followed by early
protected range-of-motion exercises. The MRI does not show evidence for a lateral collateral
ligament injury or lateral epicondylitis. As a result, lateral collateral ligament reconstruction or
debridement of the extensor origin would not be indicated for this patient.

45. A complication associate/d with using the Morrey approach (triceps reflecting) to implant a
semiconstrained total elbow arthroplastyis ?

1. Loss of elbow extensor power.


2. Implant dislocation.
3. Implant malposition.
4. Development of heterotopic ossification.
Preferred Answer :1
Numerous approaches can be used to implant a total elbow arthroplasty. The Morrey approach
identifies, transposes, and protects the ulnar nerve, and then subperiosteally reflects the triceps
off the ulna. The sleeve of tissue is very thin distally, and the triceps need to be meticulously
repaired at the time of closure. Implant dislocation and malposition are less likely with an
extensile approach, and dislocation is unlikely with a semiconstrained implant. The development
of heterotopic ossification is unrelated to the surgical approach used for elbow arthroplasty.

46. Figures 1 through 3 are the radiograph and select CT images of a 75-year-old smoker with
hypertension who sustains a ground-level fall without loss of consciousness with impact to her
left upper extremity 1 week ago. She reports living independently at home with her husband prior
to her fall. What is the most appropriate next step?

Figure 1 Figure 2 Figure 3

1. Hemiarthroplasty with biceps tenodesis


2. Sling immobilization with subsequent physical therapy
3. Open reduction and internal fixation (ORIF)
4. Reverse total shoulder arthroplasty (rTSA)

Preferred Answer :4

UI // UNAIR // UNPAD // UNHAS // UNS // UGM // UB // UNUD // USU


Shoulder and Elbow Self-Assessment Examination 2019

The radiograph and CT scans indicate a 4-part left proximal humerus fracture with tuberosity
comminution. Based upon her preinjury level of activity and current imaging studies, nonsurgical
management is unlikely to restore her ability to perform activities of daily living, including hygiene
care. There has been enthusiasm among surgeons regarding the use of the reverse shoulder
prosthesis as the primary mode of surgical treatment for certain 3- and 4-part proximal humerus
fractures. The main attribute of this implant is its ability to achieve functional shoulder forward
flexion and abduction regardless of tuberosity healing, position, and degree of comminution.
Nevertheless, repair and union of the greater tuberosity fragment during rTSA demonstrate
improved external rotation, clinical outcomes, and patient satisfaction than outcomes achieved
after tuberosity resection, nonunion, or resorption. Based upon this patient’s age and imaging
findings, an rTSA would provide pain relief and improved function with complication rates similar
to those associated with hemiarthroplasty. ORIF would not be a viable option because of the high
probability for a dysvascular head, increased risk for nonunion, and potential for revision surgery,
including arthroplasty. Hemiarthroplasty for 4-part proximal humerus fractures remains a viable
option for patients <70 years of age with minimal tuberosity comminution and an intact rotator
cuff who can comply with a postsurgical rehabilitation program. Most studies indicate significant
pain relief with this modality, with significant variation in functional outcomes. In this clinical
scenario, the patient’s injury may not be best served with hemiarthroplasty because of
uncertainty regarding functional outcome.

47. Stemless shoulder arthroplasty prostheses have recently been suggested as an alternative to
traditional stemmed replacement. Advantages of the stemless surgical technique would include?

1. Better glenoid exposure than with stemmed prosthesis.


2. Reliable use in four-part proximal humerus fracture surgery.
3. Use in proximal humeral malunion without need for osteotomy.
4. Improved long-term survivorship profile.

Preferred Answer :3
Glenoid exposure, while better than with surface replacements, is not improved over traditional
stemmed replacements. Metaphyseal comminution would make it unlikely that a stemless
implant could be used in most four-part fractures. Stemless replacement does have the unique
advantage of allowing placement of a prosthesis with a malunion without an osteotomy, as the
prosthesis is not constrained by the position of the stem. While early results are encouraging,
there is no long-term data to suggest that survivorship is increased with stemless arthroplasty.

48. A 33-year old man sustains a posterior elbow dislocation after a fall. Attempts at closed reduction
result in recurrent instability. What is the most common ligamentous injury found at the time of
surgical stabilization?

1. Midsubstance tear of the lateral ulnar collateral ligament


2. Proximal avulsion of the ulnar collateral ligament
3. Proximal avulsion of the lateral ulnar collateral ligament
4. Distal bony avulsion of the ulnar collateral ligament from the sublime tubercle

Preferred Answer :3
Classic posterior elbow dislocations result from a posterolateral rotatory mechanism, whereby
the hand is fixed (typically on the ground) while the weight of the body creates a valgus and
external rotation moment on the elbow. This results first in tearing of the lateral collateral
ligament that proceeds medially through the anterior and posterior joint capsules, ending with

UI // UNAIR // UNPAD // UNHAS // UNS // UGM // UB // UNUD // USU


Shoulder and Elbow Self-Assessment Examination 2019

potential involvement of the ulnar collateral ligament (but this is not universal). McKee and
associates assessed the lateral soft-tissue injury pattern of elbow dislocations with and without
associated fractures at the time of surgery. Injury to the lateral collateral ligament complex was
seen in every case, with avulsion from the distal humerus as the most common finding.
Midsubstance tears, proximal avulsions, and distal bony avulsions of the ulnar collateral ligament
are less common.

49. A 19-year-old, right-hand-dominant collegiate baseball pitcher reports a 4-month history of right
shoulder pain after a throwing activity. He localizes the pain primarily to the posterior aspect of
his shoulder and describes the type of pain as an aching sensation. He has been involved with
strength and conditioning with his team but denies any specific therapy other than the application
of ice after throwing and use of occasional over-the-counter anti-inflammatory drugs, neither of
which has provided relief. He denies any specific traumatic event or previous history of shoulder
problems. His pitching coach has noted a slight decrease in his throwing velocity during the last 2
months.
The patient fails nonsurgical treatment and undergoes shoulder arthroscopy. At the time of
surgery, the area marked by the asterisk in Figure 1 is visualized from the posterolateral portal.
This anatomic structure impinges on which other structure during late cocking of the throwing
phase?
Figure 1

1. Biceps tendon
2. Posterior band of the inferior glenohumeral ligament
3. Hill-Sachs lesion
4. Undersurface of the supraspinatus and infraspinatus tendons

Preferred Answer : D
This patient’s clinical presentation is consistent with internal impingement accompanied by
glenohumeral internal rotation deficit (GIRD). Although throwers may have increased external
rotation, their overall arc of motion should be the same as on the nonthrowing side. In
comparison, patients with GIRD experience a marked decrease in arc of motion, particularly in
internal rotation. Internal impingement represents a spectrum of findings that can include
superior and posterior labral tears, undersurface (articular-sided) tearing of the posterior
supraspinatus, posterior glenoid wear, and scar formation of the posterior capsule. Myers and
associates demonstrated internal impingement is associated with GIRD, although the latter by
itself may be asymptomatic and perhaps a sports-specific adaptation. However, posterior
capsular tightness can lead to posterosuperior translation of the humerus during throwing,

UI // UNAIR // UNPAD // UNHAS // UNS // UGM // UB // UNUD // USU


Shoulder and Elbow Self-Assessment Examination 2019

leading to these injuries. Internal impingement is common among overhead throwing athletes
and occurs during the late cocking and early acceleration phases of throwing. Humeral migration
during the abducted/externally rotated throwing position results in abutment of the greater
tuberosity against the posterosuperior glenoid labrum, which impinges the rotator cuff (Paley and
associates).
Pain is often posterior, but symptoms can be vague. Patients may have examination findings
consistent with rotator cuff weakness and superior labrum anterior to posterior (SLAP)/biceps
involvement. Radiographic findings can be negative, although a Bennett lesion involving
hypertrophy and mineralization of the posterior capsular injury may be seen (Wright and Paletta).
A CT scan may show glenoid retroversion (Crockett and associates), whereas MR imaging
should be reviewed for a possible partial articular-sided rotator cuff tear, SLAP tear, or increased
signal in the posterosuperior labrum, or greater tuberosity.
Treatment of this condition should be the focus on therapy, and most cases can be treated
nonsurgically. Stretching aimed at the posterior capsule (ie, sleeper stretch) has been reported
as effective (Tyler and associates, Lintner and associates). Burkhart and associates also
demonstrated that posterior capsular stretching can help to prevent throwing injuries. Because
cuff pathology may be present, physical therapy also should include rotator strengthening,
scapular stabilization, and addressing of issues related to throwing mechanics (Drakos and
associates). Kibler and associates published a comprehensive rehabilitation guideline. Surgical
intervention is reserved for those who fail 6 months of nonsurgical treatment and is directed by
intra-articular pathology (debridement vs repair of the rotator cuff and labrum) (Braun and
associates).

50. A 19-year-old, right-hand-dominant collegiate baseball pitcher reports a 4-month history of right
shoulder pain after a throwing activity. He localizes the pain primarily to the posterior aspect of
his shoulder and describes the type of pain as an aching sensation. He has been involved with
strength and conditioning with his team but denies any specific therapy other than the application
of ice after throwing and use of occasional over-the-counter anti-inflammatory drugs, neither of
which has provided relief. He denies any specific traumatic event or previous history of shoulder
problems. His pitching coach has noted a slight decrease in his throwing velocity during the last 2
months.
Which image seen during arthroscopic treatment is most likely associated with this patient's
condition?

a.

b.

UI // UNAIR // UNPAD // UNHAS // UNS // UGM // UB // UNUD // USU


Shoulder and Elbow Self-Assessment Examination 2019

c.

d.

Preferred Answer :1
This patient’s clinical presentation is consistent with internal impingement accompanied by
glenohumeral internal rotation deficit (GIRD). Although throwers may have increased external
rotation, their overall arc of motion should be the same as on the nonthrowing side. In
comparison, patients with GIRD experience a marked decrease in arc of motion, particularly in
internal rotation. Internal impingement represents a spectrum of findings that can include
superior and posterior labral tears, undersurface (articular-sided) tearing of the posterior
supraspinatus, posterior glenoid wear, and scar formation of the posterior capsule. Myers and
associates demonstrated internal impingement is associated with GIRD, although the latter by
itself may be asymptomatic and perhaps a sports-specific adaptation. However, posterior
capsular tightness can lead to posterosuperior translation of the humerus during throwing,
leading to these injuries. Internal impingement is common among overhead throwing athletes
and occurs during the late cocking and early acceleration phases of throwing. Humeral migration
during the abducted/externally rotated throwing position results in abutment of the greater
tuberosity against the posterosuperior glenoid labrum, which impinges the rotator cuff (Paley and
associates).
Pain is often posterior, but symptoms can be vague. Patients may have examination findings
consistent with rotator cuff weakness and superior labrum anterior to posterior (SLAP)/biceps
involvement. Radiographic findings can be negative, although a Bennett lesion involving
hypertrophy and mineralization of the posterior capsular injury may be seen (Wright and Paletta).
A CT scan may show glenoid retroversion (Crockett and associates), whereas MR imaging
should be reviewed for a possible partial articular-sided rotator cuff tear, SLAP tear, or increased
signal in the posterosuperior labrum, or greater tuberosity.
Treatment of this condition should be the focus on therapy, and most cases can be treated
nonsurgically. Stretching aimed at the posterior capsule (ie, sleeper stretch) has been reported
as effective (Tyler and associates, Lintner and associates). Burkhart and associates also
demonstrated that posterior capsular stretching can help to prevent throwing injuries. Because
cuff pathology may be present, physical therapy also should include rotator strengthening,
scapular stabilization, and addressing of issues related to throwing mechanics (Drakos and
associates). Kibler and associates published a comprehensive rehabilitation guideline. Surgical
intervention is reserved for those who fail 6 months of nonsurgical treatment and is directed by
intra-articular pathology (debridement vs repair of the rotator cuff and labrum) (Braun and
associates).

UI // UNAIR // UNPAD // UNHAS // UNS // UGM // UB // UNUD // USU


Shoulder and Elbow Self-Assessment Examination 2019

51. A 36-year-old right-hand-dominant man falls from his motorcycle and sustains the acute right
upper extremity injury seen in Figure 1. At surgery, an open reduction and internal fixation (ORIF)
of the ulna is performed along with attempted open reduction of the radiocapitellar joint.
However, the radial head is slightly subluxed in flexion and redislocates with elbow extension
<90°. What is the most appropriate treatment at this time?

Figure 1

1. Radial head resection


2. Casting in 90° of flexion for 3 weeks, followed by reassessment of elbow stability
3. Repair of the annular ligament
4. Revision ORIF of the ulnar fracture

Preferred Answer :4
This case is a variant of a type I Monteggia fracture according to the Bado classification, with a
segmental ulna fracture. In some cases, the radial head subluxation can be subtle, and missing
this would lead to a poor outcome. In this case, the anterior radial head dislocation is obvious,
but the segmental nature of the ulna fracture makes anatomic reduction difficult. The radial head
usually spontaneously reduces once the ulna is anatomically reduced, and no surgical treatment
to the lateral side is required. When this is not the case, a lateral approach and incision of the
annular ligament may be required for reduction. If an open reduction of the radial head is
unsuccessful, the problem is almost always residual malalignment of the ulna. Therefore, casting
and annular ligament repair will not improve reduction. A radial head resection would eliminate
the nonconcentric contact between radial head and capitellum but would not be an appropriate
treatment for this young patient who has an acute, correctable fracture deformity.

52. When performing a shoulder hemiarthroplasty for an unreconstructable proximal humerus


fracture, The Relationship Of The Repaired Greater Tuberosity To The Prosthetic Humeral Head
Should Be

1. 6 mm to 8 mm superior to the top of the humeral head.


2. 6 mm to 8 mm inferior to the top of the humeral head.
3. 1.5 cm inferior to the top of the humeral head.
4. At the same height as the top of the humeral head.

UI // UNAIR // UNPAD // UNHAS // UNS // UGM // UB // UNUD // USU


Shoulder and Elbow Self-Assessment Examination 2019

Preferred Answer : 2
The greater tuberosity lies anatomically 6 mm to 8 mm inferior to the top of the humeral head.
Normal proximal humeral anatomy must be recreated when performing a hemiarthroplasty for
fracture so as to minimize the complications associated with the greater tuberosity and maximize
functional outcomes. Tuberosity malunion and nonunion are considered the most common
reasons for poor clinical outcomes following this procedure. Placing the tuberosity too proximal
can lead to issues with impingement during shoulder abduction, and placement too distal can
increase the tension on the rotator cuff as it courses over the prosthetic humeral head.

53. A 24-year-old right-hand-dominant professional baseball pitcher has valgus extension overload
(VEO) syndrome of the right elbow, as seen in Figure 1. Which letter in the Figure 1 corresponds
to the typical area of osteophyte formation in this condition?

Figure 1

1. A
2. B
3. C
4. D

Preferred Answer :2
VEO is most commonly seen in throwers for whom valgus stress across the elbow causes
impingement of the posteromedial olecranon tip against the medial wall of the olecranon fossa.
With repeated impingement, a bony osteophyte may grow on the olecranon at the site of
impingement in this posteromedial region of the olecranon. Bony growth within the olecranon
fossa also has been seen. The distinction between this condition and ulnar collateral ligament
(UCL) injury is difficult to make, but VEO often can be distinguished from UCL injury by
determining the exact location of pain a patient experiences. With VEO, the pain typically occurs
with direct palpation of the posterior medial tip of the olecranon. The valgus extension overload
provocative test also aids in diagnosis. A supervised physical therapy program and arthroscopic
surgical decompression when nonsurgical treatment is unsuccessful are typical treatments for
this condition.
Locations C and D represent the origin and insertion, respectively, of the elbow medial collateral
ligament (MCL), and, although associated MCL pathology can exist in the setting of VEO
syndrome, osteophyte formation is not typical in these areas. Location A is the radial head, and
although the radiocapitellar joint is a known secondary stabilizer of elbow valgus stress,
osteophyte formation in this area is less likely in this clinical scenario.

UI // UNAIR // UNPAD // UNHAS // UNS // UGM // UB // UNUD // USU


Shoulder and Elbow Self-Assessment Examination 2019

54. A 45-year-old construction worker sees a surgeon 23 days after sustaining an eccentric injury to
his dominant right elbow. An MRI demonstrates a distal biceps tendon rupture with 5 cm of
proximal retraction. In the operating room, the surgeon encounters good tissue quality but finds

1. Proceed with primary repair with the elbow hyperflexed


2. Use interposition allograft to reconstruct with elbow in extension
3. Tenodese distal biceps tendon to underlying brachialis muscle
4. Forego primary repair, but perform stump debridement

that primary repair can only be performed with the elbow hyperflexed to 70°. What is the best
next step?

Preferred Answer :1
Distal biceps ruptures, although relatively less common in comparison with other upper extremity
tendon injuries, still garner considerable attention in the orthopaedic literature. The mechanism of
injury typically results from an eccentric extension load to a flexed elbow. A biceps-deficient arm
can result in up to 40% loss of supination strength and up to 80% loss of supination endurance.
A delay in diagnosis can compromise the ability to reduce the tendon back to its anatomic
insertion without having to hyperflex the elbow. Current literature confirms the ability to safely
proceed with primary repair even with the elbow flexed up to 100° without fear of developing a
flexion contracture. With time, patients can anticipate restoration of full elbow extension. An
interposition graft should be used for a poor residual tendon quality stump <4 cm in length and in
cases of delay to surgery of >6 weeks. Biceps to brachialis tendon transfer does not restore
supination strength. Isolated debridement of the distal tendon would not be an appropriate
treatment.

55. Figures 1 and 2 are the radiographs of a 47-year-old right-hand-dominant active man with a 10-
year history of progressive right elbow pain associated with stiffness. He previously underwent
collateral ligament reconstruction. He has pain throughout his range-of-motion arc, which
currently measures 20° of extension to 80° of flexion. Initial treatment with nonsteroidal anti-
inflammatory medication, physical therapy, cortisone injections, and arthroscopic debridement
has failed to provide relief of his symptoms and improvement in function.
What is the most appropriate next treatment step for this patient?

Figure 1 Figure 2

1. Total elbow arthroplasty (TEA)


2. Distal humeral replacement arthroplasty
3. Arthroscopic release with debridement
4. Soft-tissue interposition arthroplasty

UI // UNAIR // UNPAD // UNHAS // UNS // UGM // UB // UNUD // USU


Shoulder and Elbow Self-Assessment Examination 2019

Preferred Answer :4
The radiographs reveal ulnohumeralarthrosis with relative sparing of the radiocapitellar
articulation secondary to underlying osteoarthritis. Arthrosis of the elbow joint in this young and
active patient presents a treatment dilemma for the surgeon. Interposition arthroplasty allows for
improved function with pain relief and no weight-lifting restrictions, as required with TEA. This
option is an intermediate procedure that preserves bone stock and allows for conversion to a
TEA if necessary. Conventional TEA would provide pain relief with improved range of motion, but
activity limitation and lifetime weight restrictions make this an undesirable option. Arthroscopic
debridement is not an option, considering the previous failure from this modality.
Contraindications for soft-tissue interposition arthroplasty include elbow instability, active
infection, and pain without motion loss. Common complications associated with this procedure
include instability, infection, ulnar neuropathy, bone resorption, and heterotopic bone formation.

56. Figures 1 and 2 are the radiographs of a 47-year-old right-hand-dominant active man with a 10-
year history of progressive right elbow pain associated with stiffness. He previously underwent
collateral ligament reconstruction. He has pain throughout his range-of-motion arc, which
currently measures 20° of extension to 80° of flexion. Initial treatment with nonsteroidal anti-
inflammatory medication, physical therapy, cortisone injections, and arthroscopic debridement
has failed to provide relief of his symptoms and improvement in function.
What is the most appropriate treatment if instability is present at the time of evaluation?

Figure 1 Figure 2

1. TEA
2. Distal humeral replacement arthroplasty
3. Arthroscopic release with debridement
4. Soft-tissue interposition arthroplasty

Preferred Answer :1
The radiographs reveal ulnohumeralarthrosis with relative sparing of the radiocapitellar
articulation secondary to underlying osteoarthritis. Arthrosis of the elbow joint in this young and
active patient presents a treatment dilemma for the surgeon. Interposition arthroplasty allows for
improved function with pain relief and no weight-lifting restrictions, as required with TEA. This
option is an intermediate procedure that preserves bone stock and allows for conversion to a
TEA if necessary. Conventional TEA would provide pain relief with improved range of motion, but
activity limitation and lifetime weight restrictions make this an undesirable option. Arthroscopic
debridement is not an option, considering the previous failure from this modality.
Contraindications for soft-tissue interposition arthroplasty include elbow instability, active

UI // UNAIR // UNPAD // UNHAS // UNS // UGM // UB // UNUD // USU


Shoulder and Elbow Self-Assessment Examination 2019

infection, and pain without motion loss. Common complications associated with this procedure
include instability, infection, ulnar neuropathy, bone resorption, and heterotopic bone formation.

57. In the work-up of a painful shoulder arthroplasty, arthroscopic biopsy specimens that are sent for
culture and discarded after 5 days will most likely yield a false-negative result in the setting of
infection by which organism?

1. Staphylococcus epidermidis
2. Propionibacterium acnes
3. Staphylococcus aureus
4. Streptococcus pneumonia

Preferred Answer :2
It has been increasingly recognized that P acnes is a common pathogen implicated in infections
of the shoulder. Multiple studies have shown that discarding the cultures prior to 2 weeks will
result in a high rate of false-negative cultures. This is likely because P acnes is a semiobligate
anaerobe and grows slowly in the lab. As such, the clinical presentations are often nonspecific,
without overt fever or wound breakdown, and traditional infection lab values are commonly within
normal limits. In addition to S aureus, P acnes has been cited as the most common etiology for
infection following shoulder arthroplasty.

58. Figures 1 and 2 are the radiographs of a 60-year-old man with gradual onset of right shoulder
pain and motion loss 1 year after undergoing an uncomplicated right total shoulder arthroplasty
(TSA) for end-stage osteoarthritis. He denies trauma to his right shoulder and constitutional
symptoms but admits to difficulty performing activities of daily living. His surgical wound site is
benign. He demonstrates active and passive forward flexion to 90°, abduction to 60°, external
rotation to 30°, and internal rotation to the lumbosacral junction. His rotator cuff strength is
graded as normal and symmetrical to his unaffected left shoulder. Based upon the current
evaluation, what is the most appropriate next step?

Figure 1 Figure 2
1. Revision TSA
2. Infection work-up
3. CT scan of the shoulder
4. Observation and physical therapy

UI // UNAIR // UNPAD // UNHAS // UNS // UGM // UB // UNUD // USU


Shoulder and Elbow Self-Assessment Examination 2019

Preferred Answer :2
This patient’s radiographs do not reveal prosthetic loosening, osteolysis, instability, or rotator cuff
deficiency. History and examination findings suggest a possible indolent infectious process, and
it is incumbent upon the treating surgeon to obtain screening laboratory studies (CBC with
differential, ESR, C-reactive protein) and proceed with obtaining cultures (via joint aspiration or
arthroscopic soft-tissue biopsy) to rule out an infectious process. Cultures should be held for 2
weeks to evaluate for the possibility of a Propionibacterium acnes infection, which is the
predominant organism elicited from painful and stiff TSAs for which revision is required.
Immediate revision TSA is not indicated in this scenario because the components appear well
fixed and positioned. Observation and therapy in the setting of deterioration of a previously well-
functioning TSA also are not appropriate.

59. A 38-year-old man sustains a terrible triad injury consisting of an elbow dislocation, comminuted
and displaced radial head fracture, and a type I coronoid fracture. Intraoperative findings after
radial head replacement and lateral collateral ligament complex repair reveal persistent instability
consisting of medial opening on valgus stress and posteromedial subluxation of the ulnohumeral
and radiocapitellar joints. What is the best next step?

1. Medial collateral ligament repair or reconstruction


2. Reconstruction of the radial collateral ligament
3. Resection of the type I coronoid fracture and capsular repair to the remaining coronoid
4. Open reduction and buttress plating of the coronoid fracture

Preferred Answer :1
Terrible triad injuries of the elbow are common, and the management of type I coronoid tip
fractures remains controversial. Type I coronoid fractures result in only small changes in elbow
kinematics that have been shown to be uncorrected with suture repair. A type I coronoid tip
fracture is not amenable to buttress plate fixation. The radial collateral ligament is a component
of the lateral collateral ligament complex and has already been repaired. The persistent medial
laxity and posteromedial joint subluxation noted is indicative of ongoing instability. The next step
would be repair or reconstruction of the medial collateral ligament, which will normally correct the
medial instability. Articulated versus static external fixation can be considered if restoration of the
ligamentous constraint of the medial side of the elbow cannot be accomplished surgically.

60. Figure 1 is the radiograph of a 47-year-old woman who has pain and difficulty raising her arm
after playing 36 holes of golf in a weekend and now has difficulty sleeping. She denies prior
episodes of shoulder pain. Examination demonstrates guarding with any shoulder motion,
tenderness around the superolateral shoulder, and normal sensory findings.
An MRI arthrogram scan of her shoulder would most likely reveal ?

Figure 1

UI // UNAIR // UNPAD // UNHAS // UNS // UGM // UB // UNUD // USU


Shoulder and Elbow Self-Assessment Examination 2019

1. Contrast extravasation into the subacromial space.


2. Contrast in the glenoid/anterior labral interface.
3. Hyperintense heterogenous t2-signal in posterior rotator cuff.
4. Hypointense homogenous t1-signal in posterior rotator cuff.

Preferred Answer :4
Calcific tendinitis of the shoulder is a deposition of calcium carbonate apatite crystals into the
structure of the rotator cuff tendon. The crystalline form appears to progress throughout the
clinical disease process, demonstrating increasing matured stoichiometric apatite deposition
during the resorptive phase. MRI can be difficult to interpret because the signal of the calcific
lesion is frequently similar to that seen in normal supraspinatus tendon. Plain radiographs remain
the gold standard for diagnosis. Ultrasonography can be a useful ancillary study to determine the
location and size of the lesion. MRI demonstrates a hypointense homogenous signal on the T1-
and T2 fat suppressed sequences in the region of the calcific deposit. There may be some
increased T2-signal surrounding the lesion indicated localized inflammation. Primary
management of calcific tendinitis starts with nonsurgical treatment including physiotherapy and
injections, if indicated. Mixed results have been reported with extracorporeal shock wave
therapy. Surgical removal with repair of the tendon in larger lesions remains the definitive
treatment when nonsurgical modalities fail. Subacromial decompression may improve pain relief
in patients who require surgery; however, patients with decompression may take longer to fully
recover.

61. Figure 1 is the radiograph of a 47-year-old woman who has pain and difficulty raising her arm
after playing 36 holes of golf in a weekend and now has difficulty sleeping. She denies prior
episodes of shoulder pain. Examination demonstrates guarding with any shoulder motion,
tenderness around the superolateral shoulder, and normal sensory findings.
The lesion indicated in the image is comprised of ?

Figure 1

1. Calcium carbonate apatite.


2. Hyperproliferative white blood cells.
3. Hydroxyapatite crystals.
4. Degenerated tenocytes.

UI // UNAIR // UNPAD // UNHAS // UNS // UGM // UB // UNUD // USU


Shoulder and Elbow Self-Assessment Examination 2019

Preferred Answer :1
Calcific tendinitis of the shoulder is a deposition of calcium carbonate apatite crystals into the
structure of the rotator cuff tendon. The crystalline form appears to progress throughout the
clinical disease process, demonstrating increasing matured stoichiometric apatite deposition
during the resorptive phase. MRI can be difficult to interpret because the signal of the calcific
lesion is frequently similar to that seen in normal supraspinatus tendon. Plain radiographs remain
the gold standard for diagnosis. Ultrasonography can be a useful ancillary study to determine the
location and size of the lesion. MRI demonstrates a hypointense homogenous signal on the T1-
and T2 fat suppressed sequences in the region of the calcific deposit. There may be some
increased T2-signal surrounding the lesion indicated localized inflammation. Primary
management of calcific tendinitis starts with nonsurgical treatment including physiotherapy and
injections, if indicated. Mixed results have been reported with extracorporeal shock wave
therapy. Surgical removal with repair of the tendon in larger lesions remains the definitive
treatment when nonsurgical modalities fail. Subacromial decompression may improve pain relief
in patients who require surgery; however, patients with decompression may take longer to fully
recover.

62. Figure 1 is the radiograph of a 47-year-old woman who has pain and difficulty raising her arm
after playing 36 holes of golf in a weekend and now has difficulty sleeping. She denies prior
episodes of shoulder pain. Examination demonstrates guarding with any shoulder motion,
tenderness around the superolateral shoulder, and normal sensory findings.
The best initial treatment would entail ?

Figure 1

1. Physical therapy and nonsteroidal anti-inflammatory medications.


2. Open biopsy of the lesion for permanent section.
3. Manipulation under anesthesia.
4. Shoulder arthroscopy.

Preferred Answer :1
Calcific tendinitis of the shoulder is a deposition of calcium carbonate apatite crystals into the
structure of the rotator cuff tendon. The crystalline form appears to progress throughout the
clinical disease process, demonstrating increasing matured stoichiometric apatite deposition
during the resorptive phase. MRI can be difficult to interpret because the signal of the calcific
lesion is frequently similar to that seen in normal supraspinatus tendon. Plain radiographs remain
the gold standard for diagnosis. Ultrasound can be a useful ancillary study to determine the

UI // UNAIR // UNPAD // UNHAS // UNS // UGM // UB // UNUD // USU


Shoulder and Elbow Self-Assessment Examination 2019

location and size of the lesion. Primary management of calcific tendinitis starts with nonsurgical
treatment including physiotherapy and injections, if indicated. Mixed results have been reported
with extracorporeal shock wave therapy. Surgical removal with repair of the tendon in larger
lesions remains the definitive treatment when nonsurgical modalities fail. Subacromial
decompression may improve pain relief in patients who require surgery; however, patients with
decompression may take longer to fully recover.

63. A 37-year-old recreational athlete has osteoarthritis of the glenohumeral joint. He has failed
nonsurgical measures and is interested in surgical intervention but would like to avoid
arthroplasty. When performing shoulder arthroscopy for glenohumeral arthritis, which
radiographic parameter is most predictive of clinical failure?

1. Unipolar arthritis
2. >3 mm of glenohumeral joint space
3. Walch B2 glenoid morphology
4. Small inferior humeral osteophyte

Preferred Answer :3
Multiple studies have evaluated the utility of arthroscopy in the treatment of shoulder arthritis.
Despite differing levels of success, a few common characteristics have been shown to lead to a
higher probability of clinical failure. Mitchell and associates showed that shoulders with less joint
space (1.3 mm vs 2.6 mm) and Walch type B2 and C glenoids were significantly more likely to
fail than were Walch types A1, A2, and B1. Additionally, older patients (age >50 years) tended to
have worse outcomes. Skelley and associates found that isolated capsular release and
debridement had a high failure rate (conversion to total shoulder arthroplasty in 42% within 9
months) and postulated that patients undergoing concomitant procedures, such as biceps
tenodesis, may fare better. Van Theil and associates found significant risk factors for failure
included the presence of grade 4 bipolar disease, joint space <2 mm, and the presence of large
osteophytes. They had a 22% conversion to total shoulder arthroplasty at 10.1 months.

64. A 35-year-old man falls off of a roof and sustains an extra-articular supracondylar elbow fracture.
He had normal sensation in all fingers after the injury and before undergoing surgery to repair the
fracture. The ulnar nerve was not transposed but was inspected prior to wound closure. Ten days
after surgery, the patient has numbness in his small finger and is unable to cross his fingers. His
elbow range of motion is 40° to 100°. What is the next appropriate step in management?

1. Elbow splint at 40° for 6 weeks


2. Electromyography (EMG)
3. Exploration of ulnar nerve and transposition
4. Continued observation

Preferred Answer :4
This patient has an early postsurgical ulnar nerve palsy. The causes of this injury are laceration
of the nerve during surgery, entrapment of the nerve in the fracture or hardware, or traction injury
during surgery. If the orthopaedic surgeon is sure that the nerve was not lacerated at the end of
the case or entrapped in the hardware, then the nerve is probably intact and will recover.
Observation is the best treatment in this case because the nerve was checked before wound
closure. Elbow splinting has not been shown to help with postsurgical nerve recovery. EMG
findings may not be accurate this early following the injury.

UI // UNAIR // UNPAD // UNHAS // UNS // UGM // UB // UNUD // USU


Shoulder and Elbow Self-Assessment Examination 2019

65. Figures 1 and 2 show the radiograph and axial CT scan of a 56-year-old right-hand-dominant
man who sustains a right shoulder injury following a fall from a roof. He is seen in the emergency
department and placed into a sling. He denies any previous injury to the shoulder. His medical
history is significant only for hypertension. His arm is neurovascularly intact, and his deltoid is

functioning. What is the most appropriate surgical option at this point?

1. Open reduction and internal fixation (ORIF) Anatomic total shoulder arthroplasty Hemiarthroplasty
2. Reverse shoulder arthroplasty
3.
4.

Figure 1 Figure 2
Preferred Answer :3
The radiograph and axial CT image demonstrate a displaced right proximal humerus fracture,
which has a head-splitting component. The fracture line extends through the greater and lesser
tuberosities as well. The head-splitting nature of the fracture increases the risk for the
developement of avascular necrosis and, potentially, failure of fixation. In the younger patient,
<55 years, with a simple head-splitting fracture (tuberosities intact), one might consider ORIF.
However, in this case of an older patient, >55 years, with a more complex head-splitting fracture
pattern, a well-performed hemiarthroplasty provides a reliable clinical outcome. Clinical outcomes
following hemiarthroplasty rely on successful healing of the greater tuberosity. In this case, the
patient has no major medical comorbidities, has a non-comminuted greater tuberosity, and
sought treatment shortly after injury. These factors have been associated with improved
tuberosity healing. In an older patient with multiple medical comorbidities including
osteopenia/osteoporosis, lower demand or with a lengthy delay from injury to surgery, a reverse
shoulder arthroplasty might be a reasonable option. Reverse total shoulder arthroplasty has been
shown to offer predictable functional results in the setting of fractures, although with a higher
complication rate.

UI // UNAIR // UNPAD // UNHAS // UNS // UGM // UB // UNUD // USU


Shoulder and Elbow Self-Assessment Examination 2019

66. A 50-year-old pipefitter falls from a ladder at work and dislocates his non-dominant shoulder. His
MRI scan shows supraspinatus and infraspinatus tears with retraction to the glenoid. He cannot
actively raise his arm away from his side. He denies prior shoulder symptoms before his fall.
Three weeks of physical therapy have failed to improve his function. Which factor has been
demonstrated to result in a poor clinical outcome following surgical intervention?

1. The patient's age


2. he patient's gender
3. Work-related injury
4. Acute nature of the tear

Preferred Answer :3
Several studies have demonstrated that patients with work-related injuries do not do as well as
those whose injuries are not work-related after repair of the rotator cuff. This patient’s age and
gender are not negative prognostic indicators. The acute nature of the tear does not lead to an
inferior outcome.

67. A 35-year-old construction worker falls from a ladder, sustaining an injury to his elbow. An MRI
demonstrates a rupture of his distal biceps tendon. He underwent a repair through a single
transverse incision in the antecubital fossa using the fixation as seen in Figures 1 and 2.
Postoperatively, the patient reports mildly decreased sensation over the anterolateral aspect of
the forearm. Which other complication is this patient most at risk for in the early postoperative
period?

Figure 1 Figure 2

1. Proximal radioulnar synostosis


2. Heterotopic ossification
3. Radial neck fracture
4. Weak thumb and index interphalangeal joint flexion

Preferred Answer :3
Injury to the lateral antebrachial cutaneous (LABC) nerve is the most common complication
following distal biceps tendon repair, primarily due to traction. The patient demonstrates some
decreased sensation over the anterolateral forearm, consistent with an LABC neurapraxia.
Additionally, the figures show fixation with the endobutton proximal to the radial tuberosity, in the
region of the radial neck. This is associated with an increased risk of iatrogenic fracture of the
radial neck, which occurs during the drilling of the bone tunnel. Heterotopic ossification and

UI // UNAIR // UNPAD // UNHAS // UNS // UGM // UB // UNUD // USU


Shoulder and Elbow Self-Assessment Examination 2019

proximal radioulnar synostosis are more commonly associated with the dual-incision approach.
The anterior interosseous nerve is not an at-risk structure during distal biceps repair. The
posterior interosseous nerve (PIN) can be placed at risk during this procedure. The distance from
the guide pin and the nerve is decreased with more distal drilling and more radially directed
drilling. In this case, the PIN has not crossed posterior to the radius yet, making it less likely to be
injured during placement of the tunnel.

68. A 61-year-old right-hand-dominant woman falls down the stairs, resulting in a left anteroinferior
dislocation and noncomminuted greater tuberosity fracture. A closed glenohumeral reduction with
intravenous sedation is performed in the emergency department. After reduction, the greater
tuberosity fragment remains displaced by 2 mm. What is the most appropriate treatment?

1. Open reduction internal fixation with transosseous sutures


2. Arthroscopic fixation using a suture bridge technique
3. Nonsurgical treatment with early passive range of motion
4. Nonsurgical treatment with sling immobilization for 4 weeks

Preferred Answer :3
Greater tuberosity fractures and rotator cuff tears associated with a traumatic dislocation are
more commonly seen in women >60 years. Greater tuberosity fractures that are displaced <5
mm in the general population and <3 mm in laborers and professional athletes can be treated
successfully without surgery. Early passive range of motion is important to avoid stiffness.

69. A 26-year-old mixed martial arts fighter sustains a posterolateral elbow dislocation. The primary
stabilizers of the elbow joint are the ?

1. Radiocapitellar joint, the posterior band of the medial collateral ligament, and the annular
ligament.
2. Ulnohumeral joint, the anterior band of the medial collateral ligament, and the lateral ulnar
collateral ligament.
3. Radiocapitellar joint, the anterior band of the medial collateral ligament, and the radial
collateral ligament.
4. Ulnohumeral joint, the anterior band of the medial collateral ligament, and the posterior
band of the medial collateral ligament.

Preferred Answer :2
The primary stabilizers of the elbow are the ulnohumeral joint, the lateral collateral ligament
(lateral epicondyle to the crista supinatoris), and the anterior band of the medial collateral
ligament (anterior inferior medial epicondyle to the sublime tubercle). Secondary stabilizers are
the radial head, the common flexor and extensor origins, and the joint capsule. The muscles that
cross the elbow joint act as dynamic stabilizers.

70. Figures 1 and 2 are the radiographs of a 69-year-old, left-hand-dominant retired man with left
shoulder pain. The pain has been present for several years. He worked in construction but retired
3 years ago. He now reports pain interfering with activities around the house but denies recent
trauma or prior shoulder surgery. He has tried nonsteroidal anti-inflammatory drugs, but these do
not provide complete relief. He demonstrates pain and crepitus with active and passive shoulder
motion. He can actively forward flex to 100° and external rotate to 30°. Rotator cuff testing
reveals 5/5 strength and he is neurovascularly intact.

UI // UNAIR // UNPAD // UNHAS // UNS // UGM // UB // UNUD // USU


Shoulder and Elbow Self-Assessment Examination 2019

After discussion regarding surgical and nonsurgical treatment options, the patient wishes to
proceed with surgical intervention. He has done online research and has questions about which
procedure will produce the best outcome. Based on the current literature, what is the most
appropriate surgical procedure?

Figure 1 Figure 2

1. Arthroscopic glenohumeral debridement with biceps tenotomy


2. Hemiarthroplasty
3. Total shoulder arthroplasty (TSA)
4. Reverse TSA (rTSA)

Preferred Answer :3
The examination and radiograph findings are consistent with glenohumeral osteoarthritis (OA),
which is now interfering with this patient’s daily activities.
Regarding surgical treatment for glenohumeral arthritis, several studies have shown that TSA is
associated with better functional outcomes than hemiarthroplasty. A recent meta-analysis
comparing outcomes between TSA and hemiarthroplasty revealed better function after TSA
regarding pain, University of California-Los Angeles Shoulder Scores, and postsurgical forward
elevation at a minimum of 2 years (Bryant and associates). In addition, Gartsman and associates
showed that TSA resulted in better pain relief, function, strength, and patient satisfaction than
hemiarthroplasty at 3-year follow-up. The patient in this scenario exhibits good rotator cuff
strength on examination; therefore, arthroscopy with debridement or rTSA used in the setting of a
massive cuff tear would not be the best answer.
Several studies have similarly shown good results with TSA in the setting of inflammatory and
rheumatoid arthritis (Thomas and associates, Jolles and associates). Posterior glenoid wear is a
common pattern in OA and not a contraindication (Walch and associates). However, patients
with a brachial plexus root avulsion or preganglionic injury resulting in a flail arm are not
candidates for TSA because of the poor prognosis for recovery of motor and sensory deficits.
Inflammatory arthritis characteristically results in concentric glenoid wear and not the eccentric
posterior erosion seen in OA. This concentric wear results in medialization of the glenohumeral
joint line. This wear pattern can lead to severe erosion, making it difficult to achieve glenoid
resurfacing. Eccentric posterior glenoid wear would lead to posterior humeral head subluxation,
which is not as common in inflammatory arthritis. In addition, inferior humeral head osteophytes
are seen in OA. Walch and associates have developed a classification system describing glenoid
wear patterns.

UI // UNAIR // UNPAD // UNHAS // UNS // UGM // UB // UNUD // USU


Shoulder and Elbow Self-Assessment Examination 2019

During TSA, exposure to the glenohumeral joint involves subscapularis management, tendon
peel, or lesser tuberosity osteotomy. As a result, early postsurgical rehabilitation limits passive
external rotation and active internal rotation, typically for 6 weeks, to protect the subscapularis
repair. The patient in this scenario likely eccentrically contracted his subscapularis, resulting in
repair failure. Upon clinical examination, internal rotation weakness, increased passive external
rotation, and abnormal belly press or lift-off test results can be expected. One study revealed that
rupture of the subscapularis was seen in all anterior dislocations following TSA (Wirth and
Rockwood).

71. 1 and 2 are the radiographs of a 69-year-old, left-hand-dominant retired man with left shoulder
pain. The pain has been present for several years. He worked in construction but retired 3 years
ago. He now reports pain interfering with activities around the house but denies recent trauma or
prior shoulder surgery. He has tried nonsteroidal anti-inflammatory drugs, but these do not
provide complete relief. He demonstrates pain and crepitus with active and passive shoulder
motion. He can actively forward flex to 100° and external rotate to 30°. Rotator cuff testing
reveals 5/5 strength and he is neurovascularly intact.
During the patient's presurgical history and physical visit, he reports to the nurse that he has a
history of rheumatoid arthritis which is being managed by his primary care physician. With this
new information in hand, which finding is most commonly seen on imaging during presurgical
planning?
Figure 1 Figure 2

1. Glenoid medialization
2. Posterior glenoid wear
3. Posterior subluxation of the humeral head
4. Inferior osteophytes at the humeral head

Preferred Answer :1
The examination and radiograph findings are consistent with glenohumeral osteoarthritis (OA),
which is now interfering with this patient’s daily activities.
Regarding surgical treatment for glenohumeral arthritis, several studies have shown that TSA is
associated with better functional outcomes than hemiarthroplasty. A recent meta-analysis
comparing outcomes between TSA and hemiarthroplasty revealed better function after TSA
regarding pain, University of California-Los Angeles Shoulder Scores, and postsurgical forward

UI // UNAIR // UNPAD // UNHAS // UNS // UGM // UB // UNUD // USU


Shoulder and Elbow Self-Assessment Examination 2019

elevation at a minimum of 2 years (Bryant and associates). In addition, Gartsman and associates
showed that TSA resulted in better pain relief, function, strength, and patient satisfaction than
hemiarthroplasty at 3-year follow-up. The patient in this scenario exhibits good rotator cuff
strength on examination; therefore, arthroscopy with debridement or rTSA used in the setting of a
massive cuff tear would not be the best answer.
Several studies have similarly shown good results with TSA in the setting of inflammatory and
rheumatoid arthritis (Thomas and associates, Jolles and associates). Posterior glenoid wear is a
common pattern in OA and not a contraindication (Walch and associates). However, patients
with a brachial plexus root avulsion or preganglionic injury resulting in a flail arm are not
candidates for TSA because of the poor prognosis for recovery of motor and sensory deficits.
Inflammatory arthritis characteristically results in concentric glenoid wear and not the eccentric
posterior erosion seen in OA. This concentric wear results in medialization of the glenohumeral
joint line. This wear pattern can lead to severe erosion, making it difficult to achieve glenoid
resurfacing. Eccentric posterior glenoid wear would lead to posterior humeral head subluxation,
which is not as common in inflammatory arthritis. In addition, inferior humeral head osteophytes
are seen in OA. Walch and associates have developed a classification system describing glenoid
wear patterns.
During TSA, exposure to the glenohumeral joint involves subscapularis management, tendon
peel, or lesser tuberosity osteotomy. As a result, early postsurgical rehabilitation limits passive
external rotation and active internal rotation, typically for 6 weeks, to protect the subscapularis
repair. The patient in this scenario likely eccentrically contracted his subscapularis, resulting in
repair failure. Upon clinical examination, internal rotation weakness, increased passive external
rotation, and abnormal belly press or lift-off test results can be expected. One study revealed that
rupture of the subscapularis was seen in all anterior dislocations following TSA (Wirth and
Rockwood).

72. Figures 1 and 2 are the radiographs of a 69-year-old, left-hand-dominant retired man with left
shoulder pain. The pain has been present for several years. He worked in construction but retired
3 years ago. He now reports pain interfering with activities around the house but denies recent
trauma or prior shoulder surgery. He has tried nonsteroidal anti-inflammatory drugs, but these do
not provide complete relief. He demonstrates pain and crepitus with active and passive shoulder
motion. He can actively forward flex to 100° and external rotate to 30°. Rotator cuff testing
reveals 5/5 strength and he is neurovascularly intact.
Following a successful shoulder arthroplasty, the patient returns for his 1-month follow-up. At
today's visit, his wound appears benign, and he denies drainage or fevers. He reports he was
doing well until last week, when he reached out to close the car door, which resulted in new-
onset anterior shoulder pain. His radiograph from the current visit is shown in Figure 3. What is
the most likely cause of this new finding?

UI // UNAIR // UNPAD // UNHAS // UNS // UGM // UB // UNUD // USU


Shoulder and Elbow Self-Assessment Examination 2019

Figure 1 Figure 2 Figure 3

1. Supraspinatus tear
2. Subscapularis tear
3. Glenoid component
4. Infection

Preferred Answer :2
The examination and radiograph findings are consistent with glenohumeral osteoarthritis (OA),
which is now interfering with this patient’s daily activities.
Regarding surgical treatment for glenohumeral arthritis, several studies have shown that TSA is
associated with better functional outcomes than hemiarthroplasty. A recent meta-analysis
comparing outcomes between TSA and hemiarthroplasty revealed better function after TSA
regarding pain, University of California-Los Angeles Shoulder Scores, and postsurgical forward
elevation at a minimum of 2 years (Bryant and associates). In addition, Gartsman and associates
showed that TSA resulted in better pain relief, function, strength, and patient satisfaction than
hemiarthroplasty at 3-year follow-up. The patient in this scenario exhibits good rotator cuff
strength on examination; therefore, arthroscopy with debridement or rTSA used in the setting of a
massive cuff tear would not be the best answer.
Several studies have similarly shown good results with TSA in the setting of inflammatory and
rheumatoid arthritis (Thomas and associates, Jolles and associates). Posterior glenoid wear is a
common pattern in OA and not a contraindication (Walch and associates). However, patients
with a brachial plexus root avulsion or preganglionic injury resulting in a flail arm are not
candidates for TSA because of the poor prognosis for recovery of motor and sensory deficits.
Inflammatory arthritis characteristically results in concentric glenoid wear and not the eccentric
posterior erosion seen in OA. This concentric wear results in medialization of the glenohumeral
joint line. This wear pattern can lead to severe erosion, making it difficult to achieve glenoid
resurfacing. Eccentric posterior glenoid wear would lead to posterior humeral head subluxation,
which is not as common in inflammatory arthritis. In addition, inferior humeral head osteophytes
are seen in OA. Walch and associates have developed a classification system describing glenoid
wear patterns.
During TSA, exposure to the glenohumeral joint involves subscapularis management, tendon
peel, or lesser tuberosity osteotomy. As a result, early postsurgical rehabilitation limits passive
external rotation and active internal rotation, typically for 6 weeks, to protect the subscapularis
repair. The patient in this scenario likely eccentrically contracted his subscapularis, resulting in

UI // UNAIR // UNPAD // UNHAS // UNS // UGM // UB // UNUD // USU


Shoulder and Elbow Self-Assessment Examination 2019

repair failure. Upon clinical examination, internal rotation weakness, increased passive external
rotation, and abnormal belly press or lift-off test results can be expected. One study revealed that
rupture of the subscapularis was seen in all anterior dislocations following TSA (Wirth and
Rockwood).

73. Surgical management of the fracture shown in Figure 1 will have what outcome compared with
nonsurgical management in a sling?

Figure 1
1. Increased rate of radiographic union
2. Increased amount of shortening
3. Decreased risk of infection
4. Improved shoulder forward elevation

Preferred Answer :1
Multiple prospective randomized clinical trials have compared operative treatment of displaced
midshaft clavicle fractures with nonoperative management. While functional outcomes have not
consistently been improved with surgery, rates of radiographic union have consistently been
shown to improve. Fracture characteristics associated with an increased risk for nonunion of a
midshaft clavicle fracture include comminution of the fracture and significant
shortening/displacement of the fracture (>1.5-2.0 cm). Patient factors predicting nonunion include
a smoking history.

74. A 53-year-old man complains of recurrent lateral elbow pain. He was surgically treated
approximately one year ago with some improvement in his direct lateral elbow pain. He now
reports new-onset discomfort at the posterolateral elbow, as well as difficulty when pushing
himself up from a chair. On examination, he has a well-healed 6-cm incision over the lateral
epicondyle with full active and passive range of motion. He has pain with palpation along the
posterior lateral elbow and a positive posterior drawer test. Radiographs are unremarkable. What
is the best next step?

1. Platelet-rich plasma
2. Physical therapy
3. Lateral epicondyle debridement
4. Lateral collateral ligament reconstruction

Preferred Answer :4
Lateral elbow tendinopathy remains a frequently encountered pathology of the elbow. Open or
arthroscopic lateral epicondyle debridement can be considered for patients with refractory

UI // UNAIR // UNPAD // UNHAS // UNS // UGM // UB // UNUD // USU


Shoulder and Elbow Self-Assessment Examination 2019

symptoms. With either technique, the lateral collateral ligament complex of the elbow is at risk for
compromise, with excessive debridement distal and posterior to the center of rotation of the
capitellum. When injured, patients often complain of pain around the posterior lateral elbow,
which is commonly misdiagnosed as recurrent lateral epicondylitis. The push-up test
(apprehension using the supinated forearm to push up from a chair) is a typical examination
finding, along with a positive posterior drawer test. Patients may also develop posterior lateral
instability of the elbow, for which the recommended treatment is lateral collateral ligament
reconstruction.

75. Figures 1 through 4 are the radiographs of a 55-year-old healthy woman who fell down a flight of
steps while sleepwalking. When the surgeon replaces the radial head, the elbow dislocates
posteriorly at 60° of flexion as it is brought out from full flexion. What is the best next step?

Figure 1 Figure 2 Figure 3 Figure 4

1. Only repair the lateral collateral ligament (LCL).


2. Do nothing further and place the elbow in 90° of flexion.
3. Repair the posterior band of the medial collateral ligament (MCL).
4. Repair the coronoid and reassess for stability.

Preferred Answer :4
The coronoid is important for elbow stability, particularly as the elbow is moved into extension.
Repairing the LCL alone after radial head replacement in “terrible triad” injuries may suffice when
there is a type 1 coronoid fracture or an anterior capsular avulsion. For more extensive coronoid
injuries, live dynamic examination of stability is needed to determine whether repair of the
coronoid is needed. For this patient, doing nothing further will lead to immediate postsurgical
instability, and repairing the LCL complex alone will not lead to stability. The posterior band of the
MCL will not add to stability. The next step to attain stability is to repair the coronoid fracture and
re-examine the elbow for stability.

76. Figure 1 is the radiograph of a 39-year-old man who has a syncopal episode and fall. After being
cleared by the emergency department, he is referred to your office for left shoulder pain and loss
of external rotation.
What is the most likely diagnosis?

UI // UNAIR // UNPAD // UNHAS // UNS // UGM // UB // UNUD // USU


Shoulder and Elbow Self-Assessment Examination 2019

Figure 1

1. Rotator cuff tear


2. Adhesive capsulitis
3. Brachial plexus injury
4. Posterior shoulder dislocation

Preferred Answer :4
The patient has a posterior glenohumeral dislocation, as evidenced by the overlap on the initial
radiograph. While posterior dislocations are rare, they can be overlooked. A CT scan will
accurately show the lesion before proceeding to surgery. If a simple closed reduction is
performed acutely and the arm is stable after the reduction, no further intervention may be
needed, and treatment can be successful with a 2-week period of immobilization for defects
involving <30% of the humeral head. However, in this scenario, open reduction is likely and
stabilization may require a modified McLaughlin procedure or other intervention to fill in the
humeral defect. Younger male patients, those with a large humeral head defect, and those with
seizure disorder may be at highest risk for recurrence. For treatment of chronic posterior
dislocations, it may be necessary to perform shoulder arthroplasty to restore stability. Stiffness is
attributable to articular incongruity; therefore, physical therapy and capsular release are
inappropriate.

77. Figure 1 is the radiograph of a 39-year-old man who has a syncopal episode and fall. After being
cleared by the emergency department, he is referred to your office for left shoulder pain and loss
of external rotation.
What is the most appropriate next step in management?

Figure 1

UI // UNAIR // UNPAD // UNHAS // UNS // UGM // UB // UNUD // USU


Shoulder and Elbow Self-Assessment Examination 2019

1. Physical therapy
2. CT scan of shoulder
3. Arthroscopic capsular release
4. Arthroscopic Bankart repair

Preferred Answer :2
The patient has a posterior glenohumeral dislocation, as evidenced by the overlap on the initial
radiograph. While posterior dislocations are rare, they can be overlooked. A CT scan will
accurately show the lesion before proceeding to surgery. If a simple closed reduction is
performed acutely and the arm is stable after the reduction, no further intervention may be
needed, and treatment can be successful with a 2-week period of immobilization for defects
involving <30% of the humeral head. However, in this scenario, open reduction is likely and
stabilization may require a modified McLaughlin procedure or other intervention to fill in the
humeral defect. Younger male patients, those with a large humeral head defect, and those with
seizure disorder may be at highest risk for recurrence. For treatment of chronic posterior
dislocations, it may be necessary to perform shoulder arthroplasty to restore stability. Stiffness is
attributable to articular incongruity; therefore, physical therapy and capsular release are
inappropriate.

78. A 73-year old woman reports activity-related pain in the right shoulder. She had undergone
surgery on her shoulder following a fall at home. A select radiograph is given in Figure 1. The
radiographic changes seen along the glenoid neck would most likely have been minimized
through the use of what implant-related technique?

Figure 1

1. More valgus-angled humeral stem


2. More distal placement of glenoid baseplate
3. Decreased lateral offset
4. Increased number of baseplate locking screws

UI // UNAIR // UNPAD // UNHAS // UNS // UGM // UB // UNUD // USU


Shoulder and Elbow Self-Assessment Examination 2019

Preferred Answer :2
In a reverse shoulder arthroplasty, multiple designs with varying amounts of valgus neck-shaft
angles have been developed since the original 155° of the neck-shaft angle Grammont
prosthesis design. Regardless of neck-shaft angle utilized, average forward elevation flexion
improvement ranges from 78° to 131°, with no difference in dislocation rates and range of
motion. A 135° neck-shaft angle has been shown to have less scapular notching. The radiograph
demonstrates a commonly utilized reverse total shoulder replacement with a 135°-angled
humeral stem. The baseplate has not been placed at the inferior-most aspect of the glenoid with
the result being that the inferior aspect of the glenosphere lies superior to the glenoid neck. While
the clinical implications of scapular notching were previously controversial, it has more recently
been shown that advanced scapular notching is associated with a decrease in functional
outcomes and possibly baseplate loosening. Strategies for reducing the prevalence of notching
have been shown through clinical, computer simulation, and biomechanical studies. A more
valgus stem, increased superior tilt, and a more medialized offset have been shown to increase
the risk for scapular notching. The number of screws can affect initial baseplate stability but has
not been shown to have an effect on scapular notching. In this case, more distal placement of the
baseplate (or use of a larger or more laterally offset glenosphere) would have decreased the
adduction deficit and reduced impingement of the polyethylene onto the lateral scapula.

79. A 41-year-old right-hand-dominant man has been treated nonsurgically for right elbow arthritis.
His radiographs reveal end-stage ulnohumeral arthritis with complete loss of the joint space. He
reports pain during the mid-arc of elbow flexion and extension. During the last 8 years, he has
attempted activity modification, medication, physical therapy, and multiple cortisone injections.
His symptoms have progressed, resulting in constant pain, loss of a functional range of motion,
and an inability to perform many activities of daily living. Secondary to his age and activity
demands, he undergoes a soft-tissue interposition arthroplasty of his elbow with an Achilles
allograft. Which presurgical finding correlates with elevated risk for postsurgical complications?

1. Inflammatory elbow arthritis


2. A presurgical flexion-extension elbow arc of approximately 50°
3. Retained distal humerus hardware on presurgical radiographs
4. Evidence of presurgical elbow instability

Preferred Answer :4
End-stage posttraumatic or inflammatory elbow arthritis in active, high-demand patients remains
difficult to treat. Traditional total elbow arthroplasty is discouraged in this demographic secondary
to concerns about implant longevity. Soft-tissue interposition arthroplasty does not necessitate
the same activity and weight restrictions for patients after surgery and remains a reasonable
salvage procedure. Larson and Morrey published their findings on 38 patients with a mean age of
39 years following soft-tissue interposition arthroplasty for posttraumatic and inflammatory end-
stage elbow arthritis. These investigators reported a significant improvement in Mayo Elbow
Performance Score in addition to improvement in the flexion-extension arc from 51° to 97° after
surgery. They reported worse results and elevated incidence of complications for patients with
presurgical elbow instability upon examination; retained hardware from prior surgery was not
deemed a contraindication.

UI // UNAIR // UNPAD // UNHAS // UNS // UGM // UB // UNUD // USU


Shoulder and Elbow Self-Assessment Examination 2019

80. Figures 1 and 2 are the radiograph and MRI scan of a 40-year-old man who falls down a flight of
stairs. His upper arm is bruised and painful, and global weakness in the shoulder girdle function
is noted. A radiograph is ordered to rule out a fracture or dislocation. You should recommend ?

Figure 1 Figure 2

1. Immediate open reduction and internal fixation of the fracture.


2. Closed treatment with serial radiographs.
3. Fracture fragment excision and deltoid repair.
4. Rest, anti-inflammatory medications, and a home exercise program.

Preferred Answer :4
The patient has an osacromiale. The type shown is of the meso-acromion. This is not an acute
fracture; well corticated ends are seen on the axillary radiograph, and there is no bone edema on
the T2 axial MRI image. A trial of nonsurgical care that includes rest, ice, and anti-inflammatory
medications is recommended. If a patient continues to have symptoms, an arthroscopic
evaluation is needed to determine if the osacromiale is mobile and if surgical fixation is
appropriate.

81. A 51-year-old butcher has an 18-month history of recalcitrant medial elbow pain, which is
affecting his occupational demands. He describes the pain as mainly anterior and distal to the
medial epicondyle. His symptoms are exacerbated with resisted wrist flexion and forearm
pronation. On examination, he is also found to have a positive Tinel’s sign at the elbow with
weakness of intrinsic strength. He has attempted physical therapy, activity modification, bracing,
and anti-inflammatory medication without any significant improvement. Presurgical counseling
should include the understanding that ?

1. Concomitant ulnar neuropathy is a potential poor prognostic factor.


2. A change in occupation will likely be required after surgery.
3. Weakness in wrist flexion strength will result postoperatively.
4. Prior corticosteriod injections are a potential poor prognostic factor.

Preferred Answer :1
Although less common in comparison with lateral elbow tendinopathy, medial elbow tendinopathy
remains a significant cause of elbow disability. Fortunately, most patients can anticipate
resolution of symptoms with nonsurgical management. For patients with recalcitrant symptoms,
surgical intervention should be discussed as a treatment alternative. The literature reports
successful results with surgical intervention via debridement of pathologic tissue, release of the

UI // UNAIR // UNPAD // UNHAS // UNS // UGM // UB // UNUD // USU


Shoulder and Elbow Self-Assessment Examination 2019

flexor carpi radialis - pronator teres origin, and/or repair of the flexor carpi radialis - pronator teres
origin. Several authors have raised concern of the impact of concomitant ulnar neuropathy on
results following surgical treatment for medial epicondylitis. Kurvers and Verhaar and Gabel and
Morrey, among others, have reported a statistically significant association between concomitant
ulnar neuropathy and worse outcomes following surgery. Most patients can anticipate a return to
prior activity levels after surgery without any consistently reported loss of flexor/pronator strength.
Prior corticosteroid injections have not been found to impact results.

82. A 30-year-old man with diabetes sustains an acute posterior dislocation of his right shoulder after
a seizure that required emergency department reduction. You initially treat him with a sling for 4
weeks and then refer him for outpatient therapy. During his therapy sessions, the patient admits
to pain and instability symptoms during range-of-motion exercises. Repeat examination indicates
a positive posterior load-shift test and apprehension with adduction and internal rotation of the
shoulder. His CT and MRI scans are shown in Figures 1 and 2. What is the most appropriate
next step in treatment?

Figure 1 Figure 2

1. Brief period of immobilization and reinitiation of therapy


2. Open posterior capsular shift with labral repair
3. Arthroscopic capsulolabral repair with subscapularis transfer
4. Resurfacing arthroplasty with labral repair

Preferred Answer : 3
Posterior glenohumeral dislocations are much less common than anterior glenohumeral
dislocations, with a prevalence of 1.1 per 100,000 per year. There is a bimodal distribution with a
peak in young men (2.4 men to 1 woman) and a second peak in elderly people with a more
equivalent gender ratio. Posterior dislocations most commonly result from trauma, with the
remainder of events secondary to seizure activity. According to Robinson and associates, age
<40 years, dislocation during a seizure, and a large reverse Hill-Sachs lesion were all predictive
of recurrent instability. Concomitant injuries associated with posterior shoulder dislocations
include capsulolabral tears, fractures, and rotator cuff tears. Imaging studies in this patient
indicate a reverse Hill-Sachs lesion with a corresponding posterior labral tear. Because of his
persistent mechanical symptoms, continued immobilization and therapy is not appropriate. An
open capsular shift with labral repair alone would not address symptoms related to an engaging
reverse Hill-Sachs lesion and has the added morbidity from the surgical approach. Based upon
the patient’s age, activity level, and percentage of humeral head involvement, a resurfacing

UI // UNAIR // UNPAD // UNHAS // UNS // UGM // UB // UNUD // USU


Shoulder and Elbow Self-Assessment Examination 2019

arthroplasty is not recommended. A subscapularis or lesser tuberosity transfer has been used to
address symptomatic reverse Hill-Sachs lesions (20%-40% humeral head involvement)
associated with posterior shoulder dislocations. Modifications of this technique such as
arthroscopic transfer of the subscapularis tendon with posterior capsulorrhaphy have proven
beneficial. The indications for the concomitant subscapularis transfer into the defect (arthroscopic
McLaughlin) have not been as well-defined as for patients with a Hill-Sachs lesion in the setting
of recurrent anterior instability.

83. Figures 1 and 2 are the radiographs of a 48-year-old right-hand dominant man who has had
elbow pain and stiffness for many years. The patient enjoys recreational softball and weightlifting,
and the pain has gotten severe enough that it has begun to interfere with his activities of daily
living. On examination, he has a range of motion from 40° to 110°, with pain with terminal
extension and flexion, but no pain in the mid-range of motion. He had one corticosteroid injection
last year, which temporarily improved his pain but did not improve his motion. What is the most
appropriate treatment option for this patient?
Figure 1 Figure 2

1. Isolated radial head excision


2. Arthroscopic osteocapsulararthroplasty
3. Total elbow arthroplasty
4. Distal humerushemiarthroplasty

Preferred Answer :2
This patient is relatively young, active, and has evidence of elbow osteoarthritis (OA). Large
osteophytes in the anterior and posterior compartments of his elbow are limiting motion. He has
failed non-operative treatment, and surgery is indicated. Arthroscopic osteocapsulararthroplasty
allows removal of impinging osteophytes and release of hypertrophied capsule and has been
shown to be effective at relieving pain and improving motion in patients with OA. Additionally,
avoiding prosthetic replacement would allow the patient to continue his active lifestyle. An open
elbow release would be another excellent treatment option. Isolated radial head excision would
not be indicated, because it would increase contact forces across the ulnohumeral joint and
cause little improvement in symptoms. Total elbow arthroplasty is not a good option in a young,
active patient. Distal humerushemiarthroplasty has been described for certain patients with
unreconstructable distal humerus fractures, but it is not a good option for treatment of OA.

UI // UNAIR // UNPAD // UNHAS // UNS // UGM // UB // UNUD // USU


Shoulder and Elbow Self-Assessment Examination 2019

84. A 75-year-old woman with rheumatoid arthritis and a long history of oral corticosteroid use
sustains a comminuted intra-articular distal humerus fracture. What is the best surgical option?

1. Open reduction internal fixation (ORIF) with parallel plates


2. ORIF with orthogonal plates and iliac crest bone grafting
3. Total elbow arthroplasty (TEA)
4. Closed reduction and percutaneous pinning

Preferred Answer :3
TEA is the best surgical option. McKee and associates published a multicenter randomized
controlled trial comparing ORIF with TEA in elderly patients. TEA resulted in better 2-year clinical
functional scores and more predictable outcomes compared with ORIF. TEA was also likely to
result in a lower resurgical rate; one-quarter of patients with fractures randomized to ORIF could
not achieve stable fixation. Further, Frankle and associates reported a comparative study of TEA
versus ORIF in 24 elderly women. TEA outcomes were again superior to ORIF at a minimum of 2
years of follow-up. TEA was especially useful in patients with comorbidities that compromise
bone stock, including osteoporosis and oral corticosteroid use. Closed reduction and
percutaneous pinning studies have not been published on the adult population.

85. Figures 1 through 3 are the radiographs of a 45-year-old man following acute trauma.
Which radiographic finding indicates the likely need for a radial head replacement?

Figure 1 Figure 2 Figure 3

1. Two or fewer fragments of the radial head


2. Age <21 years
3. Wrist pain and asymmetry of the ipsilateral distal radioulnar joint
4. Anteromedial coronoid comminution

Preferred Answer :3
Longitudinal forearm instability is an indication for radial head replacement to prevent proximal
migration of the radial shaft. Radial head replacement is indicated in radial head fractures
involving three or more fragments. Younger age is not a contraindication or indication for radial
head replacement. Anteromedial coronoid facet fractures usually are associated with a
posteromedial rotatory mechanism that does not increase or decrease risk for radial head
fractures necessitating replacement.
Following restoration of the radial head, a lateral collateral ligament repair would be the next step
to restore stability if necessary. MCL repair would restore medial stability if stability persisted

UI // UNAIR // UNPAD // UNHAS // UNS // UGM // UB // UNUD // USU


Shoulder and Elbow Self-Assessment Examination 2019

following restoration of posterolateral laxity. Repair of type I coronoid fractures does not
substantially affect stability. Application of a hinged external fixator can restore stability in severe
cases but is usually reserved for refractory instability after ligament repair has been performed
and instability persists.
Anterior and posterior capsular repair do not significantly affect instability. MCL repair is generally
the next step to obtain stability, with application of a hinged external fixator as a last step to
maintain joint congruity.

86. Figures 1 through 3 are the radiographs of a 45-year-old man following acute trauma.
Following radial head replacement, the elbow exhibits persistent laxity to valgus stress in
extension. What is the best next step to regain stability?

Figure 1 Figure 2 Figure 3

1. Posterior capsular repair


2. Anterior capsular repair
3. Fixation of the type I coronoid fracture
4. Repair of the medial collateral ligament (MCL)

Preferred Answer :4
Longitudinal forearm instability is an indication for radial head replacement to prevent proximal
migration of the radial shaft. Radial head replacement is indicated in radial head fractures
involving three or more fragments. Younger age is not a contraindication or indication for radial
head replacement. Anteromedial coronoid facet fractures usually are associated with a
posteromedial rotatory mechanism that does not increase or decrease risk for radial head
fractures necessitating replacement.
Following restoration of the radial head, a lateral collateral ligament repair would be the next step
to restore stability if necessary. MCL repair would restore medial stability if stability persisted
following restoration of posterolateral laxity. Repair of type I coronoid fractures does not
substantially affect stability. Application of a hinged external fixator can restore stability in severe
cases but is usually reserved for refractory instability after ligament repair has been performed
and instability persists.
Anterior and posterior capsular repair do not significantly affect instability. MCL repair is generally
the next step to obtain stability, with application of a hinged external fixator as a last step to
maintain joint congruity.

87. A 54-year-old woman undergoes an interposition arthroplasty that fails and requires conversion
to a total elbow arthroplasty. She has progressive elbow pain and radiographic loosening.
Erythrocyte sedimentation rate and C-reactive protein are normal. Joint aspiration is positive

UI // UNAIR // UNPAD // UNHAS // UNS // UGM // UB // UNUD // USU


Shoulder and Elbow Self-Assessment Examination 2019

for Staphylococcus epidermidis. What surgical treatment would best optimize function and
decrease risk of recurrence?

1. Resection arthroplasty
2. Single-stage revision total elbow arthroplasty
3. Two-stage revision elbow arthroplasty
4. Aggressive arthroscopic debridement and retention of components

Preferred Answer :3
The most reliable surgical option in this case for eradicating a deep infection following a total
elbow arthroplasty is a two-stage revision. One study, however, reported that staged
reimplantation of an infected total elbow replacement could be successful in the setting of
organisms other than S epidermidis. Arthroscopic debridement is not a viable option with poorly
fixed or loose components. A single-stage revision, while considered an option in hip and knee
arthroplasty, has not been definitively proven to be an option for revision total elbow arthroplasty.
Single-stage revision has shown moderate success in the setting of Staphylococcus
aureusinfections, although with only short-term follow-up. A resection arthroplasty would likely be
successful in managing the deep infection but would not optimize the functional result. Resection
arthroplasty is best reserved for low-demand or infirm patients.

88. Figures 1 and 2 are the radiographs of a 61-year-old woman with a left elbow injury after a fall
onto her outstretched hand. She denies any previous injury to her elbow. She undergoes a
closed reduction of her elbow in the emergency department. What is the most appropriate next
step in definitive management?

Figure 1 Figure 2
1. Semiconstrained total elbow arthroplasty
2. Open reduction internal fixation (ORIF) of coronoid and radial head
3. ORIF of coronoid, radial head replacement, and lateral collateral ligament repair
4. ORIF of coronoid, radial head resection, and lateral collateral ligament repair

Preferred Answer :3
The radiographs demonstrate a coronoid fracture and a comminuted radial head fracture with
residual subluxation of the ulnohumeral joint. These findings comprise the so-called
&quot;terrible triad.&quot; The mainstay of treating this injury involves radial head fixation or
replacement, fixation of the coronoid fracture, and repair of lateral collateral ligament disruption.

UI // UNAIR // UNPAD // UNHAS // UNS // UGM // UB // UNUD // USU


Shoulder and Elbow Self-Assessment Examination 2019

Resection of the radial head in terrible triad injuries is contraindicated, as this structure serves as
a secondary restraint to valgus stress of the elbow. Fixation of the radial head and coronoid
without concomitant repair of the lateral collateral ligament will not render the elbow stable. A
semiconstrained total elbow arthroplasty has significant short- and long-term complications and
functional limitations and would be inappropriate for this clinical setting.

89. A 55-year-old man falls from a ladder and dislocates his nondominant shoulder. He undergoes
an uncomplicated closed reduction under sedation in the emergency department. Postreduction
radiographs reveal a small Hill-Sachs lesion and no other bony abnormalities. Six weeks after the
dislocation, the patient has persistent pain at rest and forward elevation and external rotation
weakness, but the remaining motor function in the extremity and sensation are intact. What is the
best next step?

1. Physical therapy with electrical stimulation and iontophoresis


2. Corticosteroid Injection
3. MRI of the shoulder
4. Electromyography (EMG) of the arm

Preferred Answer :3
For a patient >40 years of age who has persistent pain and weakness isolated to the rotator cuff
following an acute anterior shoulder dislocation, an MRI is indicated to evaluate rotator cuff
integrity. EMG is not indicated in this case because this patient has no evidence of distal motor
functional abnormality and their sensation is intact, thereby making a brachial plexus injury
unlikely. Corticosteroid injections and physical therapy modalities do not adequately address the
concern over his potential for having sustained a rotator cuff tear.

90. A 17-year-old girl develops chronic posterolateral rotatory instability (PLRI) of the elbow following
closed treatment of an elbow dislocation. Advanced imaging reveals incompetence of the lateral
collateral ligament complex, and ligament reconstruction is planned. Examination under
anesthesia is performed with the forearm in maximal supination and valgus force applied to the
elbow, demonstrated in Video 1. As the elbow is brought through a range of motion assessment,
the radial head is?

1. Dislocating posteriorly in extension and reducing in flexion.


2. Dislocating posteriorly in flexion and reducing in extension.
3. Dislocating anteriorly in extension and reducing in flexion.
4. Dislocating anteriorly in flexion and reducing in extension.

Preferred Answer :1
PLRI of the elbow is the most common form of chronic elbow instability. The mechanism occurs
following a fall onto an outstretched hand, where a valgus force is applied to the elbow and the
forearm rotates into progressive supination. This allows the radial head to translate posterior to
the capitellum, with progressive injury from lateral to medial sides of the elbow. The pivot shift
test is a useful examination maneuver to confirm the presence of PLRI. With the forearm in
maximal supination and valgus stress applied to the elbow, the radial head is forced posterior to
the capitellum as the elbow is brought into progressive extension, revealing a dimple on the

UI // UNAIR // UNPAD // UNHAS // UNS // UGM // UB // UNUD // USU


Shoulder and Elbow Self-Assessment Examination 2019

lateral aspect of the elbow. This typically occurs at roughly 30⁰ of flexion. As the elbow is flexed,
the radial head reduces.

91. What phase of overhead throwing puts the rotator cuff at most risk of injury from internal
impingement?

1. Wind up
2. Late cocking
3. Deceleration
4. Follow through

Preferred Answer :2
Internal impingement occurs when there is repetitive contact of the posterior superior aspect of
the glenoid with the humeral head causing damage to the undersurface of the supraspinatus and
anterior aspect of the infraspinatus tendons, as well as posterior superior glenoid labrum. This
occurs when the arm is in maximum abduction and external rotation such as during the late
cocking phase of the normal throwing motion. The 6 phases of throwing are wind up, early
cocking, late cocking, deceleration, and follow through. When the arm is repeatedly placed in the
abducted externally rotated position, the anterior capsule can become lax and posterior capsular
contractures can develop. When there are kinetic chain abnormalities such as scapular internal
rotation or muscle fatigue, there is exacerbation of abnormal anterior humeral head translation
and increased contact of the rotator cuff on the posterior glenoid rim, with concomitant increased
risk of injury and symptoms.

92. Placing a plate too anteriorly against the lateral aspect of the bicipital groove while performing
open reduction and internal fixation (ORIF) of a proximal humerus fracture has an increased risk
of what complication?

1. Avascular necrosis
2. Loss of fixation of the fracture
3. Malunion leading to increased retroversion of the articular surface
4. Glenoid arthrosis

Preferred Answer :1
There are two major arteries that supply the humeral head. One is the ascending branch of the
anterior humeral circumflex artery, which runs up the lateral aspect of the bicipital groove
terminating in the arcuate artery. The other is the posterior humeral circumflex artery, which more
recently has been demonstrated to supply a significant portion of the blood supply to the humeral
head. Capsular arteries also play a role in humeral head perfusion. Care should be taken to
preserve all intact arterial supply when performing ORIF, as injury to these arteries may result in
avascular necrosis. In general, the most common complications of locked plating include loss of
reduction with penetration of the joint by the screws, particularly with initial varus positioning of
the humeral head. Placement of the plate in the position described, however, should not have an
impact on any of the other complications noted.

UI // UNAIR // UNPAD // UNHAS // UNS // UGM // UB // UNUD // USU


Shoulder and Elbow Self-Assessment Examination 2019

93. Figure 1 is the radiograph of a 27-year-old bicyclist who was involved in a crash. He has an
isolated and closed injury. He is neurovascularly intact in the upper extremity. The lateral
fragment is displaced inferiorly by?

Figure 1

1. Gravity.
2. The trapezius.
3. The biceps.
4. The pectoralis minor.

Preferred Answer :1
Open reduction and internal fixation with a plate and screw construct have been demonstrated to
reduce nonunion rate and improve outcomes compared with sling immobilization for displaced
clavicle fractures. Neurovascular injury and infection risk increase, however, with surgery. In the
upright position, the weight of the extremity inferiorly displaces the lateral segment.

94. A 20-year-old collegiate pitcher sustains a medial collateral ligament (MCL) rupture of his
throwing elbow for which surgical reconstruction is necessary. The goal of surgery is anatomic
restoration of the MCL. Which statement best describes the kinematics of the native MCL?

1. The posterior bundle demonstrates the greatest change in tension from flexion to
extension.
2. The posterior bundle is isometric.
3. The anterior bundle becomes tight in flexion and lax in extension.
4. The anterior and posterior bundles are isometric.

Preferred Answer :1
The anterior bundle is the most important portion of the complex when treating valgus instability
of the elbow. The ligament originates from the anteroinferior surface of the medial epicondyle.
The anterior bundle inserts on the medial border of the coronoid at the sublime tubercle.
The anterior bundle of the medial collateral ligament (MCL) is the primary restraint to valgus
stress, and the radial head is a secondary restraint. With anterior bundle sectioning, the resultant
instability is most substantial between 60° and 70° and is lowest at full extension and full flexion.
True lateral radiographs reveal that the flexion-extension axis, or center of rotation, of the elbow
lies in the center of the trochlea and capitellum. The origin of the anterior bundle of the MCL lies
slightly posterior to the rotational center of the elbow. The anterior bundle is further divided into
an anterior band and a posterior band. The eccentric origin of these anterior bundle components

UI // UNAIR // UNPAD // UNHAS // UNS // UGM // UB // UNUD // USU


Shoulder and Elbow Self-Assessment Examination 2019

in relation to the rotational center through the trochlea creates a CAM effect during flexion and
extension. The anterior band tightens during extension, and the posterior band tightens during
flexion. This reciprocal tightening of the two functional components of the anterior bundle allows
the ligament to remain taut throughout the full range of flexion.
Cadaver dissection studies have identified the origin and insertion of both the medial and lateral
stabilizing elbow ligaments. The anterior bundle of the MCL is isometric throughout the
flexion/extension arc of motion, making Response C incorrect. The posterior bundle of the MCL
elongates with elbow flexion, so Responses B and D are incorrect. The posterior bundle of the
MCL also demonstrates the most change in length from extension to flexion of all the elbow
ligaments.

95. Figure 1 is the MRI of a 45-year-old woman with a medical history significant for rheumatoid
arthritis who returns to your office with persistent right elbow pain. Her rheumatologist has
maximized her disease-modifying anti-rheumatoid drug regimen. She complains of diffuse joint
pain and swelling. On examination, she has a pronounced joint effusion, elbow flexion arc of 45°,
and crepitus with forearm rotation. Her elbow radiograph reveals preservation of her joint space.
What is the most appropriate surgical treatment at this time?

Figure 1

1. Total elbow arthroplasty


2. Synovectomy with radial head resection
3. Synovectomy without radial head resection
4. Isolated radial head resection

Preferred Answer :3
Rheumatoid arthritis remains a common inflammatory arthropathy that can lead to progressive
synovitis of the elbow joint. Patients often present with recalcitrant elbow pain and loss of motion.
In the early stages, the joint space can be fairly well preserved. With progressive synovitis,
cartilage destruction leads to symmetric joint space narrowing and joint destruction. For the
younger patient with recalcitrant synovitis and a relatively well-preserved joint space, open or
arthroscopic synovectomy provides successful improvement for 70% to 80% of patients. In most
cases, radial head resection is not required. Synovitis that encircles the radial head and neck can
lead to pain and crepitus with forearm rotation. Preserving the radial head prevents the rapid
progression of wear at the lateral ulnohumeral joint. A total elbow replacement, while a
successful treatment modality for the older, lower demand patient with rheumatoid arthritis, would
not be appropriate for the younger patient given the significant postoperative restrictions
imparted.

UI // UNAIR // UNPAD // UNHAS // UNS // UGM // UB // UNUD // USU


Shoulder and Elbow Self-Assessment Examination 2019

96. A 45-year-old man falls from a skateboard and dislocates his elbow. After a closed reduction in
the emergency department, his elbow is carefully examined. He has positive valgus stress,
moving valgus stress, and milking maneuver tests. His elbow appears stable to varus stress and
lateral pivot shift tests. What is the most appropriate manner of immobilizing the elbow for this
patient?

1. Sling for 3 days, with early active range of motion


2. Posterior splint for 5 to 7 days, forearm in full pronation
3. Posterior splint for 5 to 7 days, forearm in neutral
4. Posterior splint for 5 to 7 days, forearm in full supination

Preferred Answer :4
Varus posteromedial rotatory instability occurs following a fall onto an outstretched hand with
axial loading and a varus stress to the elbow. This injury can result in a rupture of the posterior
band of the medial collateral ligament (MCL), fracture of the anteromedial facet of the coronoid,
and avulsion of the lateral ulnar collateral ligament (LUCL). Based on the examination findings,
this patient has an acute MCL rupture. Furthermore, the LUCL appears intact, as evidence by the
stability with varus stress. To protect the reduction in the acute setting, posterior splinting is
recommended, but placing the forearm in full supination tightens the structures medially where
the MCL is deficient. Splinting in neutral is indicated for valgus posterolateral rotatory instability,
where both the LUCL and MCL are ruptured. Splinting in full pronation is indicated for isolated
LUCL ruptures. Early active range of motion is not recommended for adults immediately after an
acute elbow dislocation, as ligamentous injury or fracture nearly always accompanies the
dislocation. In this case, the forearm should be splinted in full supination.

97. A 23-year-old minor league pitcher describes the insidious onset of posterior shoulder pain
during the late cocking phase of his throwing motion. He has gone 6 weeks without throwing, but
symptoms have returned on return to play. An MR arthrogram of the shoulder reveals fraying of
the superior labrum and proximal biceps, and a partial-thickness articular-sided supraspinatus
tear (30% tendon thickness). Figure 1 is a representative coronal MRI slice. Clinical examination
demonstrates mild weakness of the periscapular muscles, mild superior rotator cuff weakness,
and negative instability testing. Internal rotation with the arm in 90° of abduction is 40° in the
affected shoulder versus 70° in the contralateral shoulder. What is the best next step?

Figure 1

UI // UNAIR // UNPAD // UNHAS // UNS // UGM // UB // UNUD // USU


Shoulder and Elbow Self-Assessment Examination 2019

1. Intra-articular platelet rich plasma (PRP) injection


2. Therapy regimen focused on shoulder range of motion and strengthening
3. Arthroscopic surgery for rotator cuff and labral debridement
4. Arthroscopic surgery for rotator cuff repair and biceps tenodesis

Preferred Answer :2
Internal impingement is a condition that affects overhead throwing athletes, as the greater
tuberosity and articular surface of the rotator cuff contact the posterosuperior glenoid during
maximal shoulder abduction and external rotation. The etiology is typically multifactorial, but
common contributors include posterior capsular contracture, scapular dyskinesia, and subtle
anterior shoulder laxity. Nonoperative management, the mainstay of treatment, includes rest,
stretching, scapular strengthening, and proprioception. Results of surgical intervention are
variable; therefore, nonoperative measures should be exhausted first. While PRP is currently
being investigated as a biologic augmentation in a number of shoulder pathologies, it is not
considered first-line treatment.

98. A 67-year-old man with right shoulder osteoarthritis (OA) remains symptomatic despite a course
of nonsurgical treatment. A CT scan of the shoulder shows eccentric posterior glenoid wear with
10° of retroversion. What is the appropriate management of this glenoid bone loss during surgery
for an anatomic total shoulder arthroplasty?

1. In situ glenoid component implantation


2. Hemiarthroplasty
3. Eccentric reaming of glenoid
4. Posterior glenoid bone graft

Preferred Answer :3
Total shoulder arthroplasty (TSA) is superior to hemiarthroplasty for primary OA. The most
common complication of TSA is glenoid loosening and malposition, which are common causes of
glenoid failure. Glenoid malposition decreases the glenohumeral contact area and subsequently
increases contact pressures. Altering the stem version to accommodate glenoid retroversion
does not appropriately address soft-tissue balancing. A retroversion of <12° to 15° can be
corrected with eccentric reaming without excessively compromising glenoid bone stock and
risking glenoid vault penetration by the glenoid component. Posterior glenoid bone grafting may
be considered for glenoid retroversion >15°.

99. Figures 1 through 4 are the radiograph and MRI scans of a 70-year-old woman who has a 10-
year history of worsening shoulder pain. She has had multiple corticosteroids, several rounds of
physical therapy, and continues to take nonsteroidal anti-inflammatory medications. She has pain
with all activities and has to use the contralateral hand to aid in elevation of the arm. The pain
prevents her from sleeping. Her active forward elevation is 40°; her passive forward elevation is
160° with a positive lag sign. Her active external rotation with the arm at the side is 10°; her
passive external rotation is 40° with a positive lag sign. Her hornblower’s sign is negative. What
would be the most effective treatment option for this patient?

UI // UNAIR // UNPAD // UNHAS // UNS // UGM // UB // UNUD // USU


Shoulder and Elbow Self-Assessment Examination 2019

Figure 1 Figure 2 Figure 3 Figure 4

1. Arthroscopic rotator cuff repair


2. Anatomic total shoulder arthroplasty
3. Reverse total shoulder arthroplasty with latissimus transfer
4. Reverse total shoulder arthroplasty

Preferred Answer :4
This patient has failed nonsurgical measures and continues to demonstrate pseudoparalysis.
The images show proximal humeral migration with acetabularization of the acromion. The teres
minor is intact and the hornblower’s sign is negative. This patient would benefit most from a
reverse total shoulder arthroplasty. If there was severe atrophy or absence of teres minor and/or
if the hornblower’s sign was positive, then the best choice for treatment to restore external
rotation would be reverse total shoulder arthroplasty with a latissimus transfer. Ten-year follow-
up following reverse total shoulder arthroplasty for the treatment of a massive rotator cuff tear in
the setting of arthritis demonstrated >90% survivorship. All patients demonstrated improved
range of motion and pain relief, as well as improved patient-reported outcomes scores. The key
to restoration of function, especially external rotation, is the teres minor. Indications for the
addition of a latissimus transfer to reverse total shoulder arthroplasty include rotator cuff tears in
which the teres minor is absent or atrophic (as all remaining external rotators are absent).
Patients who do not regain external rotation control will have a difficult time performing activities
of daily living such as bringing their hand to their mouth. The rotator cuff in the MRI scan
demonstrates significant retraction and fatty atrophy/infiltration, making repair of the tendon a
non-viable option. An unconstrained total shoulder arthroplasty requires an intact rotator cuff, or
a repairable tendon, to minimize edge-loading of the glenoid component and potential loosening.

100. A 72-year-old man sustains a displaced four-part fracture of the proximal humerus with head split
component following a fall. A primary shoulder arthroplasty has been recommended for acute
management. In counseling the patient on pros and cons of hemiarthroplasty versus reverse
arthroplasty, what statement can be made based on the available literature?

1. The risk of tuberosity nonunion/malunion appears higher with hemiarthroplasty.


2. Functional outcomes tend to be more consistent with hemiarthroplasty.
3. Forward elevation of reverse shoulder arthroplasty depends on tuberosity union.
4. Active elevation is likely to be better following hemiarthroplasty.

Preferred Answer :1
As the indications for reverse shoulder arthroplasty have expanded, the role for shoulder
hemiarthroplasty appears to be narrowing. Several recent systematic reviews have evaluated

UI // UNAIR // UNPAD // UNHAS // UNS // UGM // UB // UNUD // USU


Shoulder and Elbow Self-Assessment Examination 2019

outcomes of shoulder hemiarthroplasty and reverse shoulder arthroplasty for acute proximal
humerus fractures. Their results suggest that reverse arthroplasty results in superior functional
results and comparable elevation, at the expense of increased complication rates and decreased
shoulder rotation. One of the benefits of reverse shoulder arthroplasty in the setting of fracture is
that forward elevation is independent of tuberosity healing and relies mainly on the deltoid
muscle. Active external rotation following a reverse total shoulder for fracture, however, does
appear to depend on successful union of the greater tuberosity. In a randomized controlled trial,
the incidence of tuberosity healing was higher and the incidence of tuberosity resorption was
lower in reverse arthroplasty compared with hemiarthroplasty. Forward elevation following a
hemiarthroplasty for fracture generally follows a bimodal distribution, whereas outcomes
following a reverse total shoulder have been more consistent.

UI // UNAIR // UNPAD // UNHAS // UNS // UGM // UB // UNUD // USU

You might also like